Download as pdf or txt
Download as pdf or txt
You are on page 1of 501

NTSE MAT Stage-1 & 2

CONTENT

Sr. No. Topic Name Page No.

1. Alphabet-Test 1- 5
2. Coding-Decoding 6-30
3. Number Series 31-45
4. Alphabet Series 46-53
5. Letter Repeating Series 54-60
6. Missing Term in Figure 61-90
7. Mathematical Operations 91-107
8. Direction Sense Test 108-118
9. Sitting Arrangement 119-127
10. Ranking - Ordering-Test 128-138
11. Blood Relation Test 139-145
12. Puzzle-Test 146-160
13. Venn Diagram 161-183
14. Syllogism 184-194
15. Analogy-1 195-212
16. Classification 213-225
17. Pyramid Test 226-238
18. Calendar Test 239-244
19. Dice-Test 245-266
20. Cubes 267-281
21. Clock 282-287
22. Figure Partition – Counting 288-300
23. Mirror - Water Images 301-326
24. Paper Cutting –Folding 327-347
25. Completion - Formation of Figures 348-364
26. Figure Embedded 365-374
27. Non-Verbal Series 375-401
28. Non-Verbal Analogy 402-427
29. Non-Verbal Classification 428-446
30. Logical Sequence of Words 447-452
31. Sequential Output Tracing 453-464
32. Arithmetical Reasoning 465-473
33. Logical Deduction 474-488
34. Data Sufficiency 489-494
35. Data Redundancy 495-499
NTSE-MENTAL ABILITY

1. ALPHABET-TEST

Alphabet Order
You have to arrange the given words in order in which they are arranged in a dictionary. In a dictionary the
words are placed in alphabetical order w.r.t the second alphabet of the words and so on (that is third
alphabet, fourth alphabet……)

Directions: (1 to 2) Arrange the given words in the correct alphabetical order.

1. Late, Long, Liver, Last, Load, Luminous, Loan, Lock


Sol. The given words can be arranged in the alphabetical order as
Last, Late, Liver, Load, Loan, Lock, Long. Luminous

2. Arrange the given words in alphabetical order and tick the one that comes last one.
Heavy, Heredity, Hesitate, Hedge, Hero. Haste, History, Hinderance

Sol. The given words can be arranged in the alphabetical order as:
Haste, Heavy, Hedge, Heredity, Hero, Hesitate, Hinderance, History Clearly, History comes last

3. Arrange the given words in the order they occur in dictionary.


1. SIGN 2. SOLID 3. SCENE 4. SIMPLE
(A) 3, 1, 2, 4 (B) 3, 1, 4, 2 (C) 3, 4, 1, 2 (D) 3, 4, 2, 1

Sol (B) The correct alphabetical order of the given words is: SCENE, SIGN, SIMPLE, SOLID. Thus,
the correct sequence is 3, 1,4, 2.

Letter-Word Problems

4. How many pairs of letter in the word DABBLE have as many letters between them as in the
Alphabet series ?
(A) Nill (B) One (C) Two (D) More than three

Sol. (D) Letters in the given Word Letters in the Alphabet


AB AB
D AB DCB
B BLE BC D E
A BB L E A BC DE

5. In the word PARADISE how many pairs of letters are there which have as many letters between
them in the word as in the alphabet ?
(A) None (B) One (C) Two (D) Three

Sol, (D) Letter in the given word Letter in the alphabet


(i) P A R PQR
(ii) A RA D ABCD
(iii) A D ISE A BCD E

1 PACE IIT & MEDICAL: Mumbai / Delhi & NCR / Goa / Akola / Kolkata / Nashik / Pune / Bokaro / Dubai
NTSE-MENTAL ABILITY

6. Number of letters skipped in between adjacent letters in the series are increased by one. Which of the
following alternatives observes this rule ?
(A) KMPTY (B) IJKOT (C) HJMQT (D) DFIJK

Sol. (A)
K L M N O P Q R ST U V W X Y
1 2 3 4
Clearly, in letter series KMPTY, the number of letters skipped in between adjacent letters in the
series are increased by one.

Alphabetical Quibble
In this type of questions. generally a letter-series is given, be it the English alphabets from A to Z or Z to A.
The candidate is then required to trace the letters satisfying certain given conditions as regards their position
in the given sequence or the sequence obtained by performing certain given operations on the given
sequence.

7. If the alphabet series is written in a reverse order, which of the following will be the seventh letter to
the left of eighth letter from your right ?
(A) L (B) M (C) O (D) P
Sol. (C) The new alphabet series is:
ZYXWVUTSRQPONMLKJIHGFEDCBA
The eight letters from the right is H.
The seventh letter to the left of H is O.

8. If the first four letters of the word ‘ANTHROPOMORPHISM' are rewritten in the reverse order
followed by the next four rewritten in the reverse order followed by the next four letters in the
reverse order and so on, which letter will be twelfth from the left end in the rewritten order?
(A) O (B) H (C) M (D) P
Sol. (C) The new arrangement is as follows HTNAOPORPROMMSIH.

Word Formation by Unscrambling Letters


In this type of questions, a set of English letters is given in a jumbled order. The candidate is required to
arrange these letters to form a meaningful word.

9. Arrange the following group of letters such that when arranged in a specific order, meaningful word
is formed
VA HYE
1 2 3 4 5
(A) 2, 3, 4, 5, 1 (B) 3, 2, 5, 1, 4 (C) 3, 5, 2, 1, 4 (D) 1, 5, 2, 3,4

Sol: (C) The given letter, when arranged in the order 3, 5, 2, 1, 4, form the word HEAVY.

Word formation using letter of a given word


10. A meaningful word starting with A is made from the first, the second, the fourth, the fifth and the
sixth letters of the word CONTRACT, which of the following is the middle letter of the word?
(A) C (B) T (C) O (D) R
Sol. (B)
The first, the second, the fourth, the fifth and sixth letters of the word CONTRACT are C, O, T, R
and A respectively. The meaningful word will be ACTOR and T will be the required letter.

2 PACE IIT & MEDICAL: Mumbai / Delhi & NCR / Goa / Akola / Kolkata / Nashik / Pune / Bokaro / Dubai
NTSE-MENTAL ABILITY

Direction: (11) In the following question, find which one word cannot be made from the letters of the given
word.

11. TEMPERAMENT
(A) METER (B) PETER (C) TENTER (D) TESTER

Sol. The word TEMPERAMENT contains all the letters of the word TESTER except S. So, the word
TESTER cannot be formed.

EXERCISE
Directions: (1 to 2) Arrange the given words in alphabetical order and tick the one that comes first.

1. (A) Grammar (B) Granary (C) Gradient (D) Grand

2. (A) Mahender (B) Mahendra (C) Maninder (D) Mahindra

3. If the following words are arranged in an alphabetical order, which word will appear in the second ?
(A) Principal (B) Principle (C) Principia (D) Principled

4. If the following words are arranged as found in the dictionary, then what will be the fourth letter
from the left in the last word?
INTIMATION, INFORMATION, INTEREST, INTERROGATION, INSTIGATION
(A) R (B) O (C) T (D) I

5. How many pairs of letters are there in the word CARROT which have as many letters between them
in the word as in the alphabet ?
(A) l (B) 2 (C) 3 (D) 4

6. How many pairs of letters are there in the word HORIZON which have as many letters between them
in the word as in the alphabet ?
(A) one (B) two (C) three (D) more than three

7. How many pairs of letters in the word BRIGHTER have as many letters between them in the Word
as in the alphabet ?
(A) 2 (B) 3 (C) 4 (D) more than 4

8. Number of letters skipped in between adjacent letters in the series decreases by two. Which of the
following series observes this rule ?
(A) FQWBG (B) HQXCF (C) TBINO (D) XFMQU

9. Number of letters skipped in between adjacent letters in the series increases by one. Which of the
following series observes this rule ?
(A)DBPUY (B) DBUYP (C) DBYPU (D) DBYUP

10. Which letter will be the midway between the fourteenth letter from the left end and nineteenth letter
from the right end of the following alphabet?
ABCDEFGHIJKLMNOPQRSTUVWXYZ
(A) I (B) K (C) M (D) G

3 PACE IIT & MEDICAL: Mumbai / Delhi & NCR / Goa / Akola / Kolkata / Nashik / Pune / Bokaro / Dubai
NTSE-MENTAL ABILITY

11. Which letter will be the sixth to the left of the eleventh letter from the right end of the alphabet ?
(A) K (B) V (C) J (D) U

12. If the alphabets were written in the reverse order, which letter will be the fifth letter to the left of the
fourteenth letter from the left?
(A) R (B) I (C) S (D) H

13. Which letter Should be ninth letter to the left of ninth letter from the right, if the first half of the
alphabet series is reversed ?
(A) D (B) E (C) F (D) I

14. If the positions of the first and sixth letters of the word BENEFICIAL are interchanged; similarly the
positions of the second and seventh letters are interchanged and so on, which letter will be third from
the right end after rearrangement?
(A) C (B) E (C) F (D) N

Directions : (15 to 17) In each of the following questions, a group of letters is given which are numbered 1,
2, 3, 4, 5 and 6. Below are given four alternatives containing combinations of these numbers.
Select that combination of numbers so that letters arranged accordingly, form a meaningful word.

15. RACET
1 2 34 5
(A) 1, 2, 3, 4, 5 (B) 3, 2, 1, 4, 5 (C) 5, 2, 3, 4, 1 (D) 5, 1, 2, 3, 4

16. RUSGA
1 2 34 5
(A) 1, 5, 4, 2, 3 (B) 5, 3, 4, 1, 2 (C) 3, 2, 4, 5, 1 (D) 4, 5, 3, 2, 1

17. RTAOUH
1 23 4 5 6
(A) 1, 3, 4, 5, 6, 2 (B) 2, 3, 6, 4, 5, 1 (C) 6, 3, 2, 4, 5, 1 (D) 3, 5, 2, 6, 4, 1

18. If a meaningful word can be formed by rearranging the letters USCALA, the first letter of the word
so
formed is the answer. If no such word can be formed the answer is X.
(A) C (B) S (C) A (D) L

19. A meaningful nine-letter English word is formed using all the alphabets given in the grid below,
starting with alphabet of a corner block, moving in clockwise direction and ending at the alphabet in
the central grid. What is the fourth alphabet of the word?

(A) o (B) d (C) t (D) m

4 PACE IIT & MEDICAL: Mumbai / Delhi & NCR / Goa / Akola / Kolkata / Nashik / Pune / Bokaro / Dubai
NTSE-MENTAL ABILITY

20. If it is possible to make a meaningful word with the third, the fourth and the eleventh letters of the
word CONTROVERSIAL, which of the following will be the last letter of that word ? If more than
one such words can be made, give M as the answer and if no such word is there, give X as the
answer.
(A) N (B) I (C) X (D) M

21. If it is possible to make a meaningful word With the second the sixth, the ninth and the twelfth letters
of the word CONTRIBUTION, which of the following will be the last letter of that word ? If more
than one such words can be made, give M as the answer and if no such word is there, give X as the
answer.
(A) N (B) O (C) X (D) M

22. A word given in Capital Letters is followed by four answer words. Out of these only one can be
formed by using the letters of the given words. Find out that word. SOMNAMBULISM
(A) NAMES (B) BASAL (C) SOUL (D) BIOME

Directions: (23 to 24) In each of the following questions, find which one word cannot be made from the
letters of the given word.

23. KALEIDOSCOPE
(A) SCALE (B) PADLOCK (C) PACKET (D) DIESEL

24. SUPERIMPOSABLE
(A) SPIRE (B) REPTILE (C) POSSIBLE (D) REPOSE

25. If Letters of alphabets are written in reverse way then which letter will be seventh letter right to Q ?
(NTSE Stage-I /Raj./2007)
(A) K (B) U (C) J (D) W

26. Select the word from given alternatives which cannot be written from the letters of EDUCATED
word
(NTSE Stage-I/Raj./2007)
(A) DUCK (B) CUTE (C) TADE (D) ACTE

27. Which name will come at 3rd place in a telephone directory from the following given names?
(NTSE Stage-I/Raj./2014)
(A) AMIT (B) AMINA (C) ALOK (D) ABHIMAN

Answer Key

1. (c) 2. (a) 3. (c) 4. (d) 5. (a) 6. (d) 7. (b) 8. (b) 9. (d) 10. (b)
11. (c) 12. (a) 13. (b) 14. (d) 15. (d) 16. (c) 17. (d) 18. (a) 19. (d) 20. (d)

21. (b) 22. (c) 23. (c) 24. (b) 25. (c) 26. (a) 27. (b)

5 PACE IIT & MEDICAL: Mumbai / Delhi & NCR / Goa / Akola / Kolkata / Nashik / Pune / Bokaro / Dubai
NTSE-MENTAL ABILITY

2. CODING-DECODING

Coding-Decoding
A Code is "a system of signals". Coding is therefore, a method of transmitting a message between sender
and receiver which cannot be understood or comprehended by a third person. The coding decoding test is
set up to judge a candidate's ability to decipher a particular word/ message and break the code to decipher
the message. In coding, actual alphabets/ words/ terms/numbers are replaced by certain other alphabets/
words/ terms/symbols etc. according to a specific rule. To solve these type of questions we have to detect the
rule and then answer the questions.

Decoding: It is a method to find the meaning of something that has been written in code.

Letter-Letter Coding
In these types of questions, the letters in a word are replaced by certain other letters according to a specific
rule to form its code. The candidate is required to detect the coding pattern / rule and answer the questions
accordingly.

1. If in any code language, KUMAR is coded as LVNBS, How is EMOTIONAL coded in that
language?
(A) FNQUJQBM (B) FNPUJPOBM
(C) GNPUJPOBM (D) GNQUJQOBM

Sol. (B)

Similarly

2. If JAPAN is coded as KCSES, then the code for CASTLE will be


(A) DCIJOB (B) DCJKRD (C) DCKMSG (D) DCVXQK

Sol. (D)

6 PACE IIT & MEDICAL: Mumbai / Delhi & NCR / Goa / Akola / Kolkata / Nashik / Pune / Bokaro / Dubai
NTSE-MENTAL ABILITY

Similarly,

3. If PAINTER is written in a code language as NCGPRGP , then REASON would be written as:
(A) PCYQMN (B) PGYQMN (C) PGYUMP (D) PGYUPM

Sol. (C)

Similarly,

4. If in any code language NATIONAL is written as MZGRLMZO than how is JAIPUR written in
that language
(A) QZRKFI (B) PZRKFI (C) QZRIFK (D) QARKFI

Sol. (A)

Similarly,

7 PACE IIT & MEDICAL: Mumbai / Delhi & NCR / Goa / Akola / Kolkata / Nashik / Pune / Bokaro / Dubai
NTSE-MENTAL ABILITY

Letter-Number Coding
In these types of questions, either numerical code values are assigned to a word or alphabetical code letters
are assigned to the numbers.

5. In a certain code, if TREE is coded as 7100, FROG as 2159, how is FREE coded in that code ?
(A) 2100 (B) 3100 (C) 1003 (D) 1002
Sol. TREE FROG
7100 2159
Hence, FREE  210 0

6. In a certain code, C is coded as 0, E as 7, T as 4,I as 9, P as 1, R as 3, and U as 5. How is 1904537


coded in that code ?
(A) PICTRUE (B) PICTURE (C) RICTPUE (D) PCTUREI

Sol. (B) 1 9 0 4 5 3 7
  
P I CTURE

7. If OX is coded as 39, what will be the code number for LION?


(A) 20 (B) 25 (C) 38 (D) 50

Sol. (D)
By their natural position in alphabet,
O  15,X  24
So, OX  15  24   39
Hence, L I O N  12  9  15  14   50

8. If AJAY is written as 1117, then in same code NAMA would be written as:
(A) 5114 (B) 5411 (C) 5141 (D) 4511

Sol. (C)

Similarly,

Substitution Coding
In these types of questions, some particular objects are assigned code names. Then a question is asked that is
to be answered in the code language.

8 PACE IIT & MEDICAL: Mumbai / Delhi & NCR / Goa / Akola / Kolkata / Nashik / Pune / Bokaro / Dubai
NTSE-MENTAL ABILITY

9. If paper is called eraser, eraser is called bag, bag is called scale, scale is called pencil and
pencil is called paper, what will a person write with?
(A) Pencil (B) Paper (C) Eraser (D) Bag

Sol. (B)
A person will write with a pencil and a 'pencil' is called 'paper’.

10. If water is called food, food is called tree, tree is called sky, sky is called wall, on which of the
following does a fruit grow ?
(A) Water (B) Food (C) Tree (D) Sky
Sol. (D)
Clearly, a fruit grows on a tree'. As given that tree' is called 'sky, a fruit grows on 'sky.

Puzzle Based Coding


In this type of questions, some messages are given in the coded language and the code for a particular word
or message is asked. To analyses Such codes, any two messages bearing a common word are picked up. The
common code Word will thus represent that word. Proceeding similarly by picking up all possible
combinations of two, the entire message can be decoded and the order for individual words found.

11. In a certain code language 389 means run very fast. 964 means come back fast and 487 means run
and come. Which digit in the language means come?
(A)7 (B) 9 (C) 4 (C) 8

Sol. (C)
In the second and third sentences, common number is 4 and common code is 'come'.
Hence, number 4 stands for 'come'.

12. In a certain code language, ken poti means good morning, hu shang means come on, and hu ken
sue means come for good. Which word in that language does mean for ?
(A) shang (B) ken (C) sue (D) hu

Sol. (C)
In the second and third statements, the common code word is 'come' and the common code word is
‘hu’. So, ‘come’ means ‘hu’. In the first and third statements, the common code word is ‘good’ and
the common code word is ‘good’ and the common word is ‘ken’ . So, ‘good’ means ‘ken’. Thus, in
third and above statements ‘for’ means ‘sue’

Column Coding
13. Decode the underlined letters in column I from the same row of choices provided under column II.
Each small letter in column II stands for some capital letter in column I. However, the small letters in
column II are not arranged in the same order as their corresponding letters in column I. The code is
the same for all the terms in column I.
Column I Column II
1. HNTBZ vbhnt

2.
CT NZB thnwv

3.
DNBZC xhvtw
4. OHNT Z tbhin
5. TZOBK nivet

9 PACE IIT & MEDICAL: Mumbai / Delhi & NCR / Goa / Akola / Kolkata / Nashik / Pune / Bokaro / Dubai
NTSE-MENTAL ABILITY

Sol. From terms 1 & 2, in column (I), NTBZ is common. From col. (Il) we have vhnt common. Hence H
= b & C = w. From term-3, NBZC have already occurred and the code for D must be a letter which
did not appear in the earlier codes. Thus, the codes for D is x. From term-4 HNTZ have occured
earlier. The letter which has not occured earlier is the code for O, that is, i. From terni-5 TZOB have
occured earlier. The letter which has not occured earlier is the code for K, that is e. The code of
underlined letters are given in the following table

Letter H C D O K
Code b w x i e

Directions : (14 to 16) In column I below, some words are given. In column II, their codes are given but
they are not arranged in the same order. Study both the columns and find out the code for the letter given in
each of the following questions from among the given alternatives. The code for a letter will be same
throughout.
Column I Column II
(i) DRGEX (a) 12567
(II) AXPRD (b) 12358
(iii) SDRKG (c) 1 2470
(iv) KLPSX (d) 34590
(v) SGPAK (e) 34780
(vi) PXDAG (f) 23578
(vii) GKSAE (g) 46780

14. What is the code used for the letter D?


(A) 2 (B) 3 (C) 5 (D) 1

15. What is the code used for the letter P?


(A) 8 (B) 5 (C) 3 (D) 2

16. What is the code used for the letter E?


(A) 4 (B) 7 (C) 8 (D) 6

Sol. (14 to 16)


14. (A) In statement (iii) and (vi), common letters are D and G and common code digit are 2 and 7.
Hence, it is clear that D and G stand for 2 and 7 but not respectively. From statement (y), it is clear
that the word has letter G and code 7 in its coding. Hence, code for G is 7 and D is 2.

15. (C) In statement (ii) and (iv), common letters are P and X and common code digit are 3 and 5.
Hence, it is clear that P and X stand for 3 and 5 but not respectively. From statement (y), it is clear
that the word has letter P and code 3 in its coding. Hence, code for P is 3.

16. (D) In statement (I) and (vii), common letters are G and E and common code digit are 6 and 7.
Hence. it is clear that G and E stand for 6 and 7 but not respectively. From statement (vi), it is clear
that the word has letter G and code 7 in its coding. Hence, code for E is 6.

Directions : (17 to 19) In each questions there is a word written in capital letters with one letter underlined
For each letter in that word there is a code written in small letters. That code is denoted by either (A), (B),
(C), (D) or (E) not in the same order. You have to find out the exact code for the underlined letter in the
word. The number of that code is the answer. Please note that the same letter appearing in other word (s)
may be coded differently.

10 PACE IIT & MEDICAL: Mumbai / Delhi & NCR / Goa / Akola / Kolkata / Nashik / Pune / Bokaro / Dubai
NTSE-MENTAL ABILITY

17. MAGIC
(A) km (B) eg (C) ik (D) ce
Sol. (C)
 'oq ', A  2  c  2  e 
M  2  o  2  q  
 'ik ', I  2  k  2  m 
'ce ', G  2  i  2  k  
 'eg '
'km 'and C  2  e  2  g 

18. QUITE
(A) hj (B) su (C) tv (D) pr
Sol. (D)
 'pr ', U  1 t  2  v 
Q  1 p  2  r   ' tv '
 'hj', T  1 s  2  u 
I  1 h  2  j   'su ' and E  1 d  2  f 
 'df ' .

19. BLAST
(A) i (B) e (C) w (D) p (E) d

Sol. (D)
B  3 is ‘e’, L  3 is 'i ', A  3 is 'd 'S  3 is ' p ' and T  3 is ‘w’.

11 PACE IIT & MEDICAL: Mumbai / Delhi & NCR / Goa / Akola / Kolkata / Nashik / Pune / Bokaro / Dubai
NTSE-MENTAL ABILITY

EXERCISE
1. If RADIO is written PYBGM, then how would OQDKNG be written in that code?
(a) MOBIEL (b) MOBLIE (c) MOIBLE (d) MOBILE

2. If TRIANGLE is coded as SSHBMHKF, then SQUARE would be


(a) RRIASF (b) RPVBSF (c) RRTBQF (d) RPVBSD

3. If CRICKETER is coded as DQJBLDUDS, them PLAYER will be coded as:


(a) QMBZFS (b) OMZZDS (c) QKBXFQ (d) QKBZDS

4. In a certain code, GIGANTIC is written as GIGTANCI. How is MIRACLES written in that code?
(a) MIRLCAES (b) MIRLACSE (c) RIMCALSE (d) RIMLCAES

5. In a certain code, INSTITUTION is written as NOITUTITSNI. How is PERFECTION written in that


code?
(a) NOICTEFREP (b) NOITCEFERP (c) NOITCEFRPE (d) NOITCEFREP

6. If POLITICS is coded as OPILITSC, then ARTICLES will be coded as:


(a) RAITLCES (b) RAITLCSE (c) NNUHPM (d) NNVHPN

7. If in a code language MENTAL is coded as 417253, then how is TEN & ANT coded in that
language?
(a) 572, 271 (b) 217, 527 (c) 572, 217 (d) 217, 572

8. If REASON is coded as 5 and BELIEVED as 7, what is the code number for GOVERNMENT?
(a) 6 (b) 8 (c) 9 (d) 10

9. If E = 5 & SAFE = 31, then PINK = ?


(a) 41 (b) 40 (c) 50 (d) 65

10. If FOX is coded as 45, what will be the code number for BOX?
(a) 41 (b) 49 (c) 55 (d) 60

11. If RAT = 42 and CAT = 57, then LATE = ?


(a) 60 (b) 70 (c) 64 (d) 74

12. If air is called water, water is called green, green is called dust, dust is called yellow and yellow is
called cloud, which of the following does fish live in?
(a) Air (b) Water (c) Green (d) Dust

13. If brightness is called darkness, darkness is called green, green is called blue, blue is called red, red is
called white and white is called yellow then what is the colour of blood?
(a) Red (b) Darkness (c) White (d) Yellow
14. If pen is called pencil, pencil is called scale, scale is called bag and bag is called book, which is used
to carry the books?
(a) Scale (b) Pen (c) Book (d) Bag

15. In a certain code language, 3a, 2b, 7c means truth is eternal, 9a, 4d, 2b, 8b means truth does not
perish. Which of the following does mean eternal in that language?
(a) 3a (b) 2b (c) 7c (d) Cannot be determined
12 PACE IIT & MEDICAL: Mumbai / Delhi & NCR / Goa / Akola / Kolkata / Nashik / Pune / Bokaro / Dubai
NTSE-MENTAL ABILITY

16. In a certain code, 256 means you are good, 637 means we are bad and 358 means good and bad.
Which of the following does represent and in that code?
(a) 2 (b) 5 (c) 8 (d) 3

17. In a certain code language, 743 means mangoes are good, 657 means eat good food and 934 means
mangoes are ripe. Which digit means ripe in that language?
(a) 5 (b) 4 (c) 9 (d) 7

Directions: (18 to 21) Below in Column I are given some words. These have been translated into a
code language. The code equivalents of the words in Column I given in Column II are not necessarily
opposite to the corresponding words. Moreover, the codes for the different letters in each word have
also not been given in the same order as these letters occur in the original word. Study the two
columns carefully and then of the four alternatives given in each question, find the one that has the
code equivalents of the letters of the word given in the question. This is your answer.

Column I Column II
SOUND abi
ADDRESS cjmv
CRUX ikmop
NET ijktv
CRONY jkgotv
CROWDY blooppv

18. What is the code used for the letter A?


(a) b (b) l (c) v (d) None of these

19. What is the code used for the letter C?


(a) j (b) k (c) l (d) None of these

20. What is the code used for the letter D?


(a) k (b) l (c) m (d) None of these

21. What is the code used for the letter N?


(a) a (b) e (c) q (d) None of these

Directions: (22 to 25) Column I contains five capital letters while column II contains five digits. Each
letter corresponds to a single digit but not necessarily in that order.

Column-I Column-II
BEIKL 61520
PNBTK 34568
XLPBE 57401
KNIXV 27396
XBNPE 45713

22. What is the value of BIKE?


(a) 5261 (b) 6125 (c) 2560 (d) None of these

23. What is the value of PIN + NIP?


(a) 423 (b) 744 (c) 777 (d) 747

13 PACE IIT & MEDICAL: Mumbai / Delhi & NCR / Goa / Akola / Kolkata / Nashik / Pune / Bokaro / Dubai
NTSE-MENTAL ABILITY

24. What is the value of BITE – KITE?


(a) 386 (b) 1000 (c) –1000 (d) None of these

25. What is the value of NIL + NINE – TEN


(a) 4364 (b) 2738 (c) 2097 (d) None of these

Directions: (26 to 30) Words in capital letters in column- I are written in small letters according to a
code language in column-II, Decode the language and find out the correct alternative for the given
word in each question. (NTSE Stage-I/Raj./2007)

Column-I Column-II
PORK oxtj
BAD dzg
WAVE sgmp
BID dzw
VAGI gswb
BAT vgd
VEX smk
ROSE yomt

26. Code for B is


(a) v (b) d (c) g (d) y

27. Code for G is


(a) b (b) z (c) y (d) s

28. Code for W is


(a) q (b) g (c) b (d) p

29. Code for letters in the word DATE are


(a) zjkm (b) xtog (c) zgvm (d) djks

30. Code for the letters in the word BEST are


(a) gzmv (b) dmyv (c) bkpd (d) gszy

Directions: (31 to 33) The words in Column-I are coded in small letters, which are written in column-
II, but the order of small letters is different. Decode the words to find out the codes for the letters and
answer the questions that follow: (NTSE Stage-II, 2007)
Column-I Column-II
TRAIN pbrsn
CRANE rmndp
DEAR rcpd
RICE bpmd

31. How would the word EAR be coded?


(a) r d m (b) d r p (c) p r s (d) d r n

32. Which is the code for the word NEAT?


(a) n d r s (b) n d r m (c) m d r s (d) n r d p

14 PACE IIT & MEDICAL: Mumbai / Delhi & NCR / Goa / Akola / Kolkata / Nashik / Pune / Bokaro / Dubai
NTSE-MENTAL ABILITY

33. Which is the code for the word TREAD?


(a) s p d r m (b) s p r d n (c) m s p r c (d) s p d r c

34. If CLOTH is called GOLD and GOLD is called PAPER and PAPER is called CAKE, then
JEWELLERY would be made up of?
(a) Gold (b) Cloth (c) Cake (d) Paper

35. In a certain code FORGET is written as DPPHCU. In the same code DOCTOR will be written as-
(a) EPDUPS (b) ROTCOD (c) BPAUMS (d) CPBUNS

36. If MALE = 31 and PLAY = 54 than CLASS = ?


(a) 35 (b) 31 (c) 54 (d) 45

Directions: (37 to 41) Words in capital letters in column I are written in small letters in a code
language in column-II. Decode the language and find out the correct alternative for the given
letter/word in each question. (NTSE Stage-I/Raj./ 2008)
Column I Column II
FISH zmkj
TEA fir
GAIN kpgf
DOG peh
ROSE cmre
NUT igq
TRAM cvif

37. Code for U is


(a) i (b) g (c) q (d) h

38. Code for M is


(a) v (b) c (c) i (d) m

39. Code for I is


(a) m (b) p (c) f (d) k

40. Code for letters in the word TRAIN is


(a) fgehk (b) fkgic (c) ipvcj (d) fvgme

41. Code for letters in the word EARN is


(a) ferk (b) gcim (c) gkrp (d) fgrc

Directions: (42 to 44) The capital letters in each of the following words are coded and written in small
letters on the right side of each word. But these letters are not in order. Find out the codes for letters
and answer the questions. (NTSE Stage-II, 2008)
Column I Column II
PROBLEM g r c a t s d
ROMAN cftxs
LAME fgat
BOLD gcdz
42. What is the code for letter A?
(a) t (b) g (c) f (d) a
15 PACE IIT & MEDICAL: Mumbai / Delhi & NCR / Goa / Akola / Kolkata / Nashik / Pune / Bokaro / Dubai
NTSE-MENTAL ABILITY

43. What is the code for letter B?


(a) g (b) c (c) d (d) z

44. What would be the code (in correct order) for the word ‘MODE’?
(a) t f z c (b) c t f z (c) f t c z (d) t c z a

45. If ‘cake is tasty’ is ‘uoi hui puri’ and milk is sweet’ is ‘dri hui coi’, Then ‘sweet is tasty’ will be
coded as: (NTSE Stage-II, 2008)
(a) coi hui puri (b) uoi hui dri (c) puri hui dri (d) uoi hui coi

46. In a certain language MONKEY is coded as 632471 and GANGA is coded as 58258. Then how is
MONGYA coded? (NTSE Stage-II, 2008)
(a) 638152 (b) 543127 (c) 632518 (d) 671854

Directions: (47 to 51) According to a code language, words in column I are given in column II. Decode
the language and choose the correct code for each of the words given in the following questions.
(NTSE Stage-I/Raj./2009)

Column-I Column-II
SET yxg
SIGN pxrk
POT yma
NINE kgpk
ROSE xgae

47. Code for letters in the word STONE are


(a) kgxmp (b) akygx (c) ayxkr (d) agxyp

48. Code for letters in the word PRINT are


(a) mekxr (b) epkya (c) epykm (d) epgmk

49. Code for letters in the word GROSS are


(a) raaex (b) exxmp (c) xerrg (d) axrex

50. Code for letters in the word SOON are


(a) kxxa (b) xkpp (c) aakx (d) kkar

51. Code for letters in the word TEN are


(a) kgy (b) gkr (c) ykm (d) eyg

Directions: (52) The capital letters in each of the following words are coded and written in small
letters, but not the same order as the letters in the word. Find the codes for letters and answer the
question. (NTSE Stage-II, 2009)
NECK:bimk
LUCK:mdnb
L I KE : m k j n

52. What would be the code in correct order for the word ‘NICE’?
(a) k j n b (b) j i k b (c) i j b m (d) i j b k

16 PACE IIT & MEDICAL: Mumbai / Delhi & NCR / Goa / Akola / Kolkata / Nashik / Pune / Bokaro / Dubai
NTSE-MENTAL ABILITY

53. If BDFHJ is written as IGECA, then how PRTVX will be written? (NTSE Stage-II, 2009)
(a) WUSQO (b) OPQRS (c) DWVEF (d) XVTRP

54. If DONKEY is written as EYDOKN, how the word CLEVER will be written?
(NTSE Stage-II, 2009)
(a) RCELVE (b) ERLCVE (c) RECLVE (d) ERCLVE

55. If BAD = 14 and DIG = 40 then HALF will be equal to (NTSE Stage-II, 2009)
(a) 45 (b) 54 (c) 82 (d) 40

Directions: (56 to 60) Some words are given in column I. These words are written in a code language
in column II. The code equivalents of the words given in column I and column II are not necessarily in
the corresponding order. Choose the correct code for the words from the given alternatives.
(NTSE Stage-II, 2011)

Column-I Column-II
i. Kahu chala na hum kuch ja
ii. Aj tak na ek ja kam
iii. Man tak pana saj ek ada
iv. Hum chala man kuch not ada
v. Hum na jai not kim ja

56. Which word will be code for word Aj?


(a) ada (b) hum (c) kuch (d) kam

57. Which word will be code for Hum?


(a) ja (b) not (c) kuch (d) ek

58. Which word will be code for word Pana?


(a) ada (b) ek (c) saj (d) not

59. Which word will be code for word Kahu?


(a) hum (b) ada (c) not (d) ja

60. Which word will be code for word Jai?


(a) ek (b) saj (c) kim (d) ja

Directions: (61 to 63) The capital letters in each of the following words are coded as figures on the
right side. Find out the codes for letters and answers the questions. (NTSE Stage-II, 2011)

17 PACE IIT & MEDICAL: Mumbai / Delhi & NCR / Goa / Akola / Kolkata / Nashik / Pune / Bokaro / Dubai
NTSE-MENTAL ABILITY

61. Which is the code for ‘STOLEN’?

62. Which is the code for ‘LOWER’?

63. Which is the code for ‘DRESSING’? (NTSE Stage-II, 2011)

Direction: (64 to 65) The cells in diagram I and sectors in diagram II contain two letters each from A to Z.

The first letters in each cell is coded by the cell shape whereas the second letter is represented by cell
shape along with a dot in it.
for example

18 PACE IIT & MEDICAL: Mumbai / Delhi & NCR / Goa / Akola / Kolkata / Nashik / Pune / Bokaro / Dubai
NTSE-MENTAL ABILITY

64. Identify the response which represents CHAIR

(a) (b) (c) (d)

65. Identify the response which represents MONKEY


(a) (b) (c) (d)

Directions: (66 to 70) Words in capital letters in column-I are written in small letters in a code
language in column-II. Decode the language in column-II. Decode the Language and find out the
correct alternative for the given letters in each questions. (NTSE Stage-I/ Raj./2012)
Column-I Column-II
HOPE vtyg
WIDE ceth
LUCK nxfl
DUST aien
SIND cmae
SOAP gapv
FEAR putj
MUST nida
HUNT mnyi
FILE cxut
PINE cmtg

66. Code for letters in the word SOLE are


(a) txza (b) fvxy (c) mtax (d) vtax

67. Code for letters in the word MENT are


(a) ndti (b) dtum (c) mdit (d) puit

68. Code for letters in the word NEWS are


(a) hmta (b) tmkh (c) fmak (d) tahv

69. Code for letters in the word STAR are


(a) ipaj (b) jami (c) paiz (d) ajkl

70. Code for letters in the word TIME are


(a) tkci (b) citd (c) ctpb (d) litm

71. In a coded language TRACE = 43251 and EARTH = 12347 then the code for FACT will be
(NTSE Stage-I/Raj./2012)
(a) 9245 (b) 9254 (c) 9425 (d) 9524

72. In a coded language SHOP = 8256, WORK = 9573 and HOME = 2541 then the code for SMOKE
will be- (NTSE Stage-I/Raj./2012)
(a) 84531 (b) 83451 (c) 84351 (d) 85431

73. In a coded language TAKE = 1790, PLOT = 5321 then code for PLATE will be-
(NTSE Stage-I/Raj./2012)
(a) 53701 (b) 53071 (c) 35710 (d) 53710

19 PACE IIT & MEDICAL: Mumbai / Delhi & NCR / Goa / Akola / Kolkata / Nashik / Pune / Bokaro / Dubai
NTSE-MENTAL ABILITY

74. In a coded language FRUT = HTWKV then FLOWER will be written as-
(a) HNQYGT (b) HGPTYN (c) HYNGPT (d) HTPNGY

Directions: (75 to 81) In each question there is a word written in capital letters with one letter
underlined. For each letter in that word there is a code written in small letters. That code is denoted
by either (a), (b), (c), (d) not in the same order. You have to find out the exact code for the underlined
letter in the word. The number of that code is the answer please note that the same letter appearing in
other word (s) may be coded differently.

75. PAGES
(a) b (b) u (c) r (d) x

76. BREAK
(a) z (b) g (c) p (d) c

77. APRIL
(a) s (b) f (c) u (d) x

78. PRISM
(a) r (b) o (c) h (d) q

79. WHICH
(a) f (b) g (c) u (d) e

80. ABOVE
(a) q (b) g (c) v (d) b

81. COVER
(a) u (b) y (c) q (d) f

Directions: (82 to 86) Words in capital letters in column-I are written in small letters in a code
language in Column-II decode the language and find out the correct alternative for the given letters in
each questions. (NTSE Stage-I/Raj./2013)

Column-I Column-II
HERO tbfw
JOIN bakp
LAZY nsvg
MINE pdkt
PART rwsx
SAURY wveos
BLUE eglt
CIGAR usqwp
WRIT wpxy
VIRUS pzwoe
QUACK jqems
PIRL wprg

82. Code for letters in the word TOIL are


(a) pxba (b) bpgn (c) bpxg (d) mpxg

20 PACE IIT & MEDICAL: Mumbai / Delhi & NCR / Goa / Akola / Kolkata / Nashik / Pune / Bokaro / Dubai
NTSE-MENTAL ABILITY

83. Code for letters in the word COST are


(a) boqx (b) xqps (c) qost (d) xqnr

84. Code for letters in the word ULCER are


(a) ggwmr (b) teqwp (c) ktegp (d) gteqw

85. Code for letters in the word SINE are


(a) ptkl (b) toka (c) ptok (d) optb

86. Code for letters in the word ARCH are


(a) frsq (b) wfsq (c) wqfp (d) sqfn
87. In a coded language NUMBER is written as in MFNYVI. Then FIGURE may be written in coded
language as- (NTSE Stage-I/Raj./2013)
(a) ERHFID (b) URTVSF (c) GJTFSF (d) URTFIV

88. In a coded language SHIFT is written as UFKDV, Then COVET may be written in coded language
as
(a) EMXCV (b) FNYDU (c) EXCUV (d) EQUDS

89. If PET = 4
LET = 3
JEY = 2
Then what is the value of XET?
(a) 1 (b) 5 (c) 6 (d) 8

90. In a coded language if HOME = 2541, SHOP = 8256, WORK = 9573, then code for SMOKE will
be- (NTSE Stage-I/Raj./2013)
(a) 85431 (b) 84531 (c) 83451 (d) 84351

91. Here are some words translated from an artificial language


mie pie is blue light
mie tie is blue berry
aie tie is rasp berry
Which words could possible mean “light fly”? (NTSE Stage-I/Raj./2013)
(a) pie zie (b) pie mie (c) aie zie (d) aie mie

92. If in a certain code, STUDENT is written as RSTEDMS, then how would TEACHER be written in
the same code?
(a) SZZDGEQ (b) SZDDGEQ (c) SDZDGDQ (d) SDZCGDQ

Direction (93 to 96): Words in capital letters in column I are written in English small letters according
to a code language in column II. Decode the language and find out the correct alternative for the given
word in each question. (NTSE Stage-I/Raj./2014)
Column I Column II
ONE cdy
TWO sqd
THREE lsgyy
FOUR dztg
FIVE zmfy
SIX rmh

21 PACE IIT & MEDICAL: Mumbai / Delhi & NCR / Goa / Akola / Kolkata / Nashik / Pune / Bokaro / Dubai
NTSE-MENTAL ABILITY

93. NET
(a) dys (b) cys (c) tcs (d) csd
94. FOX
(a) ydh (b) dhs (c) zdh (d) zgl
95. HER
(a) lgc (b) lyg (c) lgs (d) glc

96. SHE
(a) rly (b) rcy (c) rsy (d) yet

97. In a code A = 26… Z = 1 if G = 25 and MILK = 83 then, find the code for WATER:
(NTSE Stage-I/Karnatka/2014)
(a) 67 (b) 68 (c) 92 (d) 93
98. Using the total number of alphabets in your solution as a parameter, find the number that represents
G, if
A-0, B-0, C-2, D-2, E-1, F-2, G
(a) 2 (b) 3 (c) 4 (d) 5
99. If FEED is coded as 47 and TREE is coded as 91, then MEET will be coded as:
(a) 110 (b) 114 (c) 118 (d) 122
Direction (100 to 101) Following alphabets are written in a special coded language like
BLACK WHITE
0 1 23 4 5 6 789
(NTSE Stage-I/Raj./2014)
100. Then code 62830 will be written as
(a) HATCB (b) HATEC (c) HATBC (d) HATCE

101. ‘BHICK’ will be coded as


(a) 06734 (b) 6734 (c) 6743 (d) 06743
Answer Key
1. (d) 2. (c) 3. (c) 4. (b) 5. (d) 6. (b) 7. (d) 8. (c) 9. (c) 10. (a)
11. (b) 12. (c) 13. (c) 14. (c) 15. (d) 16. (c) 17. (c) 18. (b) 19. (a) 20. (d)

21. (d) 22. (a) 23. (d) 24. (c) 25. (b) 26. (b) 27. (a) 28. (d) 29. (c) 30. (b)

31. (b) 32. (a) 33. (d) 34. (d) 35. (c) 36. (c) 37. (c) 38. (a) 39. (d) 40. (b)

41. (d) 42. (c) 43. (c) 44. (d) 45. (a) 46. (c) 47. (b) 48. (c) 49. (d) 50. (c)

51. (a) 52. (d) 53. (a) 54. (d) 55. (b) 56. (d) 57. (b) 58. (c) 59. (a) 60. (c)

61. (d) 62. (b) 63. (a) 64. (b) 65. (d) 66. (d) 67. (c) 68. (a) 69. (a) 70. (b)

71. (b) 72. (a) 73. (d) 74. (a) 75. (a) 76. (a) 77. (c) 78. (a) 79. (a) 80. (d)
81. (d) 82. (c) 83. (a) 84. (d) 85. (c) 86. (b) 87. (d) 88. (a) 89. (c) 90. (b)
91. (a) 92. (c) 93. (b) 94. (c) 95. (b) 96. (a) 97. (d) 98. (d) 99. (c) 100. (a)
101. (a)
22 PACE IIT & MEDICAL: Mumbai / Delhi & NCR / Goa / Akola / Kolkata / Nashik / Pune / Bokaro / Dubai
NTSE-MENTAL ABILITY

PREVIOUS YEAR QUESTIONS

(Q. 1 to 2) Directions: In a certain code language the word BASIC has been written in four different code
language. Understanding the code, find out the correct code language for the word given in each of the
following questions.
Word Code language
Basic - (a) EDVLF
(b) CISAB
(c) YASIZ
(d) BZRHC

1. EARTH = BARTE

2. CLOUD = CKNTD

3. LEARN = OHDUQ

4. In a certain code language if

Then find the value of #


(a) 5 (b) 6 (c) 3 (d) 9

5. In a certain code language if  means 4,    means 12, @ means  ,  means  , # means +


and $ means – is used, then find   @    $     #    = ?
(a) 104 (b) 106 (c) 102 (d) 30
(NTSE-MAHARASHTRA STAGE-1-2020)

6. In the following question letters and numbers are written with a specific rule in horizontal rows. Find
the rule and decide which will be in place of question mark.

(a) 30, 41 (b) 30, 32 (c) 34, 36 (d) 35, 35

7. Write the correct alternative to replace question mark.

(a) C (b) B (c) Z (d) A


(NTSE-MAHARASHTRA STAGE-1-2020)

23 PACE IIT & MEDICAL: Mumbai / Delhi & NCR / Goa / Akola / Kolkata / Nashik / Pune / Bokaro / Dubai
NTSE-MENTAL ABILITY

(Q. 8 and 9) Directions: In the following table the digits are assigned with certain symbols. Observe them
carefully and choose the correct alternative to answer the questions.

8. How will you write the number 635104?


(a) (b)

(c) (d)

9. Which number will be expressed by


(a) 951478 (b) 958174 (c) 951847 (d) 951874

(Q. 10 and 11) Directions: In the following questions word letters are given in column I and are coded in
column II. But they are not arranged according to the order of word letters in column I. find the code
language and choose the correct alternative to answer the questions.
10. What is the code for the word PREAK?
(a) 13689 (b) 16389 (c) 16839 (d) 16489
11. 542687 code is for which word?
(a) NATURE (b) NATEUR (c) NUTRAL (d) NURTAL

12. Observe the following code language and choose the correct alternative to answer the questions.

Letters Z A W O D I Y L P C
Digits 0 1 2 3 4 5 6 7 8 9
What is the code for the word ZODIAC?
(a) 034159 (b) 034519 (c) 043951 (d) 093415
(NTSE-MAHARASHTRA STAGE-1-2020)
(Q. 13 and 14) Direction : In the following table the digits are assigned with certain symbols. Observe them
Carefully and choose the correct alternative to answer the questions.

13.

(a) (b)
(c) (d)

24 PACE IIT & MEDICAL: Mumbai / Delhi & NCR / Goa / Akola / Kolkata / Nashik / Pune / Bokaro / Dubai
NTSE-MENTAL ABILITY

14.

(a) (b) (c) (d)


(NTSE-MAHARASHTRA STAGE-1-2019)

(Q. 15 to 17) Direction : The word ACTIVE is written in four different code languages. Understanding the
code find out the correct code language for the word given in each of the following questions:
ACTIVE = (a) CEVKXG
(b) EFVKYI
(c) XZQFSB
(d) CFXNBL

15. GOLDEN = KRNFHR

16. ORANGE = LOXKDB

17. PURPLE = RWTRNG


(NTSE-MAHARASHTRA STAGE-1-2019)

(Q. 18 and 19) Direction-In the following questions numbers are given in Column I and are coded in
column II. But they are not arranged according to the order of digits in the number. Identify the code
language and choose the correct alternative to answer the questions:
Column I Column II

18. Which of the following numbers will be coded as


(a) 2165 (b) 2856 (c) 2356 (d) 2534

19. Which of the following code will be used to indicate the number 9135?
(a) (b)

(c) (d)

20. Direction : Observe the following code and answer the questions that follow:

Letters  A T M G O D N R S
Digits  9 8 7 6 5 4 3 2 1
Choose the correct code from the following alternatives for the word ‘DONAR’.
(a) 48391 (b) 54872 (c) 45392 (d) 53971
(NTSE-MAHARASHTRA STAGE-1-2019)
21. In a certain code language
3 2  29 ; 4  5  74 ;
7  3  58 ; then, 6  8  ?
(a) 134 (b) 118 (c) 116 (d) 132

25 PACE IIT & MEDICAL: Mumbai / Delhi & NCR / Goa / Akola / Kolkata / Nashik / Pune / Bokaro / Dubai
NTSE-MENTAL ABILITY

22. In a certain code language


11 + 5 = 36 ; 22 + 6 = 58;
33 + 7 = 82; then, 55 + 9 = ?
(a) 112 (b) 163 (c) 136 (d) 124
(NTSE-MAHARASHTRA STAGE-1-2018)

(Q. 23 to 25) Direction : In certain code languages the word PACK is written in four different code
languages. Understanding the code, find out the correct code language for the word given in each of the
following questions.
PACK = (a) TEGO
(b) MXZH
(c) VFGO
(d) QDHR

23. RA1N = NXFK

24. CROP = DUTW

25. SAND = WERH


(NTSE-MAHARASHTRA STAGE-1-2018)

(Q. 26 and 27) Direction: In the following questions symbol are given in column I and are coded in column
II. But they are not arranged according to the order of symbols in column I. Find the code language and
choose the correct alternative to answer the questions.

26.
(a) 143 (b) 237 (c) 549 (d) 943

27. 135 = ?

(a) (b)

(c) (d)

28. Observe the following code language and choose the correct alternative to answer the question.

947580 = ?
(a) SATPLE (b) STAPLE (c) STALPE (d) SATLPE
(NTSE-MAHARASHTRA STAGE-1-2018)

26 PACE IIT & MEDICAL: Mumbai / Delhi & NCR / Goa / Akola / Kolkata / Nashik / Pune / Bokaro / Dubai
NTSE-MENTAL ABILITY

(Q. 29 and 30) : Directions: In the following table the digits are assigned with certain symbols. Observe
them carefully and choose the correct alternative to answer the questions.

29. How will you write the number 8 7 4 6 0 5 3 ?


(a) ? ! ) ( % ^ $ (b) ! ? ) ( % ^ $ (c) ? ! ( ) ^ % $ (d) ! ? ( ) ^ % $

30. Which number will be expressed by the code ( % ! $ ₹ )  # ?


(a) 4 7 5 9 3 6 1 2 (b) 4 5 7 3 9 6 1 2 (c) 4 7 8 3 9 6 1 2 (d) 4 5 7 9 3 6 1 2
(NTSE-MAHARASHTRA STAGE-1-2018)

31. In a certain code language ZEAL = 11, written then in that language BEAT = ?
(a) 7 (b) 13 (c) 14 (d) 19
(NTSE-MAHARASHTRA STAGE-1-2017)

(Q. 32 and 33) : Directions - In a certain code language the word EXPAND has been written in four
different code languages. Understanding the code, find out the correct code language for the word given in
each of the following questions.
EXPAND =
(a) FYQBOE
(b) EYRDRI
(c) EPNXAD
(d) DWOZMC

32. (1) SOLVE=SPNYI

33. (2) LARGE = KZQFD


(NTSE-MAHARASHTRA STAGE-1-2017)

(Q. 34. and 35) : Direction -Two charts are given below:
Containing two groups of letters. In chart one the rows and columns are labelled with 0 to 4 number. In chart
two rows and columns are labelled with the numbers 5 to 9. The letter in the chart is identified firstly by its
row number and then by its column number. For example 5 is denoted by 22, 41 number.

34. Which group of words represent the word MOST?


(a) 40, 44, 22, 89 (b) 33, 20, 11, 79 (c) 21, 00, 03, 88 (d) 02, 13, 34, 56

35. Which group of words represent the word ROAD?


(a) 42, 32, 79, 58 (b) 23, 32, 98, 99 (c) 11, 13, 67, 69 (d) 04, 20, 55, 78
(NTSE-MAHARASHTRA STAGE-1-2017)
27 PACE IIT & MEDICAL: Mumbai / Delhi & NCR / Goa / Akola / Kolkata / Nashik / Pune / Bokaro / Dubai
NTSE-MENTAL ABILITY

(Q. 36 and 37) Directions: Using alphabet A to Z in sections of figure I and II a code has been created. First
letter in every section is coded according to its shape and the second letter is coded using a dot.

36. What will be the code of SMILE?

(a) (b)

(c) (d)

37. What will be the code of BUKAR?

(a) (b)

(c) (d)

38. If CH = X, then BJ = ?
(a) T (b) R (c) V (d) L
(NTSE-MAHARASHTRA STAGE-1-2016)

(Q. 39 and 40) Directions :- Two tables are given below in which two groups of alphabets are written. In
table I the rows and columns are numbered 0-4 and in table II the rows and columns are numbered 5-9. The
alphabets in the tables are represented first by their row number and then by their column number.
Eg. - R is represented as R = 10, 42.

39. Which group of number represents LEAD?


(a) 57, 65, 00, 12 (b) 75, 96, 31, 43 (c) 98, 69, 23, 14 (d) 86, 77, 41, 40

28 PACE IIT & MEDICAL: Mumbai / Delhi & NCR / Goa / Akola / Kolkata / Nashik / Pune / Bokaro / Dubai
NTSE-MENTAL ABILITY

40. Which group of numbers represents RINK?


(a) 42, 79, 30, 78 (b) 10, 68, 43, 55 (c) 24, 87, 11, 89 (d) 01, 95, 22, 67
(NTSE-MAHARASHTRA STAGE-1-2016)

(Q. 41 and 42) : Directions :- In a certain code language, the sentence XEROX COPY OF BILL WAS
SENT is written as ZGTQZ EQRA QH DKNN YCU UGPV. Using the same code language how will you
write the following words?

41. WATER?
(a) YBUFS (b) YCVGT (c) EQARR (d) EAVER

42. BLIST = ?
(a) DNKOV (b) DNKUQ (c) DNKVU (d) DNKUV
(NTSE-MAHARASHTRA STAGE-1-2015)

43. In a certain code language if SCHOOL = 24 and TEACHER = 20, then in the same code language
CLASS = ?
(a) 14 (b) 16 (c) 18 (d) 22

44. In a certain code language if CAMP is written as 9, then in the same code how will the word TEAM
be written ?
(a) 14 (b) 19 (c) 27 (d) 33
(NTSE-MAHARASHTRA STAGE-1-2015)

(Q. 45 and 46) Directions : If the sentence KEEP AWAY FROM EXPLOSIVE is written in the code
language as ICCN YUYW DPMK CVNJMQGTC, then in the same code how will you write the words
given in the following questions.

45. DANGER =?
(a) CZMPDQ (b) EBOHFS (c) BYLECP (c) FCPIGT

46. FLAME = ?
(a) DJYKC (b) EKZLD (c) GMBNF (d) HNCOG
(NTSE-MAHARASHTRA STAGE-1-2014)

47. In a certain code language if BUSH is written as 58, MARK is written as 65, then in the same code
how will you write the word PERL ?
(a) 48 (b) 51 (c) 54 (d) 57

48. In a certain code language if SMASH = 30 and DRIVE = 29, then POINT = ?
(a) 37 (b) 38 (c) 39 (d) 41
(NTSE-MAHARASHTRA STAGE-1-2014)

49. In a certain code language if 44  44 = 4444 and 34  52 = 5423, then find the value of 81  46 in the
same code language.
(a) 1648 (b) 8461 (c) 8164 (d) 4168

50. In a certain code language if 3 + 3 = 9, 6 – 3 = 2, 4  3 = 1, 3 ÷ 2 = 5, then find 24  6 ÷ 8 – 2 + 3 = ?


(a) 33 (b) 30 (c) 28 (d) 06
(NTSE-MAHARASHTRA STAGE-1-2014)

29 PACE IIT & MEDICAL: Mumbai / Delhi & NCR / Goa / Akola / Kolkata / Nashik / Pune / Bokaro / Dubai
NTSE-MENTAL ABILITY

(Q. 51 to 54) Directions :- In a certain code language word BLASTER is written in four different code
languages. Understanding the code, find the correct code language for the given question word from the
given alternatives.

BLASTER =
(a) YOZHGVI (b) EODVWHU (c) BATRLSE (d) CNDWYKY

51. PITCH = SLWFK

52. SERIES = TGUMJY

53. WONDER = DLMWVI

54. LONGEST = LNETOGS


(NTSE-MAHARASHTRA STAGE-1-2012)
F B L
55. If  , then  ?
O E P
C D C C
(a) (b) (c) (d)
E C K D

56. If FD = H, then LA = ?
(a) M (b) K (c) O (d) P

57. If GO = 32, SHE = 49, then SOME = ?


(a) 63 (b) 60 (c) 52 (d) 56
(NTSE-MAHARASHTRA STAGE-1-2012)

Answer Key

1. (c) 2. (d) 3. (a) 4. (c) 5. (b) 6. (c) 7. (b, d) 8. (b) 9. (d) 10. (b)
11. (c) 12. (b) 13. (c) 14. (b) 15. (b) 16. (c) 17. (a) 18. (c) 19. (d) 20. (c)

21. (a) 22. (c) 23. (b) 24. (d) 25. (a) 26. (d) 27. (b) 28. (b) 29. (c) 30. (b)

31. (a) 32. (b) 33. (d) 34. (d) 35. (c) 36. (c) 37. (d) 38. (a) 39. (a) 40. (d)

41. (b) 42. (d) 43. (c) 44. (c) 45. (c) 46. (a) 47. (d) 48. (a) 49. (d) 50. (b)

51. (b) 52. (d) 53. (a) 54. (c) 55. (d) 56. (b) 57. (d)

30 PACE IIT & MEDICAL: Mumbai / Delhi & NCR / Goa / Akola / Kolkata / Nashik / Pune / Bokaro / Dubai
NTSE-MENTAL ABILITY

3. NUMBER SERIES

Number series problems deal with numbers. While attempting to solve the question, you have to check the
pattern of the series. Series moves with certain mathematical operations. You have to check the pattern.
Type of questions asked in the examination :
(i) Find the missing term(s).
(ii) Find the wrong term(s).

Number Series
In this type of series, the set of given numbers in a series are related to one another in a particular pattern or
manner. The relationship between the numbers may be
● Consecutive odd/even numbers,
● Consecutive prime / composite numbers,
● Squares/cubes of some numbers with/without variation of addition or subtraction of some number.
● Sum/product difference of preceding number(s),
● Addition/subtraction/multiplication/division by some number, and
● Many more combinations of the relationship given above.

Directions : (1 to 13) Find the missing numbers:

1. 3, 5, 7, 9, 11, 13,15,17,?
(A) 14 (B) 19 (C) 15 (D) 21
Sol. (B) Each term has a common difference = + 2. Hence, next term = 17 + 2 = 19.

2. 2, 3, 5, 7, 11, ?, 17
(A) 14 (B) 13 (C) 10 (D) 12
Sol. (B) The series is made up of consecutive prime numbers. Therefore, the missing term is 13.

3. 1, 4, 9, 16, 25,?
(A) 35 (B) 36 (C) 37 (D) 49
2 22 2 2
Sol. (B) Each term is a square of 1, 2, 3, 4 and so on 1  1, 2  4, 3  9, 4  16, 5  25
Hence, next term  62  36 .

4. 2, 5, 10, 17,?
(A) 24 (B) 25 (C) 26 (D) 27
Sol. (C) Each term is a square of 1, 2, 3, 4 and so on and 1 is added to it, i.e.
2 2
12  1,  2   1,  3   1,.......  2, 5,10,17 ……..
Hence, next term   5 2  1  26

5. 2, 3, 10, 15, 26, 35, ?


(A) 48 (B) 51 (C) 49 (D) 50
2
Sol. (D) The series exhibits the pattern of + 1 , n  1 , alternately, n taking values 1, 2,…….

6. 1, 8, 9, 64, 25, 216, ? , ?


(A) 49, 64 (B) 343, 64 (C) 49, 512 (D) 343, 512
Sol. (C) Odd positioned digits are squares of 1, 3, 5 and so on, i.e. 12  1,32  9,52  25 and so on.
Similarly, even positioned digits are cubes of 2, 4, 6, etc., i.e. 23  8, 43  64, 63  216
Therefore, the next term would be 72 i.e. 49 and 83  512 respectively.
31 PACE IIT & MEDICAL: Mumbai / Delhi & NCR / Goa / Akola / Kolkata / Nashik / Pune / Bokaro / Dubai
NTSE-MENTAL ABILITY

7. 0, 7, 26, ?, 124, 215


(A) 51 (B) 37 (C) 63 (D) 16
Sol. (C) Each term is a cube of 1, 2, 3, 4 and so on and 1 subtracted from it, i.e.
13  1, 23  1,33  1, 43  1,53  1, 63  1 .
3
Therefore, the term replacing the question mark would be 4  1  64  1  63 .

8. 3, 4, 10, 33, 136?


(A) 240 (B) 430 (C) 685 (D) 820
Sol. (C) The terms of the series are, previous term 1  1 ,previous term x2+2, previous term 3  3 and so
on. Hence, the next term will be 136  5  5  680  5  685

9. 11, 15, 21, 29,?


(A) 40 (B) 41 (C) 37 (D) 39
Sol. This series consists of increasing numbers. The pattern is +4, +6, +8 ……..

10. 3, 6, 18, 72, 360’?


(A) 720 (B) 1080 (C) 1600 (D) 2160
Sol. (D)
The sequence in the given series is 2, 3, 4, 5, 6
Hence, the missing number is 360  6  2160

11. 6, 12, 7, 11, 8, 10, 9?


(A) 8 (B) 9 (C) 11 (D) 10
Sol. (B) Alternate series
(i) 6, 7, 8, 9
(ii) 12, 11, 10, ?

12. 0, 5, 22, 57, 116, ?


(A) 205 (B) 216 (C) 192 (D) 207

Sol. (A)

Hence, the next  205

13. 151,158, 172,182,?


(A) 210 (B) 193 (C) 197 (D) 203

Sol. (B) 1 + 5 + 1 = 7
The difference between 151 & 158 is seven (7)
1 + 5 + 8 = 14,
The difference between 158 & 172 is (14).
1 + 7 + 2 =10………. And so on,
 Missing term = 182 + 11 = 193.

32 PACE IIT & MEDICAL: Mumbai / Delhi & NCR / Goa / Akola / Kolkata / Nashik / Pune / Bokaro / Dubai
NTSE-MENTAL ABILITY

Directions: (14 to 15) In each of the following questions, a number series is given. After the series, below it
in the next line, a number is given followed by (P),(Q), (R), (S) and (T). You have to complete the series
starting with the number given following the sequence of the given series. Then answer the question given
below it.
14. 12 28 64 140
37 (P) (Q) (R) (S) (T)
Which number will come in place of (T) ?
(A) 1412 (B) 164 (C) 696 (D) 78

Sol. (A)

Therefore, the number 1412 will come in place of (T).

15. 2 9 57 337
3 (P) (Q) (R) (S) (T)
Which number will come in place of (Q)?
(A) 113 (B) 17 (C) 3912 (D) 8065

Sol. (A)

Therefore, the number 113 will come in place of (Q).

Direction: (16 to 18) Find the wrong term:

16. 2,5,9,11,14
(A) 2 (B) 5 (C) 9 (D) 11
Sol. (C)
Series: 3, 3, 3,..........
The next term is got by adding 3 in preceding term.
 2  3  5,5  3  8
 9 is wrong term.

17. 10, 100, 1100, 11000, 111000, 1210000.


(A) 1210000 (B) 11000 (C) 100 (D) 111000
Sol. (D)
Given series is:

33 PACE IIT & MEDICAL: Mumbai / Delhi & NCR / Goa / Akola / Kolkata / Nashik / Pune / Bokaro / Dubai
NTSE-MENTAL ABILITY

 111000 is wrong
The correct term is 121000.

18. 2, 6, 11, 17, 23, 32, 41


(A) 6 (B) 17 (C) 23 (D) 32
Sol. (C)
Given series is:

34 PACE IIT & MEDICAL: Mumbai / Delhi & NCR / Goa / Akola / Kolkata / Nashik / Pune / Bokaro / Dubai
NTSE-MENTAL ABILITY

EXERCISE-1
Directions: (1 to 50) Find the missing numbers:
1. 2, 3, 5, 7, ?
(a) 9 (b) 10 (c) 11 (d) 14

2. 0, 6, 20, 42 , ?
(a) 64 (b) 72 (c) 80 (d) 84

3. 3, 8, 35, 48, ?, 120


(a) 72 (b) 64 (c) 80 (d) 99

4. 4, 25, 64, 121, 196, ?


(a) 384 (b) 256 (c) 225 (d) 289

5. 210, 120, ?, 24, 6, 0


(a) 64 (b) 48 (c) 35 (d) 60

6. 2, 12, 36, 80, 150, ?


(a) 194 (b) 210 (c) 252 (d) 258

7. 4, 10, 22, 46, ?


(a) 56 (b) 66 (c) 76 (d) 94

8. 8, 15, 28, 53, ?


(a) 120 (b) 106 (c) 104 (d) 102

9. 4, 8, 12, 24, 36, 72, ?


(a) 98 (b) 100 (c) 144 (d) 108

10. 12, 15, 18, 21, ?


(a) 24 (b) 23 (c) 22 (d) 25

11. 3, 6, 12, 24, ?, 96


(a) 84 (b) 50 (c) 52 (d) 48

12. 2, 10, 19, 29, 40, 52, 65, 79, 94, ?


(a) 110 (b) 109 (c) 108 (d) None of these

13. 4, 7, 3, 6, 2, 5, ?
(a) 6 (b) 5 (c) 3 (d) 1

14. 4, 7, 10, 11, 22, 17, 46, 25, ?


(a) 58 (b) 69 (c) 86 (d) 94

15. 2, 2, 4, 4, 6, 8, 8, ?
(a) 10 (b) 12 (c) 14 (d) 16

16. 2, 3, 10, 15, 26, ? (NTSE Stage-I/Raj./2007)


(a) 34 (b) 35 (c) 36 (d) 37

35 PACE IIT & MEDICAL: Mumbai / Delhi & NCR / Goa / Akola / Kolkata / Nashik / Pune / Bokaro / Dubai
NTSE-MENTAL ABILITY

17. 1, 4, 27, 16, 125, 36, ? (NTSE Stage-I/Raj./2007)


(a) 216 (b) 343 (c) 64 (d) 49

18. 336, 210, 120, ?, 24, 6, 0 (NTSE Stage-I/Raj./2007)


(a) 40 (b) 50 (c) 60 (d) 70

19. 3, 4, 8, 17, 33, ? (NTSE Stage-I/Raj./2007)


(a) 58 (b) 69 (c) 49 (d) 98

20. 8, 13, 21, 34, 55, ? (NTSE Stage-I/Raj./2007)


(a) 60 (b) 68 (c) 89 (d) 76

21. 480, 480, 240, 80, 20, ? (NTSE Stage-II, 2007)


(a) 4 (b) 1 (c) 5 (d) 10

22. 1, 1, 2, 2, 3, 4, 4, 8, 5, 16, ? (NTSE Stage-II, 2007)


(a) 6 (b) 32 (c) 8 (d) 7

23. 2, 5, 11, 23, 47, ? (NTSE Stage-II, 2007)


(a) 92 (b) 90 (c) 95 (d) 91

24. 12, 21, 23, 32, 34, 43, 45, ? (NTSE Stage-II, 2007)
(a) 54 (b) 48 (c) 77 (d) 9

25. 14, 1, 21, 4, 28, 9, ? , ? (NTSE Stage-II, 2007)


(a) 9, 42 (b) 16, 35 (c) 35, 16 (d) 16, 36

26. 5, 6, 13, 26, 45, ? (NTSE Stage-I/Raj./2008)


(a) 68 (b) 74 (c) 70 (d) 82

27. 190, 94, 46, 22, 10, 4, ? (NTSE Stage-I/Raj./2008)


(a) 3 (b) 2 (c) 1 (d) 0

28. 128, 110, 90, 68, ? (NTSE Stage-I/Raj./2008)


(a) 36 (b) 42 (c) 44 (d) 48

29. 1, 2, 4, 7, ?, 16 (NTSE Stage-I/Raj./2008)


(a) 9 (b) 11 (c) 12 (d) 13

30. 6, 8, 9, 12, 14, 18, ? (NTSE Stage-I/Raj./2008)


(a) 21 (b) 19 (c) 23 (d) 20

31. 4, 9, 19, 34, 54, ? (NTSE Stage-II, 2008)


(a) 66 (b) 75 (c) 79 (d) 84

32. 31, 29, 24, 22, 17, ?, ? (NTSE Stage-II, 2008)


(a) 15, 13 (b) 10, 8 (c) 14, 12 (d) 15, 10

33. 3, 6, 11, 18, ? (NTSE Stage-II, 2008)


(a) 19 (b) 27 (c) 30 (d) 37

36 PACE IIT & MEDICAL: Mumbai / Delhi & NCR / Goa / Akola / Kolkata / Nashik / Pune / Bokaro / Dubai
NTSE-MENTAL ABILITY

34. 3, 8, 15, 24, ? (NTSE Stage-II, 2008)


(a) 30 (b) 35 (c) 36 (d) 49

35. 4, 10, 23, 50, 105, ? (NTSE Stage-I/Raj./2009)


(a) 215 (b) 210 (c) 216 (d) 439

36. 912, 303, 102, 33, ?, 3, 2 (NTSE Stage-I/Raj./2009)


(a) 12 (b) 10 (c) 8 (d) 6

37. 1, 4, 9, ?, 25, 36 (NTSE Stage-I/Raj./2009)


(a) 11 (b) 19 (c) 21 (d) 16

38. 7, 12, 22, 37, ?, 82, 112 (NTSE Stage-I/Raj./2009)


(a) 62 (b) 57 (c) 52 (d) 42

39. 11, 13, 17, 19, ?, 25 (NTSE Stage-I/Raj./2009)


(a) 20 (b) 21 (c) 23 (d) 22

40. 5, 9, 17, 33, ?, 129 (NTSE Stage-II, 2009)


(a) 72 (b) 67 (c) 65 (d) 58

41. 2, 5, 4, 10, 7, 15, 11, 20, ? (NTSE Stage-II, 2009)


(a) 12, 21 (b) 16, 25 (c) 13, 25 (d) 17, 30

42. 0, 6, 24, 60, 120, ? (NTSE Stage-II, 2009)


(a) 100 (b) 224 (c) 106 (d) 210

43. 67, 64, 50, 55, 50, 60, ? (NTSE Stage-II, 2011)
(a) 64 (b) 63 (c) 68 (d) 57

44. 27, 31, 40, 56, 81, 117, ? (NTSE Stage-II, 2011)
(a) 156 (b) 165 (c) 166 (d) 169

45. 55, 168, 57, 120, 60, 80, 62, 48, 65, 24, ?, ? (NTSE Stage-II, 2011)
(a) 69, 11 (b) 67, 8 (c) 8, 71 (d) 6, 72

46. 8, 7, 16, 5, 32, 3, 64, 1, 128, (?) (NTSE Stage-I/Raj./2012)


(a) 18 (b) 13 (c) –1 (d) 3

47. 16, 33, 65, 131, (?), 523 (NTSE Stage-I/Raj./2012)


(a) 261 (b) 521 (c) 613 (d) 721

48. 5, 2, 17, 4, (?), 6, 47, 8, 65 (NTSE Stage-I/Raj./2012)


(a) 29 (b) 30 (c) 31 (d) 32

49. 1, 2, 4, 8, (?), 32 (NTSE Stage-I/Raj./2012)


(a) 10 (b) 12 (c) 14 (d) 16

50. 2, 3, 10, 15, 26, (?) (NTSE Stage-I/Raj./2012)


(a) 36 (b) 35 (c) 39 (d) 48

37 PACE IIT & MEDICAL: Mumbai / Delhi & NCR / Goa / Akola / Kolkata / Nashik / Pune / Bokaro / Dubai
NTSE-MENTAL ABILITY

Directions: (51 to 53) In each of the following questions a number series is given. After the series,
below it in the next line, a number is given followed by (P), (Q), (R), (S) and (T). You have to complete
the series starting with the number given following the sequence of the given series. Then answer the
question given below it.

51. 2 3 8 27
5 (P) (Q) (R) (S) (T)
Which of the following numbers will come in place of (T)?
(a) 184 (b) 6 (c) 925 (d) 45

52. 5 18 48 112
7 (P) (Q) (R) (S) (T)
Which number will come in place of (S)?
(a) 172 (b) 276 (c) 270 (d) 376

53. 15 159 259 323


7 (P) (Q) (R) (S) (T)
Which of the following numbers will come in place of (R)?
(a) 251 (b) 315 (c) 176 (d) 151

Directions: (54 to 58) Find the missing numbers:

54. 2, 30, 6, 20, 12, 12, (?) (NTSE Stage-I/Raj./2013)


(a) 26 (b) 22 (c) 20 (d) 24

55. 6, 20, 36, 48, 50, (?), 0 (NTSE Stage-I/Raj./2013)


(a) 36 (b) 40 (c) 46 (d) 56

56. 7, 15, 28, 59, 114, (?) (NTSE Stage-I/Raj./2013)


(a) 243 (b) 233 (c) 213 (d) 223

57. 25, 49, 89, 145, 217, (?) (NTSE Stage-I/Raj./2013)


(a) 305 (b) 327 (c) 309 (d) 303

58. 0, 2, 2, 3, 3, 5, 8, 4, 10, (?), 5, 17 (NTSE Stage-I/Raj./2013)


(a) 6 (b) 7 (c) 9 (d) 15

59. Find the next number in sequence


0, 2, 24, 252, ? (NTSE Stage-II/Raj./2013)
(a) 620 (b) 1040 (c) 3120 (d) 5430

60. Find the next number in the sequence


6, 24, 60, 120, ? (NTSE Stage-II/Raj./2013)
(a) 180 (b) 210 (c) 240 (d) 360

61. Find the missing number in the series


2, 10, 26, ?, 242 (NTSE Stage-II/Raj./2013)
(a) 80 (b) 81 (c) 82 (d) 84

38 PACE IIT & MEDICAL: Mumbai / Delhi & NCR / Goa / Akola / Kolkata / Nashik / Pune / Bokaro / Dubai
NTSE-MENTAL ABILITY

Direction (62 to 65): In each of the questions 62 to 65 some of the numbers are missing in the given
series with one term missing shown by question mark (?). This term is one of the alternatives among
the four numbers given under it. Find the right alternative. (NTSE Stage-II/Raj./2014)

62. 5, 10, 17, 26, 37, 50, (?)


(a) 70 (b) 66 (c) 65 (d) 64

63. 6, 25, 62, 123, (?), 341


(a) 216 (b) 214 (c) 215 (d) 217

64. 5, 3, 10, 8, 17, 15, (?), 24


(a) 26 (b) 27 (c) 29 (d) 36

65. 2, 6, 12, 20, 30, (?)


(a) 40 (b) 42 (c) 44 (d) 46

66. 445, 221, 109, 53, 25, 11, ? (NTSE Stage-I/Haryana/2013)


(a) 2 (b) 4 (c) 6 (d) 8

67. 6, 15, 35, 77, 143, ? (NTSE Stage-I/Haryana/2013)


(a) 171 (b) 181 (c) 191 (d) 221

68. Find the missing number in the series.


1, 2, 2, 4, 16, ?, 65536 (NTSE Stage-I/Haryana/2013)
(a) 276 (b) 256 (c) 198 (d) 64

Directions (69 to 72): In each of the following questions write which number in sequence replaces the
question mark? (NTSE Stage-I/Maharashtra/2013)

69. ?, 17, 33, 51, 75


(a) 9 (b) 13 (c) 8 (d) 11

70. 14, 17, 24, 35, ?


(a) 49 (b) 38 (c) 50 (d) 46

71. 37, 57, 81, 99, ?


(a) 118 (b) 119 (c) 135 (d) 137

72. 25, 30, 36, 44, ?


(a) 50 (b) 43 (c) 51 (d) 47

73. 12, 22, 69, 272, 1365, ? (NTSE Stage-I/Chandigrah/2014)


(a) 8196 (b) 8184 (c) 8195 (d) 6830

1 1
74. 729,81,9,1, ,?, (NTSE Stage-I/Rajasthan/2016)
9 729
1 1 1 1
(a) (b) (c) (d)
27 81 243 486

39 PACE IIT & MEDICAL: Mumbai / Delhi & NCR / Goa / Akola / Kolkata / Nashik / Pune / Bokaro / Dubai
NTSE-MENTAL ABILITY

75. Identify the missing number in the following sequence.


2, 17, 52, _____, 206 (NTSE Stage-II, 2015)
(a) 73 (b) 85 (c) 113 (d) 184

76. Select the missing numbers in the following sequence


3, 6, 24, 30, 63, 72, ?, ?, 195, 210 (NTSE Stage-II, 2015)
(a) 117, 123 (b) 120, 132 (c) 123, 135 (d) 135, 144

77. 121, 144, 169, ?, 225, 256. (NTSE Stage-I/Rajasthan/2017)


(a) 196 (b) 296 (c) 220 (d) 222

78. 5, 10, 20, ?, 80 (NTSE Stage-I/Rajasthan/2017)


(a) 35 (b) 40 (c) 45 (d) 50

EXERCISE-2

Directions: (1 to 26) Find the wrong term of the series.

1. 3, 7, 9, 21, 27, 66, 81, 189, 243 (NTSE Stage-I/Raj./2007)


(a) 27 (b) 66 (c) 243 (d) 21

2. 27, 34, 40, 45, 49, 53, 54, 55 (NTSE Stage-I/Raj./2007)


(a) 53 (b) 45 (c) 56 (d) 34

3. 0, 2, 3, 6, 6, 20, 9, 54, 12 (NTSE Stage-I/Raj./2007)


(a) 3 (b) 6 (c) 20 (d) 54

4. 0, 2, 10, 36, 68, 130 (NTSE Stage-I/Raj./2007)


(a) 10 (b) 36 (c) 68 (d) 130

5. 9, 54, 44, 264, 254, 1520, 1514 (NTSE Stage-I/Raj./2007)


(a) 1514 (b) 1520 (c) 264 (d) 44

6. 10, 15, 26, 35, 48, 63, 82 (NTSE Stage-I/Raj./2008)


(a) 48 (b) 26 (c) 63 (d) 82

7. 3, 10, 30, 66, 127, 218 (NTSE Stage-I/Raj./2008)


(a) 3 (b) 66 (c) 30 (d) 218

8. 7, 9, 17, 42, 91, 172, 293 (NTSE Stage-I/Raj./2008)


(a) 91 (b) 42 (c) 17 (d) 9

9. 2, 12, 24, 34, 68, 78, 158, 166 (NTSE Stage-I/Raj./2008)


(a) 68 (b) 78 (c) 158 (d) 166

10. 2, 6, 10, 20, 30, 42, 56 (NTSE Stage-I/Raj./2008)


(a) 6 (b) 10 (c) 20 (d) 30

11. 7, 9, 16, 25, 41, 68, 107, 173 (NTSE Stage-II, 2008)
(a) 16 (b) 41 (c) 68 (d) 107

40 PACE IIT & MEDICAL: Mumbai / Delhi & NCR / Goa / Akola / Kolkata / Nashik / Pune / Bokaro / Dubai
NTSE-MENTAL ABILITY

12. 3, 9, 27, 82, 243 (NTSE Stage-I/Raj./2009)


(a) 27 (b) 54 (c) 82 (d) 162

13. 5, 9, 17, 35, 65, 129 (NTSE Stage-I/Raj./2009)


(a) 65 (b) 35 (c) 17 (d) 9

14. 1, 5, 6, 11, 17, 27, 45, 73 (NTSE Stage-I/Raj./2009)


(a) 27 (b) 45 (c) 17 (d) 11

15. 3, 6, 11, 18, 28, 38, 51, 66 (NTSE Stage-I/Raj./2009)


(a) 18 (b) 28 (c) 38 (d) 51

16. 320, 254, 200, 155, 122, 100, 89 (NTSE Stage-I/Raj./2009)


(a) 155 (b) 320 (c) 254 (d) 200

17. 6, 8, 9, 12, 14, 18, 22, 26, 30 (NTSE Stage-I/Raj./2012)


(a) 12 (b) 22 (c) 26 (d) 30

18. 3, 7, 9, 28, 27, 84, 81, 448, 243 (NTSE Stage-I/Raj./2012)


(a) 84 (b) 81 (c) 28 (d) 7

19. 190, 94, 46, 22, 10, 3 (NTSE Stage-I/Raj./2012)


(a) 94 (b) 46 (c) 22 (d) 3

20. 0, 5, 15, 50, 128 (NTSE Stage-I/Raj./2012)


(a) 5 (b) 15 (c) 50 (d) 128

21. 9, 63, 5, 35, 1, 8 (NTSE Stage-I/Raj./2012)


(a) 63 (b) 5 (c) 35 (d) 8

22. 89, 78, 86, 80, 85, 82, 83 (NTSE Stage-I/Raj./2013)


(a) 83 (b) 82 (c) 86 (d) 78

23. 1, 1, 3, 9, 6, 36, 10, 100, 16, 225 (NTSE Stage-I/Raj./2013)


(a) 225 (b) 16 (c) 10 (d) 9

24. 444, 300, 200, 136, 87, 84, 80 (NTSE Stage-I/Raj./2013)


(a) 300 (b) 200 (c) 136 (d) 87

25. 8, 15, 31, 61, 123, 247, 491 (NTSE Stage-I/Raj./2013)


(a) 247 (b) 491 (c) 121 (d) 61

26. 3, 6, 24, 30, 63, 72, 122, 132 (NTSE Stage-I/Raj./2013)


(a) 132 (b) 30 (c) 122 (d) 72

27. 15, 34, 71, 134, 223, 350 (NTSE Stage-I/Karnataka/2014)


(a) 71 (b) 134 (c) 223 (d) 350

41 PACE IIT & MEDICAL: Mumbai / Delhi & NCR / Goa / Akola / Kolkata / Nashik / Pune / Bokaro / Dubai
NTSE-MENTAL ABILITY

Answer Key
EXERCISE-1

1. (c) 2. (b) 3. (d) 4. (d) 5. (d) 6. (c) 7. (d) 8. (d) 9. (d) 10. (a)

11. (d) 12. (a) 13. (d) 14. (d) 15. (d) 16. (b) 17. (b) 18. (c) 19. (a) 20. (c)

21. (a) 22. (a) 23. (c) 24. (a) 25. (c) 26. (c) 27. (c) 28. (c) 29. (b) 30. (a)

31. (c) 32. (d) 33. (b) 34. (b) 35. (c) 36. (a) 37. (d) 38. (b) 39. (c) 40. (c)

41. (b) 42. (d) 43. (d) 44. (c) 45. (b) 46. (c) 47. (a) 48. (c) 49. (d) 50. (b)

51. (c) 52. (b) 53. (b) 54. (c) 55. (a) 56. (b) 57. (a) 58. (d) 59. (c) 60. (b)

61. (c) 62. (c) 63. (b) 64. (a) 65. (b) 66. (b) 67. (d) 68. (b) 69. (b) 70. (c)

71. (c) 72. (c) 73. (b) 74. (b) 75. (c) 76. (b) 77. (a) 78. (b)

EXERCISE-2

1. (b) 2. (a) 3. (c) 4. (b) 5. (b) 6. (a) 7. (c) 8. (d) 9. (c) 10. (b)

11. (c) 12. (c) 13. (b) 14. (a) 15. (b) 16. (d) 17. (b) 18. (a) 19. (d) 20. (d)

21. (d) 22. (c) 23. (b) 24. (d) 25. (a) 26. (c) 27. (b)

PREVIOUS YEAR QUESTIONS

Q.1 to 4: Directions : Which number will replace the question mark in the given series. Select the correct
number from the given alternatives.

1. 68, 54, 45, 34, 27,?


(a) 13 (b) 17
(c) 18 (d) 21

2. 18, 30, 48, 72, 96,?


(a) 96 (b) 106
(c) 115 (d) 120

3. 8, 1, 9, 10, 19, 29, ?, 77


(a) 38 (b) 48
(c) 52 (d) 56

4. 12, 32, 72, 152, ? 632


(a) 312 (b) 515
(c) 613 (d) 815
[NTSE-MAHARASHTRA STAGE-1-2020]

42 PACE IIT & MEDICAL: Mumbai / Delhi & NCR / Goa / Akola / Kolkata / Nashik / Pune / Bokaro / Dubai
NTSE-MENTAL ABILITY

Q.5 and 8: Direction : In each of the following questions, choose the correct alternative that will replace the
question mark in the given sequence

5. 4, 6, 16, 62, 308, ?


(a) 990 (b) 1721
(c) 698 (d) 1846

6. 6, 9, 18, 21, 42, 45, ?.?


(a) 90, 91 (b) 90, 92
(c) 90, 93 (d) 90, 94

7. 7, 13, 25, 43, 67 ?


(a) 97 (b) 98
(c) 99 (d) 100

8. 3624, 4363, 3644, 4563, 3664, ?


(a) 4263 (b) 4363
(c) 4536 (d) 4763
[NTSE-MAHARASHTRA STAGE-1-2019]

Q9 to 12: Direction: In each of the following questions write which term in the sequence replaces the
question mark.

9. 13, 23, 43, 83, 163, ?


(a) 326 (b) 323
(c) 321 (d) 318

10. 12, 15, 21, 24, 30, 33, ?, ?


(a) 36, 41 (b) 37, 42
(c) 38, 47 (d) 39, 51

11. 16, 40, 100, 250, ?


(a) 575 (b) 625
(c) 425 (d) 525

12. 23, 29, 47, 75, ?


(a) 87 (b) 93
(c) 110 (d) 117
[NTSE-MAHARASHTRA STAGE-1-2018]

Q.13 and 14 : Directions- In each of the following questions, write which term in sequence replaces the
question mark.

13. 2, 6, 21, 88, ?


(a) 440 (b) 356
(c) 445 (d) 352

14. 6,30, 18, 128, ?


(a) 36 (b) 38
(c) 98 (d) 90
[NTSE-MAHARASHTRA STAGE-1-2017]

43 PACE IIT & MEDICAL: Mumbai / Delhi & NCR / Goa / Akola / Kolkata / Nashik / Pune / Bokaro / Dubai
NTSE-MENTAL ABILITY

Q15 and 16 Direction:- Write which number in sequence replaces the question mark.

15. 1 1 2 2
11 ,12 ,14 ,16 ,?
9 2 7 3
(a) 8 1 (b) 19 1
3 2
(c) 20 (d) 22 1
3

16. 2, 9, 38, 155,?


(a) 314 (b) 193
(c) 623 (d) 624
[NTSE-MAHARASHTRA STAGE-1-2016]

Q17 and 18:Directions: Write which number in the given sequence replaces the question mark(?)

17. 21, 24, 30, 33, 39, 51, 57, ?


(a) 60 (b) 63
(c) 69 (d) 71

18. 197, 171, 147, 125, 105, ?


(a) 71 (b) 87
(c) 97 (d) 101
[NTSE-MAHARASHTRA STAGE-1-2015]

Q.19 to 22 : Directions:- Write which number or alphabet in sequence replaces the question mark(?)

19. 7, 15, 32, 67, 138, ?


(a) 193, (b) 209
(c) 278 (d) 281

20. 3, 14, 54, 159, 314,?


(a) 309 (b) 372
(c) 391 (d) 398

21. AYC, DVF, GSI, ?


(a) KOM (b) JRH
(c) JPL (d) IQK

22. ABY, DCX, EFU, HGT, IJQ,?


(a) LKP (b) KLO
(c) LOP (d) MLO
[NTSE-MAHARASHTRA STAGE-1-2014]

Q.23 to 26 : Directions:- Write which number or alphabet in sequence replaces the question mark(?)

23. 1234, 1240, 1246, 1258, 1268,?


(a) 1280 (b) 1284
(c) 1285 (d) 1290

44 PACE IIT & MEDICAL: Mumbai / Delhi & NCR / Goa / Akola / Kolkata / Nashik / Pune / Bokaro / Dubai
NTSE-MENTAL ABILITY

24. 21, 23, 29, 47, 75, ?


(a) 87 (b) 92
(c) 99 (d) 110

25. K16M, H19J, E22G,?


(a) D23B (b) B24D
(c) B25D (d) C24E

26. MNZA, LOYB, KPXC, JQWD,?

(a) IREV (b) HSUF


(c) GTTG (d) IRVE
[NTSE-MAHARASHTRA STAGE-1-2013]

Q.27 to 30 : Directions:- In each of the following questions write which number in sequence replaces the
question mark?

27. ?, 17, 33, 51, 75


(a) 9 (b) 13
(c) 8 (d) 11

28. 14, 17, 24, 35,?


(a) 49 (b) 38
(c) 50 (d) 46

29. 37, 57, 8, 99, ?


(a) 118 (b) 119
(c) 135 (d) 137

30. 25, 30, 36, 44,?


(a) 50 (b) 52
(c) 51 (d) 47
[NTSE-MAHARASHTRA STAGE-1-2012]

Answer Key

1.
2. (a) 3. (b) 4. (a) 5. (d) 6. (c) 7. (a) 8. (d) 9. (b) 10. (d)
(Bonus)
11. (b) 12. (c) 13. (c) 14. (b) 15. (c) 16. (d) 17. (C) 18. (b) 19.(d) 20.(a)
21. (c) 22. (a) 23 (b) 24. (d) 25. (c) 26. (d) 27.(b) 28. (c) 29.(c) 30. (c)

45 PACE IIT & MEDICAL: Mumbai / Delhi & NCR / Goa / Akola / Kolkata / Nashik / Pune / Bokaro / Dubai
NTSE-MENTAL ABILITY

4. ALPHABET-SERIES

Alphabet Series problems deals with alphabets and Alpha-Numeric. While attempting to solve the question,
you have to check the pattern of the series.
Type of questions asked in the examination :
(I) Find the missing term(s).
(ii) Find the wrong term(s).

Alphabet Series
In these types of questions, a series of single or pairs of groups of letters is given. The terms of the series
form a certain pattern as regards the position of the letters in the English alphabet.

Position of Alphabet:
(I) Alphabet in order:

(ii) Alphabet in reverse order:

Directions : (1 to 7) Find the missing term:


1. A, C, ?, G, I
(A) E (B) D (C) F (D) H
Sol. (A) Series consists of alternate letter in order .So, the missing term would be E.

2. V, T, R, ?, N,?
(A) O,M (B) P,M (C) L,P (D) P,L

Sol. (D) Given series consists of alternate letters in reverse order. So, the missing terms would be P and L

3. A, C, F, ?, O
(A) G (B) J (C) H (D) K

Sol. Hint

4. DC, DE, FE, ?, HG, HI


(A) FE (B) FG (C) GF (D) GH
Sol. (B)

5. CIR, GMV, KQZ, OUD,?


(A) RYH (B) SYH (C) SZI (D) SYI

46 PACE IIT & MEDICAL: Mumbai / Delhi & NCR / Goa / Akola / Kolkata / Nashik / Pune / Bokaro / Dubai
NTSE-MENTAL ABILITY

Sol. (B) There is a continuous difference of 4 letters between the first letter of each group, second letter
of each group and third letter of each group. So the missing term would be SYH.

6. ZSD, YTC, XUB, WVA,?


(A) VWZ (B) UVW (C) VXY (D) UWZ

Sol. (A) The first letter of each group is in continuation in backward direction. The second letter of each
group is in continuation in forward direction. The third letter of each group is in continuation in
backward direction. Therefore, the missing term would be VWZ.

7. KTE, SBM, AJU, IRC,?


(A) OZL (B) QYZ (C) QZL (D) QZK

Sol. (D)
First letter of each group differ by 8 letters. Second letter of each group differ by 8 letters. Third
letter of each group differ by 8 letters. Therefore, the missing term would be QZK.

Directions : (8 to 9) Find the wrong term (s):


8. DOU, EPV, FQW, GRX, HTY, ITZ
(A) EPV (B) FQW (C) GRX (D) HTY

Sol. (D) In every term first, second and third letter is in alphabetical order to its next term respectively.
Fourth term is not following the same rule. Hence, HTY is the wrong term and should be replaced by
HSY.

9. ABC, DGJ, HMR, NTA, SBK, ZKV


(A) DGJ (B) HMR (C) NTA (D) SBK

Sol. (C) First letter of first, second, third ……terms is moved three, four, five ……..steps forward
respectively. Similarly, second letter is moved five, six, seven …….. steps forward respectively and
third letter is moved seven, eight, nine,…….. steps forward respectively. Hence, NTA is the wrong
term and should be replaced by MTA.

Alpha-Numeric Series
A series in which both alphabets and number are used
Direction : (10) Find the missing term :

10. F3X, H7U, J15R, L310,?


(A) M46L (B) N44L (C) N63L (D) N44M

Sol. (C) The first letter of each term ¡s moved two steps forward and the last letter is moved three steps
backward to obtain the corresponding letters the next term. The numbers form the sequence
3  2  1  7,7  2  1  15,15  2  1  31,31 2  1  63 , S, the missing term would be N63L…..

11. D4V, G10T, J20R, M43P, P90N


(A) G10T (B) J20 R (C) M43 P (D) P 90 N

Sol. (A) First letter of every term is moves three steps forward in each next term. Second number of every
term of the pattern  2  1, 2  2, 2  3,...... and third letter of every term is moved two step
backward. Hence, G 10 T is the wrong term and should be replaced by G 9 T.

47 PACE IIT & MEDICAL: Mumbai / Delhi & NCR / Goa / Akola / Kolkata / Nashik / Pune / Bokaro / Dubai
NTSE-MENTAL ABILITY

EXERCISE – 1

Directions: (1 to 41) Find the missing letters:


1. B, E, H,?
(a) K (b) L (c) J (d) M

2. Y, W, U, S, Q,?
(a) A (b) P (c) O (d) B

3. AH, DL, GP, JT,?


(a) MY (b) NX (c) MX (d) NY

4. LO, IL, FI, CF,?


(a) ZB (b) AB (c) ZC (d) ZO

5. ZYX, BAZ, DCB, FED,?


(a) GHF (b) FGH (c) FFG (d) HGF

6. ATL, BUM, CVN, DWO,?


(a) EZP (b) EYQ (c) EFP (d) EXP

7. TYU, NSO, HMI,?


(a) AGC (b) CGC (c) GBC (d) BGC

8. MAAL, AALM, ALMA, LMAA,?


(a) AMLA (b) MAAL (c) AAML (d) LAAM

9. A3P, C5N, E8K, G12G,?


(a) I16D (b) I17B (c) I17D (d) J16B

10. Q1F, S2E, U6D, W21C?


(a) Y66B (b) Y44B (c) Y88B (d) Z88B

11. BYDW, FUHS, JQLO, NMPK,? (NTSE Stage-I/Raj/2007)


(a) RITG (b) RJGH (c) IRGT (d) HPIN

12. BYCXA, EVFUD, HSIRG, KPLOJ? (NTSE Stage-I/Raj/2007)


(a) MNLOL (b) NMOLM (c) QJRIP (d) PKQJO

13. UTRQU, QPNMT, MLJIS, IHFER,? (NTSE Stage-I/Raj/2007)


(a) EDCAQ (b) EDBAQ (c) IHFGP (d) KJHGP

14. CDFI, EFHK, IJLO, KLNQ, OPRU? (NTSE Stage-I/Raj/2007)


(a) QRTW (b) MNPS (c) QRST (d) RSUX

15. CFIJ, RUXY, EHKL, PSVW,? (NTSE Stage-I/Raj/2007)


(a) HILM (b) UXZA (c) SVYZ (d) MOSV

16. BCFH, ?, HILN, KLOQ, NORT (NTSE Stage-I/Raj/2008)


(a) MNQS (b) EFIK (c) NOPQ (d) PQTV

48 PACE IIT & MEDICAL: Mumbai / Delhi & NCR / Goa / Akola / Kolkata / Nashik / Pune / Bokaro / Dubai
NTSE-MENTAL ABILITY

17. KMON, NPRQ, ?, TVXW, WYAZ (NTSE Stage-I/Raj/2008)


(a) QSUT (b) QTUS (c) UWYX (d) SUWV

18. GRPT, HSQS, ITRR, ?, KVTP, LWUO (NTSE Stage-I/Raj/2008)


(a) USJQ (b) QSUJ (c) JSQR (d) JUSQ

19. XBI, JNU, VZG, HLS,? (NTSE Stage-I/Raj/2008)


(a) TYE (b) TXE (c) PTA (d) UYE

20. AYCXB, EWGVF, IUKTJ, MSORN,? (NTSE Stage-I/Raj/2008)


(a) QOPPR (b) RPSQO (c) QQSPR (d) PQRPQ

21. AAZY, DDVU, GGRQ, ?, MMJI, PPFE (NTSE Stage-II, 2008)


(a) KKMN (b) MMJN (c) KKMM (d) JJNM

22. ZDOA, VHNF, ?, NPLP, JTKU, FXJZ (NTSE Stage-II, 2008)


(a) RLKM (b) MLRK (c) RKML (d) RLMK

23. ZOA, XMF, ?, TIP, RGU, PEZ (NTSE Stage-II, 2008)


(a) YXX (b) WLL (c) UKK (d) VKK

24. CGJL, FJMO, IMPR, LPSU,? (NTSE Stage-I/Raj/2009)


(a) ORUW (b) OSVX (c) JMPR (d) KORS

25. ADCG, ?, JMLP, QTSW, UXWA (NTSE Stage-I/Raj/2009)


(a) PSRV (b) PTSV (c) PRVS (d) PVRS

26. ?, CAFH, YWBD, USXZ, QOTV (NTSE Stage-I/Raj/2009)


(a) GELJ (b) INLN (c) GEJL (d) PSNP

27. AYBX, EUFT, IQJP, MMNL,? (NTSE Stage-I/Raj/2009)


(a) QIRH (b) NLOK (c) GSHR (d) PJQI

28. PQMN, NOKL, LMIJ, ?, HIEF (NTSE Stage-I/Raj/2009)


(a) KLHI (b) MNJK (c) GHIJ (d) JKGH

29. ABHR, EFIO, IJJL, MNKI, ?, UVMC (NTSE Stage-II, 2009)


(a) QRLM (b) QRST (c) QULM (d) QRLF

30. BDF, HKN, QUY, ?, RXD (NTSE Stage-II, 2009)


(a) CHM (b) BGL (c) CIO (d) BHN

31. AZYB, CXVE, FURI,? (NTSE Stage-II, 2009)


(a) KQPL (b) JRNM (c) JQMN (d) ISPM

32. DOZ, GRC, (?), ALW, BMX (NTSE Stage-I/Raj./2012)


(a) BGL (b) LWH (c) DLT (d) GJM

33. fed, ihg, lkj, (?), rqp (NTSE Stage-I/Raj./2012)


(a) npq (b) onm (c) oqp (d) nom

49 PACE IIT & MEDICAL: Mumbai / Delhi & NCR / Goa / Akola / Kolkata / Nashik / Pune / Bokaro / Dubai
NTSE-MENTAL ABILITY

34. ABYZ, ADWZ (?), AHSZ (NTSE Stage-I/Raj./2012)


(a) AFUZ (b) AUFZ (c) ZFUA (d) ZUFA

35. VTRP, NLJH, FDBZ, XVTR, (?) (NTSE Stage-I/Raj./2012)


(a) JLPN (b) LJPN (c) NPLJ (d) PNLJ

36. OBDR, QACT, SZBV, (?), WXZZ (NTSE Stage-I/Raj./2012)


(a) WUWZ (b) YTVB (c) UYAX (d) ASVD

37. YANWY, DFMBD, IKNGI, NPMLN, (?), XZMVX (NTSE Stage-I/Raj./2013)


(a) RUMSR (b) SUNQS (c) UWNSU (d) VUMTV

38. PEXKW, RFWMU, TGVOS, VHUQQ, XITSO, (?) (NTSE Stage-I/Raj./2013)


(a) ZJSUM (b) YJSUZ (c) ZKSVJ (d) JZSTN

39. AYBZC, DWEXF, GUHVI, JSKTL, (?), POQPR (NTSE Stage-I/Raj./2013)


(a) MQDRN (b) QMONR (c) MQNRO (d) NQMOR

40. ZYYZR, ABVUN, (?), BCUTM, XWABT, CDTSL (NTSE Stage-I/Raj./2013)


(a) YXZAS (b) ZYABT (c) XWYZR (d) YXZAB

41. deb, ijg, nol, (?), xyv (NTSE Stage-I/Raj./2013)


(a) rsp (b) stp (c) rsq (d) stq

Directions: In each of the questions 42 to 44 some of the letters are missing in the given series with one
term missing shown by question mark (?). This term is one of the alternatives among the four groups
of letters given under if find the right alternative. (NTSE Stage-I/Raj./2014)

42. BEG, DGI, FIK, HKM, (?)


(a) JMO (b) KMO (c) JML (d) JNP

43. KEM, IDL, GCK, (?), CAI


(a) ECJ (b) EBK (c) FBJ (d) EBJ

44. JCME, LDOG, NEQI, (?)


(a) PFSJ (b) PESI (c) PESK (d) PFSK

45. FOX, IQV, LST, OUR,? (NTSE Stage-I/Raj./2015)


(a) RPW (b) RWP (c) QVS (d) SXU

46. qpo, nml,? (NTSE Stage-I/Raj./2015)


(a) ghf (b) ijk (c) kji (d) hgi

47. MYZ, LWX, ?, JST (NTSE Stage-I/Raj./2016)


(a) KUV (b) IQR (c) HOP (d) GMN

48. bdf, hjl _______, tvx (NTSE Stage-I/Raj./2016)


(a) nrp (b) pnr (c) nqr (d) npr

49. LO, JQ, HS,? (NTSE Stage-I/Raj./2017)


(a) FU (b) FQ (c) EV (d) DW

50 PACE IIT & MEDICAL: Mumbai / Delhi & NCR / Goa / Akola / Kolkata / Nashik / Pune / Bokaro / Dubai
NTSE-MENTAL ABILITY

50. ZXV, TRP, NLJ, ? (NTSE Stage-I/Raj./2017)


(a) HEF (b) HFD (c) EFH (d) IGE

EXERCISE – 2
Directions (1 to 10): There is a wrong term in the following numbers/ letters series. Find the wrong
term of the series
1. ABC, BCD, CDE, DEF, FEG
(a) BCD (b) CDE (c) DEF (d) FEG

2. ZOA, XMF, VKK, THP, RGU, PEZ


(a) THP (b) XMF (c) VKK (d) RGU

3. AACC, BBED, CCHE, DDMF, EEQG


(a) AACC (b) DDMF (c) BBED (d) EEQG

4. ECA, JHF, OMK, TQP, YWU


(a) ECA (b) JHF (c) TQP (d) YWU

5. DKY, FJW, HIT, JHS, LGQ


(a) FJW (b) LGQ (c) JHJ (d) HIT

6. DVG, FSI, HPK, JNM, LJO


(a) DVG (b) JNM (c) HPK (d) LJO

7. ABD, DGK, HMS, NTB, SBL, ZKW


(a) NTB (b) DGK (c) SBL (d) ZKW

8. EPV, FQW, GRX, HTY, ITZ


(a) FQW (b) GRX (c) HTY (d) ITZ

9. PON, RQP, TSR, VVT, XWV, ZYX


(a) VVT (b) TSR (c) XWV (d) RQP

10. P 3 C, R 5 F, T 8 I, V 12 L, X 18 O, Z 23 R
(a) V 1 2 L (b) X 18 O (c) Z 23 R (d) R 5 F

Answer Key
EXERCISE – 1
1. (a) 2. (c) 3. (c) 4. (c) 5. (d) 6. (d) 7. (d) 8. (b) 9. (b) 10. (c)

11. (a) 12. (b) 13. (b) 14. (a) 15. (c) 16. (b) 17. (a) 18. (d) 19. (b) 20. (c)

21. (d) 22. (d) 23. (d) 24. (b) 25. (a) 26. (c) 27. (a) 28. (d) 29. (d) 30. (a)

31. (c) 32. (b) 33. (b) 34. (a) 35. (d) 36. (c) 37. (b) 38. (a) 39. (c) 40. (a)

41. (d) 42. (a) 43. (d) 44. (d) 45. (b) 46. (c) 47. (a) 48. (d) 49. (a) 50. (b)

EXERCISE – 2
1. (d) 2. (a) 3. (b) 4. (c) 5. (d) 6. (b) 7. (a) 8. (c) 9. (a) 10. (b)

51 PACE IIT & MEDICAL: Mumbai / Delhi & NCR / Goa / Akola / Kolkata / Nashik / Pune / Bokaro / Dubai
NTSE-MENTAL ABILITY

PREVIOUS YEAR QUESTIONS

Q. 1 to 4-Direction: In each of the following questions write which correct term in sequence replaces
the question mark? (NTSE-MAHARASHTRA STAGE-1-2020)

1. CD, HI, MN,?


(a) QS (b) OP (c) RS (d) PQ

2. RD, PG, MK, IN,?


(a) ER (b) DR (c) CQ (d) DQ

3. BM26, EN70, HO120, KP176, ?


(a) NQ25O (b) NP224 (c) MQ221 (d) NQ238

4. T23C, QG24, 26NL, KP27, ?


(a) 29GV (b) 29HU (c) 27GT (d) 28HT

Q. 5 to 8-Direction: Choose the correct alternative that will replace the question mark.
(NTSE-MAHARASHTRA STAGE-1-2019)
5. JDP, NGR, RJT, VMV, ?
(a) ZPW (b) ZQY (c) ZPX (d) ZRY

6. V422 D, S719G, P1016 J, M1313M,?


(a) K1711P (b) J1610P (c) J1611P (d) I1512O

7. 29AYC, EUG33, IQ37K, ?


(a) MMO41 (b) MZB41 (c) MNP43 (d) MPO44

8. ZAB, WDE, SHI, NMA,?


(a) VEF (b) UFG (c) FUG (d) HSG

Q. 9 to 12-Direction: In the following questions choose the correct term that will replace the question mark.
(NTSE-MAHARASHTRA STAGE-1-2018)

9. AYC, EUG, JPL, CWE,?


(a) HRJ (b) IQK (c) JPL (d) KOM

10. NTS, OUT, PTS,?


(a) QWV (b) QRS (c) QTP (d) QPO

11. AMZN, BLYO, CKXP,?


(a) DQJW (b) DJWQ (c) DIWR (d) DWJQ

12. JBY, NIV, SOS, YTP,?


(a) EVM (b) BVG (c) FYL (d) FXM

52 PACE IIT & MEDICAL: Mumbai / Delhi & NCR / Goa / Akola / Kolkata / Nashik / Pune / Bokaro / Dubai
NTSE-MENTAL ABILITY

Q. 13 and 14-Direction: In each of the following questions write which term in sequence replaces the
question mark? (NTSE-MAHARASHTRA STAGE-1-2017)

13. BJ, DL, HP, PX,?


(a) FN (b) FX (c) TB (d) VD

14. AYCD, EUGH, IQKL?


(a) AYCD (b) BXDE (c) MNAB (d) MZBC

Q. 15 and 16-Directions: In each of the following questions write which number in sequence replaces the
question mark. (NTSE-MAHARASHTRA STAGE-1-2016)

15. ADY, CFW, EHU,?


(a) GIH (b) FIT (c) GJR (d) GJS

16. Y, W, T, R, O,?
(a) L (b) P (c) K (d) M

Q.17 and 18-Directions: Write the alphabets in the given sequence that will replace the question mark.
(NTSE-MAHARASHTRA STAGE-1-2015)
17. CX, EV, HS, LO,?
(a) GT (b) DW (c) BY (d) MN

18. AM, NZ, CK, PX, EI,?


(a) GF (b) VR (c) US (d) RV

(Q 19 to 22)-Direction: In each of the following questions. Write which term replaces the question mark ?
(NTSE-MAHARASHTRA STAGE-1-2012)

19. A, D, H, N, U, ? ,
(a) Z (b) G (c) F (d) H

20. A, Q, C, L, F, H, J,?
(a) E (b) B (c) F (d) D

21. BYCXW, CXDWV, EVFUT, GTHSR,?


(a) IRJQP (b) KPOLN (c) KPLON (d) JOKPO

22. FOX, GP?, HQZ


(a) Y (b) Z (c) T (d) W

Answer Key

10.
1. (c) 2. (b) 3. (d) 4. (b, d) 5. (c) 6. (b) 7. (a) 8. (b, d) 9. (c)
(bonus)
11. (b) 12. (d) 13. (a) 14. (d) 15. (d) 16. (d) 17. (d) 18. (d) 19. (b) 20. (a)

21. (c) 22. (a)

53 PACE IIT & MEDICAL: Mumbai / Delhi & NCR / Goa / Akola / Kolkata / Nashik / Pune / Bokaro / Dubai
NTSE-MENTAL ABILITY

5. LETTER REPEATING SERIES

Letter Repeating Series


These types of questions usually consist of a series of small letters which follow a certain pattern. However,
some letters are missing from the series. These missing letters are then given in a proper sequence as one of
the alternatives.
Pattern of such questions is that some letters in sequence are missing.

(i) The letters may be in cyclic order (clockwise or anti-clockwise).


(ii) To solve a problem, we have to select one of the alternatives from the given alternatives. The
alternative which gives a sequence form of letters is the choice.

Directions: (1 to 6) Which sequence of letters when placed at the blanks one after the other will complete
the given letter series ?

1. a_ab_ba_a_ab
(A) babb (B) abba (C) baba (D) aabb
Sol. (A) we proceed step by step to solve the above series :

Steps:
1. We have two letters a and b making the series.
2. The first blank space should be filled in by ‘b’ so that we have one a followed by one ‘b’.
3. Second blank space should be filled in by ‘a’ so that the same pattern followed till end.

2. a_cab_ a_c_bc
(A) bbac (B) abab (C) abba (D) bcba
Sol. (D)

Series is abc/ abc/ abc/ abc. So, pattern abc is repeated.

3. _abb_a_ baa_ b
(A) baba (B) abba (C) aabb (D) aaab
Sol. (C) Series is aabb/ aabb/ aabb. So, pattern aabb is repeated.

4. ba_cb_b_ bab_
(A) acbb (B) bcaa (C) cabb (D) bacc
Sol. (D)
The series is b a b c/b a b c/b a b c So, pattern babc is repeated

5. ab_aa_caab_c_abb_c
(A) bbcaa (B) bcbca (C) cabac (D) cbbac
Sol. (D) Series is abc/ aabc/ aabbc / aabbcc

6. bc_b_c_b_ccb
(A) cbcb (B) bbcb (C) cbbc (D) bcbc

Sol. (A) Series is bccb/ bccb/ bccb. So, pattern bccb is repeated.

54 PACE IIT & MEDICAL: Mumbai / Delhi & NCR / Goa / Akola / Kolkata / Nashik / Pune / Bokaro / Dubai
NTSE-MENTAL ABILITY

Direction : (7 to 9) The series given below is based on the letter series, In the series, some letters are
missing. Select the correct alternative. If more than five letters are missing, select the last five letters of the
series.
7. x_xxy_x_xy_yxx_ _ yy_y
(A) xyyyy (B) xxyyx (C) yxxyx (D) xyxyx
Sol. (A) The pattern of series is xy/xxyy/xxxyyy

8. xyzu_yz_v_ _ uv_ _ _ _ _ _ _
(A) uvxyz (B) vuzyx (C) uvzyx (D) vuxyz

Sol. (A)The series is x y z u v/y z u v x/z u v x y/u v x y z


Thus the letters are written in a cyclic order.

9. abcd_bc_e_ _de_ _ _ _ _ _ _
(A) deabc (B) edcba (C) decba (D) edabc

Sol. (A)The series is a b c d e/ b c d e a / c d e a b/ b e a b c


Thus the letters are written in a cyclic order.

Directions : (10 to 11) There is a letter series in the first row and a number series in the second row. Each
number in the number series stands for a letter in the letter series. Since in each of that series some term are
missing you have to find out as to what those terms are, and answer the questions based on these as given
below in the series.

10. _miax_irxa__ma______
4 _ 5 _ 7 3 _ _ _ 6 _ _ _ _ _ _ _ _ _ _. The last five term of the letter series are
(A) r m x i a (B) x m r a i (C) x r m a i (D) r m i x a

Sol. (D) a=6, i= 5,m=3, r=4 and x=7 the letter series runs as rmiax mirxa irmax rmixa. By taking the
letter in the groups of five, we find that first letter of the first group (i.e. r) is the third letter of the
second group and the last two letters have interchanged their positions. The same rule applies in
others groups also.

11. a _ h_ _ c _ n e _ h _ e a c _ _ _ _ _
21 _ 4 3 _ 5 _ _ 2 5 4 _ _ _ _ _ _ _ _
The last five terms in the series are
(A) 32524 (B) 43215 (C) 25314 (D) 32541

Sol. (B) By taking a = 2, c = 1, n = 4, h = 5 and e = 3, the numbers series runs as 21543 15432 54321 43215.
If first digit of a group of five digits is placed as the last digit, we obtain the second group of five
digits and so on.

Direction : (12) In the following question, three sequences of letter/numbers are given which correspond to
each other in some way. In the given question, you have to find out the letter/numerals that come in the
vacant places marked by (?). These are given as one of the four alternatives under the question. Mark your
answer as instructed.

12. CB__D_ BABCCB


__2 354_ _ ? ? ? ?
p_ p q_r_ q _ _ _ _
(A) 4554 (B) 4334 (C) 4224 (D) 2552

55 PACE IIT & MEDICAL: Mumbai / Delhi & NCR / Goa / Akola / Kolkata / Nashik / Pune / Bokaro / Dubai
NTSE-MENTAL ABILITY

Sol. (C) Comparing the positions of the capital letters, numbers and small letters, we find p corresponds
to C and 2 corresponds to p. So, p and 2 correspond to C. q corresponds to A and 3 corresponds to q.
So, q and 3 corresponds to A. Also, 5 corresponds to D. So, the remaining number i.e., 4 corresponds
to B. So, BCCB corresponds to 4, 2, 2, 4.

EXERCISE

Directions: (1 to 6) Which sequence of letters when placed at the blanks one after the other will
complete the given letter series?

1. _ a a b b _ a b b a _b
(a) b a b (b) a b a (c) b b a (d) b a a

2. a_b aa_baa_ba
(a) a a b (b) b a b (c) b b a (d) b b b

3. _b aa_ba_ aab_
(a) b a b a (b) b b a a (c) a b b b (d) b b a b

4. b a b b b _ b _ b _ bb
(a) b b a (b) b a a (c) a b a (d) a a a

5. _ ha _ hach _ c _ _
(a) ccaha (b) achac (c) chaaa (d) aaach

6. m _ l _ ml _ m _ llm
(a) lmmm (b) lmlm (c) lmml (d) mllm

Directions: (7 to 8) The series given below are based on the letter series. In each of these series, some
letters are missing. Select the correct alternative. If more than five letters are missing, select the last
five letters of the series.

7. abcd _ bc _ e _ _ de _ _ _ _ _ _
(a) deabc (b) edcba (c) decba (d) edabc

8. _ _ r _ tqrptsrpqst _ _ _ _ _ _
(a) pqrts (b) pqtrs (c) pqrst (d) qrpst

Directions: (9 to 10) There is a letter series in the first row and a number series in the second row.
Each number in the number series stands for a letter in the letter series. Since in each of that series
some term are missing you have to find out as to what those terms are, and answer the questions
based on these as given below in the series.
9. n_gf_t_fhtn__t_b_f
13_2450_4__3______
The last five terms of the number series are
(a) 50123 (b) 40321 (c) 40231 (d) 51302

10. _ m ye__ ylx_ ylm _ _ l____


46_586___57_658_____
The last five terms of the number series are
(a) 46758 (b) 74658 (c) 76485 (d) 46785
56 PACE IIT & MEDICAL: Mumbai / Delhi & NCR / Goa / Akola / Kolkata / Nashik / Pune / Bokaro / Dubai
NTSE-MENTAL ABILITY

Directions: (11 to 34) Which sequence of letters when placed at the blanks one after the other will
complete the given letter series?

11. a _ b b _ caab _ ccaa _ bcc (NTSE Stage-I/Raj./2007)


(a) bacb (b) acbb (c) caba (d) abba

12. ab _ aa _ bbb _ aaa _ bbba (NTSE Stage-I/Raj./2007)


(a) baab (b) abab (c) aaab (d) abba

13. abca _ bcaab _ aa _ caa _ c (NTSE Stage-I/Raj./2007)


(a) abac (b) abba (c) acbb (d) ccaa

14. abb _ baa _ a _ bab _ aba (NTSE Stage-I/Raj./2007)


(a) ccac (b) abab (c) aabb (d) abba

15. bc _ b _ c _ b _ ccb (NTSE Stage-I/Raj./2007)


(a) cbcb (b) cbbc (c) bbcb (d) bcbc

16. ab _ ab _ ababb _ b _ ab (NTSE Stage-I/Raj./2008)


(a) baaa (b) abbb (c) aaab (d) baab

17. p _ rp _ qr _ qrr _ qrp (NTSE Stage-I/Raj./2008)


(a) qqpq (b) ppqq (c) qpqp (d) qqpp

18. mq _ sqm _ qssq _ m _ ssqmm _ s (NTSE Stage-I/Raj./2008)


(a) qsmsq (b) qmsmq (c) smmqq (d) sqmss

19. aa _ bb _ aa _ abbbb _ a (NTSE Stage-I/Raj./2008)


(a) abab (b) baba (c) aabb (d) bbaa

20. aaa _ bb _ aab _ baaa _ bb (NTSE Stage-I/Raj./2008)


(a) bbaa (b) babb (c) baab (d) abab

21. aba_a_ab a_ aba_ab (NTSE Stage-II, 2008)


(a) a a a a (b) b b b b (c) a b a a (d) b a b a

22. _lmnok_mno kl_ noklm_o (NTSE Stage-II, 2008)


(a) l k l m (b) l l m m (c) k l m n (d) m l m n

23. k_ kklll_ mm_ m_ n_ n (NTSE Stage-II, 2008)


(a) l k n m n (b) l k m n n (c) l l m m n (d) k l m n n

24. bc _ b _ c _ b _ ccb (NTSE Stage-I/Raj./2009)


(a) cbcb (b) bbcb (c) cbbc (d) bcbc

25. ab _ baa _ cb _ ab _ ba (NTSE Stage-I/Raj./2009)


(a) cacb (b) bacb (c) cbac (d) abcb

26. ab _ ba _ abab _ _ (NTSE Stage-I/Raj./2009)


(a) aaaa (b) babb (c) aaba (d) aaab

57 PACE IIT & MEDICAL: Mumbai / Delhi & NCR / Goa / Akola / Kolkata / Nashik / Pune / Bokaro / Dubai
NTSE-MENTAL ABILITY

27. _ bc _ _ bb _ aabc (NTSE Stage-I/Raj./2009)


(a) acac (b) babc (c) abab (d) aacc

28. ab _ bca _ a _ a _ c (NTSE Stage-I/Raj./2009)


(a) abba (b) ccbb (c) bacb (d) abcc

29. B _ N _ ZF _ HNT _ FBH _ TZ (NTSE Stage-II, 2009)


(a) H B T Z N F (b) H T B F N Z (c) T B Z H N F (d) H T B Z N F

30. U _ T _ K _ P _ CKUP _ CK (NTSE Stage-II, 2009)


(a) T C P T U (b) C T P U T (c) P C U T T (d) P U T C T

31. J _ HL _ K _ IH _ LKJI _ LLK (NTSE Stage-II, 2009)


(a) H I L J L (b) I L J L H (c) L J L H I (d) J L H I L

32. _ ab _ a _ bb _ ab _ a _ (NTSE Stage-I/Raj./2012)


(a) abaaba (b) babbba (c) aabbab (d) bbaabb

33. _ _ a bb _ bba _ bab _ a _ (NTSE Stage-I/Raj./2012)


(a) abaaba (b) aabbaa (c) bbabbb (d) bbaabb

34. _ ac _ ca _ aca _ a _ a _ (NTSE Stage-I/Raj./2012)


(a) cacaca (b) aaaccc (c) acacac (d) cacccc

Directions: (35 to 37) In each of the following questions, three sequences of letter/numbers are given
which correspond to each other in some way. In each question, you have to find out the letter/
numerals that come in the vacant places marked
35. _AC_BD_CDCD
2_41_14____
rs_qr_p????
(a) p q p q (b) p r p r (c) r q r q (d) r s r s

36. A_BAC_D_BCDC
_4_3_2_5????
dc__bacb____
(a) 2 4 5 4 (b) 2 5 4 5 (c) 3 4 5 4 (d) 4 5 2 5

37. _ADACB__BDCC
24__2353____
p__q__rs????
(a) p r s s (b) p s r r (c) r p s s (d) s r p p

Directions: (38 to 40) These questions are based on letter series in which some of the letters are
missing. The missing letters are given in the proper sequence in one of the alternatives among the four
given under each question. Find out the correct alternatives for each question.
38. ab _ acc _ _ da _ bba _ (NTSE Stage-I/Raj./2013)
(a) cdabc (b) badaa (c) cdbcd (d) dbacd

39. abb _ _ ab _ b _ bba _ a (NTSE Stage-I/Raj./2013)


(a) bbbab (b) babba (c) abaab (d) bbabb
58 PACE IIT & MEDICAL: Mumbai / Delhi & NCR / Goa / Akola / Kolkata / Nashik / Pune / Bokaro / Dubai
NTSE-MENTAL ABILITY

40. b _ a _ bab _ ab _ a (NTSE Stage-I/Raj./2013)


(a) baba (b) babb (c) abab (d) abba

41. In the following letter sequence, some of the letters are missing. These are given in order as one of
the alternatives below. Choose the correct alternative.
 _   _  _    _ ... (NTSE Stage-II, 2013)
(a)  (b)  (c)  (d) 

42. ca _ cab _ ab _ _ bc _ (NTSE Stage-I/Raj./2014)


(a) bccaa (b) accab (c) bacca (d) abaca

Answer Key

1. (d) 2. (d) 3. (c) 4. (c) 5. (a) 6. (b) 7. (a) 8. (a) 9. (d) 10. (d)
11. (b) 12. (a) 13. (c) 14. (d) 15. (a) 16. (a) 17. (d) 18. (c) 19. (d) 20. (b)

21. (b) 22. (c) 23. (d) 24. (a) 25. (c) 26. (a) 27. (a) 28. (b) 29. (d) 30. (c)

31. (b) 32. (a) 33. (c) 34. (d) 35. (a) 36. (b) 37. (d) 38. (b) 39. (a) 40. (c)

41. (b) 42. (a)

PREVIOUS YEAR QUESTIONS

Q.1 to 2- Direction: A rhythmic arrangement of letters is given. The missing letters appear in the same
order in one of the alternative answers. Find the correct alternative.

1. p _ rsqr _ _ rs _ q _ pqr
(a) qrspq (b) qrppr
(c) qspps (d) qsqpr

2. a _ cb _ ac _ _ ab _
(a) bcabb (b) bcaab
(c) bacbc (d) bcabc
[NTSE-MAHARASHTRA STAGE-1-2020]

Q.3 and 4. Direction: A rhythmic arrangement of letter is given. The missing letters appear in the same
order in one of the alternative answer. Choose the correct alternative.

3. ab _ bc _ c _ ba _ c
(a) baac (b) aabb
(c) caab (d) aaab

4. abb _ baa _ _ bb _ b _ ab
(a) bbaha (b) abaaa
(c) abbba (d) ababa
[NTSE-MAHARASHTRA STAGE-1-2019]

59 PACE IIT & MEDICAL: Mumbai / Delhi & NCR / Goa / Akola / Kolkata / Nashik / Pune / Bokaro / Dubai
NTSE-MENTAL ABILITY

Q.5 and Q.6 Direction: A rhythmic arrangement of alphabet is given. The missing alphabet appears in the
same order in one of the alternative answer, choose the correct alternative.

5. _b c_ab_ ca ab c
(a) a c b (b) b a b
(c) a b a (d) a a c

6. a b b _ b a a _a _ b a b _ a b
(a) abab (b) ccac
(c) aabb (d) abba
[NTSE-MAHARASHTRA STAGE-1-2018]

7. A rhythmic arrangement of numbers given. The missing numbers appear in the same order in one of
the alternative answer. Find the correct alternative.
0 – 0100 – 10 –1111 – –
(a) 01011 (b) 01101
(c) 0 1111 (d) 01110
[NTSE-MAHARASHTRA STAGE-1-2017]

8. A rhythmic arrangement of alphabets is given. The missing letter alphabets appear in the same order
as in one of the alternative answers. Find the correct alternative.
_ bcdbc _ dcabd _ bcdbc _ dc _ bd
(a) aaaaa (b) bbbbb
(c) ccccc (d) ddddd
[NTSE-MAHARASHTRA STAGE-1-2016]

9. A rhythmic arrangement of alphabets is given. The missing letter alphabets appear in the same order
in one of the alternative answers. Find the correct alternative :
_ bc _ ca _ aba _ c_ ca
(a) abbcc (b) bacba
(c) bbbcc (d) abcbb
[NTSE-MAHARASHTRA STAGE-1-2013]

Q.10 and 11  Directions: - A rhythmic arrangement of alphabets or numbers is given. The missing letter
alphabets or numbers appear in the same order in one of the alternative answer. Find the correct alternative.

10. ab _ d _ abcc _ ab _ _da


(a) cdbdc (b) cddbc
(c) cbddc (d) cddcb

11. 01 _ 110_ _ 01_00_0_10


(a) 001011 (b) 001101
(c) 010110 (d) 001111

Answer Key

1. (c) 2. (d) 3. (c) 4. (b) 5. (a) 6. (d) 7. (c) 8. (a) 9. (d) 10. (b)
11. (d)

60 PACE IIT & MEDICAL: Mumbai / Delhi & NCR / Goa / Akola / Kolkata / Nashik / Pune / Bokaro / Dubai
NTSE-MENTAL ABILITY

6. MISSING TERM IN FIGURE

Missing Term in Figure


In such type of questions, a figure, a set of figures, an arrangement or a matrix is given each of which bears
certain characters, be it numbers, letters or a group or combination of letters or numbers, following a certain
pattern.

Directions : (1 to 10) Find the missing term/number(s) :

1.

(A) 125 (B) 25 (C) 625 (D) 156

Sol. (C)
2
Clearly 1  3   16
2 2
15  6    21  441
2 2
10  5   15   225
 missing number in figure
2 2
12  13   25   625

2.

(A) 64 (B) 36 (C) 34 (D) 60

Sol. (A) Moving clockwise, in every quarter region, value of numbers gets doubled.
2  2  4,8  2  16,16  2  32,32  2  64
128  2  256

3.
7 9 11
2 3 2
53 90 ?
(A) 120 (B) 100 (C) 125 (D) 64
2 2
Sol. In the first column, 7  2  53
In the second column, 92  32  90
So, missing number, 112  22  125

61 PACE IIT & MEDICAL: Mumbai / Delhi & NCR / Goa / Akola / Kolkata / Nashik / Pune / Bokaro / Dubai
NTSE-MENTAL ABILITY

4.
B C ?
O Q S
M N R
(A) A (B) D (C) G (D) P
Sol. (A)
In each column, the sum of top & bottom letter is equal to the order of the middle letter in that
column.

5.

(A) 10 (B) 9 (C) 8 (D) 6

Sol. (B)
In the first set, 2  2  4
In the second set, 3  9  27
 2  x  18, x  9

(A) 16 (B) 14 (C) 20 (D) 22

Sol. (B)
In the first diagram,  3  4  6   72 and the number on the sides twice as the number on the opposite

vertex. 3  2  6,6  2  12,4  2  8 .In the second diagram, 1 2  3 and the number on the sides
twice as the number on the opposite vertex 3  2  6, 2  2  4,1 2  2
 In the Third diagram,  7  4  5  140 and the number on the sides twice as the number on the
opposite vertex. 7  2  14,5  2  10, 4  2  8 .

7.

(A) 0 (B) 5 (C) 10 (D) 15

Sol. (B) In first figure, 14  6  2  2  16d


In second figure, 12  5  31  14
 In third figure, 11  4  10 1  5

62 PACE IIT & MEDICAL: Mumbai / Delhi & NCR / Goa / Akola / Kolkata / Nashik / Pune / Bokaro / Dubai
NTSE-MENTAL ABILITY

8.

(A) 262 (B) 622 (C) 631 (D) 631

Sol. (B)
In first figure,  915  364  551
In second figure,  789  543  246
 In third figure, missing number
  863  241  622

9.

(A) 5 (B) 19 (C) 27 (D) 89

Sol. (D)
In first figure,  6  3   5 15  18  75  93 . In second figure,  4  8  18  1  32  18  50
 In third figure, missing number
  9  6   7  5  54  35  89

10. Which one number can be placed at the sign of interrogation?

(A) 5 (B) 6 (C) 8 (D) 9

Sol. (D)
In first figure, 27  63  3  93
In second figure, 38  37  3  79
So, in third figure, 16  42  x  67, x  9

63 PACE IIT & MEDICAL: Mumbai / Delhi & NCR / Goa / Akola / Kolkata / Nashik / Pune / Bokaro / Dubai
NTSE-MENTAL ABILITY

EXERCISE

Directions: (1 to 83) Find the missing numbers:

1.

(a) 125 (b) 216 (c) 121 (d) 225

2.

(a) 6 (b) 8 (c) 10 (d) 14

3.

(a) 75 (b) 26 (c) 25 (d) 20

4.

(a) 49 (b) 50 (c) 48 (d) 55

5.

(a) 47 (b) 45 (c) 37 (d) 35

6. 3  5  7  8  5783, 9  5  3  2  5329, ? ? 8  5  2853.


(a) 2  3 (b) 3  2 (c) 4  3 (d) 8  3

64 PACE IIT & MEDICAL: Mumbai / Delhi & NCR / Goa / Akola / Kolkata / Nashik / Pune / Bokaro / Dubai
NTSE-MENTAL ABILITY

7.
5 6 12 42

7 5 7 42

3 4 10 22

5 ? 12 32
(a) 3 (b) 4 (c) 5 (d) 6

8.

(a) 169 (b) 168 (c) 85 (d) 706

9.
7 9 11

2 3 2

51 84 ?
(a) 125 (b) 100 (c) 123 (d) 64

10.
1 2 3

1 8 ?

1 4 9

(a) 6 (b) 27 (c) 30 (d) 12

11.

(a) 40 (b) 32 (c) 35 (d) 30

12.

(a) Z (b) A (c) E (d) F

65 PACE IIT & MEDICAL: Mumbai / Delhi & NCR / Goa / Akola / Kolkata / Nashik / Pune / Bokaro / Dubai
NTSE-MENTAL ABILITY

13.

(a) 15 (b) 20 (c) 25 (d) 40

14.
963 2 844

464 ? 903
(a) 1 (b) 2 (c) 3 (d) 4

15.

(a) 72 (b) 70 (c) 68 (d) 66


16.

(a) 262 (b) 274 (c) 320 (d) 132

17.

(NTSE Stage – I / Raj./2007)


(a) 3 (b) 4 (c) 5 (d) 6

18.

(NTSE Stage – I / Raj./2007)


(a) 6 (b) 7 (c) 8 (d) 9
19.

(NTSE Stage – I / Raj./2007)


(a) 4.5 (b) 5.0 (c) 8.5 (d) 7.0
66 PACE IIT & MEDICAL: Mumbai / Delhi & NCR / Goa / Akola / Kolkata / Nashik / Pune / Bokaro / Dubai
NTSE-MENTAL ABILITY

20.

(NTSE Stage – I / Raj./2007)


(a) 36 (b) 38 (c) 48 (d) 42

21.

(NTSE Stage – I / Raj./2007)


(a) 11 (b) 13 (c) 15 (d) 17

22.
11 12 13

18 15 16

25 ? 21
(NTSE Stage – I / Raj./2007)
(a) 19 (b) 18 (c) 20 (d) 21

23.
13 8 10

16 22 31

21 ? 9
(NTSE Stage – I / Raj./2007)
(a) 20 (b) 21 (c) 22 (d) 23

24.
1 4 ?

64 9 16

49 36 25
(NTSE Stage – I / Raj./2007)
(a) 5 (b) 40 (c) 45 (d) 81

25.
24 3 15

? 0 48

80 63 35
(NTSE Stage – I / Raj./2007)
(a) 7 (b) 8 (c) 9 (d) 10
67 PACE IIT & MEDICAL: Mumbai / Delhi & NCR / Goa / Akola / Kolkata / Nashik / Pune / Bokaro / Dubai
NTSE-MENTAL ABILITY

26.
1 26 6

21 5 31

16 ? 11
(NTSE Stage – I / Raj./2007)
(a) 27 (b) 31 (c) 36 (d) 41

27.
B Y X

D W ?

F U T
(NTSE Stage-II, 2007)
(a) V (b) C (c) U (d) E

28.
Z B X

Y C V

? D T
(NTSE Stage-II, 2007)
(a) W (b) X (c) Y (d) Z

29.
E ? O

C H M

A F K
(NTSE Stage-II, 2007)
(a) L (b) J (c) G (d) N

30.
N O P

S R ?

T U V
(NTSE Stage-II, 2007)
(a) T (b) K (c) F (d) Q

31.
6 21 36

9 45 81

7 (?) 49
(NTSE Stage – I / Raj./2008)
(a) 32 (b) 28 (c) 35 (d) 56
68 PACE IIT & MEDICAL: Mumbai / Delhi & NCR / Goa / Akola / Kolkata / Nashik / Pune / Bokaro / Dubai
NTSE-MENTAL ABILITY

32.
3 5 16

7 9 32

11 13 (?)
(NTSE Stage – I / Raj./2008)
(a) 48 (b) 64 (c) 24 (d) 143

33.
1 5 9

4 8 12

7 (?) 15
(NTSE Stage – I / Raj./2008)
(a) 11 (b) 12 (c) 13 (d) 16

34.
3 4 43

7 5 57

9 11 (?)
(NTSE Stage – I / Raj./2008)
(a) 34 (b) 75 (c) 119 (d) 911

35.
8 11 15

25 34 46

74 101 (?)
(NTSE Stage – I / Raj./2008)
(a) 138 (b) 139 (c) 140 (d) 137

36.

(NTSE Stage – I / Raj./2008)


(a) 9 (b) 15 (c) 18 (d) 12

37.

(NTSE Stage – I / Raj./2008)


(a) 6 (b) 15 (c) 30 (d) 47

69 PACE IIT & MEDICAL: Mumbai / Delhi & NCR / Goa / Akola / Kolkata / Nashik / Pune / Bokaro / Dubai
NTSE-MENTAL ABILITY

38.

(NTSE Stage – I / Raj./2008)


(a) 2 (b) 38 (c) 27 (d) 18

39.

(NTSE Stage – I / Raj./2008)


(a) 3 (b) 2 (c) 5 (d) 4

40.

(NTSE Stage – I / Raj./2008)


(a) 120 (b) 59 (c) 62 (d) 22

41.

(NTSE Stage-II, 2008)


(a) 25 (b) 37 (c) 40 (d) 42

42.

(NTSE Stage-II, 2008)


(a) 85 (b) 81 (c) 75 (d) 64

43.

(NTSE Stage-II, 2008)


(a) 9 (b) 11 (c) 14 (d) 16
44.

(NTSE Stage-II, 2008)


(a) 12 (b) 9 (c) 8 (d) 6
70 PACE IIT & MEDICAL: Mumbai / Delhi & NCR / Goa / Akola / Kolkata / Nashik / Pune / Bokaro / Dubai
NTSE-MENTAL ABILITY

45.

(NTSE Stage-II, 2008)


(a) 15 (b) 13 (c) 9 (d) 8

46.
6 8 4

9 12 6

15 20 (?)
(NTSE Stage-II, 2008)
(a) 5 (b) 10 (c) 20 (d) 25

47.
7 8 9

7 15 24

7 (?) 46
(NTSE Stage-II, 2008)
(a) 33 (b) 23 (c) 22 (d) 14

48.
78 ? 97

43 67 58

35 13 39
(NTSE Stage-II, 2008)
(a) 84 (b) 80 (c) 54 (d) 48

49.

(NTSE Stage – I / Raj./2009)


(a) 4 (b) 9 (c) 6 (d) 8

50.

(NTSE Stage – I / Raj./2009)


(a) 9 (b) 8 (c) 6 (d) 5

71 PACE IIT & MEDICAL: Mumbai / Delhi & NCR / Goa / Akola / Kolkata / Nashik / Pune / Bokaro / Dubai
NTSE-MENTAL ABILITY

51.

(NTSE Stage – I / Raj./2009)


(a) 26 (b) 20 (c) 11 (d) 10

52.

(NTSE Stage – I / Raj./2009)


(a) 64 (b) 100 (c) 81 (d) 144

53.

(NTSE Stage – I / Raj./2009)


(a) 14 (b) 13 (c) 12 (d) 11

2 3 32

1 7 71

9 1 ?
54. (NTSE Stage – I / Raj./2009)
(a) 23 (b) 103 (c) 91 (d) 19

23 48 25

46 92 ?

31 60 29
55. (NTSE Stage – I / Raj./2009)
(a) 54 (b) 4 (c) 46 (d) 138

56.
3 5 4

8 14 11

25 ? 34
(NTSE Stage – I / Raj./2009)
(a) 44 (b) 43 (c) 42 (d) 41

72 PACE IIT & MEDICAL: Mumbai / Delhi & NCR / Goa / Akola / Kolkata / Nashik / Pune / Bokaro / Dubai
NTSE-MENTAL ABILITY

57.
30 5 23

8 2 26

? 7 3
(NTSE Stage – I / Raj./2009)
(a) 33 (b) 38 (c) 25 (d) 27

58.
? 31 5

17 2 41

47 11 23
(NTSE Stage – I / Raj./2009)
(a) 59 (b) 53 (c) 43 (d) 37

59.

(NTSE Stage-II, 2009)


(a) 20 (b) 40 (c) 45 (d) 60

60.

(NTSE Stage-II, 2009)


(a) 4 (b) 5 (c) 6 (d) 8

61.

(NTSE Stage-II, 2009)


(a) 13 (b) 17 (c) 29 (d) 31

62.
12 8 10 7

14 16 13 15

9 11 ? 13
(NTSE Stage-II, 2009)
(a) 12 (b) 11 (c) 8 (d) 15

73 PACE IIT & MEDICAL: Mumbai / Delhi & NCR / Goa / Akola / Kolkata / Nashik / Pune / Bokaro / Dubai
NTSE-MENTAL ABILITY

63.
2 3 25

3 5 64

4 ? 144
(NTSE Stage-II, 2009)
(a) 6 (b) 7 (c) 8 (d) 9

64.

(NTSE Stage-II, 2009)


(a) 3 (b) 4 (c) 5 (d) 13

65.
2 4 9

6 4 25

8 4 ?
(NTSE Stage-II, 2009)
(a) 41 (b) 36 (c) 32 (d) 12

66.

(NTSE Stage-I/Raj./2012)
(a) 80 (b) 88 (c) 800 (d)

67.

(NTSE Stage-I/Raj./2012)
(a) 9 (b) 10 (c) 11 (d) 12

68.

(NTSE Stage-I/Raj./2012)
(a) 12 (b) 14 (c) 16 (d) 20

74 PACE IIT & MEDICAL: Mumbai / Delhi & NCR / Goa / Akola / Kolkata / Nashik / Pune / Bokaro / Dubai
NTSE-MENTAL ABILITY

69.

(NTSE Stage-I/Raj./2012)
(a) 270 (b) 196 (c) 256 (d) 320

70.

(NTSE Stage-I/Raj./2012)
(a) 3 (b) 4 (c) 5 (d) 6

71.
4 9 20

8 5 14

10 3 ?
(NTSE Stage-I/Raj./2012)
(a) 8 (b) 11 (c) 14 (d) 15

72.
? 1 1

9 4 4

2 3 5
(NTSE Stage-I/Raj./2012)
(a) 8 (b) 10 (c) 14 (d) 16

73.
8 10 9

? 15 28

7 12 13
(NTSE Stage-I/Raj./2012)
(a) 9 (b) 10 (c) 11 (d) 12

74.
36 43 49

55 ? 9

17 30 169
(NTSE Stage-I/Raj./2012)
(a) 49 (b) 58 (c) 76 (d) 77
75 PACE IIT & MEDICAL: Mumbai / Delhi & NCR / Goa / Akola / Kolkata / Nashik / Pune / Bokaro / Dubai
NTSE-MENTAL ABILITY

75.
5 8 7

11 17 15

21 33 ?
(NTSE Stage-I/Raj./2012)
(a) 29 (b) 31 (c) 33 (d) 38

76.

(NTSE Stage-I/Raj./2013)
(a) 144 (b) 136 (c) 135 (d) 124

77.

(NTSE Stage-I/Raj./2013)
(a) 102 (b) 152 (c) 162 (d) 172

78.

(NTSE Stage-I/Raj./2013)
(a) 91 (b) 108 (c) 116 (d) 119

79.

(NTSE Stage-I/Raj./2013)
(a) 10.25 (b) 10.50 (c) 11.25 (d) 11.50

80.
2 72 56

? 0 42

12 20 30
(NTSE Stage-I/Raj./2013)
(a) 4 (b) 6 (c) 8 (d) 10

76 PACE IIT & MEDICAL: Mumbai / Delhi & NCR / Goa / Akola / Kolkata / Nashik / Pune / Bokaro / Dubai
NTSE-MENTAL ABILITY

81.
91 64 73

84 76 61

25 60 ?
(NTSE Stage-I/Raj./2013)
(a) 66 (b) 68 (c) 69 (d) 71
82.
7 32 ?

31 8 25

11 24 9
(NTSE Stage-I/Raj./2013)
(a) 50 (b) 48 (c) 47 (d) 51

83.
4 20 25

27 81 9

11 44 ?
(NTSE Stage-I/Raj./2013)
(a) 4 (b) 16 (c) 30 (d) 55

84. Fill in the missing number


-C 2B -3A

2A ? -B

-3C -A -2B
(NTSE Stage-II, 2013)
(a) –3C (b) –2C (c) 3C (d) 2B

85. Find the number in the position of “?”

(NTSE Stage-II, 2013)


(a) 41 (b) 45 (c) 50 (d) 52

77 PACE IIT & MEDICAL: Mumbai / Delhi & NCR / Goa / Akola / Kolkata / Nashik / Pune / Bokaro / Dubai
NTSE-MENTAL ABILITY

86. Identify the number in the position of ‘?’

(NTSE Stage-II, 2013)


(a) 2 (b) 3 (c) 5 (d) 6

87. Find the letter to be placed in place of ‘?’ in the figure given.

(NTSE Stage-II, 2013)


(a) M (b) N (c) Q (d) R

88. Identify the number corresponding to the ‘?’

(NTSE Stage-II, 2013)


(a) 3 (b) 5 (c) 7 (d) 8

Directions (89 to 91):In each of the following figures numbers are written according to some patterns
and one number is missing, shown by questions mark. Find the missing number that replaces the
question mark. (NTSE Stage-I/Haryana/2013)

89.

(a) 84 (b) 195 (c) 240 (d) 275

78 PACE IIT & MEDICAL: Mumbai / Delhi & NCR / Goa / Akola / Kolkata / Nashik / Pune / Bokaro / Dubai
NTSE-MENTAL ABILITY

90.

(NTSE Stage-I/Haryana/2015)
(a) 1 (b) 2 (c) 6 (d) 10

91.

(NTSE Stage-I/Haryana/2015)

(a) 06 (b) 09 (c) 12 (d) 18


92.
C E B D F

24 120 6 60 ?
(NTSE Stage-I/Haryana/2015)
(a) 210 (b) 310 (c) 410 (d) 510

93. Which letter replaces the question mark?

(NTSE Stage-II, 2015)


(a) L (b) N (c) P (d) R

94. From among the four alternatives given below, which number replaces the question mark?

(NTSE Stage-II, 2015)


(a) 9 (b) 10 (c) 18 (d) 23

79 PACE IIT & MEDICAL: Mumbai / Delhi & NCR / Goa / Akola / Kolkata / Nashik / Pune / Bokaro / Dubai
NTSE-MENTAL ABILITY

Answer Key

1. (b) 2. (c) 3. (b) 4. (b) 5. (c) 6. (b) 7. (b) 8. (b) 9. (c) 10. (b)
11. (a) 12. (c) 13. (b) 14. (b) 15. (b) 16. (a) 17. (c) 18. (b) 19. (b) 20. (a)

21. (b) 22. (b) 23. (a) 24. (d) 25. (b) 26. (c) 27. (a) 28. (b) 29. (b) 30. (d)

31. (b) 32. (a) 33. (a) 34. (c) 35. (d) 36. (c) 37. (b) 38. (d) 39. (a) 40. (b)

41. (c) 42. (c) 43. (b) 44. (c) 45. (d) 46. (b) 47. (c) 48. (b) 49. (c) 50. (d)

51. (d) 52. (b) 53. (a) 54. (d) 55. (c) 56. (b) 57. (b) 58. (a) 59. (b) 60. (c)

61. (b) 62. (a) 63. (c) 64. (d) 65. (b) 66. (c) 67. (a) 68. (c) 69. (b) 70. (d)

71. (b) 72. (d) 73. (c) 74. (b) 75. (a) 76. (a) 77. (c) 78. (b) 79. (c) 80. (b)
81. (a) 82. (c) 83. (b) 84. (c) 85. (a) 86. (c) 87. (a) 88. (a) 89. (b) 90. (b)
91. (c) 92. (a) 93. (b) 94. (b)

PREVIOUS YEAR QUESTIONS

Q.1 and 2-Directions: In the following figure numbers are written with a specific rule. Find the rule and
decide which alternative will be in place of a question mark.

1.

(a) 140 (b) 220


(c) 320 (d) 500

2.

(a) 19 (b) 23
(c) 31 (d) 25
[NTSE-MAHARASHTRA STAGE-1-2020]

80 PACE IIT & MEDICAL: Mumbai / Delhi & NCR / Goa / Akola / Kolkata / Nashik / Pune / Bokaro / Dubai
NTSE-MENTAL ABILITY

Q.3 to 5-Directions: There is a certain relationship between the numbers that are given in the following
figure. According to that relationship, which alternative will replace the question mark?

3.

(a) 75 (b) 240


(c) 360 (d) 400
4.

(a) 32 (b) 40
(c) 64 (d) 80
5.

(a) 114 (b) 83


(c) 72 (d) 60
[NTSE-MAHARASHTRA STAGE-1-2020]

Q 6 and 7 Directions: In the following questions the numbers outside the bracket are related to number
inside the bracket in a specific manner. From the given alternative find the right number which matches and
will replace the question mark..

6. 78 (20) 82
37 (12) 59
45 (?) 91
(a) 13 (b) 17
(c) 19 (d) 23
7. 95(53)87
152(82)58
76(?)174
(a) 46 (b) 93
(c) 89 (d) 78
[NTSE-MAHARASHTRA STAGE-1-2020]
81 PACE IIT & MEDICAL: Mumbai / Delhi & NCR / Goa / Akola / Kolkata / Nashik / Pune / Bokaro / Dubai
NTSE-MENTAL ABILITY

Q.8 and 9: Direction: Identify the rule in the following arrangement of numbers. Choose the correct
alternative that will replace the question mark.

8.

(a) 185 (b) 68


(c) 78 (d) 93

9.

(a) 54 (b) 73
(c) 92 (d) 108
[NTSE-MAHARASHTRA STAGE-1-2019]

Q.10 to 12: Direction: There is a specific rule in the following arrangement of numbers. Study that rule
carefully. According to the rule choose the correct alternative for the questions that follow.

10.

(a) 30 (b) 32
(c) 34 (d) 52

11.

(a) 57 (b) 84
(c) 98 (d) 121

82 PACE IIT & MEDICAL: Mumbai / Delhi & NCR / Goa / Akola / Kolkata / Nashik / Pune / Bokaro / Dubai
NTSE-MENTAL ABILITY

12.

(a) 216 (b) 126


(c) 113 (d) 93

13. In the following question the numbers and letters in each horizontal line related to each other by a
specific rule. Identify the rule and choose the correct alternative to replace the question mark.
FJ 25 16 NS
LZ 25 196 SX
NQ ? ? WY
(a) 4.9 (b) 9.4
(c) 18, 169 (d) 31,256

14. Choose the correct alternative to replace the question mark

(a) F (b) T
(c) U (d) S
[NTSE-MAHARASHTRA STAGE-1-2019]

Q. 15 and 16 Direction: In the following question in every row the numbers outside the bracket and inside
the bracket are related to each other in a specific manner. From the given alternative choose the correct
alternative that will replace the question mark..

15. 17 (68) 28
11 (22) 14
49 (?) 9
(a) 56 (b) 105
(c) 147 (d) 63

16. 24 (7) 67
53 (6) 25
82 (?) 35
(a) 11 (b) 10
(c) 9 (d) 8
[NTSE-MAHARASHTRA STAGE-1-2019]

83 PACE IIT & MEDICAL: Mumbai / Delhi & NCR / Goa / Akola / Kolkata / Nashik / Pune / Bokaro / Dubai
NTSE-MENTAL ABILITY

Q.17 and 18 Direction: There is a specific relationship between the numbers that are given in the following
figures. On the basis of the relationship choose the correct alternative to replace the question mark

17.

(a) 210 (b) 266


(c) 288 (d) 318
18.

(a) 473 (b) 623


(c) 389 (d) 584
[NTSE-MAHARASHTRA STAGE-1-2018]

Q.19 and 21: Direction: Observe the arrangement of numbers that is given below. There is a specific rule in
that arrangement. Study that rule carefully and choose the correct alternative to replace the question mark.

19.

(a) 11 (b) 13
(c) 15 (d) 17

20.

(a) 125 (b) 90


(c) 105 (d) 225

21.

(a) 33 (b) 81
(c) 243 (d) 42
[NTSE-MAHARASHTRA STAGE-1-2018]
84 PACE IIT & MEDICAL: Mumbai / Delhi & NCR / Goa / Akola / Kolkata / Nashik / Pune / Bokaro / Dubai
NTSE-MENTAL ABILITY

22. In the following question there is a specific rule between the letter and the numbers in each
horizontal row. Identify the rule and choose the correct alternative to replace the question mark.

(a) 9, 29 (b) 18,210


(c) 179, 239 (d) 203, 181

23. Choose the correct alternative to replace the questions mark

(a) H (b) I
(c) J (d) K
[NTSE-MAHARASHTRA STAGE-1-2018]

Q.24 and 25 Direction: In the following questions in every row the numbers outside the bracket are related
to the number inside the bracket in a specific manner. From the given alternatives choose the number that
will replace the question mark.
24. 37 (46) 60
121(74)158
318 (?) 269
(a) 184 (b) 248
(c) 98 (d) 79

25. 35 (36) 73
64 (38) 27
43 (?) 58
(a) 39 (b) 52
(c) 101 (d) 119
[NTSE-MAHARASHTRA STAGE-1-2018]

26. Find out the correct number from the given alternatives to replace the question mark.

(a) 132 (b) 262


(c) 274 (d) 320
[NTSE-MAHARASHTRA STAGE-1-2017]

85 PACE IIT & MEDICAL: Mumbai / Delhi & NCR / Goa / Akola / Kolkata / Nashik / Pune / Bokaro / Dubai
NTSE-MENTAL ABILITY

Q.27 and 28: Directions: In the following square numbers are written with a specific rule. Find the rule and
decide which alternative will be in place of question mark.

27.

(a) 49 (b) 58
(c) 71 (d) 102
28.

(a) -1 (b) 12
(c) 41 (d) 63 [2017]

29. In the following question in every row the numbers outside the bracket are related to number inside
the bracket in a specific manner. From the given alternatives find the right number which matches
and will replace the question mark.
64 (87) 343 49(76) 216 36(?) 512
(a) 68 (b) 59
(c) 52 (d) 48 [2017]

Q.30 and 17 Directions: - Find the correct alternative which will replace the question mark.
30.

(a) 24 (b) 30
(c) 36 (d) 40

31.

(a) 25 (b) 37
(c) 98 (d) 129 [2016]

86 PACE IIT & MEDICAL: Mumbai / Delhi & NCR / Goa / Akola / Kolkata / Nashik / Pune / Bokaro / Dubai
NTSE-MENTAL ABILITY

32. In the following question, there is a specific relationship between the numbers inside the bracket and
outside the bracket in each row. Find the relation and select the correct alternative to replace the
question mark.
3 (45) 6
7 (70) 4
5 (?) 8
(a) 84 (b) 78
(c) 94 (d) 100 [2016]

33. Direction – In the following example one number in each row is in bracket. Numbers outside the
bracket have some specific relation with the number inside the bracket. Find the relation and replace
the question mark from the alternatives given below:
37 (33) 43
61 (24) 29
57 (?) 13
(a) 17 (b) 19
(c) 35 (d) 38

34. Direction – In the following table, replace the question mark by choosing the correct alternative
given below

(a) 48 (b) 57
(c) 67 (d) 83 [2015]

Q.35 and 36: Directions: Replace the question mark by choosing the correct alternative from given below:

(a) 42 (b) 56
(c) 64 (d) 72

36.

(a) 61 (b) 63
(c) 66 (d) 68

87 PACE IIT & MEDICAL: Mumbai / Delhi & NCR / Goa / Akola / Kolkata / Nashik / Pune / Bokaro / Dubai
NTSE-MENTAL ABILITY

Q. 37 and 38 : Directions:- Find the correct alternative which will replace the question mark (?)

37.

(a) 12 (b) 14
(c) 15 (d) 16

38.

(a) 89 (b) 97
(c) 132 (d) 178 [2014]

39. Directions - The number given in the bracket has specific relationship with the numbers outside the
bracket. Find the relationship and replace the question mark from the alternatives given below.
45 (13) 72
32 (11) 56
63 (?) 28
(a) 10 (b) 12
(c) 13 (d) 14

40. Directions – In the following table replace the question mark (?) from the alternatives given below:

7 2 53
3 6 45
5 4 ?
(a) 39 (b) 41
(c) 51 (d) 63 [2014]

Q.41 and 42  Directions: - Find the correct alternative which will replace the question mark.

41.

(a) 262 (b) 622


(c) 631 (d) 824

88 PACE IIT & MEDICAL: Mumbai / Delhi & NCR / Goa / Akola / Kolkata / Nashik / Pune / Bokaro / Dubai
NTSE-MENTAL ABILITY

42.

(a) 5 (b) 19
(c) 27 (d) 89 [2013]

43. Which number replaces the question mark?

(a) 146 (b) 113


(c) 74 (d) 92 [2012]

44. In the following question in every row the numbers outside the bracket are related to number inside
the bracket in a specific manner. From the given alternatives find the right number which matches
and will replace the question mark.
52 (47) 91
26 (26) 78
65 (?) 39
(a) 45 (b) 82
(c) 53 (d) 34 [2012]

45. Choose from the alternatives which number will replace the question mark?

(a) 5 (b) 6
(c) 7 (d) 8

Q.46 and 47  Directions:- In the following square number are written with a specific rule. Find the rule
and decide which alternative will be in place of question mark?

46.
102 30 31
199 ? 43
60 162 19

(a) 67 (b) 71
(c) 75 (d) 79

89 PACE IIT & MEDICAL: Mumbai / Delhi & NCR / Goa / Akola / Kolkata / Nashik / Pune / Bokaro / Dubai
NTSE-MENTAL ABILITY

47.
32 13 37
11 71 ?
58 17 25
(a) 39 (b) 43
(c) 47 (d) 51 [2012]

Answer Key

1. (d) 2. (c) 3. (b) 4. (c) 5. (c) 6. (b) 7. (a) 8. (b) 9. (d) 10. (c)
11. (d) 12. (b) 13. (a) 14. (c) 15. (d) 16 (c) 17. (c) 18. (d) 19. (a) 20.(d)
22.
21. (b) 23. (a) 24. (c) 25. (b) 26.(b) 27. (b) 28. (c) 29. (a) 30.(b)
(Bonus)
31. (a) 32. (d) 33.(d) 34. (c) 35. (d) 36. (c) 37.(c) 38.(d) 39. (c) 40. (b)
41.(b) 42. (d) 43. (c) 44. (c) 45. (c) 46.(d) 47.(c)

90 PACE IIT & MEDICAL: Mumbai / Delhi & NCR / Goa / Akola / Kolkata / Nashik / Pune / Bokaro / Dubai
NTSE-MENTAL ABILITY

7. MATHEMATICAL OPERATIONS

This section deals with questions on simple mathematical operation. There are four fundamental operations,
namely : Addition i.e. + ; Subtraction i.e. -; Multiplication i.e., x; and Division i.e., ÷.
There are also statements such as less than i.e. < ; greater than i.e. > ; equal to i.e. = ; and not equal to i.e. 
etc.
Such operations are represented by symbols different from the usual ones. The candidate has to make a
substitution of real signs and solve the equation accordingly.
While attempting to solve a mathematical expression, proceed according to the rule BODMAS - that is,
Brackets, Of, Division, Multiplication, Addition & Subtraction. We can perform addition or subtraction in
any order.

Application of BODMAS

1. What is the value of (48  12) ÷ 4 + 6÷2 3  ?


(A) 10 (B) 0.6 (C) 2 (D) 18

Sol. (D)
Given expression
  48  12   4  6  2  3
 36  4  6  2  3 (Solving Bracket)
 9  3 3 (Solving Division)
 99 (Solving Multiplication)
 18 (Solving Addition)

2. What is the value of 1260  15  7  ?


(A) 12 (B) 58 (C) 122 (D) 588

Sol. (A) Given expression  1260  7  84  7  12


15

Problems - Solving by Substitution


In these type of questions, you are provided with substitutes for various mathematical symbols or numerals
followed by a questions involving calculations of an expression or choosing the correct/incorrect equations.
The candidate is required to put in the real signs or numerals in the given equation and then solve the
questions required.

3. If ‘+’ mean, ‘x’, ' ' mean ' ' , ' ' means ' ' and ' x ' means ' ' , then what will be the value of
16  64  4  4  3  ?
(A) 20 (B) 52 (C) 12 (D) None of these

Sol. (A)
Using the correct symbols,
16  64  4  4  3  16  16  12  20

4. If L denotes x,M denotes  , P denotes + and Q denotes  , then 8P36M6Q 6 M 2L3  ?


(A) 13 (B)  1 (C) 14 1 (D) 5
6 6 2

91 PACE IIT & MEDICAL: Mumbai / Delhi & NCR / Goa / Akola / Kolkata / Nashik / Pune / Bokaro / Dubai
NTSE-MENTAL ABILITY

Sol. (D) Using the correct symbols


Given expression
 8  36  6  6  2  3
 8  6  3 3  5

5. It being given that : > denotes +, < denotes –.+ denotes ÷, – denotes , = denotes less than’ and x
denotes ‘greater than’, find which of the following is a correct statement.
(A) 3 + 2 > 4 = 9 + 3 < 1 (B) 3 > 2 > 4 = 18 + 3 < 2
(C) 3 > 2 < 4 x 8 + 4 < 2 (D) 3 + 2 < 4 x 9 + 3 < 3

Sol. (C)
Using proper notations, we have :
(A) given statement is 3 ÷ 2 + 4 < 9 ÷ 3—1 or 11  2 , which is not true.
2
(B) given statement is 3 + 2 + 4 < 18 ÷ 3—2 or 9 < 4, which is not true.
(C) given statement is 3 + 2 — 4 > 8 ÷ 4 — 2 or 1 > 0, which is true.
(D) given statement is 3 ÷ 2 — 4 > 9 ÷ 3 — 3 or  5  0 , which is not true.
2

Interchange of signs and numbers


6. Find out to sign to be interchanged for making the given equation correct
10  10  10  10  10  10
(A)  and  (B) + and x (C) ÷ and x (D) + and ÷

Sol. (B) By making the interchanges given in (A), the equation as 10  10 ÷10 + 10 10 = 10
or 109 = 10 which is false
By making the interchanges given in (B), the equation as 10 10 10 10  10  10 or 10  10
which is true
By making the interchanges given in (C), the equation as 10  10 10  10 10  10
or 109 = 10 which is false
By making the interchanges given in (D), the equation as 10 10  10 10 10  10
Or 89  10 which is false

7. Which one of the four interchanges in signs and numbers would make the given equation correct?
35 2  4
(A) + and  , 2 and 3 (B) + and  , 2 and 5

(C) + and , 3 and 5 (D) None of these

Sol. (C)
By making the interchange given in (A), we get the equation as 2  5 + 3 = 4 or O = 4, which is false.
By making the interchange given in (B), we get the equation as 3  2 + 5 = 4 or 6 = 4, which is false.
By making the interchange given in (C), we get the equation as 5  3 + 2 = 4 or 4 = 4, which is true.

Deriving the appropriate conclusion


In these type of questions, certain relations between different sets of elements is given (in terms of ‘less
than’, ‘greater than’ or ‘equal to’) using either the real symbols or substituted symbols. To analyses the
given statements and then decide which of the relations given as alternatives follows from those given in the
statements.

92 PACE IIT & MEDICAL: Mumbai / Delhi & NCR / Goa / Akola / Kolkata / Nashik / Pune / Bokaro / Dubai
NTSE-MENTAL ABILITY

Directions : (8 to 9) In the following questions :


 means ‘is equal to’
 means ‘is not equal to’
+ means ‘is greater than’
 means ‘is less than’
 means ‘is not greater than’
+ means ‘is not less than’
Now select the correct alternative in each of the following questions :

8. a  b  c implies
(A) a  b + c (B) b + a  c (C) c  b + a (D) b + a + c

Sol. (B) With the notations given,


we have : a  b  c means a  b  c
From option (A), a  b + c means a < b > c, this is not true. From option (B), b + a  c means b > a
<c, this is true. From option (C), c  b + a means c  b > a, this is not true. From option (D), b + a
÷c means b > a  c, this is not true.

9. a + b + c does not imply 


(A) b  a + c (B) c  b  a (C) c  a + b (D) b  a  c

Sol. (D) With the notations given,


we have: a + b + c means a > b > c, this is true
From option (A), b  a + c means b < a > c, this is true
From option (B), c  b  a means c < b < a, this is true.
From option (C), c  a + b means c < a > b, this is true.
From option (D), b  a  c means b < a < c, this is not true.

10. Which of the following conclusions is correct according to the given expressions and symbols?
A :  B: C :  D : E :  F : 
Expression (pEq) and (qEr)
(A) pEr (B) pFr (C) rBp (D) rBq

Sol. (A)
pEq and qEr  p  q and q  r  p  r  p Er

11. If A+ D > C + E, C + D = 2B and B + E > C + D, it necessarily follows that


(A) A + B > 2D (B) B + D > C + E (C) A+ D > B + E (D) A+ D > B + C

Sol. (D)
AD  CE
 A  D   2B  D   E  C  D  2B 
 A  D  B  E  B  D
 A  D  C  D  B  D
 A D  BC

Direction : (12) In answering the questions below, use the following information :
X  Y means divide X by Y
X  Y means multiply X by Y
93 PACE IIT & MEDICAL: Mumbai / Delhi & NCR / Goa / Akola / Kolkata / Nashik / Pune / Bokaro / Dubai
NTSE-MENTAL ABILITY

X  Y means subtract Y from X


X  Y means add Y to X

12. One-fifth of one-tenth of two-third of a number X is given by


(A) X  1  5   1  10    2  3

  
(B) X 1  5 1  10  2  3
(C) X 1  51  10  2  3
(D) can’t be determined

Sol. (A)
1 1 2
X     X  1  5  1  10    2  3
5 10 3
Directions : (13 to 16) In the following questions, the symbols, ©, ©, =, * and * are used with the following
meanings
‘A © B’ means ‘A is greater than B’ ;
‘A © B’ means ‘A is greater than or equal to B’;
‘A = B’ means ‘A is equal to B’:
‘A * B’ means ‘A is smaller than B’:
‘A * B’ means ‘A s either smaller than or equal to B’;
Now in each of the following questions, assuming the given statements to be true, find which of the two
conclusions I and II given below them is/are definitely true.
Give answer (A) if only conclusion I is true; (B) if only conclusion ii is true; (C) if either I or II is true;
(D) if neither I nor II is true.

13. Statements: S©T, M * K,T=K


Conclusions: I. T© M
II. T= M

Sol. (C) Given statements : S > T, M  K, T = K.


Relation between T and M :
T = K,K  M  T  M  T > M or T = M
 T © M or T = M
So, either I or II is true.

14. Statements: S * M, M © L, L © P
Conclusions: I. S  P
II. S* L
Sol. (D) Given statements: < , > , ≥
I. Relation between S and P:
S  M,M  L, L  P  no definite conclusion So, I is not true.
II. Relation between S and L:
S  M, M  L  no definite conclusion
So, II is also not true.

15. Statements: U  V, V* N, R * U
Conclusion: I. R*N
II. U © N
94 PACE IIT & MEDICAL: Mumbai / Delhi & NCR / Goa / Akola / Kolkata / Nashik / Pune / Bokaro / Dubai
NTSE-MENTAL ABILITY

Sol. (A) Given statements: U  V, V  N, R  U


I. Relation between R and N:
R  U, U  V,V  N  R  N i.e.R * N
So, I is true
II. Relation between U and N:
U  V, V  N  U  N i.e. U * N
So, U © N i.e. U  N is not true
Thus, II is false

16. Statements: E © U, C* E, C © B
Conclusion: I. U  C
II. E© B
Sol. (B) Given statements: E  U,C  E,C  B
I. Relation between U and C:
U  E,E  C  no definite conclusion.
So, I is not true.
II. Relation between E and B:
E  C,C  B  E  B i.e. E©B
So, II is true

17. What is the value of A if each letter represents a different digit ?


A 3B
B
__________

217B
(A) 3 (B) 4 (C) 5 (D) 7
Sol. (B)
In the following question the value of A and B are 4 and 5 respectively.
435
5
________________

2175

Directions: (18 to 20) Read the following statement. Seven digits are chosen from among 0, 1,……9 and
each is represented by a different letter in the subtraction problem below :
SLIDE
–DEAN
3 6 5 1

18. What digit does S represent?


(A) 0 (B) 1 (C) 2 (D) 3

19. What digit does N represent?


(A) 0 (B) 1 (C) 2 (D) 3

20. What digit does E represent?


(A) 0 (B) 1 (C) 2 (D) 3

95 PACE IIT & MEDICAL: Mumbai / Delhi & NCR / Goa / Akola / Kolkata / Nashik / Pune / Bokaro / Dubai
NTSE-MENTAL ABILITY

Sol. (18 to 20):


S has to 1. Then D (under L) must be greater than6. D = 7, L= 0, A = 2 or. D =8, L= 1, A= 3. or,
D = 9, L = 2 or 3, A= 3 or 4.
If L was carried over from D to E than N= 9 and, D (Over A) would be 6 more than A, But this
cannot be true (from the above inference about D, L, A), hence E is not 0. E is 1 more than N.
E 1 E  2  N 1
 E  2, E  3  N  2
18. (B) S  1

19. (C) N  2

20. (D) E  3

21. If 4 # 2 @ 3 = 6 l8 # 6 @ 4 = l2, then what will be the value to 24 # 3 @ 7?


(A) 21 (B) 27 (C) 72 (D) 56

Sol. (D) Here sign # implies division and sign @ implies multiplication 4  2  3  6,18  6  4  12
Therefore, 24 ÷ 3 x 7 = 56.

22. In the following question some numbers are given in the shape of figures

What is the value of ?


(A) 9 (B) 6 (C) 3 (D) 2
Sol. (A)
Clearly, the numbers are given in the shape of figures are as follow

So, 10  5  2,10  2  5
5  2  7.2  9  18 Hence the value of is 9.

96 PACE IIT & MEDICAL: Mumbai / Delhi & NCR / Goa / Akola / Kolkata / Nashik / Pune / Bokaro / Dubai
NTSE-MENTAL ABILITY

EXERCISE

1. The value of 1001  11 of 13 is:


(a) 7 (b) 91 (c) 143 (d) 169

6  6  6  6  6
2. The value of is equal to
44444
3 4 6
(a) 1 (b) (c) (d) 3
2 13 13

3. What mathematical operation should come at the place of ‘?’ in the equation:
2 ? 6 – 12  4 + 2 = 11
(a) + (b) – (c)  (d) 

4. If  means  , – means  ,  means + and + means –, then (3 – 15  19)  8 + 6 = ?


(a) 8 (b) 4 (c) 2 (d) – 2

5. If a means ‘plus’, b means ‘minus’, c means ‘multiplied by’ and d means ‘divided by’ then 16c 12b
6d 2a 17 = ?
(a) 65 (b) 55 (c) 216 (d) 206

6. If L denoted  , M denotes  , P denotes + and Q denotes – , then which of the following statements
is true?
3 173
(a) 32 P8 L 16 Q 4 =  (b) 6 M 18 Q 26 L 13 P 7 =
2 13
38
(c) 11 M 34 L 17 Q 8 L3 = (d) 9 P 9 L 9 Q 9 M 9 = –71
3
Directions: (7 to 9) Answer the questions of the basis of the information given below. If ‘$’ represents ‘++,

‘,’ represents ‘–‘, ‘#’ represents ‘  ’ and ‘@’ represents ‘l’ then answer the following questions based

on the above given representation.

7. What is the value of 4 # 3 $ 10 @ 5 $ 8 # 2, 18 ?


(a) 10 (b) 12 (c) 6.8 (d) 11.2

8. Which of the following has the value equivalent of


5$6#2$8@4?
(a) 4 # 7, 12 $ 2 # 1 (b) 8 # 2, 3 $ 6 @ 3 (c) 8 @ 2, 3 $ 6 # 3 (d) 4 $ 7, 12 $ 2 # 1

9. Which of the given value sis greater than


7 $ 3, 2 $ 12 @ 4 ?
(a) 4 # 3 $ 6 @ 3, 4 (b) 5 # 2, 8 @ 4 $ 3 # 3, 7
(c) 6 # 3, 18 @ 2 $ 1 # 2 (d) 9 @ 3 $ 6 # 2, 2 # 1

10. Find out the sign to be interchanged for making the given equation correct:
5 – 9  45 + 15  3 = 5
(a) + and – (b)  and + (c)  and  (d)  and –

97 PACE IIT & MEDICAL: Mumbai / Delhi & NCR / Goa / Akola / Kolkata / Nashik / Pune / Bokaro / Dubai
NTSE-MENTAL ABILITY

11. Which one of the four interchanges in signs and numbers would make the given equation correct?
6  4 + 2 = 16
(a) + and  , 2 and 4 (b) + and  , 2 and 6 (c) + and  , 4 and 6 (d) none of these

Directions: (12 to 13) for the following questions


 means “bigger than”
 means “smaller than”
O means “equal to”
 means “addition”
= means “subtraction”

12. If a  c and b  d O c, then


(a) d  a (b) a O a (c) b  c (d) d  a

13. If a = b  d = c and a O c, then


(a) d  b (b) d  b (c) b O d (d) a  d

Directions: 14 to 15) some symbols are given below. These symbols denote some relationships between
number.
 = greater than
 = equal to
 = not less than
 = less than
+ = not greater than
 = not equal to

14. a  b  c does not mean:


(a) c  b + a (b) b  c  a (c) c  b  a (d) a  b  c

15. b  c  a means
(a) a  b  c (b) c  b  a (c) c  b  a (d) b a  c
Directions: (16 to 17) The following symbols have been used
 stands for equal to
< stands for not equal to
– stands for greater than
+ stands for not greater than
> stands for less than
= stands for not less than

16. If p = q + r, then it is possible that


(a) p  q – r (b) p + q – r (c) p – q – r (d) p < q < r

17. If p > q  r, then it is possible that


(a) p + q + r (b) p = r – q (c) p  q + r (d) p = q – r

Directions : (18 to 22) In the following questions some letters stands for arithmetic sign as indicated below.
The remaining letters have their serial numbers in the Alphabets. Decode the letters into number and sign to
decide correct alternative (NTSE stage-I /Raj/2007)
 
A = , E = –, O = , U = +

98 PACE IIT & MEDICAL: Mumbai / Delhi & NCR / Goa / Akola / Kolkata / Nashik / Pune / Bokaro / Dubai
NTSE-MENTAL ABILITY

18. TEF
(a) 14 (b) 12 (c) 16 (d) 18

19. SETUH
(a) 5 (b) 6 (c) 7 (d) 8

20. GULAB
(a) 26 (b) 28 (c) 31 (d) 38

21. NACED
(a) 32 (b) 36 (c) 39 (d) 38

22. RUFOBEG
(a) 14 (b) 16 (c) 12 (d) 18

Directions : (23 to 27) In the following questions some numbers are given in the shape of figures. Finding
the values of the figures give the correct answer of the questions. (NTSE Stage-l /Raj/2008)

23.
(a) 0 (b) 3 (c) 4 (d) 6

24.
(a) 5 (b) 7 (c) 8 (d) 9

25.
(a) 0 (b) 3 (c) 5 (d) 6

26.
(a) 3 (b) 6 (c) 8 (d) 24

27.
(a) 3 (b) 4 (c) 5 (d) 7

28. Given that ACT ÷ AT = 11, find out which of the following numbers does not stand for CAT to
fulfill the above equation (NTSE Stage-II, 2011)
(a) 246 (b) 615 (c) 624 (d) 835

99 PACE IIT & MEDICAL: Mumbai / Delhi & NCR / Goa / Akola / Kolkata / Nashik / Pune / Bokaro / Dubai
NTSE-MENTAL ABILITY

29. Given the following subtraction problem. find out which of the following number does not stand for
CART. (NTSE Stage-Il, 2011)

(a) 6420 (b) 7531 (c) 8420 (d) 9753

30. If ‘–‘ means ‘multiplied by’, ‘  ’ means ‘plus’, ‘+‘ means ‘divided by’ and ‘  ‘ means ‘minus’ then
14 – 10  4  16 + 8 = ? (NTSE Stage-II, 2011)
(a) 142 (b) 134 (c) 6 (d) 5

31. If ‘+‘ means ‘multiplied by’, ‘–‘ means ‘divided by’ ‘  ’, means ‘plus’ and ‘  ’ means ‘minus’ then
(18 + 10  20) – 8 + 6 =? (NTSE Stage-Il, 2011)
(a) 92 (b) 35 (c) 19 (d) 26

32. If ‘+’ means ‘divided by’, ‘–‘ means ‘multiplied’, ‘  ’ means ‘plus’ and ‘÷‘ means ‘minus then
(280 +10  20) –8 ÷ 6 = ? (NTSE Stage-II, 2011)
(a) 378 (b) 258 (c) 70 (d) 64

Directions : (33 to 35) The following questions are based on the given matrix. The value of each letter is the
product of its row and column number e.g. the value of Z’ is 3  4 = 12.

Answer the following questions.


(NTSE Stage-II, 2011)
33. Find the letters which make the least total among the alternatives.
(a) DKA (b) FHY (c) ODX (d) VTM

34. What is the total of GREAT?


(a) 8 (b) 10 (c) 12 (d) 14

35. Find the letters which make the highest total among the alternatives.
(a) PLOT (b) PLAN (c) PLAY (d) PLUS

Directions : (36 to 39) In the following questions some relations are written by particular indicators as
shown below
 = (Greater than)
= (Not less than)
 = Not equal to)
= (Equal to)
+ = (Not greater than)
= (Less than)

100 PACE IIT & MEDICAL: Mumbai / Delhi & NCR / Goa / Akola / Kolkata / Nashik / Pune / Bokaro / Dubai
NTSE-MENTAL ABILITY

Find out the correct answer for each question.


(NTSE Stage-I/Raj./ 2012)
36. If x y  z it is not possible
(a) x  y z (b) x + y  z (c) x  y  z (d) x y z

37. If x y z it is not possible


(a) x y Z (b) x  y + z (c) x + y z (d) x + y z

38. If x y  z, it is possible
(a) x  y z (b) x y+Z (c) x y Z (d) x yz

39. If x  y z, it is not possible


(a) x  y + z (b) x  y  z (c) x yz (d) x + y + z

40. Find the correct inference according to given premises and symbols :
A : Not greater than B : Greater than
C : Not equal to D : Equal to
E : Not less than F : Less than
Premises : (pCm) and (pAm)
(A) pBm (B) pDm (C) pEm (D) pFm

Direction : (41 to 42) In the following question find the relationship that can definitely be deducted on the
basis two relationship given. The symbols used are as follows :
means greater than, A means less than, – means not equal to, means equal to

41. If 8A A 6B and 3B A 4C, therefore


(a) C A (b) C A (c) 2C +A (d) C 2A

42. If B D, D C, C A and B + A, therefore


(a) C D (b) D C (c) C – B (d) Can’t be determined

43. If A+ B > C + D, B + E = 2C and C + D > B + E, it necessarily follows that:


(a) A + B > 2C (b) + B > 2D (c) A + B > 2E (d) A > C

Direction : (44) In answering the questions below, use the following information :
X  Y means divide X by Y
X  Y means multiply X by Y
X # Y means subtract Y from X
X  Y means add Y to X

44. A receives X number of balls. He gives 10% of his ball to B, 15% of his ball to C and 12% of his ball
to D. How many balls does he have with him now?
(a) X  X  10  100   X  15  100  # X  12  100 
  
(b) X  X  10  100  X 15  100  X  1  100   
(c) X #  X  10 100   X  15 100   X  12 100  
(d) None of these
101 PACE IIT & MEDICAL: Mumbai / Delhi & NCR / Goa / Akola / Kolkata / Nashik / Pune / Bokaro / Dubai
NTSE-MENTAL ABILITY

Direction : (45 to 46) In the question given below, use the following notations :
A “B means ‘add B to A’;
A ‘B means ‘subtract B from A’;
A @ B means ‘divide A by B’;
A, B means ‘multiply A by B’;
Now, answer the following question.

45. The time taken by two running trains in crossing each other is calculated by dividing the sum of the
lengths of two trains by the total speed of the two trains. If the length of the first train is L1, the
length of the second train is L2, the speed of the first train isV1 and the speed of the second train is
V2, which of the following expressions would represent the time taken ?
(a) (L1 " L2), (V1 " V2) (b) (L1 " L2) @ (V1 " V2)
" "
(c) [(L1 L2) @ (V1 V2)], 60 (d) (L1 " L2) @ (V1 " V2)

46. The total airfare is calculated by adding 15% basic fare as fuel surcharge, 2% of basic fare as IATA
charges and Rs.200 as airport tax to the basic fare. If the basic fare of a section is B, which of the
following will represent the total fare?
(a) B ' (B, 15) @ 100 " (B, 2) @ 100 " 200
(b) B " (B, 15) @ 100 " (B, 2)@ 100 " 100
(c) B " (B, 15) @ 100 " (B, 2)@ 100 " 200
(d) B " (B, 15) @ 100 " (B, 2) @ 100 " 100

47. If then
(a) 690 (b) 689 (c) 780 (d) 789

48. Directions : (48 to 52) In the following questions the symbols $, @,  ,  and  are used with the
following meaning.
A $ B means A is greater than B
A @ B means A is either greater than or equal to B
A  B means is A is equal to B
A  B means A is smaller than B
A  B means A is either smaller than or equal to B Now is each of the following questions assuming
the given statements to be true, find which of the two conclusions I and Il given below them is/are
definitely true? Give answer (A) if only conclusions I is true, (B) if only conclusion II is true (C) if
neither I nor II is true (D) if both I and Il are true.

48. Statements: P @ Q, M  N, N  Q
Conclusions: I. P $ M
II. N  P

49. Statements: D  X, F @ Y, D $ F
Conclusions: I. X @ Y
II. Y  D

50. Statements: M  P, S $ T, M @ T
Conclusions: I. T  P
II. S  T

102 PACE IIT & MEDICAL: Mumbai / Delhi & NCR / Goa / Akola / Kolkata / Nashik / Pune / Bokaro / Dubai
NTSE-MENTAL ABILITY

51. Statements: U  V, X $ W, U  W
Conclusions: I. W $ V
II. U  X
52. Statements: G $ H, J  K, H  K
Conclusions: I. G $ K
II. J  K

Directions : (53 to 57) In the following questions the symbol @ @ , =, and are used with following
meaning:
P @ Q  P is greater than Q
P @ Q  P is either greater or equal to Q
P Q  P is smaller than Q
P Q  P is either smaller than or equal to Q
P = Q  P is equal to Q
Now in each of the following questions, assuming the given statement, to be true, find which of the
two conclusions I and II given below them is/are definitely true.
Give answer (A) if only conclusion I is true, give answer (B) it only conclusion Il is true, give
answer (C) if either I or II is true, give answer (D) if neither nor II is true, give answer (E) if both I
and II are true.

53. Statement : B @ V, K C, C B
Conclusions: I. V @ C
II. N @ K

54. Statement : K @ T, S = K, T R
Conclusions: I. S @ R
II. T = R

55. Statement : U = M, P @ U, M @ B
Conclusions: I. P = B
II. P @ B

56. Statement : L @ N, J P, P @ L
Conclusions: I. J = L
II. P = N

57. Statement : H @ G, D @ E, H = E
Conclusions: I. D @ H
II. G D

58. In the correctly worked out multiplication problem at the below, each letter represent a different
digit. What is the value of B?

(a) 1 (b) 2 (c) 4 (d) 5


103 PACE IIT & MEDICAL: Mumbai / Delhi & NCR / Goa / Akola / Kolkata / Nashik / Pune / Bokaro / Dubai
NTSE-MENTAL ABILITY

Directions: (59 to 61) In the following questions find out the digits corresponding to the letters representing
those digits in the multiplication give below.

59. b stand for :


(a) 6 (b) 7 (c) 8 (d) 9

60. c stands for:


(a) 7 (b) 6 (c) 5 (d) 4

61. d stands for:


(a) 2 (b) 3 (c) 4 (d) 5

62. What is the value of A if each letter represents a different digit?

(a) 3 (b) 4 (c) 5 (d) 7

63. Find the 4 digit number ABCD such that ABCD  9 = DCBA.
(a) 1089 (b) 9801 (c) both A and B (d) none of these

64.

I. Here  (dot) represents multiplication


II. Every letter has unique value from 0-10
III. O < Y
Then the value of ‘O’ is
(a) 3 (b) 4 (c) 8 (d) 2

Directions (65 to 70) Digit 7 does not occur in the following multiplication. Find the missing digits denoted
by different letters.

65. What is the sum of the digits of Multiplicand ?


(a) 11 (b) 12 (c) 9 (d) 10

66. What is the sum of the digits of Multiplier?


(a) 7 (b) 9 (c) 11 (d) 12

104 PACE IIT & MEDICAL: Mumbai / Delhi & NCR / Goa / Akola / Kolkata / Nashik / Pune / Bokaro / Dubai
NTSE-MENTAL ABILITY

67. What is the sum of the digits of Row No. 1?


(a)11 (b) 13 (c) 15 (d) 17

68. What is the sum of the digits of Row No. 2?


(a) 11 (b) 13 (c) 15 (d) 17

69. What is the sum of the digits of Row No, 3?


(a) 13 (b) 21 (c) 15 (d) 17

70. What is the sum of the digits of the product?


(a) 21 (b) 23 (c) 29 (d) 25

Directions : (71 to 73) In the following questions some relations are written by particular indicators as
shown below: (NTSE Stage-I /Raj./2013)
O = Greater than
+ = Equal to
 = Not equal to
= Not greater than
 = Not less than
 = Less than
Find out the correct answer for each question.
71. If p  q O r, it is possible that
(a) p  q  r (b) p  q  r (c) p  q  r (d) p  q  r

72. If p  q  r, it is not possible that


(a) p  q  r (b) p  q  r (c) p  q  r (d) p  q  r

73. If p  q  r , it is not possible that


(a) p  q  r (b) p  q  r (c) p  qOr (d) pO q  r

73. If 27 * 3 = 243 (NTSE Stage-I/Raj./2013)


5 * 4 = 80
Then what is the value of 3 * 7 ?
(a) 84 (b) 147 (c) 63 (d) 23

74. In this multiplication question the five letters represent five different digits. What are the actual
figures? There is no zero.

(A) M = 3, E = 9, A = 7, T = 4, S = 8 (B) M = 3, E = 9, A = 7, T = 8, S = 4
(C) M = 4, E = 3, A = 9, T = 7, S = 8 (D) M = 4, E = 9, A = 3, T = 7, S = 8

76. Which symbol replaces the ‘?’ Figure below represent a balance. (NTSE Stage-II/2013)

(a)  (b) O (c)  (d) O


105 PACE IIT & MEDICAL: Mumbai / Delhi & NCR / Goa / Akola / Kolkata / Nashik / Pune / Bokaro / Dubai
NTSE-MENTAL ABILITY

77. If 23*52 = 48, then 43*35 = ? (NTSE Stage-I/Raj./2014)


(a) 78 (b) 98 (c) 96 (d) 69

78. If 54/32 = 4, 36/42 = 3, 92/22 = 7 then what is 28/33 = ? (NTSE Stage-II/2015)


(a) 5 (b) 6 (c) 4 (d) 9

Answer Key

1. (a) 2. (c) 3. (c) 4. (c) 5. (d) 6. (d) 7. (b) 8. (c) 9. (d) 10. (d)
11. (c) 12. (d) 13. (a) 14. (d) 15. (d) 16. (d) 17. (a) 18. (a) 19. (c) 20. (c)

21. (d) 22. (a) 23. (c) 24. (d) 25. (a) 26. (c) 27. (d) 28. (a) 29. (c) 30. (a)

31. (c) 32. (a) 33. (c) 34. (c) 35. (d) 36. (a) 37. (d) 38. (c) 39. (b) 40. (d)

41. (a) 42. (a) 43. (a) 44. (c) 45. (b) 46. (c) 47. (b) 48. (b) 49. (c) 50. (a)

51. (c) 52. (a) 53. (b) 54. (d) 55. (c) 56. (d) 57. (e) 58. (b) 59. (c) 60. (a)

61. (c) 62. (b) 63. (a) 64. (d) 65. (a) 66. (a) 67. (b) 68. (b) 69. (c) 70. (b)

71. (a) 72. (d) 73. (c) 74. (b) 75. (b) 76. (c) 77. (c) 78. (c)

PREVIOUS YEAR QUESTIONS


1. If according to mathematical code language 8 ÷ 2 = 70, 9 ÷3 = 87, 10 ÷ 4 = 106 then 7 ÷5 = ?
(a) 65 (b) 58
(c) 51 (d) 63

2. If according to mathematical code 9 + 2 = 36, 8 + 3 = 72, 7 + 4 = 112 then 6 + 5 = ?


(a) 84 (b) 130
(c) 75 (d) 150
[NTSE-MAHARASHTRA STAGE-1-2020]
3. If in a mathematical code language
    9,    13,    11,and     12 , then find the value of  from the following
alternatives.
(a) 5 (b) 7
(c) 6 (d) 8

4. In a certain code language if S  rs  35, E S  30, rs  U  63and U  #  36 then find the value of #
(a) 6 (b) 4
(c) 5 (d) 9
[NTSE-MAHARASHTRA STAGE-1-2019]
5. In a mathematical code language
88 – 7 = 39, 77 – 6 = 41, 99 – 5 = 74, then 55 – 4= ?
(a) 31 (b) 39
(c) 49 (d) 34

106 PACE IIT & MEDICAL: Mumbai / Delhi & NCR / Goa / Akola / Kolkata / Nashik / Pune / Bokaro / Dubai
NTSE-MENTAL ABILITY

6. In a mathematical code language


8 + 6 = 42, 7 + 5= 30, 9 + 3 = 24, then 6 + 4=?
(a) 27 (b) 20
(c) 22 (d) 24
[NTSE-MAHARASHTRA STAGE-1-2019]

7. In a mathematical language of  means +,  means , means  and means  then find the
value of the following expression 3  518 3  23  ?
(a) -6 (b) 0
(c) 6 (d) -3

8. In a mathematical language if  means,  means ,  means  and  means  then find the value of
the following expression 4  8  2  5  7  ?
(a) 33 (b) 23
(c) 17 (d) -4
[NTSE-MAHARASHTRA STAGE-1-2018]
9. In a mathematical language if + means ÷, - , means x, ÷ means + and x means - then
 200  5   25  (20  5)  10  ?
(a) 125 (b) 100
(c) 155 (d) 40

10. In a certain code language if 3 + 3 = 9, 6 – 3 = 2, 4  3  1, 3  2  1, then find the value of


5  4  24  8 6 in the same code language?
(a) 9 (b) 11
(c) 17 (d) 29

11. In the following example, if the numbers 3 and 5 are interchanged and the numbers 7 and 9 are
interchanged, then find the value of the following expression:
93  5  7  3  25  ?
(a) 95 (b) 73
(c) 57 (d) 75

Answer Key

1. (c) 2. (d) 3. (a, d) 4. (b) 5. (b) 6. (b) 7. (c) 8. (c) 9. (c) 10. (b)
11. (d)

107 PACE IIT & MEDICAL: Mumbai / Delhi & NCR / Goa / Akola / Kolkata / Nashik / Pune / Bokaro / Dubai
NTSE-MENTAL ABILITY

8. DIRECTION SENSE TEST

There are four directions such as North, South, East and West. The word NEWS came from North, East,
West and South. There are four regions :
(i) North-East (ii) North-West (iii) South-East (iv) South-West

The directions OP, OS, QQ and OR are: North-East direction; North-West direction; South-West direction;
and South-East direction respectively.

NOTE : The candidate must distinguish between the regions and directions, i.e., between North-East region
and North-East direction. If you move with your face Eastwards, your left hand is towards North and your
right hand is towards South. Similarly the positions of the directions of the hands can be fixed when you
move in any of the other three directions.

1. Mohan travels 7 km Eastwards, then he turns right and travels 3 km and further turns right again and
travels 11 km. How far is he from the starting point?
(A) 5 (B) 14 (C) 21 (D) 23

Sol. (A)
The movement of Mohan from A to D as shown in figure. Now,
DE   CD  CE    CD  AB  11  7   4km
Required distance  AD  DE 2  AE 2
2 2
DE2  BC2   4    3  5km

2. A man walks 9 km due East and then 12 km due South. How far is from the starting point?
(A) 15 km (B) 6 km (C) 7 km (D) None of these

Sol. (A) The movement of man from A to C as shown in figure. Clearly, required distance
AC  AB2  BC2
 92  122  15km

108 PACE IIT & MEDICAL: Mumbai / Delhi & NCR / Goa / Akola / Kolkata / Nashik / Pune / Bokaro / Dubai
NTSE-MENTAL ABILITY

3. The time on the watch is quarter to three. If the minute-hand points to North-East, in which direction
does the hour hand point?
(A) South-West (B) South-East (C) North-West (D) North-East

Sol. (A) The required position of clock as shown in figure. Clearly, if minute hand points towards North-
East then hour hand will point towards South-West

4. If I stands in up side down position with my face pointing Northwards, in what direction will my
right-hand point ?
(A) East (B) West (C) North (D) South

Sol. When I stand on my head with my face pointing Northwards. Clearly, my left hand will point
towards East and my right hand will point towards West.

5. From his house, Rajan went 25 km to the North. Then he turned West and covered 20 km. Then
he turned South and covered 15 km. Finally, turning to East, he covered 20 km. In which direction
was he from his house ?
(A) East (B) West (C) North (D) South
Sol. (C)
The movements of Rajan are as shown in fig.(A to B,B to C, C to D and D to E) Clearly, his final
position is E which is to the North of his house at A.

6. Raj walked 20 metre towards South. Then he turned to his left and walked 25 metre. He then

turned to his left and walked 20 metre. He again turned to his right and walked 10 metre. At what
distance is he from the starting point and in which direction ?
(A) 35 metre, East (B) 35 metre, North (C) 40 metre, East (D) 60 metre, East

Sol. (A)
The movements of Raj are as shown in figure
 Raj’s distance from the starting point A

109 PACE IIT & MEDICAL: Mumbai / Delhi & NCR / Goa / Akola / Kolkata / Nashik / Pune / Bokaro / Dubai
NTSE-MENTAL ABILITY

=AE = (AD + DE)= (BC + DE)= (25 + 10)m = 35m.


So, E is to the East of A:

7. If South-East becomes North, North-East becomes West and so on, what will South become ?
(A) North-East (B) South – West (C) South (D) North-West

Sol. (A) The initial and the final position of the directions as shown in the figure. Clearly, as observed in
the figure, South will become North-East.

8. A man is facing North-West. He turns 90  in the clockwise direction and then 135° in the anti-
clockwise direction. Which direction is he facing now ?

(A) East (B) West (C) North (D) South


Sol. (B) As shown in the Fig. the man faces in the direction CA. After moving 900 clockwise, the man
faces in the direction OB. On moving to 135° anti-clockwise, he faces in the direction OC, which is
West.

9. A man is facing North-West. He turns 90° in the clockwise direction, then 180° in the anti-clockwise
direction and then another 90° in the same direction. Which direction is he facing now?
(A) South (B) South-West (C) West (D) South-East
Sol. (D) As shown in fig. the man initially faces in the direction CA. On moving 90° clockwise, he faces
in the direction OB. On further moving 180° anti-clockwise, he faces in the direction OC. Finally on
moving 90° anti-clockwise, he faces in the direction CD, which is South-East.

110 PACE IIT & MEDICAL: Mumbai / Delhi & NCR / Goa / Akola / Kolkata / Nashik / Pune / Bokaro / Dubai
NTSE-MENTAL ABILITY

10. While facing East you turn to your left and walk 10 m then turn to your left and walk 10 m, and now
you turn 45° towards your right and go straight to cover 25 m. Now, in which direction are you from
your starting point?
(A) North-East (B) South-West (C) South-East (D) North-West

Sol. (D) The movement from to D as shown in the figure. Clearly, the final position i.e. D is to North-
West of A.

11. I run along the sides of a square field ABCD where C is to the North-East of A and D is to the
South-East of B. Starting from A in anti-clockwise direction, in which direction shall I be running
after crossing C ?
(A) East (B) West (C) North (D) South

Sol. (B) The required position of the square field ABCD as shown in the figure. Starting from A in the
anti-clockwise direction. I will move from A to D, D to C, C to B and B to A. Clearly, I will be
running in the West direction after crossing C.

12. A and B start walking in opposite directions. A walked 5 km, B walked 6 km. Thereafter both turned
to their right and walked 2 km. They turned to right again and walked 3 km, again turned to right and
walked 2 km. How much distance apart are they from each other?
(A) 2km (B) 13 km (C) 3km (D) 5km

Sol. (D) A and B he facing North and South direction respectively at initial position O. The movement of
A from O to W (O to X, X to Y, Y to Z and Z to W)and the movement of B from O to R (O to P, P
to Q, Q to S, S to R) as shown in figure. Clearly, required distance = RW
 RO  OW   OP  PR    OX  XW 
  6  3   5  3  3  2  5km

111 PACE IIT & MEDICAL: Mumbai / Delhi & NCR / Goa / Akola / Kolkata / Nashik / Pune / Bokaro / Dubai
NTSE-MENTAL ABILITY

EXERCISE

1. Anil left home and cycled 10 km Southwards, turned right and cycled 5 km & turned right and
cycled 10 km and turned left and cycled 10 km. How many kilometer will he have to cycle to reach
his home straight ?
(A) 10 km (B) l5 km (C) 20 km (D) 25 km

2. ‘A’ travelled Westwards 5 km, turned left and travelled 3 km, turned right and travelled 9 km. He
then travelled North 3 km. How far was ‘A’ from the starting point now?
(A) 3 km (B) 5 km (C) 10 km (D) 14 km

3. Amar travels one km due East, then 5 km due South, then 2 km due East and finally 9 km due North.
How far is from the starting point?
(A) 16 km (8) 8 km (C) 6 km (D) 5 km

4. Amit walks 2 km South, turned right and walked 1 km, again turned North and walked 5 km, turned
East and walked 5 km. How far is he from the starting point?
(A) 3 km (B) 7km (C) 5 km (D) 6 km

5. A watch reads 4 : 30. If the minute-hand points to East, in which direction does the hour-hand point?
(A) North-East (B) South-East (C) North-West (D) North

6. A man walked 3 metre towards North, turned West and walked 2 metre then turned North and
walked 1 metre and finally turned East and walked 5 metre. How far is he from the starting point?
(A) 5 metre (B) 8 metre (C) 10 metre (D) 12 inetre

7. Amit faces towards North. Turning to his right he walks 25 metre. He then turns to his left and walks
30 metre. Then moves 25 metre to his right. He then turns to his right again and walks 55 metre.
Finally, he turns to the right and moves 40 metre. In which direction is he now from his starting
point?
(A) South-West (B) South (C) North-West (D) South-East

8. Kishen walks 10 km towards North. From there, he walks 6 km towards South. Then, he walks
3 km towards East. How far and in which direction is he with reference to his starting point?
(A) 5 km, North (B) 5 km, North-East
(C) 7 km, East (D) 7 km, West

9. A man was facing East. He took Three Paces forward, turned right, walked another two paces
and then turned right again, took three paces and turned about. Which direction was he last facing?
(A) East (B) North (C) South (D) None of these

10. I am facing South. I turn right and walk 20 m. Then I turn right again and walk 10 m. Then I turn left
and walk 10 m and then turning right walk 20 m. Then I turn right again and walk 60 m. In Which
direction am I from the starting point?
(A) North (B) North-West (C) East (D) North-East

11. Kurnar stands with his face pointing to the South-East direction. He walked 15 metre and then turned
Northwards and walked another 12 metre. How far was he then from the starting point ?
(A) 12 metre (B) 10 metre (C) 5 metre (D) None of these

112 PACE IIT & MEDICAL: Mumbai / Delhi & NCR / Goa / Akola / Kolkata / Nashik / Pune / Bokaro / Dubai
NTSE-MENTAL ABILITY

12. I went 15 m to the North, then turned West and covered 10 m, then turned South, and covered 5 m
then turned East and covered 10 m. In which direction am I now from my house?
(A) North (B) South (C) East (D) West

13. At my house I am facing West, then 1 turn left and go10 m, then I turn 90o anti-clockwise and go
5 m, and then I go 5 m to the South and from there 5 m to the West. In which direction am I from my
house?
(A) East (B) West (C) North (D) South

14. Sanjay went 70 metre in the East before turning to his right. He went 10 metre before turning to his
right again and went 10 metre from this point. From here he went 90 metre to the North. How far
was he from the starting point?
(A) 80 metre (B) 100 metre (C) 140 metre (D) 260 metre

15. A and B start walking from the same point. A goes North and covers 3 km: then turns right and
covers 4 km. B goes west and covers 5 km, then turns right and covers 3 km. How far apart are they
from each other?
(A) 10 km (B) 9 km (C) 8 km (D) 5 km

16. L is to South-West of K, M is to the East of L and South-East of K and N is to the North of M in line
with LK. In which direction of K is N located?
(A) North (B) East (C) South-East (D) North-East

17. Ramesh starts walking from his house at 4 PM facing towards sun and walks 5 km. Then he turns
left and walks 8 km. Again turning to left he walks 13 km and reaches Mohan’s house. In which
direction Ramesh’s house is from Mohan’s house? (NTSE Stage-I/Raj./2008)
(A) North-West (B) North-East (C) South-West (D) South-East

18. Vinod travelled 6 km South from the starting point D, then turned right and moved 4 km and again
turned right and travelled 6 km and turned left and travelled 8 km. Find out how many kilometer he
has to cover to reach his starting point D. (NTSE Stage-II, 2008)
(A) 10 km (B) 12 km. (C) 14 km (D) 16 km

19. Suresh moves a distance of 7 km form a place P towards North, then turns left arid walks 4 km,
again turns towards right and walks 3 km, then again turns right and walks 2 km to reach his
destination Q. Which direction is he facing now? (NTSE Stage-II, 2008)
(A)West (B) East (C) North-West (D) South

20. A child goes 50 metre towards South and then turning to his right, he goes 50 metre. Then, turning to
his left, he goes 30 metre. Again he turns to his left and goes 50 metre. How far is he from his initial
position ? (NTSE Stage-II, 2008)
(A) 30 metre (B) 40 metre (C) 50 metre (D) 80 metre

21. Ganesh moves 20 metres towards East from his house. Then he turns left 3 times each time covering
a distance of 20 meters. Finally he takes 2 successive right turns, each time covering a distance of 20
metres. In which direction is he with respect to his house? (NTSE Stage-II, 2009)
(A) North (B) North-East (C) South-West (D) North-West

22. Ashok is facing North. He turns 45 degrees in the clockwise direction and then turns 90 degrees in
the anticlockwise direction. Finally, he turns back. Which direction is he facing now?
(NTSE Stage-II, 2009
(A) South-East (B) South-West (C) North-East (D) North-West
113 PACE IIT & MEDICAL: Mumbai / Delhi & NCR / Goa / Akola / Kolkata / Nashik / Pune / Bokaro / Dubai
NTSE-MENTAL ABILITY

23. Shabnam’s school bus picks her up from her house and takes two left turns and one right turn to
reach her school. If the bus is facing East, white reaching the school, which direction was the bus
facing at her home? (NTSE Stage-II, 2009)
(A) North (B) South (C) East (D) West

24. One morning Ram and Shyam were talking to each other face to face. If Shyam’s shadow was
exactly to the right of Ram, which direction Shyam was facing ? (NTSE Stage-II, 2009)
(A) South (B) East (C) West (D) North

Directions : (25 to 29) Study the maze given below and answer the questions that follow.
(NTSE Stage-II,2009)

25. How many ‘turns will ‘X’ take to come out of the maze ?
(A) 36 (B) 38 (C) 40 (D) 42

26. How many turns will ‘V’ take to come out of the maze?
(A) 10 (B) 8 (C) 7 (D) 6

27. How many times ‘X’ has to go towards South, if he wants to meet ‘Z’?
(A) 5 (B) 6 (C) 8 (D) 10

28. How many times ‘W’ has to go towards East, if he wants to meet ‘V’?
(A) 5 (B) 6 (C) 7 (D) 8

29. How many turns will ‘Y’ take to meet ‘X’?


(A) 42 (B) 40 (C) 39 (D) 37

30. Prakash moves 40 km in the direction of North then he turns to right and moves 50 km. After this
he turns to right and moves 30 km. Again he turns to right and moves 50 km.
How many kilometers away is he from the starting point? (NTSE Stage-I, 2009)
(A) 40 (B) 10 (C) 50 (D) 60

31. Shalini is standing at the South-East comer of a rectangular field. She starts crossing the field
diagonally. After walking half the distance, she turns right, walks some distance and turns left.
Which direction is Shalini facing now? (NTSE Stage-II, 2011)
(A) South - East (B) South- West (C) North- East (D) North-West

114 PACE IIT & MEDICAL: Mumbai / Delhi & NCR / Goa / Akola / Kolkata / Nashik / Pune / Bokaro / Dubai
NTSE-MENTAL ABILITY

32. One morning after sunrise, Seema was standing facing a pole. The shadow of the pole fell exactly
to her right. Which direction was she facing ? (NTSE Stage-II, 2011)
(A) North (B) South (C) West (D) East

33. Rahul travels 10 km to the North. He turns to the right and walks 5 km. Then again he turns to his
right and moves 10 km forward. How many km away from starting point is he?
(NTSE Stage-II,2011)
(A) 26 km (B) 19 km (C) 13 km (D) 5 km

34. Ram starts from a point P, drives 2 km towards North. He then turns to his left and drives 3 km and
after taking another turn to his left the drives 2 km, and finishes at point Q. After the first turn in
which direction Ram will be driving ? (NTSE Stage-II, 2011)
(A) West (B) North (C) East (D) South

35. Rakesh starts from A and walks towards East to B. He turns south and walks to C. Then he turns
north-west and walks to D. Finally he turns south west and comes to E. Which of the answer figures
shows the exact path he traced? (NTSE Stage-II, 2011)

36. Pankaj starts from A and walks northeast to B. He turns west and walks to C. Then turns south and
walks to D. He then turns east and Walks to E. Finally he turns south-west and Walks to F.
Which of the answer figures exactly shows the path Pankaj traced ? (NTSE Stage-II,2011)

115 PACE IIT & MEDICAL: Mumbai / Delhi & NCR / Goa / Akola / Kolkata / Nashik / Pune / Bokaro / Dubai
NTSE-MENTAL ABILITY

37. A sprinter goes off the starting block for 100m run and at that instant the second-hand of a stop
watch had pointed towards North. He touches the finishing line exactly after 12 seconds. In Which
direction did the second hand point when he just crossed the finishing line ?
(NTSE Stage-II, 2013)
(A) 18o North of East (B) 18° East of North
(C) 72° North of East (D) 82° East of North

38. I left home at 3:00pm and returned at 3:48pm. The clock was rotated by 45 o, so that when I left, the
hour-hand of a clock was pointing along the south-east direction. In which direction would the hour
hand point when I returned? (NTSE Stage-II, 2013)
(A) 15° East of South (B) 21° East of South
(C) 63° South of East (D) 27° South of East

39. Madhu walks 15 metres towards north, then she turns left at 90o and walk 30 metres, then turns right
at 90o and walks 25 metres. How far, she is form the starting point and in which direction?
(NTSE Stage-II, 2015)
(A) 55 mt., north-east (B) 50 mt., north-west
(C) 60 mt., north (D) 50 mt., west

40. A ship navigating in the Indian Ocean is hit by a sea storm and drifts as follows:
40 km North
28 km northwest
36 km west
52 km south and 29 km south east
The ship had finally drifted in ________ direction from its original position. (NTSE Stage-II, 2015)
(A) South West (B) South (C) West (D) South East

41. Ramesh travels 3 km to east then moves to right and travels 5 km and at the end he again moves right
and travels 15 km. Then the distance and direction of initial point from Ramesh is
(NTSE Stage-I/Raj./2017)
(A) 13 km south - west (B) 13 km north-east
(C) 12 km north-east (D) 12 km south-east

Answer Key

1. (b) 2. (d) 3. (d) 4. (c) 5. (a) 6. (a) 7. (d) 8. (b) 9. (a) 10. (d)
11. (d) 12. (a) 13. (d) 14. (b) 15. (b) 16. (d) 17. (a) 18. (b) 19. (b) 20. (d)

21. (d) 22. (a) 23. (b) 24. (d) 25. (b) 26. (c) 27. (b) 28. (c) 29. (d) 30. (b)

31. (d) 32. (b) 33. (d) 34. (a) 35. (c) 36. (d) 37. (a) 38. (b) 39. (b) 40. (a)

41. (b)

116 PACE IIT & MEDICAL: Mumbai / Delhi & NCR / Goa / Akola / Kolkata / Nashik / Pune / Bokaro / Dubai
NTSE-MENTAL ABILITY

PREVIOUS YEAR QUESTIONS

1. A map was so placed that north-west becomes south then what will east become?
(a) South-west (b) North
(c) North-east (d) West

2. A map was so placed that south-east becomes west then what will north-east become?
(a) South-west (b) West
(c) North-west (d) South
[NTSE-MAHARASHTRA STAGE-1-2020]

Q.3 and 4 Direction Pradyumna walked 12 km west. Then he turned right and walked 5 km. Again he
turned right and walked 4 km. Finally he Again turned right and walked 11 km. Then

3. At the end, which direction Pradyumna is facing?


(a) North (b) East
(c) South (d) West

4. At what distance is Pradyumna now from the original place?


(a) 8 km (b) 6 km
(c) 12 km (d) 10 km
[NTSE-MAHARASHTRA STAGE-1-2019]

Q.5 and 6: Direction: Ajit walked 5 km. East from A. After turning left he walked 3 km. Then he turned in
South-east direction and walked 5 km. Then he turned west and walked 4 km. Finally he turned left and
walked 12 km. Then

5. How far Ajit is from his original place?


(a) 13 km (b) 17 km
(c) 18 km (d) 7 km

6. Ajit is facing which direction now?


(a) East (b) South
(c) West (d) North
[NTSE-MAHARASHTRA STAGE-1-2018]

7. Seem went 9 km to west. She turned to right and went 7 km. She turned to left and went 8 km. From
there she turned back and went 11 km. Then she turned to right are went 7km.
How much distance is she from origin? (Seema turns every time in 900 angle)
(a) 9 km (b) 6 km
(c) 3 km (d) 7km [2017]

8. A farmer travels 20 m north from his house. He then turns east and walks 6m, from there he again
south and walk 12 m. How far is he from his original position?
(a) 6 meter (b) 8 meter
(c) 10 meter (d) 14 meter [2016]

0
9. If a man facing east rotates in clockwise direction through 45 and later in anticlockwise direction
through 2700 , then which direction is he facing?
(a) South-east (b) West
(c) South-west (d) South
117 PACE IIT & MEDICAL: Mumbai / Delhi & NCR / Goa / Akola / Kolkata / Nashik / Pune / Bokaro / Dubai
NTSE-MENTAL ABILITY

10. Meghana travels 10 km towards south, then turns left and travels 35 km, again turns left and travels
40 km, then she turns right and travels 5 km and reaches her workplace to the bank. What is the
distance between Meghana’s house and bank?
(a) 50 km (b) 39 km
(c) 40 km (d) 30 km [2016]

11. Jay starts moving towards South and walks 20 m. Then the turns towards South-West direction and
walks 25m. Now he turns to North and walks 24 m. After taking a left turn at this point, he walks
8m. From that he turns to North and walks 12 m. Now at what distance Jay is standing from the
starting point?
(a) 25 m (b) 17 m
(c) 15m (d) 12m [2015]

12. Yash starts moving towards South and walks for 12 m. Then he takes the left turn and walks for 8 m.
Again he takes a right turn and walks for 24 m. Now he takes a turn to North-east direction and
walks 25 m. From that he takes a turn towards North and walk 20 m. Find the distance between the
start point and the finishing point of his movement.
(a) 12 m (b) 15m
(c) 17m (d) 25m [2014]

0
13. Prakash is standing facing East. After turning 180 in anticlockwise direction, he travels straight for
8 km. Turning to right he travels 2 km, then again turns right and travels 11 km, again turns right and
covers 6km. How far is he from his starting point?
(a) 10 km (b) 3 km
(c) 5 km (d) 13 km [2013]

14. Shrikant from a certain place went 4 km to East. He turned left and went 1 km . Then he turned right
and went 2 km. Again he turned right and walked a
distance of 9 km. Then at what distance is he from his original position?
(a) 4km (b) 10km
(c) 9 km (d) 6km [2012]

Answer Key

1. (a) 2. (b) 3. (c) 4. (d) 5. (a) 6.(b) 7. (b) 8. (c) 9. (c) 10. (a)
11. (b) 12. (c) 13.(c) 14. (b)

118 PACE IIT & MEDICAL: Mumbai / Delhi & NCR / Goa / Akola / Kolkata / Nashik / Pune / Bokaro / Dubai
NTSE-MENTAL ABILITY

9. SEATING ARRANGEMENT

Seating Arrangements
In these type of questions, some clues regarding seating or placing sequence (linear or circular or
rectangular) of some persons or items is given. The candidate is required to form the proper sequence using
these clues and answer the given questions accordingly.

Linear Seating Arrangement

1. Who among P, Q, R, S and T is in the exactly middle while standing in a line ?


(i) Q is to the immediate right of T.
(ii) S is exactly between P and T.
(iii) Q is exactly between T and R.
(A) P (B) Q (C) R (D) T

Sol. (D) On the basis of information, we have the arrangement of the persons in a line as under:

Directions: (2 to 4) Study the given information and answer the question that following.
Five persons are standing in a queue. One of the two persons at the extreme ends is a Professor and the other
is a Businessman. An Advocate is standing to the right of a student. A Author is to the immediate left of the
businessman. The Student is between the Professor and the Advocate.

2. Counting from the left the Author is at which place ?


(A) First (B) Second (C) Third (D) Fourth

3. Which of the following is in the exactly middle of the queue?


(A) Professor (B) Advocate (C) Student (D) Businessman

4. If Advocate and the Businessman exchange their positions, also the Author and the Student, then
who will be standing to the left of the student?
(A) Author (B) Businessman (C) Professor (D) Advocate

Sol. (2 to 4) On the basis of the information given in the question, we have the arrangement of standing
order of persons as per figure.

2. (D) Author is fourth from the left.

3. (B) Advocate is in the middle of queue.

4. (B) Businessman will be to the left of Student if Advocate and Businessman, Author and Student
exchange their positions.

Circular Seating Arrangements

5. Five persons were playing cards game sitting in a circle all facing the centre. Mukund was to the
immediate left of Rajesh, Vijay was to the right of Anil and between Anil and Nagesh. Who was to
the immediate right of Nagesh ?
(A) Vijay (B) Rajesh (C) Anil (D) Mukund
119 PACE IIT & MEDICAL: Mumbai / Delhi & NCR / Goa / Akola / Kolkata / Nashik / Pune / Bokaro / Dubai
NTSE-MENTAL ABILITY

Sol. (D)

Directions : (6 to 8) Read the following information and answer the questions that follow:
(i) Six friends A, B, C, D, E and F are sitting in a closed circle facing the centre.
(ii) E is to the immediate left of D.
(iii) C is exactly between A and B.
(iv) F is between E and A.

6. Who is to the immediate left of B ?


(A) A (B) C (C) D (D) E

7. Who is to the immediate right of C ?


(A)A (B) B (C) D (D) E

8. Which of the above given statements is superfluous ?


(A) (i) (B) (ii) (C) (iii) (D) All are required

Sol. (6 to 8) Clearly, in the circle the arrangement is as shown:

6. (C) Clearly, D is to the immediate left of B.

7. (A) Clearly, A is to the immediate right of C.

8. (D) Since all the statements are necessary to determine the arrangement, none of them is superfluous.

120 PACE IIT & MEDICAL: Mumbai / Delhi & NCR / Goa / Akola / Kolkata / Nashik / Pune / Bokaro / Dubai
NTSE-MENTAL ABILITY

EXERCISE

Directions : (1 to 4) Study the given information and answer the questions that following.
(i) P, Q, R, S T, U and V are sitting in a row facing East.
(ii) R is on the immediate right of S.
(iii) Q is at an extreme end and has T as his immediate neighbour.
(iv) V s exactly between T and U.
(y) S is sitting third from the South end.

1. Who is sitting to the immediate right of T?


(a) P (b) V (c) S (d) U

2. Which of the following pairs of people are sitting at the extreme ends?
(a) PQ (b) PS (c) QR (d) UB

3. Name the person who is at the third place from the North end.
(a) T (b) U (c) V (d) S

4. Immediately between which of the following pairs of people is S sitting?


(a) PR (b) PU (c) RT (d) RU

Directions : (5 to 6) Are based on the information given below :


A group of seven singers, facing the audience, are standing in a line on the stage as follows :
(i) D is to the immediate right of C.
(ii) F is to the immediate neighbours of G.
(iii) B is to the immediate left of F.
(iv) E is to the immediate left of A.
(v) C and B have one singer between them.
(vi) A and D have one singer between them.

5. Which of the following pairs of singer are sitting at the extreme ends?
(a) D, C (b) F, A (c) E, G (d) E, F

6. Immediately between which of the following pairs of singer is D sitting?


(a) A, B (b) F, A (c) B, C (d) E, F

Directions : (7 to 8) Answer the questions based on the following information.


6 men R, S, T, U, V and W sat around a circular table playing cards. It was noticed that no two men
the initial letters of whose names are adjacent in the alphabetical order, sat next to each other, U was
opposite of R. V was not to the immediate right of R.

7. Who sat to the immediate left of R?


(a) S (b) T (c) V (d) W

8. Who sat to the immediate right of R?


(a) S (b) T (c) V (d) W

Directions : (9 to 13) Study the following information and answer the given questions that following.
(i) Eight friends A, B, C, D, E, F, G and H are seated in a circle facing centre.
(ii) D is between B and G and F is to the immediate neighbours of A and H.
(iii) E is second to the right of A.

121 PACE IIT & MEDICAL: Mumbai / Delhi & NCR / Goa / Akola / Kolkata / Nashik / Pune / Bokaro / Dubai
NTSE-MENTAL ABILITY

9. Which of the following is A’s position?


(a) Immediate left of F. (b) Immediate right of F.
(c) Exactly between E and F. (d) Can’t be determined.

10. Which of the following is C’s position ?


(a) Exactly between E and A. (b) Between G and E.
(c) Second to the left of B. (d) Can’t be determined.

11. Who are the immediate neighbours of D ?


(a) B and C (b) C and E (c) B and G (d) B and G or B and H

12. If the positions of B and G and D and A are nterchanged then who is sitting between B and G in
new position.
(a) D (b) A (c) H (d) E

13. If B is sitting opposite to C and H is sitting opposite to E then find who is sitting opposite to F ?
(a) B (b) G (c) A (d) D

Directions: (14 to 16) Six Persons P, Q, R, S, T and U are sitting in a circle facing one another front to
front. P is sitting ¡n front of Q, Q is sitting to the immediate right of T and immediate left of R, P is to
the left of U and right of S.

14. Who is sitting opposite to R ?


(a) P (b) Q (c) S (d) U

15. Who is sitting opposite to S ?


(a) U (b) T (c) R (d) Q

16. Who is sitting between P and R?


(a) S (b) T (c) U (d) Q

Directions : (17 to 21) Read the following information carefully and answer the questions given below
(I) Eight persons E, F, G H, I, J, K and L are seated around a square table- two on each side.
(II) There are three lady members and they are not seated next to each other.
(III) J is to the immediate neighbours of L and F.
(IV) G is between I and F.
(V) H, a lady member, is second to the left of J
(VI) L, a male member, is seated opposite of E, a lady member,
(VII) There is a lady member between F and I.

17. Who among the following is to the immediate neighbours of E and H?


(a) F (b) I (c) J (d) None of these

18. How many persons are seated between K and F?


(a) One (b) Two (c) Three (d) Cannot be determined

19. Who among the following are the three lady members ?
(a) E, G and J (b) E, H and G (c) G, H and J (d) Cannot be determined

20. Who among the following is to the immediate left of F?


(a) G (b) I (c) J (d) Cannot be determined

122 PACE IIT & MEDICAL: Mumbai / Delhi & NCR / Goa / Akola / Kolkata / Nashik / Pune / Bokaro / Dubai
NTSE-MENTAL ABILITY

21. Which of the following is true about J?


(a) J is a male member (b) J is a female member
(c) Sex of J cannot be determined (d) Position of J cannot be determined

Directions : (22 to 26) Read the following information carefully and answer the questions :
(i) Six flats on a floor in two rows, facing east and west are allotted to Q, R, S, T. U and V.
(ii) R gets east side facing and not next to T.
(iii) T and V get diagonally opposite fiats.
(iv) S next to V gets a west facing flat.
(v) U gets a east facing flat. (NTSE Stage-II, 2008/2009)

22. Which of the following combination gets west facing flats ?


(a) SQR (b) RTS (c) STU (d) QSV

23. Whose flat is between R and T?


(a) Q (b) S (c) U (d) V

24. If the flats of U and Q are interchanged then whose flat will be opposite to the flat of U ?
(a) V (b) Q (c) T (d) S

25. The flats of which pair other than T and V are diagonally opposite to each other?
(a) TQ (b) US (c) TU (d) RQ

26. Whose flat is between Q and V?


(a) R (b) T (c) U (d) S

27. Six friends L, M, N, O, P and Q are seated in a circle facing the centre. Q is between L and O. N
is between P and M. P is not between O and N. O is second to the left of N.
Who is between L and N? (NTSE Stage-II, 2009)
(a) M (b) P (c) O (d) N

28. Six students are sitting in a row. A is sitting between B and C. B is sitting next to E. E is sitting
next to O who is sitting on the extreme left and D is sitting next to C. Who two are sitting adjacent
to B? (NTSE Stage-II, 2009)

Directions: (29 to 31) Study the information carefully and then choose the correct alternative to
answer the questions. five friends A, B, C, D and E are sitting on a bench.
(1)A is sitting next to B.
(2) C is sitting next to D.
(3) D is not sitting with E.
(4)E ¡son the left end of the bench.
(5) C is on second position from the right.
(6) A is on the right side of B and to the right side of E.
(7) A and C are sitting together. (NTSE Stage-II,2010)

29. Where is A sitting?


(a) Between B and D (b) Between D and C
(c) Between C and E (d) Between B and C

30. C is sitting between


(a) B and D (b) A and E (c) D and E (d) A and D

123 PACE IIT & MEDICAL: Mumbai / Delhi & NCR / Goa / Akola / Kolkata / Nashik / Pune / Bokaro / Dubai
NTSE-MENTAL ABILITY

Directions : (32 to 33) Read the following information carefully and answer the questions based on it
by selecting the correct option from the given alternatives.
(j) S, T, U, V, W, X and Y are sitting along a wall facing North direction
(ii) U is on the immediate light of V
(iii) T is at an extreme end and has W as his neighbour.
(iv) Y is between W and X.
(y) V is sitting third from the other end. (NTSE Stage-II,2011)

32. Who is sitting on the left of X?


(a) T (b) Y (c) W (d) U

33. Where is S sitting?


(a) Extreme East (b) Extreme West
(c) In the middle (d) Second from the East end

34. Six persons R Q, R, S. T and U are sitting in two rows, three n each as per following information.
(NTSE Stage-II,2011)
T is not at the end of any row.
S is second of the left of U.
R the neighbour of T is sitting diagonally opposite to S.
Q is the neighbour of U
Which of the following are sitting diagonally opposite to each other.
(a) P and R (b) S and P (c) P and U (d) None of these

35. X, Y. Z and P are sitting around a circular table and discussing their trades.
(i) X, sits opposite to cook
(ii) Y, sits right to the barber
(iii) The washerman is on the left of the tailor
(iv) R sits opposite Z
What are the trades of X and Y? (NTSE Stage-I / Raj./ 2013)
(a) Tailor and Barber (b) Barber and Cook
(c) Tailor and Cook (d) Tailor and washerman

36. Six persons A, B. C, D, E and F are standing in a circle. B is between F and C, A Is between E and
D, F is to the left of D. Who is between A and F? (NTSE Stage-I/Raj./ 2013)
(a) B (b) C (c) D (d) E

37. Six students are sitting in a row. K is sitting between V and R. V s sitting next to M. M is sitting
next to B who is sitting on the extreme left and Q is sitting next to R.
Who are sitting adjacent to V? (NTSE Stage-I/ Raj./ 2013)
(a) R and Q (b) B and M (c) K and R (d) M and K

Directions (38 to 42): These questions are based on the following information:
, , , ,, , ,  are sitting on a merry-go-round facing at the centre.  is second to the left on  who
is third to the left of ,  is fourth to the right of  who is immediate neighbor of  .  is not a
neighbour of  . or  is not a neighbour of  . (NTSE Stage-II,2013)

38. Who is third to the left of  .


(a)  (b)  (c)  (d) 

124 PACE IIT & MEDICAL: Mumbai / Delhi & NCR / Goa / Akola / Kolkata / Nashik / Pune / Bokaro / Dubai
NTSE-MENTAL ABILITY

39. In which of the following pairs is the first person sitting to the immediate right of the second per
son ?
(a) ,  (b) , (c) ,  (d) , 

40. What is  's position with respect to p?


(a) Third towards right (b) Third towards left
(c) Second towards right (d) Second towards left

41. Who is sitting between  and  ?


(a) Both  and  (b) Both  and  (c) Only  (d) Only 

42. How many of them are sitting between and ?


(a) 0 or 6 (b) 1 or 5 (c) 2 or 4 (d) 3

43. A, B, C, D and E are sitting on a bench. A s sitting next to B, C is sitting next to D, D is not sitting
next to E, who is sitting on the left end of the bench. C is on the second position from the right. A
is to the right of B and E. Counting from the left m which position is A sitting?
(NTSE Stage-II,2013)
(a) 2
(b) 3
(c) 5
(d) Cannot be determined from the given conditions.

Answer Key

1. (b) 2. (a) 3. (c) 4. (d) 5. (c) 6. (c) 7. (c) 8. (b) 9. (b) 10. (a)
11. (c) 12. (b) 13. (b) 14. (d) 15. (b) 16. (a) 17. (d) 18. (c) 19. (b) 20. (c)
28.
21. (a) 22. (d) 23. (c) 24. (c) 25. (d) 26. (d) 27. (b) 29. (d) 30. (d)
(EA)
31. (b) 32. (b) 33. (a) 34. (c) 35. (c) 36. (c) 37. (d) 38. (c) 39. (b) 40. (a)

41. (c) 42. (d) 43. (b)

PREVIOUS YEAR QUESTIONS

Q.1 to 3 -Directions: A, B, C, D, E, F, G, H eight friends are sitting around the circular table. C is sitting in
front of A and H is sitting at the first position to the left of A. F is in front of H. Whereas D is at the first
position to right of E and C is at the first position to left of E. A is between G and H. choose the correct for
the questions given below. (NTSE-MAHARASHTRA STAGE-1-2020)

1. Who is sitting between B and C?


(a) D (b) G (c) F (d) H

2. Who is sitting between A and B?


(a) D (b) G (c) E (d) F

125 PACE IIT & MEDICAL: Mumbai / Delhi & NCR / Goa / Akola / Kolkata / Nashik / Pune / Bokaro / Dubai
NTSE-MENTAL ABILITY

3. If B and D would have interchanged the places then who will be sitting at the first position to the left
of B?
(a) G (b) C (c) F (d) E

Q.4 to 6 -Directions : A B, C, D, E and F are sitting at each corner of a hexagonal table A and D are facing
opposite direction. B is sitting to the left of D. D is sitting next to C and E is sitting to the other side of C.
(NTSE-MAHARASHTRA STAGE-1-2020)
4. Who is sitting opposite to F?
(a) C (b) E (c) D (d) B

5. If the persons sitting in opposite direction interchange their places, then who will be sitting in
between D and F.
(a) E (b) A (c) B (d) C
6. If only A and D interchange their place who will be in between B and C?
(a) A (b) F (c) E (d) D
Q.7 to 9 : Direction : Eight players are standing to play ‘Standing Kho-Kho.Geeta is at the third place to the
right of Mahesh. Amar is to the first place to the right of Geeta. Asha is to the fourth place to the left of
Geeta. Radha is at central place between Parag and Asha. Meena is at the central place between Geeta and
Hemant. Then (NTSE-MAHARASHTRA STAGE-1-2018)

7. Who is standing opposite to Hemant ?


(a) Amar (b) Parag (c) Radha (d) Geeta

8. Who is standing between Parag and Geeta ?


(a) Hemant (b) Mahesh (c) Amar (d) Meena

9. Who is at the fourth place to the left of Geeta ?


(a) Mahesh (b) Asha (c) Paiag (d) Radha

10. Six teachers of a workshop have sat down for a photo session as shown below. A is sitting ¡n between
K and S. M is at a comer. There is no one sitting in between N and S. Then where is the person ‘J’
sitting? (NTSE-MAHARASHTRA STAGE-1-2017)
(a) At the central position between K and M
(b) At the central position between N and A
(c) At the central position between S and K.
(d) At the central position between M and A.

11. On a playground J, K, L, M, N, O, P, Q, R are sitting in one row to watch a cricket match. L is at the
right side of M and is occupying third place from N at the right side. K is sitting either at first or last
position. Q is in between O and P. O is sitting at the third position at the left side of K. O is sitting
next to ‘J’ at the right side. Who is sitting at the centre among them?
(NTSE-MAHARASHTRA STAGE-1-2017)
(a) L (b) O (c) J (d) Q

12. The seven boys Sunil, Anil, Harshal, Shubham, Kishore, Ujwal and Omsai are sitting in a row.
Harshal is sitting in between Shubham and Sunil. Kishore is in between Ujwal and Omsai. Anil is
sitting in between Shubham and Ujwal. Sunil and Omsai are sitting at the two ends. Then
Shubham is between which of the two boys ? (NTSE-MAHARASHTRA STAGE-1-2015)
(a) Harshal and Anil (b) Sunil and Anil
(c) Anil and Kishore (d) Ujwal and Anil

126 PACE IIT & MEDICAL: Mumbai / Delhi & NCR / Goa / Akola / Kolkata / Nashik / Pune / Bokaro / Dubai
NTSE-MENTAL ABILITY

Q. 13 and 14. Directions : Six friends A, B, C, D, E, F are sitting around a round table. F is sitting in
between A and D. C is sitting in between E and B. E is not sitting between D and C. D is at second position
to the left of C. Answer the following questions. (NTSE-MAHARASHTRA STAGE-1-2014)

13. Who is sitting in between A and C ?


(a) B (b) E (c) D (d) Cannot be said

14. What is the position of D ?


(a) Second position to the right of A (b) The next to the right of B
(c) Third position to the left of B (d) Fourth position to the right of A

15. A, B, C and D ladies are sitting around a table talking about different occupations. A is seated before
a Cook. B is to the right of Wadawala, and D is seated before C. Dhobi is seated to the left of Tailor.
Then what are the occupations of A and B ? (NTSE-MAHARASHTRA STAGE-1-2012)
(a) Tailor and Wadawala (b) Wadawala and Cook
(c) Tailor and Dhobi (d) Tailor and Cook

Answer Key

1. (c) 2. (b) 3. (d) 4. (a) 5. (c) 6. (a) 7. (b) 8. (c) 9. (b) 10. (a)
11. (c) 12. (a) 13. (b) 14. (a) 15. (d)

127 PACE IIT & MEDICAL: Mumbai / Delhi & NCR / Goa / Akola / Kolkata / Nashik / Pune / Bokaro / Dubai
NTSE-MENTAL ABILITY

10. RANKING & ORDERING –TEST

Number-Quibble
In these type of questions, generally a set, group or series of numerals is given and the candidate is required
to find out how many times a number satisfying the conditions, specified in the question occurs.

1. How many 5s are there in the following sequence which are immediately followed by 3 but not
immediately preceded by 7 ?
8953253855687335775365335738
(A) One (B) Two (C) Three (D) Four
Sol. 8 9 5 3 2 5 3 8 5 5 6 8 7 3 3 5 7 7 5 3 6 5 3 3 5 7 3 8 clearly, there are three such numbers.

2. In the following number series how many 8’s are there which are exactly divisible by the numbers
which are preceded and followed by it ?
824517284842282698454832843183
(A) 1 (B) 2 (C) 3 (D) 4
Sol. (D) As per the question
824517284842282698454832843183
Thus, four such numbers are there

Alphabetical Quibble
In these type of questions, generally a letter-series is given, be ¡t the English alphabets from A to Z or a
randomised sequence of letters. The candidate is then required to trace the letters satisfying certain given
conditions as regards their position in the given sequence or the sequence obtained by performing certain
given operations on the given sequence.

3. How many W’s are there in the following series which are immediately followed by W but not
immediately preceded by K ?
DWWDHKVDWZDWWWDDWKWWDKKDHC
(A) One (B) Two (C) Three (D) None
Sol. (C)
Clearly, W’s satisfying the given conditions can be marked as under
DWWDHKVDWZDWWDDWKWWDKKDHC

4 How many L’s are there which do not have R preceding them and also do not have T following
them?
ZQSTLRMNQNRTUVXRLTASLTWRSLT
(A) 1 (B) 2 (C) 3 (D) 5
Sol. (C)
ZQSTLRMNQNRTUVXRLTASLTQRSLT

Alpha-Numeric Quibble
In these type of questions a jumped sequence of some letters, numbers and symbols is giver and the
candidate is required to find out how many times a number or a letter or a symbol satisfying the conditions,
specified in the question occurs.

Directions : (5 to 6) Study the following arrangement of symbols, letters and numbers to answer the
questions given below it :
f 2 KS75  $PLV8@MUE6  QG@93&TY 

128 PACE IIT & MEDICAL: Mumbai / Delhi & NCR / Goa / Akola / Kolkata / Nashik / Pune / Bokaro / Dubai
NTSE-MENTAL ABILITY

5. How many such letters are there in the arrangement each of which is either immediately preceded by
a symbol or immediately followed by a number, but not both ?
(A) Three (B) Four (C) Five (D) None of these
Sol. (D) The letters satisfying the given conditions are shown as under :
  f 2 K S 75  $PLV8@MUE6  QG@93&TY  clearly, there are 8 such letters.

6. How many such consonants are there in the above arrangement each of which is immediately
followed by a consonant but not immediately preceded by a symbol ?
(A) Nil (B) One (C) Two (D) Three

Sol. (B) We know that of the 26 letters of English alphabet, five letters namely A, E, I, O, U are vowels,
while remaining are consonants. The consonants satisfying the given conditions may be shown as
under :
   F 2 K S 7 5  $ P L V 8@ M U E 6  Q G © 9 3 & T Y 
Clearly, there is only one such consonant.

Ranking Test
In these type of questions, generally the ranks of a person both from the top and from the bottom are
mentioned and the total number of persons is asked. However, sometimes this question is put in the form of
a puzzle of interchanging seats by two persons.

Useful Tips
1. Position of person from upward
= [Total number of persons - position of person from down] + 1

2. Position of person from downward


= [Total number of persons - position of person from up] + 1

3. Position of person from right


= [Total number of persons - position of person from left] +1

4. Position of person from left


= [Total number of persons - position of person from right] + 1

5. Total number of persons


= [Position of person from upward/right + position of person from downward / left]  1

7. In a row of girls, Mridula is 18th from the right and Sanjana is 18th from the left. If both of them
exchange their position, Sanjana becomes 25th from the left, how many girls are there in the row?
(A) 40 (B) 41 (C) 42 (D) 35

Sol. (C) Sanjana’s new position is 25th from left. But it is the same as Mridula’s earlier position which is
18th from the right. Then total total number of girls are = (rank from left+ rank from right) 1
 18  25   1  43  1  42

8. Anil and Sunil are ranked seventh and eleventh respectively from the top in a class of 31 students.
What will be their respective ranks from the bottom in the class ?
(A) 20th and 24 th (B) 24th and 20th (C) 25th and 21st (D) 26th and 22

Sol. (C) Number of students behind Anil in rank


129 PACE IIT & MEDICAL: Mumbai / Delhi & NCR / Goa / Akola / Kolkata / Nashik / Pune / Bokaro / Dubai
NTSE-MENTAL ABILITY

  31  7   24
So, Anil is 25th from the bottom.
Number of students behind Sunil in rank
= (31  11) = 20
So, Sunil is 21st from the bottom.

Ordering Test
In such type of questions, clues are given regarding comparisons among a set of persons or things with
respect to their qualities. The candidate is required to analyses the whole information, form a proper
ascending/descending sequence and then answer the given questions accordingly.

9. A is shorter than B but much taller than E. C is the tallest and D is shorter than A and taller than E.
Which one is the shortest ?
(A) A (B) E (C) B (D) D
Sol. (B) According to the given statement E < D < A < B < C

10. A is richer than B.


C is richer than A.
D is richer than C.
E is the richest of all.
If they are made to sit in the above degree of richness who will be in the middle position (central
position) ?
(A) A (B) B (C) C (D) D

Sol. (C) According to the given statement

C is in the middle position.

11. Ashok is older than Suresh, Vivek is not youngest. Raju is older than Ashok and Vivek is not older
than Suresh. Which one of the following statements must necessarily be true ?
I. Vivek is as old as Ashok
II. Vivek is as old as Raju
III. Vivek is as old as Suresh
IV. Suresh is younger than Vivek
(A) Only I (B) Only II (C) Only III (D) Either III or IV

Sol. (C) According to the given statement V = S < A< R

Directions: (12 to 13) Read following information carefully and answer the questions given below it :
(i) P, Q, R, S and T are five friends.
(ii) Q is elder to T.
(iii) R is younger to P.
(iv) P is younger to T.
(v) S is elder to P.

12. Who among the following is the eldest?


(A) P (B) Q (C) S (D) Data inadequate

130 PACE IIT & MEDICAL: Mumbai / Delhi & NCR / Goa / Akola / Kolkata / Nashik / Pune / Bokaro / Dubai
NTSE-MENTAL ABILITY

13. Who among the following is the youngest ?


(A) P (B) R (C) T (D) Data inadequate

Sol. (12 to 13) Thus, sequence of their age becomes


Q > T > S > P > R or Q > S > T > P > R
or S > Q > T > P > R

12. (D) According to a given statement there are three sequences, as follows:
Q > T > S > P > R or Q > S > T > P > R
or S > Q > T > P > R
Hence the answer is data inadequate

13. (B) Clearly, R is the youngest.

EXERCISE

1. How many even numbers are there in the following sequence of numbers which are immediately
followed by an odd number as well as immediately preceded by an even number ?
86768932753422355228119
(a) One (b) Three (c) Five (d) None of these

2. In the following series, how many times the sum of two consecutive numbers results an even
number?
12346342593674123676543
(a) 3 (b) 4 (c) 5 (d) None of these

3. In the following number series, how many 8’s are there which are immediately preceded by a
number which does not divide it but followed by a number which divides it ?
28283858853282384715838286
(a) 1 (b) 2 (c) 3 (d) 4

4. In the following series of numbers, find out how many times, 1, 3 and 7 have appeared together, 7
being in the middle and 1 and 3 on either side of 7?
2973173771331738571377173906
(a) One (b) Two (c) Three (d) Four

5. The positions of how many digits in the number 423157698 will remain unchanged after the digits
within the number are arranged in ascending order?
(a) None (b) One (c) Two (d) Three

6. In the following series how many C’s are there which are immediately followed by ‘Y’ but not
immediately preceded by ‘J’ ?
JCDYJCYOJHCYYYCIJWCYACY
(a) One (b) Two (c) Three (d) Four

Directions : (8 to 10) These questions are based on the following arrangement of symbols, letters and
numbers.

131 PACE IIT & MEDICAL: Mumbai / Delhi & NCR / Goa / Akola / Kolkata / Nashik / Pune / Bokaro / Dubai
NTSE-MENTAL ABILITY

8. How many such numbers are there in the arrangement each of which is immediately preceded by a
symbol but not immediately followed by a letter?
(a) Nil (b) One (c) Two (d) Three

9. How many such symbols are there in the above arrangement each of which is either immediately
followed by a number or immediately preceded by a letter?
(a) One (b) Two (c) Three (d) Four

10. How many such letters are there in the arrangement each of which is immediately followed by a
number but not immediately preceded by a symbol?
(a) Nil (b) One (c) Two (d) Three

11. In a Class Vidya ranks 7th from the top, Diya is 7 ranks ahead of Megha and 3 ranks behind Vidya.
Sushma who is 4th from the bottom, is 32 ranks behind Meg ha. How many students are there in
the class?
(a) 52 (b) 49 (c) 50 (d) None of these

12. In a queue of boys Sohan is 9th from the back. Ramesh’s place is 8th from the front. Radhey is
standing in the middle of the two. What could be the minimum number of boys standing in the
queue?
(a) 8 (b)10 (c) 12 (d) 14

13. If all the numbers from 7 to 59, which are divisible by 3 are arranged in descending order then
which number will be at 10th place from the bottom ?
(a) 36 (b) 39 (c) 30 (d) 27

14. In a row of girls, Rina and Mona occupy the ninth place from the right end and tenth place from
the left end, respectively, If they interchanged their places, Rina and Mona occupy seventeenth
place from the right and eighteenth place from the left, respectively. How many girls are there in
the row?
(a) 25 (b) 26 (c) 27 (d) Data inadequate

15. In a class of 35 students, Kunal is placed seventh from the bottom where as Sonali is placed ninth
from the top. Pulkit is placed exactly in between the two. What is Kunars position from Pulkit?
(a) 9 (b) 10 (c) 11 (d)13

16. In a queue, A is eighteenth from the front while B is sixteenth from the back. If C is twenty-fifth
from the front and is exactly in the middle of A and B, then how many persons are there in the
queue?
(a) 45 (b) 46 (c) 47 (d) 48

17. Students line up in a queue in which Ashish stands fifteenth from the left and Sachin is seventh
from the right. If they interchange their places. Sachin would be fifteenth from the right. How
many students are there in the queue ?
(a) 21 (b) 22 (c) 29 (d) None of these

18. Nitin was counting down from 32. Sumit was counting upwards, the numbers starting from I
and he was calling out only the odd numbers. What common number will they call out at the
same time if they were calling out at the same speed ?
(a) 19 (b) 21
(c) 22 (d) They will not call out the same number

132 PACE IIT & MEDICAL: Mumbai / Delhi & NCR / Goa / Akola / Kolkata / Nashik / Pune / Bokaro / Dubai
NTSE-MENTAL ABILITY

19. In a group of six children. Q is taller than P but not as tall as L. M is taller than N and O, but not as
tall as R Who is the shortest among them ?
(a) N (b) O (c) M (d) Data inadequate

20. R earns more than H but not as much as T, M earns more than R. Who earns least among them ?
(a) H (b) R (c) T (d) M

Direction: (21 to 23) Read the following information and answer the questions. A was punctual. B
came after him and C immediately before B. D was earliest but E was last of al getting to school.

21. Who arrived first at school?


(a) A (b) B (c) C (d) D

22. Who arrived second?


(a) A (b) B (c) C (d) D

23. Who came third?


(a) A (b) B (c) C (d) D

Directions : (24 to 28) Read the following information and answer the questions given below it :
Alka is older than Mala. Gopal is older than Mala but younger than Alka. Kapil is younger than Ram
and Mala. Mala is older than Ram.

24. Whose age is between Gopal and Ram ?


(a) Mala (b) Kapil (c) Alka (d) None of these

25. Whose age is between Mala and Kapil?


(a) Gopal (b) Ram (c) Alka (d) None of these

26. Whose age is exactly in the middle of all the five?


(a) Mala (b) Gopal (c) Ram (d) Alka

27. Who is the eldest?


(a) Alka (b) Mala (c) Kapil (d) Gopal

28. Who is the youngest?


(a) Mala (b) Ram (c) Alka (d) Kapil

29. How many odd numbers are in the following number series, which does not contains just after
it an odd number, but contains an even number just before it? (NTSE Stage- I/ Raj./ 2007)
35814976159234852
(a) 2 (b) 3 (c) 4 (d) 5

30. If all the even numbers in between numbers from 32 to 51 are arranged in a row then number at
Fifth position from right, will be (NTSE Stage-I /Raj./2007)
(a) 36 (b) 40 (c) 42 (d) 48

31. How many A are in the given letter series which does not has B just before it but has C just
after it? (NTSE Stage-I /Raj./ 2007)
DACBACDACBCACBACBADC
(a)3 (b) 4 (c) 5 (d) 6

133 PACE IIT & MEDICAL: Mumbai / Delhi & NCR / Goa / Akola / Kolkata / Nashik / Pune / Bokaro / Dubai
NTSE-MENTAL ABILITY

32. How many 9’s are there which come after 6 or multiple of 3?
699969796979592989696999979896978 (NTSE stage-II,2007)
(a)7 (b) 9 (c) 11 (d) 6

33. Shyama ranked 12th from the top and 28th from the bottom among those children who passed
the examination. Eight children failed in the examination while five children did not appear in
the examination. How many children were there in the class? (NTSE Stage-II,2007)
(a) 50 (b) 51 (c) 52 (d) 53

34. If you write all the numbers from 201 to 250, then how many times will you write the numeral 2 ?
(NTSE Stage-II, 2007)
(a) 70 (b) 65 (c) 25 (d) 80

35. How many c’s are there in between two consonants in the following series ?
cabcdcdcecf ocicjckcck (NTSE Stage-II,2007)
(a) 4 (b) 5 (c) 6 (d) 11

36. How many times ‘+’ comes before ‘  ’ and after ‘  ’?


                  
                           (NTSE Stage-II,2007)
(a) 6 (b) 5 (c) 4 (d) 7

37. There are five friends A, B, C, D and E. A is shorter than B but taller then E, C is tallest, D is little
shorter than B and little taller than A. If they are standing in the order of their heights who will be
in the middle ? (NTSE Stage-I/Raj./2008)
(a) D (b) C (c) A (d) B

38. In the following series how many 4’s are preceded by 5 but not followed by 2 or 3.
(NTSE Stage-I/Raj./2008)
(a) 2 (b) 4 (c) 3 (d) 5

39. In a class the rank of Sohan is sixteen from the top and forty-nine from the bottom. The total
number of students in the class are- (NTSE Stage-I/Raj./2008)
(a) 64 (b) 65 (c) 66 (d) 67

40. In the following series how many times 7, 8, 9 appear together when 7 being in the middle?
7287921789167421797828972 (NTSE Stage-I/Raj./2009)
(a) 1 (b) 2 (c) 3 (d) 4

Directions: (41 to 43) Study the following number line and answer the questions that follow.
2591725197395246819752731978526541978201097851973516972173
7951

41. How many times is number ‘3’ preceded by number ‘7’ and not followed by an even number?
(a) 6 (b) 5 (c) 4 (d) 3

42. How many times is number 5 followed by 1 or 2, but not preceded by 8?


(a) 4 (b) 5 (c) 6 (d) 7

43. How many times is number 9 preceded by 1 and succeeded by 7 which is not followed by 5?
(a) 3 (b) 4 (c) 5 (d) 6
134 PACE IIT & MEDICAL: Mumbai / Delhi & NCR / Goa / Akola / Kolkata / Nashik / Pune / Bokaro / Dubai
NTSE-MENTAL ABILITY

44. Raju (a) is taller than Sonu (b). Ravi (c) is taller than Monu (d) but shorter than Raja (E). Sonu is
shorter than Monu and Monu is taller than Raju. Who is the tallest? Write the code of the boy in
your answer script. (NTSE Stage-II, 2009)

45. Three persons A, B and C are standing in a queue. There are 5 persons in between A and B and
eight persons between B and C. If there are 3 persons ahead of C and 21 persons behind A, what
would be the minimum number of persons in the queue? (NTSE Stage-II, 2009)

46. In a queue, Ramesh is the 7th from the back, Suresh is standing 6th from the front and only Shyam
is standing in between the two. Find the minimum number of boys standing in the queue?
(NTSE Stage-II, 2011)
(a) 8 (b) 10 (c) 12 (d) 14

47. In a queue of 10 persons, A is standing on 7th from front and B is standing 6 th from back. 3 persons
entered the queue disrupting the position of A and B. Find the pair of numbers indicating minimum
possible disruption in the position from the front? (NTSE Stage-II, 2011)
(a) 6, 10 (b) 6, 9 (c) 7, 10 (d) 7, 9

48. In the following how many times is rectangle preceded by a circle but following by a triangle?
(NTSE Stage-II, 2011)

(a) 1 (b) 2 (c) 3 (d) 4

49. Which figure has been used most of the times? (NTSE Stage-II, 2011)

(a) (b) (c) (d)

50. How many 6’s are there in the following sequence which are followed by 3 and preceded by 8?
94863271869813689786313684335 (NTSE Stage-I/Raj./2012)
(a) 2 (b) 3 (c) 4 (d) 5

51. In a dairy, there are 60 cows and buffalos. The number of cows is twice that of buffalos. Buffalo X
ranked seventeenth in terms of milk delivered. If there are 9 cows ahead of Buffalo X, how many
buffalos are after in rank in terms of milk delivered? (NTSE Stage-11,2013)
(a) 10 (b) 11 (c) 12 (d) 13

52. Amongst five friends, Lata, Alka, Rani. Asha and Sadhana. Lata is older than only three of her
friends. Alka is younger to Asha and Lata. Rani is older than only Sadhana. Who amongst them is
the eldest? (NTSE Stage-11, 2013)
(a) Asha (b) Lata (c) Alka (d) Sadhana

135 PACE IIT & MEDICAL: Mumbai / Delhi & NCR / Goa / Akola / Kolkata / Nashik / Pune / Bokaro / Dubai
NTSE-MENTAL ABILITY

53. If the following numbers are written in ascending order, the sum of the digits of middle number
will be
810, 912 ,910 , 809, 781, 673, 573 (NTSE Stage-I/Raj./2014)
(a) 9 (b) 12 (c) 17 (d) 13

54. How many 3s are there in the following figure series which are just preceded by 6 but not
immediately followed by 7?
323743563746389635183724286395 (NTSE Stage-I/Raj./2014)
(a) 1 (b) 2 (c) 3 (d) 4

Answer Key

1. (d) 2. (b) 3. (c) 4. (c) 5. (b) 6. (c) 7. (b) 8. (b) 9. (d) 10. (c)
11. (a) 12. (b) 13. (a) 14. (b) 15. (b) 16. (c) 17. (c) 18. (d) 19. (d) 20. (a)

21. (d) 22. (a) 23. (c) 24. (a) 25. (b) 26. (a) 27. (a) 28. (d) 29. (b) 30. (c)

31. (a) 32. (c) 33. (c) 34. (b) 35. (b) 36. (a) 37. (a) 38. (b) 39. (a) 40. (b)

41. (c) 42. (b) 43. (b) 44. (e) 45. (28) 46. (b) 47. (b) 48. (a) 49. (a) 50. (a)

51. (c) 52. (a) 53. (c) 54. (c)

PREVIOUS YEAR QUESTIONS

1. ABZYCDXWEFVUGHTSIJRQKLPOMN
Observe the letter series and observe letter which is at the central place of letters which is at 8 th place
from the left and at 13th place from right, find the serial number of that letter from left?
(a) 13 (b) 14
(c) 16 (d) 11
[NTSE-MAHARASHTRA STAGE-1-2020]

Q.2 and 3-Directions: In a row Pradyuman is twelfth from front and Sarvesh is Twenty Fifth from behind.
Rahul is exactly at the centre place between Pradyuman and Sarvesh. There are 70 persons in the row then.

2. Rahul is standing at which place from front?


(a) 29 (b) 33
(c) 17 (d) 42

3. Rahul is at which place from behind?


(a) 29 (b) 42
(c) 33 (d) 17
[NTSE-MAHARASHTRA STAGE-1-2020]
4. ABCDEFGHIJKLMNOPQRSTUVWXYZ
From the above alphabets which word will be formed from the given alternatives if the meaningful
word formed by the 5 th and 10th letter from the right and 1 st and 5th letter from the left is written in
the reverse order
(a) VEAS (b) SAEV
(c) AVES (d) EVAS
[NTSE-MAHARASHTRA STAGE-1-2019]
136 PACE IIT & MEDICAL: Mumbai / Delhi & NCR / Goa / Akola / Kolkata / Nashik / Pune / Bokaro / Dubai
NTSE-MENTAL ABILITY

Q.5 and 6 Direction In a queue, Amruta is at the 11th place from front. Suneeta is at 26th place from behind.
Sapna is at the central place between Amruta and Suneeta. If there are 60 person in the queue, then

5. At which place Sapna is standing from the front?


(a) 12 (b) 24
(c) 23 (d) 26

6. At which place Sapna is standing from behind?


(a) 37 (b) 38
(c) 23 (d) 39
[NTSE-MAHARASHTRA STAGE-1-2019]

7. ABCDEFGHIJKLMZYXYWVUTSRQPON
From the above letter series find the letter which is at the 6 th position to the right side of the letter
which is at the centre position of the letter which are at the 11th place from the left and 14th place
from the right.
(a) U (b) F
(c) Y (d) L
[NTSE-MAHARASHTRA STAGE-1-2018]
Q.8 & 9-Direction: In a queue, Suneeta is at the tenth place from front. Subhash is at 25th place from
behind. Gargi is standing at the central place between Suneeta and Subhash. There are 50 persons in the
queen. Then

8. Gargi is standing at which place from front?


(a) 20 (b) 19
(c) 18 (d) 17

9. Gargi is at which place from behind?


(a) 31 (b) 32
(c) 33 (d) 34
[NTSE-MAHARASHTRA STAGE-1-2018]
10. Alphabet series
xcwmvcxwmwmxcxwmxm
In the given alphabet series how many times m is succeeded by w and proceeded by w?
(a) 1 (b) 0
(c) 2 (d) 3
[NTSE-MAHARASHTRA STAGE-1-2016]

11. In a row of girls Seema and Reema are ninth from right and tenth from left respectively. If they
interchange their position then Seema and Reema are seventeenth from right and eighteenth from left
respectively. Find the total number of girls in the row.
(a) 27 (b) 26
(c) 25 (d) 20
[NTSE-MAHARASHTRA STAGE-1-2016]

12. Harshal is standing at the centre of a row of boys. Ujwal is eighth to the left of Harshal. Shubham is
standing twenty first to the right of Ujwal, but he is thirteenth to the left of Kishore. Find the total
number of boys standing in a row.
(a) 27 (b) 35
(c) 52 (d) 53
[NTSE-MAHARASHTRA STAGE-1-2015]

137 PACE IIT & MEDICAL: Mumbai / Delhi & NCR / Goa / Akola / Kolkata / Nashik / Pune / Bokaro / Dubai
NTSE-MENTAL ABILITY

13. ambccmamcbcabacamcmamcb. In the given alphabet series how many times ‘m’ is preceded by ‘a’
and succeeded by ‘c’.
(a) 6 (b) 3
(c) 5 (d) 4
[NTSE-MAHARASHTRA STAGE-1-2013]

14. Vivek is standing at the centre of a row of boys. Subhash is sixth to the right of Vivek. Niwas is
standing fifteenth to the left of Subhash. Niwas is standing eleventh to the right of Yogesh. Find the
total number of boys standing in a row.
(a) 38 (b) 41
(c) 40 (d) 42
[NTSE-MAHARASHTRA STAGE-1-2013]

15. In a class of eighth standard, among students passed in Maths, Rekhee stands serially 21st and 34th
from bottom. 4 students were absent and 9 students failed. If there are 37 boys in the class, find the
number of girls in the class.
(a) 30 (b) 55
(c) 68 (d) 69
[NTSE-MAHARASHTRA STAGE-1-2012]

16. In the given number series how many times ‘9’ is preceded by 6 and 6 is preceded by 9?
36996869888963968969
(a) 0 (b) 1
(c) 2 (d) 3
[NTSE-MAHARASHTRA STAGE-1-2012]

17. In the given alphabetic series how many time is k succeeded by 1 and h succeeded by k?
dbhfklthfbhkltbdbhtdbbdhk
(a) 3 (b) 2
(c) 1 (d) 0
[NTSE-MAHARASHTRA STAGE-1-2012]

Answer Key

4. 7.
1. (d) 2. (a) 3. (b) 5. (c) 6. (b) 8. (c) 9. (c) 10. (a)
(bonus) (Bonus)
11. (b) 12. (d) 13. (b) 14. (b) 15. (a) 16. (b)

138 PACE IIT & MEDICAL: Mumbai / Delhi & NCR / Goa / Akola / Kolkata / Nashik / Pune / Bokaro / Dubai
NTSE-MENTAL ABILITY

11. BLOOD RELATIONS

Problems on Blood Relations involve analysis of information showing blood relationship among members
of a family. In the questions, a chain of relationship is given in the form of information and on the basis of
these information relation between any two members of the chain is asked. Students are supposed to be
familiar with the knowledge of different relationship in a family.

Grandfather’s son Father or uncle


Grandmother’s son Father or uncle
Grandfather’s only son Father
Grandmother’s only son Father
Mother’s or Father’s mother Grandmother
Mother’s or Father’s father Grandfather
Grandfather’s only daughter-in-law Mother
Grandmother’s only daughter-in-law Mother
Mother’s or Father’s son Brother
Mother’s or Father’s daughter Sister
Mother’s or Father’s brother Uncle
Mother’s or Father’s sister Aunt
Husband’s or wife’s sister Sister-in-law
Husband’s or wife’s brother Brother-in-law
Son’s wife Daughter-in-law
Daughter’s husband Son-in-law
Brother’s son or Sister’s son Nephew
Brother’s daughter or Sister’s daughter Niece
uncle or Aunt’s son or daughter Cousin
Sister’s husband Brother-in-law
Brother’s wife Sister-in-law

Remark: A relation on the mother’s side is called ‘maternal’ while that on the father’s side is called
paternal’. Thus, mother’s brother is ‘maternal uncle’ while father’s brother is ‘paternal uncle’.

Note: To build a family tree, certain standard notations are used to indicate a relationship between the
members of the family.
‘+’ stand for male person.
' ' stand for female person.
' /  ' male or female person.

'  ' stand for married couple.

139 PACE IIT & MEDICAL: Mumbai / Delhi & NCR / Goa / Akola / Kolkata / Nashik / Pune / Bokaro / Dubai
NTSE-MENTAL ABILITY

Tree Diagram of Blood Assuming Your Self as Male

Tree Diagram of Blood Relation Assuming Yourself as Female

140 PACE IIT & MEDICAL: Mumbai / Delhi & NCR / Goa / Akola / Kolkata / Nashik / Pune / Bokaro / Dubai
NTSE-MENTAL ABILITY

Direct-Relationship
In these type of questions, around about description is given in the form of certain small relationship and
you required to analyses the whole chain of relations and decipher the direct relationship between the
persons concerned.

1. Pointing towards a man in the photograph, Archana said, “He is the son of the only son of my
grandmother”. How is that man related to Archana ?
(A) Cousin (B) Nephew (C) Brother (D) Son

Sol. (C) Only son of Archana’s grandfather means Archana’s father & his son is Archana’s brother.

2. Pointing to a photograph, a lady tells Amit, “I am the only daughter of this lady and her son is your
material uncle.” How is the speaker related to Amit’s father ?
(A) Sister-in-law (B) Wife (C) Either (A) or (B) (D) Neither (A) nor (B)

Sol. (B) The lady who is talking to Amit is the daughter of the lady in the photograph. The son of that
lady who is the brother of the lady who is talking to Amit. The brother of this lady is the maternal
uncle of Amit. The lady is the mother of Amit & wife of Amit’s Father.

Relation Puzzle
In these type of questions, mutual blood relations of more than two persons are mentioned. The candidate is
required to analysis the given information, work out a family chart and then answer the given questions.

3. Rohit and Rohan are brothers. Soniya and Sunita are sisters. Rohit’s son is Sunita’s brother. How
is Rohan related to Soniya.
(A) Father (B) Brother (C) Grand Father (D) Uncle

Sol. (D) Rohit’s son is Sunita’s brother means Rohit is Sunita’s father. Rohit and Rohan are brothers.
Sunita and Soniya are sisters. So, Rohan is the uncle of Soniya.

Directions : (4 to 6) P, Q, R, S, T, U, V & W are the family members. Q is the sister of V and V is the
brother of R. P whose’s father is W, is husband of T. S is the husband of Q and U is the son of V. P is the
father of Q.

4. How U is related with T ?


(A) Son (B) Mother (C) Grandson (D) Nephew

5. How S is related with R?


(A) Son (B) uncle (C) Brother-in-law (D) Brother

6. How W is related with R?


(A) Grandfather (B) uncle (C) Son (D) Brother

141 PACE IIT & MEDICAL: Mumbai / Delhi & NCR / Goa / Akola / Kolkata / Nashik / Pune / Bokaro / Dubai
NTSE-MENTAL ABILITY

4. (C) U is son of V and V is son of T.


 U is grandson of T.

5. (C) S is husband of Q and Q is sister of R.


 is brother -in- law of R.

6. (A) R is son of P and P is son of W.


 is grandfather of R.

EXERCISE
1. Amit said, “This girl is the wife of the grandson of my mother.” How is Amit related to the girl?
(a) Father (b) Father-in-law (c) Grandfather (d) Husband

2. Introducing a girl, Vipin said, “Her mother is the only daughter of my mother-in-law. “How is Vipin
related to the girl?
(a) Uncle (b) Father (c) Brother (d) Husband

3. Showing the lady in the park, Vineet said, “She is the daughter of my grand father’s only son”. Hos
is Vineet related to that lady?
(a) Brother (b) Cousin (c) Father (d) Uncle

4. Pointing to a man, Amit said, “His son is my son’s uncle”. How is that man related to Amit?
(a) Brother (b) Uncle (c) Father (d) Grandfather

5. Pointing to a man Asha said, “His brother’s father is the only son of my grand father”. How Asha is
related to the man.
(a) Mother (b) Aunt (c) Sister (d) Daughter

6. If P is the brother of the son of Q’s Son, how is P related to Q?


(a) Son (b) Brother (c) Cousin (d) Grandson

7. Pointing to a photograph, a man says to his friend, “She is the grand-daughter of the elder brother of
my father”. How is the girl in the photography related to the man?
(a) Niece (b) Sister (c) Aunt (d) Sister-in-law

8. Pointing to a girl in the photography, Amar said, “Her mother’s brother is the only son of my
mother’s father’. How is the girl’s mother related to Amar?
(a) Mother (b) Sister (c) Sister-in-law (d) Grandmother

9. T is the son of P.S is the son of Q. T is married to R. R is Q’s daughter. How is S related to T?
(a) Brother (b) Uncle (c) Father-in-law (d) Brother-in-law

Directions: (10 to 12) Read the following information and answer the questions given below:
A is the father of B who is the sister of C, C is the husband of P and L is mother of C.

10. How P is related to L?


(a) Daughter-in-law (b) Sister-in-law (c) Mother-in-law (d) Daughter

11. How B is related to L?


(a) Daughter (b) Sister (c) Son (d) Brother
142 PACE IIT & MEDICAL: Mumbai / Delhi & NCR / Goa / Akola / Kolkata / Nashik / Pune / Bokaro / Dubai
NTSE-MENTAL ABILITY

12. How A is related to L?


(a) Wife (b) Father (c) Husband (d) Son

13. If Nishant is the brother of Brijesh, Mitali is the sister of Nishant, Jai is the brother of Purnima,
Purnima is the daughter of Brijesh. Who is the uncle of Jai?
(a) Brijesh (b) Mitali (c) Nishant (d) Purnima

14. If A is B’s brother, B is C’s sister and C is D’s father then D is A’s.
(a) Brother (b) Sister (c) Nephew (d) Data inadequate

Directions: (15 to 18) Read the following information carefully and answer the questions given below:
There are six children playing football namely A, B, C, D, E and F. A and E are brothers. F is the
sister of E. C is the only one of A’s uncle. B and D are the daughters of C’s father.

15. How is C related to F?


(a) Cousin (b) Brother (c) Son (d) Uncle

16. How many male players are there?


(a) One (b) Three (c) Five (d) Six

17. How many female players are there?


(a) Two (b) Three (c) Five (d) One

18. How is D related to A?


(a) Uncle (b) Sister (c) Niece (d) Cousin

19. If P $ Q means P is the father of Q, P # Q means P is mother of Q and P * Q means P is the sister of
Q. Then how Q related to N if N # L $ P * Q.
(a) Grandson (b) Granddaughter (c) Nephew (d) Data inadequate

20. S – T means S is the mother of T, S  T means S is the father of T, S  T means S is the brother of T.
Which of the following represents M is the son of Q?
(a) M  Q  R (b) M  Q  R (c) Q  M  R (d) Q  M  R

Directions: (21 to 24) Study the following information to answer the given questions.
A + B means A is father of B.
A – B means A is wife of B.
A  B means A is brother of B.
A  B means A is daughter of B.

21. If P  R  S  Q, which of the following is true?


(a) P is daughter of Q (b) Q is aunt of P (c) P is aunt of Q (d) P is mother of Q

22. If P  R  Q, which of the following is true


(a) P is mother of Q (b) Q is daughter of P (c) P is aunt of Q (d) P is sister of Q

23. If P  R  Q, which of the following is true?


(a) P is uncle of Q (b) P is father of Q (c) P is brother of Q (d) P is son of Q

143 PACE IIT & MEDICAL: Mumbai / Delhi & NCR / Goa / Akola / Kolkata / Nashik / Pune / Bokaro / Dubai
NTSE-MENTAL ABILITY

24. If P  R  Q, which of the following is true


(a) P is brother in law of Q (b) P is brother of Q
(c) P is uncle of Q (d) P is father of Q

25. Sita introducing a man to her husband Ram, told “Grand father of his son is only a son of my
Grandfather” how Sita is related to that man. (NTSE Stage-I/Raj./2007)
(a) Daughter-in-law (b) Sister (c) Daughter (d) Mother-in-law

26. Introducing Radha, Kalpana said “her mother is alone daughter of my mother”. How Kalpana is
related to Radha? (NTSE Stage-I/Raj./2007)
(a) Mother (b) Sister (c) Niece (d) Daughter

27. If A + B means A is the brother of B, A – B means A is the father of B, A  B means A is the wife of
B, then P  R  S  T means (NTSE Stage-II, 2007)
(a) P is the mother of T (b) T is the sister of P
(c) R is the grand father of T (d) T and R are sisters

28. If M is Father of N, L is Brother of M and P is Mother of L, then what is the relation of N with P?
(NTSE Stage-I/Raj./2008)
(a) Grand - Daughter (b) Grand-Son (c) Father (d) Data inadequate

29. Shanker is father of ‘B’ but ‘B’ is not son of Shanker then what is the relationship of ‘B’ to Shanker?
(NTSE Stage-I/Raj./2012)
(a) Grand daughter (b) Mother (c) Daughter (d) Grand Son
30. Sachin is the brother of the son of Ajit’s son. Then what is the relationship of Sachin to Ajit?
(NTSE Stage-I/Raj./2012)
(a) Brother (b) Cousin (c) Nephew (d) Grandson

31. A and B are brother. C and D are sisters. A’s son in D’s brother. How is B related to C?
(NTSE Stage-I/Raj./2013)
(a) Brother (b) Father (c) Uncle (d) Son

Directions: (32) Read the following information carefully and answer the question given below
A + B means A is the daughter of B;
A – B means A is the husband of B;
A  B means A is the brother of B (NTSE Stage-I/Raj./2013)

32. If P  Q  R, which one of the following is true?


(a) R is the mother of P (b) R is the sister-in-law of P
(c) R is the aunt of P (d) R is the mother-in-law of P

33. Ramesh is father of Mohan and Shyam is son of Mohan. What is the relation between Ramesh and
Shyam? (NTSE Stage-I/Raj./2014)
(a) Son (b) Brother (c) Grandfather (d) Father

34. B is father of C but C is not son of B. What is the relation of C with B? (NTSE Stage-I/Raj./2014)
(a) Sister (b) Brother (c) Daughter (d) Nephew

35. A and B are brothers, C and D are sisters. The son of A is brother of D. Then the relation of B with C
is:
(a) Husband (b) Brother (c) Uncle (d) Nephew
144 PACE IIT & MEDICAL: Mumbai / Delhi & NCR / Goa / Akola / Kolkata / Nashik / Pune / Bokaro / Dubai
NTSE-MENTAL ABILITY

36. A is uncle of B, B is daughter of C, C is the wife of D’s son. Then how is A related to D?
(NTSE Stage-I/Raj./2016)
(a) Son (b) Brother (c) Father (d) Maternal uncle

37. Sailesh introduces Mahipal as the son of the only brother of his father’s wife. How is Mahipal related
to Sailesh? (NTSE Stage-I/Raj./2017)
(a) Cousin (b) Son (c) Maternal uncle (d) Son-in-law

Answer Key

1. (b) 2. (b) 3. (a) 4. (c) 5. (c) 6. (d) 7. (a) 8. (a) 9. (d) 10. (a)
11. (a) 12. (c) 13. (c) 14. (d) 15. (a) 16. (b) 17. (b) 18. (d) 19. (d) 20. (c)

21. (c) 22. (a) 23. (d) 24. (a) 25. (b) 26. (a) 27. (a) 28. (d) 29. (c) 30. (d)

31. (c) 32. (a) 33. (c) 34. (c) 35. (c) 36. (a) 37. (a)

145 PACE IIT & MEDICAL: Mumbai / Delhi & NCR / Goa / Akola / Kolkata / Nashik / Pune / Bokaro / Dubai
NTSE-MENTAL ABILITY

12. PUZZLE TEST

This chapter comprises of questions given in the form of puzzles, involving certain number of items, persons
or things. You are required to analyses the given information of clue and answer the questions accordingly.

Hints for Handling the Questions:


 Generally, several conditions in the form of information are given with the question. So, do not make
hurry to mix all the given information, instead go step by step.

 To avoid confusion while solving such questions, you should symbolize persons, items by dot, lines
etc.

Classification Type Puzzles


This type consists of questions in which certain items belonging to different groups or possessing
different qualities are given along with some clues with the help of which the candidate is required to
group and analyses the given items and answer the questions accordingly.

Directions : (1 to 3) Read following information carefully and answer the questions given below it :
(i) A and B are good in Biology & Chemistry.
(ii) A & C are good in Biology & Physics.
(iii) C, D & E are good in Physics & History.
(iv) C & E are good in Physics & Mathematic.
(v) D & B are good in Chemistry & History.

1. Who is good in Physics, History & Mathematics but not in Biology?


(A) D (B) C (C) A (D) E

2. Who is good in Physics, History, Mathematics & Biology?


(A) C (B) E (C) D (D) B

3. Who is good in Physics, Chemistry & History?


(A) C (B) E (C) D (D) B

Sol. (1 to 3) By given following table :

1. (D) Clearly, E is good in Physics, History & Mathematics but not in Biology.

2. (A) Clearly, C is good ¡n Physics, History, Mathematics & Biology

3 (C) Clearly, D is good in Physics, Chemistry & History.

146 PACE IIT & MEDICAL: Mumbai / Delhi & NCR / Goa / Akola / Kolkata / Nashik / Pune / Bokaro / Dubai
NTSE-MENTAL ABILITY

Family Based Puzzles


In these type of questions, some clause are given regarding relationship among different members of a
family, together with their professions, qualities, dresses, preferences etc. The candidate is required to
analyses the whole information and then answer the given questions accordingly.

Directions : (4 to 5) Read the given information carefully and answer the questions that follow :
Ratan, Anil, Pinku and Gaurav are brothers of Rakhi, Sangeeta, Pooja and Saroj, not necessarily in that
order. Each boy has one sister and the names of brothers and sisters do not begin with the same letter. Pinku
and Gaurav are not Saroj’s or Sangeeta’s brothers. Saroj is not Ratan’s sister.

4. Pooja’s brother is ?
(A) Ratan (B) Anil (C) Pinku (D) Gaurav

5. Which of the following are brother and sister?


(A) Ratan and Pooja (B) Anil and Saroj
(C) Pinku and Sangeeta (D) Gaurav and Rakhi

Sol. : (4 to 5) As given that the names of brothers and sisters do not begin with the same letter and Pinku
and Gaurav are not Saroj or Sangeeta’s brothers, Pinku cannot be the brother of Pooja and Hence
he is the brother of Rakhi.
Now we have that Gaurav cannot be the brother of Saroj, Sangeeta or Rakhi. Therefore Gaurav is the
brother of Pooja. As given that Saroj is not Ratan’s sister and Rakhi and Pooja can also not be the
sister’s of Ratan (From above conclusions), Ratan is the brother of Sangeeta. Anil will have to be the
brother of saroj as this is the only valid combination left. Therefore, we have this table finally.

Brother Sister
Pinku Rakhi
Gaurav Pooja
Ratan Sangeeta
Anil Saroj

4. (D) Clearly, Pooja’s brother is Gaurav.

5. (B) Clearly, Anil and Saroj are brother and sister.

Directions : (6 to 10)
Read the following information carefully and answer the questions given below it:
There are five men A, B, C, D and E and six women P, Q, R, S, T and U. A, B and R are advocates; C, D,P,
Q and S are doctors and the rest are teachers. Some teams are to be selected from amongst these eleven
persons subject to the following conditions :
A, P and U have to be together.
B cannot go with D or R.
E and Q have to be together.
C and T have to be together.
D and P cannot go together.
C cannot go with Q.

6. If the team is to consist of two male advocates, two lady doctors and one teacher, the members
(A) A B P Q U (B) A B P U S (C) A P R S U (D) B E Q R S

Sol. (B)
147 PACE IIT & MEDICAL: Mumbai / Delhi & NCR / Goa / Akola / Kolkata / Nashik / Pune / Bokaro / Dubai
NTSE-MENTAL ABILITY

The male advocates are A and B, lady doctors are P, Q and S ; teachers are E, T and U.
Now, A and B will be selected. A, P and U have to be together. Now, we have to select one lady
doctor more. It can be Q or S. But Q and E have to be together. Since E is not selected, so S will be
selected. Thus, the team is ABPUS

7. If the team is to consist of one advocate, two doctors, three teachers and C may not go with T,
(A) A E P Q S U (B) A E P Q T U (C) B E Q S T U (D) E Q R S T U
Sol. (B)
The advocates are A, B and R ; doctors are C, D, P, Q, S ; teachers are E, T and U. The team consists
of 3 teachers i.e. E, T,U . Now A, P and U have to be together. E and Q have to be together, E and Q
have to be together. Thus, the team is A E P Q T U

8. If the team is to consist of one male advocate, one male doctor, one lady doctor and two teachers the
members of the team are :
(A)A E P Q S U (B)A E P Q T U (C) A D E PU (D) B C E Q U

Sol. (A)
The male advocates are A and B; male doctors are C and D; lady doctors are P,Q and S; teachers are
E,T and U. If A is selected, P and U will be selected. D and P cannot go together. So, a male doctor
C will be selected C and T have to be together. Thus, the term is A C P T U. If B is selected, D will
not be selected. So, male doctor C will be chosen. C and T have to be together. Now, the second
teacher to be selected is E or U. But, U cannot go without A. So, E will be selected. E and Q have to
be together. Thus, the team can also be B C E Q T

9. If the team is to consist of one advocate, three doctors and one male teacher, the members of the
team are:
(A)A D P S U (B) C D R S T (C) D E Q R S (D) D E Q R T

Sol. (C) The advocates are A, B and R ; the doctors are C, D, P, Q and S ; male teacher is E. Clearly, E
will be selected. E and Q have to be together. C and Q cannot be together. So, C will not be selected.
P also cannot be selected because U is not selected. So, two other doctors D and S will be selected. P
is not selected, so A will not be selected. D is selected, so B cannot be selected.
Thus, the team is D E Q R S.

10. If the team is to consist of two advocates, doctors, two teachers and not more than three ladies, the
members of the team are :
(A) A B C PT U (B) A C P R T U (C) A E P Q R T (D) B C E Q R T

Sol. (A) A C P R T U and A E P Q R T are wrong because each of these combinations consists of four
ladies. B C E Q R T is incorrect because B and R cannot go together.

11. A vagabond runs out of cigarettes. He searches for the stubs, having learnt that 7 stubs can make a
new cigarette, good enough to be smoked, he gathers 49 stubs, If he smokes 1 cigarette every three -
quarters of an hour, how long will his supply last ?
(A) 5.25 hr (B) 6 hr (C) 4.5 hr (D) 3 hr

49
Sol. (B) He has got   7 cigarettes.
7
3
 The duration of time he will take to smoke these 7 cigarette  7  hr  5.25 hr  i.e.5hr and15 min 
4
Now note that after he has smoked these 7 cigarettes, he will collect 7 more stubs( one form each),
148 PACE IIT & MEDICAL: Mumbai / Delhi & NCR / Goa / Akola / Kolkata / Nashik / Pune / Bokaro / Dubai
NTSE-MENTAL ABILITY

3
form which he will be able to make another cigarette. This will take him another hr (45 min) to
4
smoke. Therefore, total time taken  6hr

Directions: (12 to 13) Read the following information and answer the questions that follow.
There are 70 clerks working with M/s. Jha Lal Khanna & Co. chartered accountants, of which 30 are
female.
(i) 30 clerks are married.
(ii) 24 clerks are above 25 years of age
(iii) 19 Married clerks are above 25 years of age among them 7 are males.
(iv) 12 males are above 25 years of age
(v) 15 males are married.

12. How many unmarried girls are there ?


(A) 12 (B) 15 (C) 18 (D) 10

13. How many of these unmarried girls are above 25?


(A) 12 (B) 15 (C) 4 (D) 0

Sol. (12 to 13): From the given data, we can make the following table with the help of which rest of the
questions can be solved very easily.

Male(40) Female(30)
Above 25
Married 7 12
Unmarried 5 0
Below 25
married 8 3
unmarried 20 15
Total 40 30

12. There are 15 unmarried girls.

13. In these 15 unmarried girls no one is above 25.

149 PACE IIT & MEDICAL: Mumbai / Delhi & NCR / Goa / Akola / Kolkata / Nashik / Pune / Bokaro / Dubai
NTSE-MENTAL ABILITY

EXERCISE

Directions : (1 to 5) Five students A, B, C) D & E of a class secured distinction in four subjects English,
Science, Maths and Sanskrit as: A and C secured distinction in English and Science. B, C and E secured
distinction in Science and Sanskrit) while A, D and E Secured distinction in English and Maths then find out
the correct alternatives for each question. (NTSE Stage-I/Raj./2007)

1. The student who secured distinction in all the four subjects is


(A) A (B) E (C) C (D) D

2. The students who secured distinction in English, Science and Maths are
(A) A, B (B) D, E (C) A, E (D) C, D

3. The Student who secured distinction in three subjects except Maths is


(A) A (B) B (C) C (D) D

4. The Student who secured distinction ¡n Sanskrit and Science but not in Maths and English Is
(A) B (B) C (C) D (D) E

5. The Student who secured distinction in only English and Maths is


(A) C (B) B (C) D (D) A

Directions: (6 to 10) Road the following information carefully and answer the questions given below it.
(i) Five professors (Dr. Joshi, Dr. Davar, Dr. Natrajan, Dr. Choudhary and Dr. Zia) teach five different
subjects (zoology, physics, botany, geology and history) in four universities (Delhi, Gujarat, Mumbai
and Osmania). Do not assume any specific order.
(ii) Dr. Choudhary teaches zoology in Mumbai University.
(iii) Dr. Natrajan is neither in Osmania University nor in Delhi University and he teaches neither geology
nor history.
(iv) Dr. Zia teaches physics but neither in Mumbai University nor in Osmania University.
(v) Dr. Joshi teaches history in Delhi University.
(vi) Two professors are from Gujarat University.
(vii) One professor teaches only one subject and in one University only.

6. Who teaches geology?


(A) Dr Natrajan (B) Dr. Zia (C) Dr. Davar (D) Dr. Joshi

7. Which university is Dr. Zia from?


(A) Gujarat (B) Mumbai (C) Delhi (D) Osmania

8. Who teaches botany?


(A) Dr. Zia (B) Dr. Davar (C) Dr. Joshi (D) Dr. Natrajan

9. Who is from Osmania University?


(A) Dr. Natrajan (B) Dr. Davar (C) Dr. Joshi (D) Dr. Zia

10. Which of the following combinations is correct?


(A) Delhi University - Dr. Zia (B) Dr. Choudhary – geology
(C) Dr. Davar - Mumbai University (D) Dr. Natranjan - Gujarat University

150 PACE IIT & MEDICAL: Mumbai / Delhi & NCR / Goa / Akola / Kolkata / Nashik / Pune / Bokaro / Dubai
NTSE-MENTAL ABILITY

Directions : (11 to 15) Study the following information carefully and answer the question given below it:
(i) A B,C, D, E and F are six members in a family in which there are two married couples.
(ii) E, a teacher is married to the doctor who is C and F.
(iii) B, the lawyer is married to A
(iv) A has one son and one grandson.
(v) Of the two married ladies one is a housewife.
(vi) There is also one student and one male Engineer in the family.

11. How is A related to C?


(A) Grand-father (B) Mother (C) Sister (D) Grand mother

12. Who among the following is the housewife?


(A) A (B) B (C) D (D) None of these

13. How is C related to F?


(A) Brother (B) Sister (C) Brother or Sister (D) None of these

14. Which of the following represents the group of females in the family ?
(A) ADC (B) ADF (C) BEC (D) Data inadequate

15. Which of the following Is true about the grand daughter in the family ?
(A) She is a lawyer (B) She is a student
(C) She is an engineer (D) None of these

Directions : (16 to 20) Study the following information carefully and answer the questions given below it:
There are five friends A, B, C, D and E. Two of them are businessmen while the other three belong to
different occupations viz. medical, engineer and legal. One businessman and the lawyer stay in the same
locality S, while the other three stay in three different localities P, Q and R. Two of these five persons are
Hindus while the remaining three come from three different communities viz. Muslim, Christian and Shikh.
The lawyer is the oldest in age while one of the businessmen who runs a factory is the youngest. The other
businessman is a cloth merchant and age wise lies between the doctor and the lawyer. D is a cloth merchant
and stays in locality S while E is a Muslim and stays in locality R. The doctor is a Christian and stays in
locality P, B is a Shikh while A is a Hindu and runs a factory.

16. Who stays in locality Q?


(A) A (B) B (C) C (D) E

17. What is E’s occupation ?


(A) Business (B) Engineer (C) Lawyer (D) Doctor

18. Agewise who among the following lies between A and C?


(A) Lawyer (B) Doctor (C) Cloth merchant (D) Engineer

19. What is B’s occupation ?


(A) Business (B) Engineer (C) Lawyer (D) Doctor

20. What is C’s occupation?


(A) Doctor (B) Lawyer (C) Engineer (D) Business

151 PACE IIT & MEDICAL: Mumbai / Delhi & NCR / Goa / Akola / Kolkata / Nashik / Pune / Bokaro / Dubai
NTSE-MENTAL ABILITY

Directions : (21 to 25) Read the following information carefully and answer the questions that follow.
I. There are six students (A, B, C, D, E and F) n a group. Each student can opt for only three choices
out of the six which are music, reading, Painting, badminton, cricket and tennis.
Il. A, C and F like reading.
III. D does not like badminton, but likes music.
IV. Both B and E like painting and music.
V. A and D do not like painting, but they like cricket.
VI. All students except one like badminton.
VII. Two students like tennis.
VIII. F does not like cricket, music and tennis.

21. Which pair of students has the same combination of choices ?


(A) A and C (B) C and D (C) B and E (D) D

22. Who among the following students likes both tennis and cricket?
(A) A and B (B) C (C) B and D (D) D and F

23. How many students like painting and badminton?


(A) 1 (B) 2 (C) 3 (D) 4

24. Who among the following do not like music?


(A) A, C and D (B) A, B and C (C) A, C and F (D) B, D and F

25. Which of the following is the most popular choice?


(A) Tennis (B) Badminton (C) Reading (D) Painting

Directions: (26 to 30) Next questions are based upon the information given below. Study the information
carefully and then choose the correct alternative. A, B, C, D, E and F are members of a family. There are
two married couples in the family. B is a businessman and father of E. F is a teacher and grandfather of C. D
is grandmother of E and she is a domestic lady. C is daughter of A. In the family there are two students, one
businessmen, one teacher, one domestic lady and one engineer. (NTSE Stage-II, 2009)

26. Which of the following are married couples?


(A) FD, AB (B) FC, AB (C) EC, BF (D) FD, AC

27. What is the profession of A?


(A) Student (B) Teacher (C) Engineer (D) Domestic lady

28. Who is the husband of A?


(A) D (B) C (C) E (D) B

29. Which one among the following is the group of male persons ?
(A) EF (B) BFE (C) BF (D) ADE

30. Who is the sister of E?


(A) C (B) F (C) B (D) A

152 PACE IIT & MEDICAL: Mumbai / Delhi & NCR / Goa / Akola / Kolkata / Nashik / Pune / Bokaro / Dubai
NTSE-MENTAL ABILITY

Directions : (31 to 34) Read the following information carefully and answer the questions based on it by
selecting the correct option from the given alternatives.
Five students K, L. M. N and O study in a class of these-
(i) K and L study Physics and Chemistry
(ii) M and L study Physics and Mathematics
(iii) N and K study Biology and Chemistry
(iv) O and L study Anthropology and Civics
(v) Q and N study Chemistry and Mathematics (NTSE Stage-II, 2011)

31. Who among the students studies maximum number of subjects?


(A) O (B) N (C) L (D) K

32. Who among the students studies minimum number of subjects?


(A) O (B) N (C) K (D) M

33. Who among the students studies only four subjects ?


(A) M (B) O (C) N (D) K

34. Which of the following pairs studies Chemistry and Civics?


(A) K and N (B) L and M (C) L and O (D) M and N

Directions : (35 to 39) Those questions based on following Information.


Rajesh, Sudhir and Mohan play football, hockey and cricket Rajesh, Rakesh and Mohan play hockey, cricket
and Basketball, Rajesh. Sudhir, Naresh and Mohan play football arid cricket. (NTSE Stage-II, 2011)

35. Which two boys play all the games?


(A) Rajesh and Sudhir (B) Rajesh and Rakesh
(C) Sudhir and Mohan (D) Rajesh and Mohan

36. Which game is played by all the boys?


(A) Basket Ball (B) Cricket (C) Football (D) Hockey

37. Who does not play football?


(A) Rakesh (B) Sudhir (C) Naresh (D) Mohan

38. Which two games are not played by Naresh?


(A) Cricket and Football (B) Cricket and hockey
(C) Hockey and Basketball (D) Football and Basketball

39. Who do not play basketball?


(A) Rajesh and Mohan (B) Sudhir and Naresh
(C) Rakesh and Rajesh (D) Mohan and Rakesh

Directions : (40 to 44) A, B, C, D, E and F are members of a family. Amongst them there are lawyer,
doctor, teacher, salesman, engineer and an accountant. There are two married couples in the family. ‘D’ who
is a salesman is married to the lady teacher. Doctor is married to the lawyer. ‘F’ who is an accountant is son
of ‘B’ and also brother of ‘E’. ‘C’ who is lawyer is daughter in law of ‘A. E is an unmarried engineer. ‘A’ is
grandmother of ‘F’. (NTSE Stage-II, 2011)

40. What is the profession of B?


(A) Salesman (B) Doctor (C) Lawyer (D) Teacher

153 PACE IIT & MEDICAL: Mumbai / Delhi & NCR / Goa / Akola / Kolkata / Nashik / Pune / Bokaro / Dubai
NTSE-MENTAL ABILITY

41. What is the profession of A?


(A) Teacher (B) Doctor (D) Lawyer (D) Engineer

42. What is the relation of B with D?


(A) Brother (B) Grandson (C) Son (D) Father

43. What is the relation of D with F?


(A) Husband (B) Brother (C) Father (D) Grandfather

44. Which of the following is a married couple?


(A) C and D (B) A and B (C) B and C (D) D and B

Directions : (45 to 49) Read the following paragraph carefully and choose the correct alternative.
The office staff of XYZ corporation presently consists of three females A, B, C and five males
D, E, F, G and H. The management is planning to open a new office in another city using three males
and two females of the present staff. To do so they plan to separate certain individuals who do
not function well together. The following guidelines were established
I. Females A and C are not to be together
II. C and E should be separated
III. D and G should be separated
IV. D and F should not be part of a team.

45. If A is chosen to be moved, which of the following cannot be a team?


(A) ABDEH (B) ABDGH (C) ABEFH (D) ABEGH
46. If C and F are to be moved to the new office, how many combinations are possible?
(A) 1 (B) 2 (C) 3 (D) 4
47. If C is chosen to the new office, which number of the staff cannot be chosen to go with C?
(A) B (B) D (C) F (D) G

48. Under the guidelines, which of the following must be chosen to go to the new office?
(A) B (B) D (C) E (D) G

49. If D goes to the new office, which of the following is/are true ?
I. C cannot be chosen
II. A cannot be chosen
III. H must be chosen
(A) I only (B) II only (C) I and II only (D) I and III only

Directions : (50 to 54) Study the following information carefully and answer the questions that follow :
A team of five is to be selected from amongst five boys A, B, C, D and E and four girls P, Q, R and S.
Some criteria for selection are :
A and S have to be together
P cannot be put with R
D and Q cannot go together
C and E have to be together.
R cannot be put with B.
Unless otherwise stated, these criteria are applicable to all the questions below :

50. If two of the members have to be boys, the team will consist of :
(A) A B S P Q (B) A D S Q R (C) B D S R Q (D) C E S P Q

154 PACE IIT & MEDICAL: Mumbai / Delhi & NCR / Goa / Akola / Kolkata / Nashik / Pune / Bokaro / Dubai
NTSE-MENTAL ABILITY

51. If R be one of the members, the other members of the team are :
(A) P S A D (B) Q S A D (C) Q S C E (D) S A C E

52. If two of the members are girls and D is one of the members, the members of the team other than D
are :
(A) P Q B C (B) P Q C E (C) P S A B (D) P S C E

53. If A and C are members, the other members of the team cannot be :
(A) B E S (B) D E S (C) E S P (D) P Q E

54. If including P at least three members are girls, the members of the team other than P are:
(A) Q S A B (B) Q S B D (C) Q S C E (D) R S A D

Direction : (55) The ages of Mandar, Shivku, Pawan and Chandra are 32, 21, 35 and 29 years, not in order,
whenever asked they lie of their own age but tell the truth about others.
(i) Pawan says, “My age is 32 and Mandar’s age is not 35”
(ii) Shivku says, “My age is not 29 and Pawan’s age in not 21”
(iii) Mandar says, My age is 32.

55. What is Chandra’s age?


(A) 32 years (B) 35 years (C) 29 years (D) 21 years

56. You have 8 similar looking coins. 7 of them weigh the same. One of them is less in weight. You
have a scale. You can put coins on both sides of the scale and it’ll tell you which side is heavier or
will stay in the middle if both sides weigh the same. What is the minimum number of weighing
required to find out the odd coin.
(A) 2 (B) 3 (C) 4 (D) 6

57. Two players X and O play a game of “doughts and crosses” on a 3  3 grid. The purpose of the game
is for a player to get 3 symbols belonging to the player in a straight line (vertically, horizontally or
diagonally). Each player marks one symbol on his or her turn.
After two moves, (1 turn each), the grid looks as follows with X to play next.
Where should X put his symbol next so that he will always win this game finally regard less of how
well O plays ? (NTSE Stage-II, 2013)

(A) Bottom row right corner


(B) Bottom row middle cell
(C) Middle row left most cell.
(D) It is not possible to always ensure X wins if O plays carefully

155 PACE IIT & MEDICAL: Mumbai / Delhi & NCR / Goa / Akola / Kolkata / Nashik / Pune / Bokaro / Dubai
NTSE-MENTAL ABILITY

58. An electrical circuit for a set of 4 lights depends on a system of switches A, B, C and D. When these
switches work they have the following effect on the lights. They each change the state of two lights
(i.e. on becomes off and off becomes on). The lights that each switch controls are as follows.

In configuration I shown below, switches CBDA are activated in turn, resulting in configuration 2.
One switch did not work and had no effect at all. Which was that switch?

(NTSE Stage-II, 2013)


(A) A (B) B (C) C (D) D

Direction (Question 59-63) : A, B, C, E, F, G and H are seven employees in an organisation working in the
departments of administration, Accounts and Operations. There are at least two employees in each
department. There are three females, one in each department. Each of seven employees earns different
amount. The only bearded employee F Works in administration and his only other colleague G earns the
maximum. C, the least earner works in accounts. B and E are brothers and do not work in the same
department. A, husband of H. works in Accounts and earns more than each of F, B and E. The wife in the
couple earns more than the husband. (NTSE Stage-II, 2015)

59. Which of the following is a group of females:


(A) GCE (B) GEH (C) GCH (D) GHB

60. In which department(s) do three people work?


(A) Operations (B) Accounts
(C) Operation or Account (D) Data inadequate

61. What will be position of A from the top when they are arranged in descending order of their income?
(A) Second (B) Third (C) Fourth (D) Fifth

62. In which of the following department does B work?


(A) Operations (B) Accounts (C) Administration (D) Data inadequate

63. Which of the following statements is definitely true?


(A) B earns less than F and H (B) F earns more than B and E
(C) B earns more than E and C (D) B earns less than A and H

156 PACE IIT & MEDICAL: Mumbai / Delhi & NCR / Goa / Akola / Kolkata / Nashik / Pune / Bokaro / Dubai
NTSE-MENTAL ABILITY

Answer Key

1. (b) 2. (c) 3. (c) 4. (a) 5. (c) 6. (c) 7. (a) 8. (d) 9. (b) 10. (d)
11. (d) 12. (a) 13. (c) 14. (d) 15. (b) 16. (a) 17. (b) 18. (d) 19. (c) 20. (a)

21. (c) 22. (d) 23. (c) 24. (c) 25. (b) 26. (a) 27. (c) 28. (d) 29. (c) 30. (a)

31. (c) 32. (d) 33. (b) 34. (c) 35. (d) 36. (b) 37. (a) 38. (c) 39. (b) 40. (b)

41. (a) 42. (c) 43. (d) 44. (c) 45. (b) 46. (a) 47. (b) 48. (a) 49. (d) 50. (a)

51. (d) 52. (c) 53. (d) 54. (a) 55. (a) 56. (a) 57. (a) 58. (c) 59. (c) 60. (b)

61. (b) 62. (d) 63. (d)

PREVIOUS YEAR QUESTIONS

Q.1 to 3: Directions: Sushil, Vipin, Prashant, Amar are four class friends. Sushil does not like to dance.
Vipin likes only music and dance. Only three of them like dance and craft. Prashant likes all subjects except
music. Sushil is a master in drawing and music.
1. Amar likes which subjects?
(a) music and craft (b) dance and drawing
(c) dance and craft (d) music and drawing

2. Which subject Vipin, Prashant and Amar likes?


(a) drawing (b) music
(c) dance (d) craft

3. Who likes drawing?


(a) Sushil and Vipin (b) Vipin and Prashant
(c) Sushil and Prashant (d) Prashant and Amar
[NTSE-MAHARASHTRA STAGE-1-2020]

Q.4 and 6: Direction : Atul, Tushar, Nishant and Amar are four players. Except Nishant all play cricket.
Atul plays only cricket and football. Only three players play football.Tushar plays all the games except kho-
kho.Only one player does not play kabaddi. Only Nishant does not play football. Nishant and Amar are
in Kho-Kho.

4. Which game Tushar, Nishat and Amar play?


(a) Kabaddi (b) Kho-Kho
(c) Cricket (d) Football

5. Who plays all the games?


(a) Atul (b) Tushar
(c) Nishant (d) Amar

6. Which game is played by only two players?


(a) Cricket (b) Kabaddi
(c) Football (d) Kho-Kho
[NTSE-MAHARASHTRA STAGE-1-2019]

157 PACE IIT & MEDICAL: Mumbai / Delhi & NCR / Goa / Akola / Kolkata / Nashik / Pune / Bokaro / Dubai
NTSE-MENTAL ABILITY

Q.7 to 9: Direction: Meena, Sarika, Geeta and Neeta are four friends. They like different flowers. Geeta and
Sarika like Marigold. Except Meena all the friends like ‘Jai’ Meena and Sarika like ‘Champak’ and “Mogra’
Geeta likes all the flowers except ‘Mogra.’ Neeta does not like only ‘Marigold’ Meena likes only two types
of flowers then

7. Who likes all types of flowers?


(a) Neeta (b) Sarika
(c) Meena (d) Geeta

8. Name the type of flower, that all the friends like?


(a) Marigold (b) Mogra
(c) Champak (d) Jui

9. Who do not like ‘Marigold’?


(a) Meena and Sarika (b) Meena and Geeta
(c) Meena and Neeta (d) Sarika and Neeta
[NTSE-MAHARASHTRA STAGE-1-2018]

Q.10 and 11: Directions- Madhav and Govind play Hockey and Volleyball. Hemant and Madhav play
Hockey and Baseball. Ramesh and Govind play Cricket and Volleyball. Hemant, Ramesh and Anant play
Football and Baseball. Then, answer the following questions.

10. Who plays Hockey, Cricket and Volleyball?


(a) Madhav (b) Govind
(c) Hemant (d) Anant

11. Who does not play Baseball? Choose the correct alternative.
(a) Govind (b) Hemant
(c) Madhav (d) Ramesh
[NTSE-MAHARASHTRA STAGE-1-2017]

Q.12 and 13  Directions:-Answer the following questions based on the information given below:
(1) Bhanudas, Gopal, Amar, Akhil and Chaitanya each practice one of the professions farming, Lawyer,
Doctor, Teacher, Photographer.
(2) Akhil who is richer than Amar and Chaitanya is a lawyer.
(3) Farmer is the richest of all.
(4) Occupation of the poorest is photography.
(5) Amar is a doctor and Gopal is a photographer.
(6) Akhil is more richer than Amar but less richer than Bhanudas.
Then

12. What is the occupation of Chaitanya?


(a) Lawyer (b) Doctor
(c) Farmer (d) Teacher

13. Who among the following is farmer?


(a) Akhil (b) Bhanudas
(c) Gopal (d) Chaitanya
[NTSE-MAHARASHTRA STAGE-1-2016]

158 PACE IIT & MEDICAL: Mumbai / Delhi & NCR / Goa / Akola / Kolkata / Nashik / Pune / Bokaro / Dubai
NTSE-MENTAL ABILITY

Q.14 and 15  Directions : -From the information given below, answer the following questions:
(1) A painter knows Hindi.
(2) A farmer, an advocate and a teacher can speak English fluently.
(3) Except the teacher, the other three know Gujarati.
(4) Out of Marathi and Hindi, The advocate and the farmer speak only Marathi, but the teacher can
speak both the languages.

14. Who know Marathi as well as Gujarati?


(a) Painter and the advocate (b) The farmer and the teacher
(c) The advocate and the farmer (d) The teacher and the painter

15. Who knows Hindi but not English?


(a) The farmer (b) The teacher
(c) The advocate (d) The painter
[NTSE-MAHARASHTRA STAGE-1-2015]

Q16 and 17  Directions:- The information about the students wearing T-shirts having numbers 7, 8, 9 and
10 given below. Read the given information and answer the following questions.
(i) The players having number 7 and 9 play Football as well as Cricket
(ii) The fat player having number 8 participates in all the games except Kho-Kho and Football.
(iii) The player carrying number 9 is very short. He is expert in Kabaddi. The player having number 10 is
very slim. He participates in all the games except Football.
(iv) The player having number 7 is very tall.

16. Which game can be played by one and only one player?
(a) Kabaddi (b) Kho-Kho
(c) Cricket (d) Football

17. Name the game, which is played by the slim player but cannot be played by the fat player?
(a) Cricket (b) Football
(c) Kho-Kho (d) Kabaddi
[NTSE-MAHARASHTRA STAGE-1-2014]
18. A, B, C, D, E and F are six families living in different houses in a row. F and D are neighbours of B.
E is neighbor of A and C. A is not a neighbor of F or D also if C is not a neighbor of D, then who is
the neighbor of F
(a) B and C (b) B and F
(c) B and D (d) only B

Q.19 and 20  Directions:- Suresh and Urmila are good at singing. Suman being expert in elocution sings
sweetly. Parag being an orator is also good at acting and singing. Vijaya a dramatist, also delivers good
lectures. Pictures drawn by Parag and Suresh are in great demand. Find answers to the following questions
from the given alternatives.
19. Who has all the skills?
(a) Suman (b) Urmila
(c) Vijaya (d) Parag

20. Who is not an expert in elocution?


(a) Suman and Vijaya (b) Vijaya
(c) Suresh and Urmila (d) Urmila
[NTSE-MAHARASHTRA STAGE-1-2013]
159 PACE IIT & MEDICAL: Mumbai / Delhi & NCR / Goa / Akola / Kolkata / Nashik / Pune / Bokaro / Dubai
NTSE-MENTAL ABILITY

21. Kamal and Lalit both went to Jaipur and Srinagar. Karan and Lalit had also been to Nagpur. Vivek
and Kamal both returned from Solapur. Then, who has not been to Nagpur.?
(a) Kamal-Lalit (b) Kamal-Karan
(c) Vivek (d) Kamal-Vivek
[NTSE-MAHARASHTRA STAGE-1-2012]

Q.22 to 24  Directions: Suman and Komal are experts in acting and singing. Geeta is well versed in
singing and debating. Suman and Preeti and experts in drawing. Komal and Shweta are well versed in
drawing and debating. Komal and Preeti have a good knowledge of sculpture. Then answer the following
questions.

22. Who knows only drawing and debating?


(a) Geeta (b) Preeti
(c) Suman (d) Shweta

23. Who knows only singing and debating?


(a) Suman (b) Geeta
(c) Shweta (d) Preeti

24. Who knows all art forms?


(a) Geeta (b) Komal
(c) Preeti (d) Suman
[NTSE-MAHARASHTRA STAGE-1-2012]

Answer Key

1. (c) 2. (c) 3. (c) 4. (a) 5. (d) 6. (d) 7. (b) 8. (c) 9. (c) 10. (b)
11. (a) 12. (d) 13. (b) 14. (c) 15. (d) 16. (b) 17. (c) 18. (a) 19.(d) 20.(c)
21. (d) 22. (d) 23 (b) 24. (b)

160 PACE IIT & MEDICAL: Mumbai / Delhi & NCR / Goa / Akola / Kolkata / Nashik / Pune / Bokaro / Dubai
NTSE-MENTAL ABILITY

13. VENN DIAGRAM

1. An object is called a subset of another object, if former is a part of latter and such relation is shown
by two concentric circles.
(i) Pencil, Stationary
(ii) Spinach, Vegetable
(iii) Chair, Furniture
It is very clear from the above relationship that one object is a part of other, and hence all such
relationships can be represented by figure below -

2. An object is said to have an intersection with another object, when two objects share something in
common.
(i) Surgeon, Males
(ii) Politicians, Indian
(iii) Educated, Unemployed

All the three relationships given above have something in common as some surgeons can be male
and some female, some politicians may be Indian and some may belong to other countries, educated
may be employed and unemployed as well. And all the three relationships can be represented by
figure above.

3. Two objects are said to be disjoint when neither one is subset of another nor they share anything in
common. In other words, totally unrelated object fall under this type of relationship
(i) Furniture, Car
(ii) Copy, Cloth
(iii) Tool, Shirt

It is clear from the above relationship that both the objects are unrelated to each other, and hence
can be represented diagrammatically, as shown in figure above. From the above discussion we
observe that representation of relationship of two objects is not typical if students follow the above
points. But representation of three objects diagrammatically pose slight problem before the students.
A variety of such relationship is being discussed in the following examples.

Directions : (1 to 6) Each of these questions given below contains three group of things. You are to choose
from the following four numbered diagrams, a diagram that depicts the correct relationship among the three
groups of thing in each question.

161 PACE IIT & MEDICAL: Mumbai / Delhi & NCR / Goa / Akola / Kolkata / Nashik / Pune / Bokaro / Dubai
NTSE-MENTAL ABILITY

1. Moon, Earth, Universe

(A) (B)

(C) (D)

Sol. (A) Moon and Earth are the parts of universe and therefore are subsets of universe.

2. India, Pakistan, Asia

(A) (B)

(C) (D)

Sol. (A) India and Pakistan are the subsets of Asia.

3. Batsman, Cricket, Stick

(A) (B)

(C) (D)

Sol. (D) Batsman is a subset of Cricket and Stick is something unrelated to Cricket.

162 PACE IIT & MEDICAL: Mumbai / Delhi & NCR / Goa / Akola / Kolkata / Nashik / Pune / Bokaro / Dubai
NTSE-MENTAL ABILITY

4. Book, Pen, Pencil

(A) (B)

(C) (D)

Sol. (C) Book, Pen, Pencil are neither subset of one another nor have anything in common.

5. Which of the following diagrams correctly represents the relationship among Tennis fans,
Cricket players and Students.

(A) (B)

(C) (D)

Sol. (A) From the relationship given in the question, we observe that each of the objects carries
something in common to one another. A Tennis fan can be a cricket player as well as student.
Hence Diagram (A) represents this relationship.

6 Which of the following diagrams correctly represents the relationship among smokers, bidi
smokers, cancer patients?

(A) (B)

(C) (D)

Sol. (B) Bidi smokers is a subset of smokers and cancer patient may be a smoker, bidi smoker and
non-smoker. Hence third object shares a common relationship with first and second object as well.

Directions : (7 to 11) In the following diagram, three classes of population are represented by three figures.
The triangle represents the school teachers, the square represents the married persons and the circle
represents the persons living in joint families.
163 PACE IIT & MEDICAL: Mumbai / Delhi & NCR / Goa / Akola / Kolkata / Nashik / Pune / Bokaro / Dubai
NTSE-MENTAL ABILITY

7. Married persons living in joint families but not working as school teachers are represented by
(A) C (B) F (C) D (D) A

8. Persons who live in joint families, are unmarried and who do not work as school teachers are
represented b
(A) C (B) B (C) E (D) D

9. Married teachers living in joint families are represented by


(A) C (B) B (C) D (D) A

10. School teachers who are married but do in joint families are represented by
(A) C (B) F (C) A (D) D

11. School teachers who are neither married nor do live in joint families are represented by
(A) F (B) C (C) B (D) A

Sol. (Ex. 7 to Ex. 11)

7. (C) Married persons living in joint families are presented by the region common to the square
and the circle i.e., D and B. But, according to the given conditions, the persons should not be
school teachers. So, B ¡s to be excluded. Hence, the required condition is denoted by region D.

8. (C) Persons living in joint families are represented by the circle. According to the given conditions,
the persons should be unmarried and not working as school teachers. So, the region should not be
a part of either the square of the triangle. Thus, the given conditions are satisfied by the region E.

9. (B) Married teachers are represented by the region common to the square and the triangle i.e., B and
C. But, according to the given conditions, the persons should be living in joint families. So, the
required region should be a part of the circle. Since B lies inside the circle, so the given conditions
are satisfied by the persons denoted by the region B.

10. (A) As in the above question, married teachers are represented by B and C. But, here, the given
conditions lay down that the persons should not be living in joint families. So, the required region
should lie outside the circle. Since C lies outside the circle, so the given conditions are satisfied by
the persons denoted by the region C.

11. (A) School teachers are represented by the triangle. But according to the given conditions, persons
are neither married nor do they live in joint families. So, the region should not be a part of either the
square or the circle. Such a region is F.

Dot Situation
The problems on dot situation involve the search of similar conditions in the alternative figures as indicated
in the problem figure. The problem figure contains dots placed in the spaces enclosed between the
combinations of square, triangle, rectangle and circle. Selecting one of these dots we observe the region in

164 PACE IIT & MEDICAL: Mumbai / Delhi & NCR / Goa / Akola / Kolkata / Nashik / Pune / Bokaro / Dubai
NTSE-MENTAL ABILITY

which this dot in enclosed i.e. to which of the four figures (circle, square, rectangle and triangle) is this
region common. Then we look for such a region in the four alternatives. Once we have found it. We repeat
the procedure for other dots, if any. The alternative figure which contains all such regions is the answer.

Directions : (1 to 3) In each of the following questions, there is a diagram marked (X), with one or more
dots placed in it. The diagram is followed by four other figures, marked (A), (B), (C) and (D) only one of
which is such as to make possible the placement of the alternative in each case.

1.

(A) (B)

(C) (D)

Sol. (C) In figure (X), the dot lies in the region common to the circle and the triangle only. Such a region
is present in figure (C) only.

2.

(A) (B)

(C) (D)

Sol. (A) In figure (X), one of the dots is placed in the region common to the circle and the triangle and
the other dot is placed in the region common to the triangle and the square.
From amongst the figures (A), (B), (C) and (D),only figure (A) has both the regions, one common to
circle and triangle and the other common to triangle and square.
165 PACE IIT & MEDICAL: Mumbai / Delhi & NCR / Goa / Akola / Kolkata / Nashik / Pune / Bokaro / Dubai
NTSE-MENTAL ABILITY

3.

(A) (B)

(C) (D)

Sol. (C) Figure (X), contains one dot in the square only, another dot in the region common to the square
and the triangle only and the third dot in the region common to the circle and the triangle. Figure (A)
does contain a region which lies in the square alone. Figures (B) and (D) do not contain any region
common to the circle and the triangle. Only figure (C) contains all the three types of regions.

EXERCISE
Directions: (1 to 7) Each question below has three items having certain relationship among them. The same
relationship is expressed by sets of each circle representing one item irrespective of its size. Match the items
with right set of circles.

1. Computer skilled, Graduates, Employed

2. Vegetable, Apple, Spinach

166 PACE IIT & MEDICAL: Mumbai / Delhi & NCR / Goa / Akola / Kolkata / Nashik / Pune / Bokaro / Dubai
NTSE-MENTAL ABILITY

3. Clever, Punctual, Poor

4. Copper, Cobalt, Silver

5. Doctor, Lawyer, Male

6. Man, Husband, Son

,
7. Female Medicine Physician

Directions: (8 to 12) Study the figure below and answer the following questions.

167 PACE IIT & MEDICAL: Mumbai / Delhi & NCR / Goa / Akola / Kolkata / Nashik / Pune / Bokaro / Dubai
NTSE-MENTAL ABILITY

8. Find out the number of families which have all the four things mentioned in the diagram.
(A) 40 (B) 30 (C) 35 (D) 20

9. Find out the number of families which have scooters.


(A) 145 (B) 100 (C) 188 (D) 240

10. Find out the number of families which have V.C.R. and T.V. both
(A) 84 (B) 24 (C) 104 (D) 100

11. Find out the number of families which have only one thing, that is, either V.C.R. or T.V, or Scooter
or Maruti.
(A) 160 (B) 184 (C) 225 (D) 254

12. Find out the number of families which have T.V. and scooter both but have neither V.C.R. nor
Maruti.
(A) 15 (B) 30 (C) 4 (D) 50

Directions : (13 to 16) Read the following Information carefully and answer the questions based on them :
The circle represents poor boys, the square educated boys, the triangle represents the boys who are
employed somewhere and the rectangle represents those who help in the family business. Each section of the
diagram is numbered.

13. Which number represents those poor boys who help in family business but are not educated or
employed elsewhere?
(A) 2 (8) 3 (C) 4 (D) 5

14. Which number represents the group of educated poor boys who are employed somewhere but do
not help in family business ?
(A) 3 (B) 11 (C) 2 (D) None of these

15. Which section does number 12 represent?


(A) Uneducated poor boys who do not help in family business
(B) Educated poor boys employed in service
(C) Uneducated boys who help in family business
(D) Educated poor boys who help In family business.

16. Which number represents (hat Section of poor boys who are neither educated nor are in any
employment or have any family business?
(A) 5 (8) 1 (C) 11 (D) 12

Directions : (17 to 21) Following questions are based on the Venn diagram given below in which the
triangle stands for lady, the rectangle stands for doctors, the circle stands for teachers and the, square stands
for engineers. Find out (ho correct answer of each question from the alternatives given under it.
(NTSE Stage-I/Raj./ 2007)

168 PACE IIT & MEDICAL: Mumbai / Delhi & NCR / Goa / Akola / Kolkata / Nashik / Pune / Bokaro / Dubai
NTSE-MENTAL ABILITY

17. How many persons are engineers as well as do teaching job?


(A) 5 (B) 8 (C) 13 (D) 7

18. The numbers of lady doctors who are neither engineer nor teachers are
(A) 0 (B) 7 (C) 10 (D) 11

19. The number of engineers who are neither doctors nor lady teachers are
(A) 4 (B) 11 (C) 9 (D) 2

20. The numbers of lady doctors who are engineers but teach children of the village are
(A) 7 (B) 8 (C) 10 (D) 15

21. The numbers of gents teachers who are neither doctors nor engineers are
(A) 12 (B) 9 (C) 5 (D) 4

Directions : (22 to 26) The following questions are based on the diagram given below. In the diagram,
circle represents tall children, the square, obese children, rectangle, the energetic children and the triangle,
girl children. Study the diagram and answer the questions that follow. (NTSE Stage-II, 2007)

22. Which area represents girls who are tall and obese but not energetic?
(A) 2 (B) 3 (C) 4 (D) 5

23. Which of the following areas represents the tall, obese and energetic girls ?
(A) Only 3 (B) 3 and 4 (C) 2 and 3 (D) 2, 3 and 4

24. Which of the following area represents children who are tail, obese and energetic but are not girls?
(A) 3 (B) 2 (C) 4 (D) 9

25. Which areas represent energetic children who are not obese?
(A) 1, 13 and 15 (B) 13 and 15 (C) 1, 11 and 15 (D) 6.13 and 15

169 PACE IIT & MEDICAL: Mumbai / Delhi & NCR / Goa / Akola / Kolkata / Nashik / Pune / Bokaro / Dubai
NTSE-MENTAL ABILITY

26. Which of the following areas represent obese and energetic children, who are neither girls nor tall?
(A) 2 and 12 (B) 3 and 10 (C) 2 and 3 (D) 11 and 12

Directions : (27 to 31) Which of the Venn diagrams given in the alternatives best represents the relation
between the given items?

27. Doctors, Engineers, Lawyers

28. Books, Newspaper, Words

29. Boys, Students, Players

30. Animals, cows, grass-eating animals

31. Mammals, Tigers, Cows

170 PACE IIT & MEDICAL: Mumbai / Delhi & NCR / Goa / Akola / Kolkata / Nashik / Pune / Bokaro / Dubai
NTSE-MENTAL ABILITY

Directions : (32 to 36) The following questions are based on the figure given below. In this figure the
rectangle represents artists, the circle represents players and the triangle represents doctors, The numbers in
different sections refer to the number of persons in that area. A few questions are then asked based upon this
information. (NTSE Stage-II, 2008)

32. How many players are neither artists nor doctors?


(A) 35 (B) 28 (C) 24 (D) 18

33. How many doctors are players but not artists?


(A) 4 (B) 5 (C) 20 (D) 25

34. What percentage of doctors has at least one more interest - either arts or sports in life?
(A) 55% (B) 50% (C) 45% (D) 40%

35. How many players are artists but not doctors?


(A) 28 (B) 25 (C) 18 (D) 5

36. In terms of number, arrange artists, players and doctors in decreasing order (Those with maximum
number first, with minimum number last)
(A) Players, Artists, Doctors (B) Players, Doctors Artists
(C) Artists, Doctors Players (D) Mists, Players Doctors

Directions (37 to 38) Which of the Venn diagrams given in the alternatives best represents the relation
between the given items? (NTSE Stage-II, 2008)

37. Mothers. Engineers, Females

38. Crows. Birds, Snakes

171 PACE IIT & MEDICAL: Mumbai / Delhi & NCR / Goa / Akola / Kolkata / Nashik / Pune / Bokaro / Dubai
NTSE-MENTAL ABILITY

Directions : (39 to 40) Questions are based on Venn Diagram. Circle stands for rural, Triangle stands for
educated, square stands for hard-working and Rectangle stands for intelligent persons, Numbers given
represent serial number of the area. (NTSE Stage-I/ Raj, /2009)

39. Which area represents “Intelligent, hard-working and educated but not rural” persons ?
(A) 12 (B) 10 (C) 4 (D) 3

40. Which area represents Hard-working and intelligent but neither rural nor educated” persons?
(A) 11 (B) 5 (C) 3 (D) 4

Directions : (41 to 42) The numbers in different section of the overlapping circles indicate the number of
people who speak different language. Answer the questions that follow. (NTSE Stage-II, 2009)

41. How many people speak only two languages?


(A) 19 (B) 21 (C) 24 (D) 30

42. How many cannot speak all the three languages?


(A) 21 (B) 36 (C) 57 (D) 60

Directions : (43 to 45) In each of the following questions, patterns of circles are used to represent
relationship among different items. Each circle represent an Item. Which of the patterns represents the
relationship among the items given In the question ? (NTSE Stage-II, 2009)

43. House, Kitchen. Bedroom

44. Animal, Insect, Cockroach

172 PACE IIT & MEDICAL: Mumbai / Delhi & NCR / Goa / Akola / Kolkata / Nashik / Pune / Bokaro / Dubai
NTSE-MENTAL ABILITY

45. Players, Human Beings. Students

Directions : (46 to 47) In the following diagram ‘I’ represents Indians ‘S’ represent scientist and ‘P’
represents politicians. (NTSE Stage-I/Raj./2012)

46. Indians those are politicians but not scientist will be


(A) a (B) b (C) d (D) f

47. Scientists which are politician but not Indians will be


(A) d (B) e (C) f (D) g

Directions: (48 to 50) Following four diagrams marked A, B, C and D are given as alternatives. Choose the
best suitable alternative diagram marked A, B, C and D, the one that represent the best relationship amongst
the three given groups. (NTSE Stage-I/Raj./2013)

48. Birds, Crows. Cats

49. Snakes, Land creatures Water eveatures

50. Males, Nephews, Nieces

51. In the figure, the circle represents youth, the triangle represents footballers and the rectangle
represents athletes. Which letter(s) represent(s) athletes among youths who are not footballers?
(NTSE Stage-II, 2013)

(A) g (B) g and c (C) f (D) f and d

173 PACE IIT & MEDICAL: Mumbai / Delhi & NCR / Goa / Akola / Kolkata / Nashik / Pune / Bokaro / Dubai
NTSE-MENTAL ABILITY

52. Which of the following diagram/sets indicate the relation between women, mothers and parents?
(NTSE Stage-II, 2013)

Directions (53 to 54) : These questions are based on the following diagram. In this diagram there are three
sets U, A and B. Which represent the married persons, educated people and teachers respectively live in a
colony. The total population of the colony is 300. (NTSE Stage-I/Raj./2014)

53. Number is unmarried persons in the colony is


(A) 40 (B) 112 (C) 124 (D) 125

54. Number of educated people who is not teacher is


(A)120 (B) 112 (C) 124 (D) 125

55. Which of the following Venn diagrams correctly represents cows, horses and animals?
(NTSE Stage-I/Raj./2014)

56. Which of the following Venn diagrams correctly represents rectangle, quadrilateral and polygon?
(NTSE Stage-I/Raj./2014)

174 PACE IIT & MEDICAL: Mumbai / Delhi & NCR / Goa / Akola / Kolkata / Nashik / Pune / Bokaro / Dubai
NTSE-MENTAL ABILITY

57. Which of the following Venn diagrams correctly represents intelligence, ability and honesty?
(NTSE Stage-I/Raj./2016)

58. Which of the following Venn diagrams correctly represents quadrilateral, rectangle and square?
(NTSE Stage-I/Raj./2017)

59. Which of the following Venn diagrams correctly represents female, mother and doctor?
(NTSE Stage-I/Raj./2017)

Direction (60 - 61) Give answer according to the following Venn diagram :
(NTSE Stage-I/Raj./2017)

60. Total number of teachers is


(A) 12 (B) 31 (C) 19 (D) 22

61. How many artists are there who are Writer but not teacher?
(A) 8 (B) 14 (C) 13 (D) 7

175 PACE IIT & MEDICAL: Mumbai / Delhi & NCR / Goa / Akola / Kolkata / Nashik / Pune / Bokaro / Dubai
NTSE-MENTAL ABILITY

Direction : (62 to 70) In the following question a dot is placed in the figure marked as (X), this figure is
followed by four alternatives marked as (A), (B). (C) and (D). One out of these four Options contains the
common region to circle, Square, triangle and rectangle similar to that of mar1ed by dot in figure (X). Select
that option.
62.

63.

64.

176 PACE IIT & MEDICAL: Mumbai / Delhi & NCR / Goa / Akola / Kolkata / Nashik / Pune / Bokaro / Dubai
NTSE-MENTAL ABILITY

65.

66.

67.

177 PACE IIT & MEDICAL: Mumbai / Delhi & NCR / Goa / Akola / Kolkata / Nashik / Pune / Bokaro / Dubai
NTSE-MENTAL ABILITY

68.

(NTSE Stage-II, 2011)


69.

(NTSE Stage-II, 2011)

70.

(NTSE Stage-II, 2011)

178 PACE IIT & MEDICAL: Mumbai / Delhi & NCR / Goa / Akola / Kolkata / Nashik / Pune / Bokaro / Dubai
NTSE-MENTAL ABILITY

Answer Key

1. (b) 2. (c) 3. (a) 4. (b) 5. (d) 6. (c) 7. (b) 8. (d) 9. (c) 10. (d)
11. (c) 12. (b) 13. (d) 14. (d) 15. (a) 16. (d) 17. (c) 18. (a) 19. (b) 20. (b)

21. (d) 22. (c) 23. (a) 24. (b) 25. (b) 26. (d) 27. (b) 28. (b) 29. (a) 30. (c)

31. (d) 32. (a) 33. (b) 34. (d) 35. (c) 36. (a) 37. (d) 38. (c) 39. (a) 40. (d)

41. (b) 42. (c) 43. (b) 44. (a) 45. (d) 46. (b) 47. (c) 48. (d) 49. (a) 50. (d)

51. (a) 52. (a) 53. (a) 54. (b) 55. (a) 56. (a) 57. (c) 58. (a) 59. (c) 60. (b)

61. (a) 62. (a) 63. (d) 64. (d) 65. (b) 66. (a) 67. (a) 68. (d) 69. (b) 70. (c)

PREVIOUS YEAR QUESTIONS

Q.1 to 3: Directions: The numbers in the figure show the number of tourists from different states. Observe
the figure and choose the answer from given alternatives for the following questions.

1. How many tourists visited all three states?


(a) 119 (b) 108
(c) 21 (d) 195

2. Find the number of tourists visiting only two states.


(a) 93 (b) 98
(c) 87 (d) 139

3. Find number of tourists who visited Kerala and Rajasthan but not visited Maharashtra
(a) 139 (b) 185
(c) 206 (d) 232
[NTSE MAHARASHTRA Stage-1-2020]
Q.4 to 6: Direction: In the adjacent figure the numbers represent the number of artists in direction acts.
Observe the diagram carefully and choose an alternative to answer the question.

179 PACE IIT & MEDICAL: Mumbai / Delhi & NCR / Goa / Akola / Kolkata / Nashik / Pune / Bokaro / Dubai
NTSE-MENTAL ABILITY

4. How many artists are experts in all the arts?


(a) 23 (b) 10
(c) 14 (d) 33

5. How many artists are good at ‘acting’?


(a) 35 (b) 77
(c) 67 (d) 32

6. How many artists are good in only two arts?


(a) 65 (b) 97
(c) 83 (d) 71
[NTSE MAHARASHTRA Stage-1-2019]

Q.7 and 9: Directions: Observe the following venn diagram and choose the correct alternative to answer
the question.

7. How many employees play only kabaddi and cricket?


(a) 24 (b) 72
(c) 58 (d) 31

8. How many players play only one game?


(a) 151 (b) 216
(c) 183 (d) 210

9. How many players do not play cricket and football?


(a) 24 (b) 58
(c) 64 (d) 72
[NTSE MAHARASHTRA Stage-1-2018]

Q.10 and 11: Directions- In the following diagram, three geometrical figures have been drawn intersecting
each other. The labels have been given to different parts. Each figure represent a specific group of people.
Observe the figure closely and answer the questions that follow.

180 PACE IIT & MEDICAL: Mumbai / Delhi & NCR / Goa / Akola / Kolkata / Nashik / Pune / Bokaro / Dubai
NTSE-MENTAL ABILITY

10. How many employee players do private business?


(a) b (b) a
(c) c (d) d

11. How many players are unemployed?


(a) b (b) d
(c) a (d) c
[NTSE MAHARASHTRA Stage-1-2017]

Q.12 and 13:  Directions : - In the following Venn diagram rectangle represents no. of poets, triangle
represents no. of novelists and circle represents no. of dramatists. Answer the following questions:

12. How many are novelists but not dramatists?


(a) 7 (b) 9
(c) 8 (d) 3

13. How many are dramatists but not poets?


(a) 5 (b) 8
(c) 13 (d) 22
[NTSE MAHARASHTRA Stage-1-2016]

14. Some students of a village school stay in the village itself. Some students come from neighbouring
villages. Some students study in English medium, while some study in Marathi medium. Of the
following figures which represents the above information?

(a) (b)

(c) (d)

[NTSE MAHARASHTRA Stage-1-2016]


181 PACE IIT & MEDICAL: Mumbai / Delhi & NCR / Goa / Akola / Kolkata / Nashik / Pune / Bokaro / Dubai
NTSE-MENTAL ABILITY

Q.15 and 16  Directions:- In a University some students are learning Sanskrit, Japanese, Japanese,
German and French. In the following Venn diagram the circle represents the students studying Sanskrit,
square represents the students studying German, triangle represents the students studying Japanese and the
eclipse represents the students studying French. Observe the figure and answer the equation by choosing
correct alternative given below:

15. How many students studying Japanese learn one more language except German?
(a) 51 (b) 18
(c) 33 (d) 43

16. How many students studying German are learning one more language along with Sanskrit?
(a) 11 (b) 15
(c) 26 (d) 37
[NTSE MAHARASHTRA Stage-1-2015]

Q.17 and 18  Directions:- In the following Venn diagram the number of students who like Science,
Mathematics, and Language is given. Observe the figure and answer the following questions:

17. Find the number of students who like Mathematics but not Language.
(a) 15 (b) 14
(c) 11 (d) 10

18. How many students, who like Science and Mathematics do not like Language?
(a) 4 (b) 8
(c) 11 (d) 5
[NTSE MAHARASHTRA Stage-1-2014]

Q.19 to 21:  Directions:- Information was gathered based on observations of readers of three newspapers
A, B and C. 26% people read newspaper A, 25% people read newspaper B, 14% people read newspaper C,
11% people read both newspapers A and B, 10% people read both newspapers B and C, 9% people read
both newspapers A and C and 0% people read only newspaper C. Based on this information answer the
following questions.
182 PACE IIT & MEDICAL: Mumbai / Delhi & NCR / Goa / Akola / Kolkata / Nashik / Pune / Bokaro / Dubai
NTSE-MENTAL ABILITY

19. What is the average of readers of all three newspapers?


(a) 1 (b) 4
(c) 5 (d) 6
20. What is the average of people who read both A and B newspaper but do not read C newspaper?
(a) 2 (b) 4
(c) 5 (d) 6
21. What is the average of number of readers who read at least 0 newspaper?
(a) 40 (b) 50
(c) 60 (d) 65
[NTSE MAHARASHTRA Stage-1-2013]
Q.22 and 23  Directions: - In the following Venn diagram, if square represents students playing cricket,
circle represents students playing kabaddi and triangle represents students playing volleyball, then answer
the following questions.

22. How many students play only kabaddi?


(a) 1 (b) 3
(c) 4 (d) 14

23. How many students play cricket, kabaddi as well as volleyball?


(a) 2 (b) 6
(c) 4 (d) 37
[NTSE MAHARASHTRA Stage-1-2012]
24. A is a group of triangles
B is a group of acute angled triangles
C is a group of isosceles triangles
D is a group of equilateral triangles
Then, which of the following figure represents the group A, B, C, D.
(a) (b) (c) (d)

[NTSE MAHARASHTRA Stage-1-2012]


Answer Key

1. (c) 2. (d) 3. (b) 4. (b) 5. (Bonus) 6(c) 7. (d) 8. (d) 9. (b) 10.(d)
11.(c) 12.(b) 13.(a) 14.(a) 15.(d) 16.(b) 17.(b) 18.(d) 19.(c) 20.(d)
21.(c) 22.(c) 23.(a) 24.(a)

183 PACE IIT & MEDICAL: Mumbai / Delhi & NCR / Goa / Akola / Kolkata / Nashik / Pune / Bokaro / Dubai
NTSE-MENTAL ABILITY

14. SYLLOGISM

Syllogism literally means to stay together, giving us the sense of putting two propositions or premises
together in order to draw a logical conclusion from them.

Venn-Diagram Representation of Four Propositions

1. Universal Affirmative (A) : All S are P.

There are two possibilities to represent the relation between S and P given by universal affirmative
proposition “All S are P”. This can also be understood with the help of set theory.
Case I. S = {a,b,c} P={a,b,c,d}
Case II. S = {a, b, c} P = {a, b, c}
Case I is represented by Fig. (i) and Case II is represented by Fig. (ii). In both these cases, we see
that every element of set S is also the element of set P. Hence, we can definitely say that the above
two figures show “All S are P”.

2. Universal Negative (E): No S are P

There is only one possibility of drawing the relationship between S and P.


S = {a, b, c} P = {d, e, f}
From the above two sets, it is clear that none of the elements of S is the element of set P.

3. Particular Affirmative (I): Some S are P

184 PACE IIT & MEDICAL: Mumbai / Delhi & NCR / Goa / Akola / Kolkata / Nashik / Pune / Bokaro / Dubai
NTSE-MENTAL ABILITY

There are three possible representations given by Fig. (iv), Fig. (v) and Fig. (vi) depicting particular
affirmative proposition “Some S are P”. This can be supported with the help of following sets:
Case I.S={a,b,c,d} P={c,d,e,f}
Case II.S={a,b} P={a,b,c,d}
Case III. S = {a, b, c, d} P = {a, b}
In all the three cases we find that some of the elements of S are also the elements of set P

4. Particular Negative (0): Some S are not P

The particular negative proposition “Some S are not P” can be represented with the help of three
possible figures given in (vii), (viii) and (ix). Venn-diagram representations of the above
propositions can be supported by way of following sets:
Case I. S = {a, b, c} P = {c, d, e}
Case II. S = {a, b, c, d} P = {c, d}
Case III. S = {a, b, c} P = {d, e, f}
In all the three cases, we find that there are some elements in set S which are not elements of set P.
Hence all the cases along with the respective figures support the proposition “Some S are nor P”.

Venn-Diagram Representation
Immediate inferences drawn from each type of propositions (A, E, I, O). One of the important points to be
noted while drawing inference from Venn diagrams is that all possibilities of Venn-diagrams should be
taken in account. Let us now discuss each type of proposition in relation to the pictorial representation.

1. A-Type-All S are P
It is clear from the A type of proposition that all S are contained in P. Therefore, circle representing
S will be either inside or equal to circle representing P. However, in both the cases, conclusions
(Some P are S) and (Some S are P) are true This case can be understood clearly by taking two sets in
all possible ways.

(I) S = {1,2,3} P = (1,2,3,4,5)


(ii) S = {1,2,3} P = {1,2,3}
The above cases show the all the possibilities of two sets S and P showing the relationship between
each other as represented by the proposition. All S are P. Now in both the cases we see that set {2,3}
is the part of set S and also of set R Hence, it is clear that inference (Some S are P) is true from this
relationship. Likewise set {2, 3} is the part of set P and also of set S. Therefore, it is also clear that

185 PACE IIT & MEDICAL: Mumbai / Delhi & NCR / Goa / Akola / Kolkata / Nashik / Pune / Bokaro / Dubai
NTSE-MENTAL ABILITY

inference (Some P are S) is true. Inference (Some P are not S) is not valid because it is true not from
case (I) but false from case (ii). Inference (All P are S) is not valid because it is true from case (ii)
and false from case (i).

2. E- Type-No S is P
There is only one possibility of Venn-diagram representation of E-type proposition. The relationship
can also be shown by two sets S={12,3} and P ={4, 5,6}. From these two sets. We see that set{2,3}is
the part of S but not of set P. It implies that inference (Some S are not P) is true. Similarly, set {5,6}
is the part of set P but not of set S. This means that interference (Some P are not S) is true
Therefore, on the basis of E-type proposition, we can, draw following immediate interferences.

(i) No P is S. (ii) Some S are not P. (iii) Some P are not S.


Any other immediate inference drawn from E-type proposition is not valid

3. I-Type -Some S are P


This Proposition gives rise to many possible representations of Venn-diagrams and hence most of the
inferences drawn therefrom are invalid and doubtful This relationship can be shown by following
sets and respective Venn-diagrams.
(i) S={1,2,3,4} P = {3,4,5,6}

Set {3,4} is the part of set S as well as set P, hence some S are P
(ii) S={1,2,3,4} P={1,2}

Set {1,2} is the part of set S as well as set P, hence some S are P.
(iii) S ={ 1,2} P ={1,2,3,4}

186 PACE IIT & MEDICAL: Mumbai / Delhi & NCR / Goa / Akola / Kolkata / Nashik / Pune / Bokaro / Dubai
NTSE-MENTAL ABILITY

Set {1 ,2} is the part of set S as well as set P, hence some S are P.
The above three combinations of sets and respective diagrams show the relationship between S and P
as represented by l-type proposition. From all the possible combinations, it is clear that inference
(Some P are S) is true. Inference (Some S are not P} is true from combinations (I) and (ii). But it is
not true from combinations (iii). Therefore, inference (Some S are not P) is not a valid inference
drawn from the above proposition.

4. 0-Type-Some S are not P


From this proposition no immediate inference can be drawn. Let us discuss this proposition in the
light of Venn-diagram representation.
(i) S = {1, 2, 3, 4} P = {3,4,5,6}

Set {1,2} is the part of set S but not of set P, hence this shows the relationship represented by the
proposition ‘Some S are not P.
(ii) S = {1 ,2,3} P = {4,5,6}

Set {2, 3) is the part of set S but not of set P, hence this shows the relation represented by the
proposition ‘Some S are not P’.
(iii) S = {1,2,3,4,5} P = {4,5}

Set {1, 2, 3} is the part of set S but not of set P hence denotes proposition ‘Some S are not P’. On the
basis of all possible combinations showing relationship between S and P, no valid inference can be
drawn.

Inference - Some S are P is true from case


(i) and (ii) but not true from case (ii) and hence it is an invalid inference. Inference - Some P are not S is true
from case (i) and (ii) but not true from case (iii) and hence it is an invalid inference.

Students should note that if an inference is true, it has to comply with or follow all the possible pictorial
representation of Venn-diagrams.

187 PACE IIT & MEDICAL: Mumbai / Delhi & NCR / Goa / Akola / Kolkata / Nashik / Pune / Bokaro / Dubai
NTSE-MENTAL ABILITY

Directions : (1 to 5) In these types of questions, two statements followed by two conclusions, I and II, are
given. You have to take the given two statements to be true even if they seem to be at variance from
commonly known facts. Read the conclusions and then decide which of the given conclusions logically
follows from the two given statements, disregarding commonly known facts.

Mark the answer


(A) if only I follows
(B) if only II follows
(C) if both I and II follow
(D) if neither I nor II follows

1. Statements : Some players are dancers All dancers are tall.


Conclusions: I. Some players are tall.
II. All players are tall.

Sol. (A)

So some players (those who are dancers) are tall.

2. Statements: Anil is a good sportsman.


Sportsmen are healthy.

Conclusions: I. All healthy persons are sportsmen.


II. Anil is healthy.

Sol. (B)

3. Statements: Businessmen married only fair girls.


Anisha is fair.
Conclusions: I. Anisha was married to a businessman.
II. Anisha was not married to a businessman.
Sol. (D)

So Anisha may or may not be married to a businessman.


188 PACE IIT & MEDICAL: Mumbai / Delhi & NCR / Goa / Akola / Kolkata / Nashik / Pune / Bokaro / Dubai
NTSE-MENTAL ABILITY

4. Statements: All apples are oranges.


Some oranges are grapes.
Conclusions: I. Some apples are grapes.
II. Some grapes are apples.
Sol. (D)

So grapes may or may not be applies.

5. Statements: All men are bachelors.


Some men are educated
Conclusion: I. Some bachelors are educated
II. Some educated are bachelors

Sol. (C)

Here some bachelors are definitely educated. So both conclusions follow.

EXERCISE
Directions: (1 to 10) In each question, two statements are followed by two conclusions, I and II. You have
to take the given two statements to be true even if they seem to be at variance from commonly known facts.
Read the conclusions and then decide which of the given conclusions logically follows from the given
conclusions logically follows from the two given statements, disregarding commonly known facts, Mark the
answer
(a) If only conclusion I follows
(b) If only conclusion II follows
(c) If both I and II follow
(d) Id none follows

1. Statements: Some clouds are thunder.


All clouds are rain.
Conclusions: I. Those clouds which are not thunder, are also rain,
II. Those clouds which are not thunder, are not necessarily rain.

2. Statements: Some tins are pens.


Some pens are rods.
Conclusions: I. No rod is tin.
II. All tins are rods.

3. Statements : Some sabres are bombs.


Key is a sabre.
Conclusions: I. Some bombs are sabres.
II. Some keys are bombs.
189 PACE IIT & MEDICAL: Mumbai / Delhi & NCR / Goa / Akola / Kolkata / Nashik / Pune / Bokaro / Dubai
NTSE-MENTAL ABILITY

4. Statements : All lenses are horns.


No horn is colourful.
Conclusions: I. Some lenses are colourful.
II. No lens is colourful.

5. Statements: All flirts are smart.


All smart are rotten.
Conclusions: I. All rotten are flirts.
II. All smarts are flirts.

6. Statements: All puppies are dogs.


All dogs are trained.
Conclusions: I. Some trained are puppies.
II. All trained are puppies.

7. Statements: All advocates are obese.


Some obese are priests.
Conclusions: I. Some advocates are priests.
II. Some priests are advocates.

8. Statements: Some psychiatrists are singers.


All singers are tall.
Conclusions: I. Some psychiatrists are tall.
II. All psychiatrists are tall.

9. Statements: All coins are crows.


Some crows are pens.
Conclusions: I. No pen is coin.
II. All coins are pens.

10. Statements: All magicians are dumb.


Some magicians are educated.
Conclusions: I. Some dumb are magicians.
II. Some educated are dumb

Directions: (11 to 18) In each question, two or more statements are followed by two or more than two
conclusions. You have to take the given statements to be true even if they appear to be at variance with
commonly known facts, and then decide which of the conclusions logically follow(s) from the given
statements. For each question, mark out an appropriate answer choice that you think is correct.
11. Statements: 1. All bibs are red.
2. All red are flowers.
3. No flowers are tablets
Conclusions: I. Some flowers are bibs
II. No tablets are bibs
III. Some red are bibs
IV. Some tablets are red.
(A) I, II and III follow (B) II, III 8nd IV follow
(C) Only I and II follow (D) Only I and III follow

190 PACE IIT & MEDICAL: Mumbai / Delhi & NCR / Goa / Akola / Kolkata / Nashik / Pune / Bokaro / Dubai
NTSE-MENTAL ABILITY

12. Statements: 1. All pencils are birds.


2. All birds are skies.
3. All skies are hills.
Conclusions: I. All pencils are hills.
II. All hills are birds
III. All skies are pencils.
IV. All birds are hills.
(A) Only I and II follow (B) Only land III follow
(C) Only III and IV follow (D) None of these

13. Statements: 1. All big are novels.


2. Some novels are willows.
3. No willows is pencil.
Conclusions: I. Some willow are big.
II. Some novels are pencils.
III. No willow is big.
IV. Some novels are not pencils.
(A) I and either II or IV follow (B) I, III and IV follow
(C) I, II and III follow (D) Either I or III and IV follows

14. Statements: 1. Some spoons are bowls.


2. All bowls are knives.
3. All knives are forks.
Conclusions: I. All spoons are forks.
II. All bowls are forks.
III. Some knives are bowls.
IV. Some forks are spoons.

(A) Only II and III follow (B) Only II and IV follow


(C) Only III and IV follow (D) None of these

15. Statements: 1. All benches are desks.


2. Some desks are roads.
3. All roads are pillars.
Conclusions: I. Some pillars are benches.
II. Some pillars are desks.
III. Some roads are benches.
IV. No pillar is bench.

(A) None follows (B) Only either I or IV, and III follow
(C) Only either I or IV follows (D) Only either I or IV, and II follow

16. Statements : 1. All lamps are books.


2. No book is coloured.
Conclusions: I. Some lamps are coloured.
II. No lamp is coloured. (NTSE Stage-II, 2010)

(A) Only conclusion I follows. (B) Only conclusion II follows


(C) Both I and II follow. (D) Neither I nor II follows.

191 PACE IIT & MEDICAL: Mumbai / Delhi & NCR / Goa / Akola / Kolkata / Nashik / Pune / Bokaro / Dubai
NTSE-MENTAL ABILITY

17. Statements: 1. All envelopes are umbrellas.


2. All umbrellas are chalks.
Conclusions: I. Some chalks are envelopes.
II. Some umbrellas are not envelopes. (NTSE Stage-II, 2010)

(A) Only conclusion I follows. (B) Only conclusion II follows.


(C) Both I and II follow (D) Neither I nor II follows.

18. Statements: 1. All dogs are reptiles.


2. Some cats are reptiles.
Conclusions: I. Some dogs are cats.
II. Some cats are not reptiles. (NTSE Stage-II, 2010)

(A) Only conclusion I follows (R) Only conclusion II follows


(C) Both I and II follow. (D) Neither I nor II follows.

19. All the students passed the examination.


Some of the students are girls.
Select the correct alternative. (NTSE Stage-II, 2011)
(A) Some of the boys passed the examination
(B) All the girl students failed in the examination
(C) None of the boys passed the examination
(D) No girl student failed in the examination

Directions (20 to 21) : Take the given statement(s) as true and decide which of the conclusion logically
follows from the statements. (NTSE Stage-II, 2013)

20. Statement: All Actors are Musicians.


No Musician is a Singer.
Some Singers are Dancers.
Some Dancers are Musicians.
Conclusions: I. Some Actors are Singers
II. Some Dancers are Actors
III. No Actor is a Singer
(A) Only conclusion I follows. (B) Only conclusion III follows.
(C) Exactly one of conclusion I, III follows. (D) Only conclusion II follows.

21. Statement: All Clocks are Alarms.


No Clocks are Cuckoos.
Conclusion: 1. Some Alarms are Birds.
II. No Clock is a Bird
III. All Birds are Alarms
(A) Only conclusion I follows. (B) Only conclusion II follows.
(C) Only conclusion III follows. (D) Both conclusion II and III follows.

Directions (22 to 24): In each of the questions below are given two statements and two conclusions
numbered 1 and II. You have to take the given two statements to be true even if they seem to be at variance
from commonly known facts. Read the conclusions and then decide which of the given conclusions logically
follows from the two given statements. (NTSE Stage-I/Raj./ 2014)

192 PACE IIT & MEDICAL: Mumbai / Delhi & NCR / Goa / Akola / Kolkata / Nashik / Pune / Bokaro / Dubai
NTSE-MENTAL ABILITY

22. Statements: (I) Some books are pencils.


(II) Some pencils are pens.

Conclusions: I. All books are pens.


II. Some pens are books.
(A) Only conclusion I is true.
(B) Only conclusion II is true
(C) Conclusions I and II both are true
(D) Neither conclusion I nor conclusion II are true.

23. Statements: (I) Some men are educated.


(II) Educated persons prefer small families.
Conclusions: I. All small families are educated.
II. Some men prefer small families
(A) Only conclusion I is true.
(B) Only conclusion II is true
(C) Conclusions I and II both are true
(D) Neither conclusion I nor conclusion II are true.

24. Statements: (i) some hens are cows


(ii) All cows are horses
Conclusions: I. some horses are hens
II. Some hens are horses
(A) Only conclusion I is true.
(B) Only conclusion II is true
(C) Conclusions I and II both are true
(D) Neither conclusion I nor conclusion II are true.

25. Statements: All students are boys


No boy is dull
Conclusions: I. There is no girl student in the class
II. No student is dull
(A) Only conclusion I is true.
(B) Only conclusion II is true
(C) Conclusions I and II both are true
(D) Neither conclusion I nor conclusion II are true. (NTSE Stage-I/Raj./ 2015)

Direction: (26 to 28) In each of the questions below two statements and two conclusions numbered I and II
are given. You have to take the given two statements to be true even if they seem to be at variance from
commonly known facts. Read the conclusions and then decide which of the given conclusions logically
follows from the two given statements.
(NTSE Stage-I/Raj./ 2017)

26. Statements: (i) All dancers are singers.


(ii) All singers are teachers.
Conclusions: I. All dancers are teachers.
Il. Some singers are dancers.
(A) Only conclusion I is true
(B ) Only conclusion II is true
(C) Both conclusions I and II are true
(D) Neither conclusion I nor conclusion II is true

193 PACE IIT & MEDICAL: Mumbai / Delhi & NCR / Goa / Akola / Kolkata / Nashik / Pune / Bokaro / Dubai
NTSE-MENTAL ABILITY

27. Statements : (i) Some fruits are aflg0es.


(ii) Some fruits are not guavas
Conclusions: I. All fruits are mangoes.
II. All mangoes are fruits.
(A) Only conclusion I is true
(B) Only conclusion II is true
(C) Both conclusions I and II are true
(D) Neither conclusion I nor conclusion II is true

28. Statements (i) No horse is dog.


(ii) All dogs are elephants.
Conclusions I. No elephant is horse.
II. Some elephants are dogs.
(A) Only conclusion I is true
(B) Only conclusion II is true
(C) Both conclusions I and II are true
(D) Neither conclusion I nor conclusion II is true

Answer Key

1. (a) 2. (d) 3. (a) 4. (b) 5. (d) 6. (a) 7. (d) 8. (a) 9. (d) 10. (c)
11. (a) 12. (d) 13. (d) 14. (d) 15. (d) 16. (b) 17. (a) 18. (d) 19. (d) 20. (b)

21. (a) 22. (d) 23. (b) 24. (c) 25. (c) 26. (c) 27. (d) 28. (b)

194 PACE IIT & MEDICAL: Mumbai / Delhi & NCR / Goa / Akola / Kolkata / Nashik / Pune / Bokaro / Dubai
NTSE-MENTAL ABILITY

15. ANALOGY

‘Analogy’ means ‘Similarity’. A particular relationship is given and another similar relationship has to be
identified from the alternatives provided. Analogy tests are, therefore, meant to test a candidate’s overall
knowledge, power of reasoning and ability to think concisely and accurately.

Kinds of Relations

(a) Study & Topic Relationship:

1. Botany Plants
(Botany means study of Plants)
Zoology : Animals
Pathology : Diseases
Cardiology : Heart
Astronomy : Planets
Astrology : Future
Cytology : Cells
Geology : Earth
Mycology : Fungi
Craniology : Skill
Ornithology : Birds
Nephrology : Kidney
Pedology : Soil
Phycology : Algae
Histology : Tissues
Nidology : Nests
Seismology : Earthquakes
Penology : Punishment
Archaeology : Artifacts
Ecology : Environment
Anthology : Collection of Poems
Trigonometry : Triangles
Paleography : Writings
Semantics : Language
Hematology : Blood
Malacology : Molluscs
Bryology : Bryophytes
Taxonomy : Classification
Orography : Mountains

(b) Worker & Tool Relationship:

2. Laborer : Spade
(Spade is a tool used by a Laborers)
Carpenter : Saw Wood cutter : Axe
Blacksmith : Anvil Soldier : Gun
Tailor : Needle Chef: Knife
Farmer : Plough Author: Pen
Warrior: Sword Sculptor : Chisel

195 PACE IIT & MEDICAL: Mumbai / Delhi & NCR / Goa / Akola / Kolkata / Nashik / Pune / Bokaro / Dubai
NTSE-MENTAL ABILITY

Mason : Plumb line Jockey: Tack


Gardener : Harrow Surgeon : Scalpel
Astronomer : Telescope Lumberjack : Axe
Painter : Brush Violinist : Bow
Barber : Scissors Butcher : Chopper
Doctor : Stethoscope Cobbler : Awl

(c) Tool & Action Relationship:

3. Needle : Saw
(A needle is used for sewing)
Knife : Cut Gun: Short
Pen : Write Microscope: Magnify
Spanner : Grip Sword : Slaughter
Filter : Purity Spade : Dig
Mattock: Dig Steering : Drive
Spoon : Feed Chisel : Carve
Axe: Grind Shield : Guard
Auger: Bore Oar: Row
Loudspeaker : Amplify Shovel: Scoop

(d) Worker and Working place:

4. Chef : Kitchen
(Chef works in a Kitchen)
Farmer : Field Warrior : Baffle field
Engineer : Site Sailor : Ship
Pilot : Cockpit Beautician : Parlor
Actor: Stage Mechanic : Garage
Painter : Gallery Grocer: Grocery
Waiter : Restaurant Gambler : Casino
Servant : House Worker: Factory
Umpire : Pitch Teacher : School
Artist : Theatre Doctor : Hospital
Clerk : Office Driver : Cabin
Scientist : Laboratory Lawyer: Court

(e) Worker & Product:

5 Poet: Poem
(Poet writes poem)
Farmer : Crop Author: Book
Cobbler: Shoes Mason : Wall
Editor : Newspaper Hunter: Prey
Carpenter : Furniture Butcher: Meat
Gold Smith : Ornaments Producer : Film
Architect : Design Tailor : Clothes
Teacher : Educations Dramatist : Play
Choreographer : Ballet Judge : Justice

196 PACE IIT & MEDICAL: Mumbai / Delhi & NCR / Goa / Akola / Kolkata / Nashik / Pune / Bokaro / Dubai
NTSE-MENTAL ABILITY

(f) Product and Raw Material:


6 Cloth : Fibre
(Cloth is made of Fibre)
Paper: Pulp Book: Paper
Jewellery: Gold Oil: Seed
Road : Asphalt Sack : Jute
Metal : Ore Fabric : Yarn
Pullover: Wool Furniture: Wood
Butter: Milk Wine : Grapes
Omelette : Egg Rubber : Latex
Wall : Brick Shoes : Leather
Prism : Glass Linen : Flax
Jaggery : Sugarcane Chair : Cane

(g) Instrument & Measurement:


7. Scale : Length
(Scale is an Instruments used to measure length)
Ammeter : Current
Balance : Mass
Thermometer : Temperature
Odometer : Speed
Hygrometer : Humidity
Screw gauge : Thickness
Seismograph : Earthquake
Anemometer : Wind
Barometer : Pressure
Taseometer : Strains
Raingauge : Rain
Sphygmomanometer : Blood Pressure

(h) Quantity & unit:


8. Time : second
(Seconds is the unit of Time)
Force: Newton Length : Meter
Energy: Joule Work : Joule
Current : Ampere Volume : Litre
Temperature : Degrees Potential : Volt
Mass : Kilogram Pressure : Pascal
Area : Hectare Conductivity: Mho
Resistance : Ohm Angle : Radians
Magnetic field : Oersted Power : Watt
Luminosity : Candela Frequency: Hertz

(i) Animal & Young ones:


9. Dog : Puppy
(Puppy is the young one of Dog)
Lion : Cub Man : Child
Hen : Chicken Sheep : Lamb
Cow: Calf Cat: Kitten
Duck : Duckling Horse : Pony! Calf
Insect : Larva Stallion : Colt

197 PACE IIT & MEDICAL: Mumbai / Delhi & NCR / Goa / Akola / Kolkata / Nashik / Pune / Bokaro / Dubai
NTSE-MENTAL ABILITY

Butterfly: Caterpillar Frog : Tadpole


Pig : Farrow Tortoise : Turtle

(j) Male & Female:


10. Tiger : Tigress
(Tigress is Female tiger )
Son : Daughter Gentleman : Lady
Nephew : Niece Drone: Bee
Dog : Bitch Stage: Doe
Sorcerer : Sorceress Horse: Mare
Lion : Lioness

11. Vacant: Empty


(Empty means almost the same as Vacant)
Substitute : Replace Blend : Mix
House : Home Solicit : Request
Flaw: Defect Fierce: Violent
Dearth: Scarcity Ban: Prohibition
Mend: Repair Assign: Allot
Abduct: Kidnap Sedate: Calm
Alight: Descend Presage: Predict
Pressure: Assume Fallacy: illusion
Dissipate: Squander Brim: Edge
Dissipate: Squander Haughty: Proud

(I) Word & Antonym:

12. Good: Bad


Cruel : Kind Best : Worst
Sink: Float Strong : Weak
Initial : Final Start: End
Ignore : Notice Advance: Retreat
Create : Destroy Gentle : Harsh
Deep ; Shallow Affirm: Deny
Kindle : Extinguish Mourn : Rejoice
Cordial : Hostile Kindle :Extinguish
Chaos : Peace Fresh : Stale
Condense : Expand Lend : Borrow
Gradual : Abrupt (Sudden)

(m) Words & Intensity:


13. Quarrel : War
Anger: Rage Kindle : Burn
Error : Blunder Wish : Desire
Sink : Drown Unhappy: Sad
Famous : Renowned Moist : Drench
Crime : Sin Refuse : Deny

198 PACE IIT & MEDICAL: Mumbai / Delhi & NCR / Goa / Akola / Kolkata / Nashik / Pune / Bokaro / Dubai
NTSE-MENTAL ABILITY

Word Analogy
Direction : (1 to 3) In the following questions, choose the words that show the same relationship as given
in the each questions.

1. Match is related to Win in the same way as Examination is related to


(A) Write (B) Appear (C) Success (D) Attempt

Sol. (C) One of outcomes of ‘Match’ is ‘Victory’. Likewise ‘Success’ is one of the outcomes of
‘Examination’.

2. Boat is related to Oar in the same way as Bicycle is related to


(A) Road (B) Wheel (C) Seat (D) Paddle

Sol. (D) ‘Oar’ is a devise used to push a ‘Boat’. Likewise ‘Paddle’ is used to push the ‘Bicycle’.

3. As Earthquake is related to Seismograph in the same way, Milk is related to which ?


(A) Hydrometer (B) Hygrometer (C) Spherometer (D) Lactometer

Sol. (D) ‘Seismograph’ is an instrument by which intensity of ‘Earthquake’ Is measured. In same manner
the purity of milk is measured by lactometer.

Directions : (4 to 5) In each of the following questions, there is a certain relationship between two given
words on one side of :: and one word is give on another side of:: while another word is to be found from the
given alternatives, having the same relation with this word as the words of the given pair bear. Choose the
correct alternatives.

4. Ice: Water:: Solid:?


(A) Liquid (B) Solution (C) Matter (D) Matte

Sol. (A) First is made from second.

5. Tea : Leaves: : Coffee:?


(A) Leaves (B) Beverage (C) Seeds (D) Plant

Sol. (C) Latter is the original form of the former.

Directions : (6 to 7) In each of the following questions, there are two terms to the left of the sign :: which
are related in some way. Obtain the same relationship between the term to the right of the sign :: from one
of the four alternatives given under it.

6. Factory: Production::
(A) School : Education (B) Teacher : Discipline
(C) Mother: Love (D) Plants : Fruit

Sol. (A) Production is carried out in a factory similarly education is given by in a school.

7. Mosquito: Malaria ::
(A) Tobacoo : Cancer (B) Road : Accident
(C) Housefly: Food (D) Soil : Erosion

Sol. (A) Second is the causes of first.

199 PACE IIT & MEDICAL: Mumbai / Delhi & NCR / Goa / Akola / Kolkata / Nashik / Pune / Bokaro / Dubai
NTSE-MENTAL ABILITY

Directions : (8 to 9) In each of the following questions, a group of three interrelated words is given. Choose
a word from the given alternatives, that belongs to the same group.

8. Potato: Carrot: Radish


(A) Tomato (B) Spinach (C) Sesame (D) Groundnut

Sol. (D) Groundnut [All of these grow under the ground.

9. Basket: Pail: Pan


(A) Spoon (B)Bowl (C) Fork (D) Knife

Sol. (B) Bowl [These are the things for keeping something].

Directions : (10 to 11) Three words are given in each question, which have something in common among
themselves. Out of the four given alternatives, choose the most appropriate description about these three
words.

10. Spinach : Fenugreek : Celery


(A) These are cactus plant (B) These are wild flowers
(C) These are wild plants (D) These are leafy vegetables

Sol. (D) These are leafy vegetables.

11. Petrol : Phosphorus : Cooking gas


(A) They are fuels (B) They are highly inflammable
(C) They can’t be sold without permit (D) India has to import them

Sol. (B) They are highly inflammable.

Number Analogy

This section deals with four types of questions.

1. Choosing a number related to a given number in the same manner as the two numbers of another
given pair are related to each other;

2. Choosing a similarly related pair as the given number pair on the basis of the relation between
the numbers in each pair.

3. Choosing a number similar to a group of numbers on the basis of certain common properties that
they possess;

4. Choosing a number set similar to a given number set.

Directions : (12 to 14) In each of the following questions, there is a certain relation between two given
number on one side of : : and one number is given on another side of: : while another number is to be found
from the given alternatives, having the same relation with this number as the numbers of the given pair bear.
Choose the best alternative.

12. 3 : 24 :: 5 : ?
(A) 120 (B) 45 (C) 124 (D) 27

200 PACE IIT & MEDICAL: Mumbai / Delhi & NCR / Goa / Akola / Kolkata / Nashik / Pune / Bokaro / Dubai
NTSE-MENTAL ABILITY

Sol. (A) 33  3  24 similarly 53  5  120

13. 11: 1331 :12:?


(A) 1782 (B) 1728 (C) 1792 (D) 1772

Sol. (B) 113  1331 similarly123  1728

14. 6:18::4:?
(A) 2 (B) 6 (C) 8 (D) 16

Sol. (C) 62 = 36 36 + 2 = 18 similarly


42  16,16  2
Directions : (15 to 16) Question consists of a pair of numbers that have a certain relationship to each other,
followed by four other pairs of numbers given as alternatives. Select the pair in which the numbers are
similarly related as in the given pair.

15. 5:18
(A) 30:96 (B) 21; 66 (C) 19:61 (D)11:35

Sol. The relationship is x :  3x  3 

16. 11:1210
(A) 8:448 (B) 6:2160 (C) 7:1029 (D) 9:729

Sol. (A) The relationship is x : x 2  x  1

Direction : (17 to 18) In each of the following questions, choose that set of numbers from the alternative
sets that is similar to the given set ?

17. Give group : (5, 6, 22)


(A)4,8,2 (B)3,8,26 (C) 8, 9, 34 (D) 7, 9, 48

Sol. (C) The pattern is 5  6  11  2  22


So, the same pattern is 8  9  17  2  34

18. Given set: (8, 3,2)


(A) (10,6,5) (B) (63,8, 3) (C) (95, 24, 5) (D) (168, 15, 4)

Sol. (B)
2 2
In each set, 1st number   2n number   1 ; 2nd number   3rd number   1 .

Directions : (19 to 20) In each of the following questions, choose one number which is similar to the
numbers in the given set ?

Ex 19. Given set: 192,282,372


(A) 453 (B) 461 (C) 236 (D) 425

Sol. In all the numbers, the sum of digits is 12.

201 PACE IIT & MEDICAL: Mumbai / Delhi & NCR / Goa / Akola / Kolkata / Nashik / Pune / Bokaro / Dubai
NTSE-MENTAL ABILITY

20. Given set: 363, 489, 579


(A) 562 (B) 471 (C) 382 (D) 281

Sol. (B) Number Sum of digits New sum of digits


363  3  6  3  12  1  2  3
489  4  8  9  21  2  1  3
579  5  7  9  21  2  1  3
471  4  7  1  12  1  2  3

Directions : (21 to 24) In each of the following questions, there are two terms to the left of the sign:: which
are related in some way. Obtain the same relationship between the term to the right of the sign:: from one of
the four alternatives given under it.

21. MANTEL: NAMLET :: VANITY:? .


(A) NAVYIT (B) NAVYTI (C) NAVIYI (D)AVNTIY
Sol. (B) Group of three letters is reversed.

22. LXNU: NYPV::QTBR:?


(A) RUSD (B) SDSU (C) SUDS (D) RSUD

Sol. (C) Second term is obtained from the first by moving its first and third letters two steps forward
while the second and fourth letters one step forward.

23. CG : El :: FJ : ?
(A) JK (B) IJ (C) LM (D) GK

Sol. (D) Letter groups consist of 2 letters in alphabetic order skipping 3 letters immediately following.

24. DFHJ : LNPR :: ? : BDFH


(A) VXZT (B) UVXZ (C) TXVZ (D) TVXZ

Sol. (D) All the letters of the second term are moved eight steps backward to obtain the first term.

202 PACE IIT & MEDICAL: Mumbai / Delhi & NCR / Goa / Akola / Kolkata / Nashik / Pune / Bokaro / Dubai
NTSE-MENTAL ABILITY

EXERCISE

Directions: (1 to 4) In the following questions, choose the words that show the same relationship as
given in the each questions.

1. Heart is related to Blood in the same way as Lung is related to


(a) Oxygen (b) Chest (c) Purification (d) Circulation

2. Disease is related to Pathology in the same way as Planet is related to


(a) Sun (b) Satellite (c) Astrology (d) Astronomy

3. Hour is related to second in the same way as Tertiary is related to


(a) Ordinary (b) Secondary (c) Primary (d) Intermediary

4. Flower is to a Bouquet as Minister is to a.


(a) Voter (b) Cabinet (c) Constituency (d) Department

Directions: (5 to 7) In each of the following questions, there is a certain relationship between two given
words on one side of : : and one word is given on another side of : : while another word is to be found
from the given alternatives, having the same relation with this word as the word of the given pair
bear. Choose the correct alternatives.

5. Mirror : Reflection : : Water : ?


(a) Conduction (b) Reflection (c) Refraction (d) Defraction

6. Oil : Seed : : Butter: ?


(a) Goat (b) Cow (c) Milk (d) Ghee

7. Cobbler : Leather : : Carpenter : ?


(a) Furniture (b) Wood (c) Hammer (d) Chair

Directions: (8 to 10) In each of the following questions, there are two terms to the left of the sign : :
which are related in some way. Obtain the same relationship between the term to the right of the sign
: : from one of the four alternatives given under it

8. Water : Oxygen
(a) Helium : Nitrogen (b) Salt : Sodium (c) Tree : Plant (d) Food : Hunger

9. Manager : Cabin
(a) Driver: Train (b) Captain : Desk (c) Pilot : Cockpit (d) Servant : Hospital

10. Aeroplane : Hanger


(a) Train: yard (b) Train: Plat form (c) Train : Rail (d) Train : Railway station

Direction: (11) In the following questions, a group of three interrelated words is given. Choose a word
from the given alternatives, that belongs to the same group.

11. Botany : Zoology : Cardiology


(a) Morphology (b) Seismology (c) Pedology (d) Taxonomy

203 PACE IIT & MEDICAL: Mumbai / Delhi & NCR / Goa / Akola / Kolkata / Nashik / Pune / Bokaro / Dubai
NTSE-MENTAL ABILITY

Directions: (12 to 13) Three words are given in each question, which have something in common
among themselves. Out of the four given alternatives, choose the most appropriate description about
these three words.

12. Food : Cyclone : Earthquakes


(a) They take toll of many lives (b) They move from one place to another
(c) They are caused by low pressure (d) They are natural calamities

13. Water : Glass : Mirror


(a) They are without colour (b) Reflection can be seen in them
(c) They are used in day to day life (d) They are transparent

Directions: (14 to 17) In each of the following questions, there is a certain relation between two given
number on one side of : : and one number is given on another side of : : while another number is to be
found from the given alternatives, having the same relation with his number as the numbers of the
given pair bear. Choose the best alternative.

14. 425 : 2 : : 613: ?


(a) 1 (b) 2 (c) 3 (d) 4

15. 7584 : 4251 : : 4673 : ?


(a) 1367 (b) 1340 (c) 1530 (d) None of these

16. 225 : 257 : : 289:?


(a) 301 (b) 316 (c) 320 (d) 325

17. 9 : 79 : : 100 : ?
(a) 901 (b) 1009 (c) 9889 (d) 9998

Directions: (18 to 20) Question consists of a pair of numbers that have a certain relationship to each
other, followed by four other pairs of numbers given as alternatives. Select the pair in which the
numbers are similarly related as in the given pair.

18. 12 : 144
(a) 22 : 464 (b) 20 : 400 (c) 15 : 135 (d) 10 : 140

19. 5 : 35
(a) 7 : 77 (b) 9 : 45 (c) 11 : 55 (d) 3 : 24

20. 7 : 24
(a) 30 : 100 (b) 23 : 72 (c) 19 : 58 (d) 11 : 43

Directions: (21 to 23) In each of the following questions, choose that set of numbers from the
alternative sets that is similar to the given set?

21. Given set: (8, 15, 24)


(a) (6, 13, 21) (b) (10, 17, 28) (c) (11, 18, 27) (d) (13, 20, 32)

22. Given set: (2, 14, 16)


(a) (2, 7, 8) (b) (2, 9, 16) (c) (3, 21, 24) (d) (4, 15, 18)

204 PACE IIT & MEDICAL: Mumbai / Delhi & NCR / Goa / Akola / Kolkata / Nashik / Pune / Bokaro / Dubai
NTSE-MENTAL ABILITY

23. Given set : (3, 18, 36)


(a) (2, 10, 16) (b) (4, 24, 48) (c) (6, 42, 48) (d) (12, 72, 96)

Directions: (24 to 26) In each of the following questions, choose one number which is similar to the
numbers in the given set?

24. Given set: 282, 354, 444


(a) 453 (b) 317 (c) 337 (d) 265

25. Given set: 992, 733, 845


(a) 114 (b) 326 (c) 425 (d) 947

26. Given set: 134, 246, 358


(a) 372 (b) 459 (c) 572 (d) 684

Directions: (27 to 80) In each of the following questions, there are two terms to the left of the sign : :
which are related in some way. Obtain the same relationship between the term to the right of the sign
: : from one of the four alternatives given under it.

27. ACF : EDG : : IEH: ?


(a) OFJ (b) OFI (c) GHI (d) LMN

28. FG : LM : : ? : ?
(a) NO : TU (b) HI : RS (c) GH : KL (d) DE : BA

29. AEZ : EIY : : IOX : ?


(a) UYZ (b) AEX (c) EIX (d) OUW

30. ? : IJGH : : STQR : NOLM


(a) OPMN (b) NOLM (c) MNKL (d) LMNO

31. KLQM : CFMK : : NRPT : ?


(a) FLLR (b) HIJH (c) FLTM (d) RLTM

32. CEHG : XVST : : ? : OMJK


(a) LMNO (b) LNQP (c) NLPQ (d) OPMN

33. APOC : ? : : ITSK : MVUN


(a) DRQH (b) ERQF (c) EQRG (d) DQRH

34. NM : QP : : ZY:?
(a) MQ (b) YW (c) CB (d) YZ

35. BACE : DACE : : FACE : ?


(a) HACE (b) LACE (c) CASE (d) NACE

36. ZA : YB : : XC : ?
(a) YZ (b) NM (c) BC (d) WD

37. ACEG : ? : : BDFH : KMOQ


(a) LMNO (b) JLNP (c) JNLO (d) JLON

205 PACE IIT & MEDICAL: Mumbai / Delhi & NCR / Goa / Akola / Kolkata / Nashik / Pune / Bokaro / Dubai
NTSE-MENTAL ABILITY

38. XAYZ : PSRQ : : JMKL : ? (NTSE Stage-I/Raj./2007)


(a) UVXZ (b) YZVX (c) WZYX (d) WXYZ

39. PQRR : RQPR : : CDEE : ? (NTSE Stage-I/Raj./2007)


(a) EDCE (b) EECD (c) ECDE (d) CDEF

40. ADCB : ZWXY : : GJIH : ? (NTSE Stage-I/Raj./2007)


(a) TQRS (b) PSRQ (c) SVUT (d) TSRQ

41. GiL : J I O : : LnQ : ? (NTSE Stage-I/Raj./2007)


(a) O p S (b) P r U (c) O r U (d) O q T

42. DFJL : OQUW : : EGKM : ? (NTSE Stage-I/Raj./2007)


(a) NPTU (b) JLPR (c) PRVX (d) RTWY

43. BDG : YWT : : DFI: ? (NTSE Stage-II, 2007)


(a) XVS (b) WUR (c) WTR (d) TQN

44. RVLP : CGWA : : ? : EIYC (NTSE Stage-II, 2007)


(a) BFZW (b) DHXB (c) DFXZ (d) FHMN

45. LOQ : GJL : : ? : ADF (NTSE Stage-I/Raj./2008)


(a) BEG (b) FIK (c) PSW (d) CEG

46. BCE : FGM : : ABD : ? (NTSE Stage-I/Raj./2008)


(a) EDK (b) DEH (c) EFL (d) CBA

47. DEF : IKM : : NOP : ? (NTSE Stage-I/Raj./2008)


(a) UVX (b) SUW (c) GHI (d) USW

48. TPVOE : UOWNF : : POISE : ? (NTSE Stage-I/Raj./2008)


(a) QNTHF (b) QJNRF (c) QPTJY (d) QNJRF

49. EGIK : HJLN : : SUWY : ? (NTSE Stage-I/Raj./2008)


(a) VXZB (b) VXBZ (c) VBXZ (d) VZXB

50. YOU : QWA : : SIO : ? (NTSE Stage-II, 2008)


(a) VRS (b) PKU (c) KQU (d) TZB

51. APOC : ? : : ITSK : MVUN (NTSE Stage-II, 2008)


(a) DRQH (b) EQRH (c) DRQF (d) ERQF

52. RVX : IDF : : YSP : ? (NTSE Stage-II, 2008)


(a) JJF (b) GJG (c) FKF (d) FFK

53. AAEI : PPTX : : GGKO : ? (NTSE Stage-II, 2008)


(a) LLVQ (b) QQRS (c) VVZD (d) JJPT

54. 7 : 42 : 9 : ? (NTSE Stage-II, 2008)


(a) 56 (b) 72 (c) 82 (d) 86

206 PACE IIT & MEDICAL: Mumbai / Delhi & NCR / Goa / Akola / Kolkata / Nashik / Pune / Bokaro / Dubai
NTSE-MENTAL ABILITY

3 16 4
55. : :: : ? (NTSE Stage-II, 2008)
2 81 3
27 81 54 27
(a) (b) (c) (d)
64 256 128 128

56. 6 : 35 : : 11 : ? (NTSE Stage-II, 2008)


(a) 121 (b) 120 (c) 56 (d) 54

57. EGIK : HJLN : : SUWY : ? (NTSE Stage-I/Raj./2009)


(a) VXZB (b) VXBZ (c) VBXZ (d) VZXB

58. DFGE : IKLJ : : NPQO : ? (NTSE Stage-I/Raj./2009)


(a) STVU (b) SUTV (c) SUVT (d) SVTU

59. dIk : gLn : : oTv : ? (NTSE Stage-I/Raj./2009)


(a) fKm (b) rWy (c) pUw (d) RwY

60. ABC : BCD : : PQR : ? (NTSE Stage-I/Raj./2009)


(a) RQS (b) SRQ (c) QRS (d) RQP

61. MKig : jhFD : : RPnl : ? (NTSE Stage-I/Raj./2009)


(a) OMki (b) oMKi (c) OmkI (d) omKI

62. 64 : 16 : : 27:? (NTSE Stage-I/Raj./2009)


(a) 1 (b) 4 (c) 8 (d) 9

63. 9 : 125 : : 16: ? (NTSE Stage-I/Raj./2009)


(a) 216 (b) 210 (c) 204 (d) 198

64. 6 : ? : : 20 : 130 (NTSE Stage-I/Raj./2009)


(a) 24 (b) 30 (c) 36 (d) 40

65. ? : 6 : : 12 : 20 (NTSE Stage-I/Raj./2009)


(a) 0 (b) 2 (c) 3 (d) 4

66. 10 : 26 : : 50 : ? (NTSE Stage-I/Raj./2009)


(a) 142 (b) 132 (c) 122 (d) 112

67. CEHJ : XVSQ : : EGJL : ? (NTSE Stage-II, 2009)


(a) VTQO (b) VTRP (c) WTRO (d) VSQN

68. CFKN : AIGS : : ? : DOLX (NTSE Stage-II, 2009)


(a) BRHC (b) FMQT (c) FLPS (d) FLHS

69. DFIM : WSPN : : ? : YWTP (NTSE Stage-II, 2009)


(a) CGJL (b) BFIK (c) BDHJ (d) BFJL

70. AZCX : DWGT : : BYEV : ? (NTSE Stage-II, 2009)


(a) CXEV (b) AEZV (c) FUJQ (d) GTJQ

207 PACE IIT & MEDICAL: Mumbai / Delhi & NCR / Goa / Akola / Kolkata / Nashik / Pune / Bokaro / Dubai
NTSE-MENTAL ABILITY

71. 18 : 81 : : 22 : ? (NTSE Stage-II, 2009)


(a) 98 (b) 121 (c) 212 (d) 222

72. 531 : 81 : : 642 : ? (NTSE Stage-II, 2009)


(a) 91 (b) 111 (c) 144 (d) 196

73. 7 : 42 : : 9 : ? (NTSE Stage-II, 2011)


(a) 75 (b) 65 (c) 46 (d) 72

74. 8 : 32 : : 12 : ? (NTSE Stage-II, 2011)


(a) 52 (b) 68 (c) 72 (d) 75

75. DGOT : JKUX : : FINP : ? (NTSE Stage-II, 2011)


(a) KMTU (b) LNTS (c) LMTT (d) MNTU

76. EIKR : HMPX : : GKMT : ? (NTSE Stage-II, 2011)


(a) KORY (b) JORZ (c) JNSZ (d) INQZ

77. FILP : AEIN : : IKNT : ? (NTSE Stage-II, 2011)


(a) CGLR (b) EGJR (c) DGKR (d) DHLS

78. ILNT : FHIN : : KOSV : ? (NTSE Stage-II, 2011)


(a) IKMN (b) HKOQ (c) ILNR (d) HKNP

79. 03 : 10 : : 08 : (?) (NTSE Stage-I/Raj./2012)


(a) 17 (b) 16 (c) 14 (d) 13

80. 01 : 08 : : (?) : 125 (NTSE Stage-I/Raj./2012)


(a) 10 (b) 12 (c) 15 (d) 16

81. 08 : 28 : : (?) : 65 (NTSE Stage-I/Raj./2013)


(a) 9 (b) 12 (c) 15 (d) 18

82. 35 : 91 : : 189 : (?) (NTSE Stage-I/Raj./2013)


(a) 343 (b) 341 (c) 280 (d) 210

7 3 19
83. : :: :  ?  (NTSE Stage-I/Raj./2013)
11 17 23
25 29 23 29
(a) (b) (c) (d)
27 31 29 33

208 PACE IIT & MEDICAL: Mumbai / Delhi & NCR / Goa / Akola / Kolkata / Nashik / Pune / Bokaro / Dubai
NTSE-MENTAL ABILITY

Answer Key

1. (a) 2. (d) 3. (c) 4. (b) 5. (c) 6. (c) 7. (b) 8. (b) 9. (c) 10. (a)
11. (a) 12. (d) 13. (c) 14. (a) 15. (b) 16. (d) 17. (d) 18. (b) 19. (a) 20. (b)

21. (c) 22. (c) 23. (b) 24. (a) 25. (c) 26. (b) 27. (b) 28. (a) 29. (d) 30. (b)

31. (a) 32. (b) 33. (b) 34. (c) 35. (a) 36. (d) 37. (b) 38. (c) 39. (a) 40. (a)

41. (d) 42. (c) 43. (b) 44. (b) 45. (b) 46. (c) 47. (b) 48. (d) 49. (a) 50. (c)

51. (d) 52. (b) 53. (c) 54. (b) 55. (b) 56. (b) 57. (a) 58. (c) 59. (b) 60. (c)

61. (d) 62. (d) 63. (a) 64. (b) 65. (b) 66. (c) 67. (a) 68. (c) 69. (b) 70. (c)

71. (b) 72. (c) 73. (d) 74. (c) 75. (c) 76. (b) 77. (c) 78. (d) 79. (a) 80. (d)
81. (c) 82. (b) 83. (b)

PREVIOUS YEARS QUESTIONS

1. In the following question there is a specific relationship between the first and second term. The same
relationship exists between the third and fourth term. Considering the relationship select correct
alternative to replace question mark: (NTSE-MAHARASHTRA STAGE-1-2020)
4 : 80 :: 21: ?
(a) 9702 (b) 8702 (c) 8820 (d) 421

(Q.2 to 4) Directions : In the following questions there is a specific relation between first and second term.
The same relationship exists between third and fourth term, which will replace the question mark. Select the
correct alternative from the given alternatives. (NTSE-MAHARASHTRA STAGE-1-2020)

2. EJOT : VQLG : : BGLQ : ?


(a) DINS (b) RMHC (c) SNID (d) EJOT

3. FJUL : BOQQ : : LHRX : ?


(a) BKPR (b) MNCC (c) HRYY (d) HMNC

4. QPRS : TUWV : ; JIKL : ?


(a) MNOP (b) NMOP (c) MNPO (d) NMPO

5. In the following question there is a specific relation between first and second term. The same
relationship exists between third and the fourth term. Considering the same relationship choose the
correct alternative that will replace the question mark. (NTSE-MAHARASHTRA STAGE-1-2019)
11529 : 72135 : : 152943 : ?
(a) 213549 (b 223649 (c) 224194 (d) 215049

(Q.6 to 8) Directions : In each of the following questions there is a specific relationship between the first
and the second term. The same relationship exists between the third and the fourth term. Find the relation
and choose the correct answer to replace the question mark. (NTSE-MAHARASHTRA STAGE-1-2019)

209 PACE IIT & MEDICAL: Mumbai / Delhi & NCR / Goa / Akola / Kolkata / Nashik / Pune / Bokaro / Dubai
NTSE-MENTAL ABILITY

6. KMF : LLH : : R,S : ?


(a) SLR (b) SLU (c) SSU (d) SUS

7. ADE : FGJ : : KNO : ?


(a) PQR (b) PQT (c) RQP (d) TPR

8. ? : ALKLO : : WOULD : TLRIA


(a) BLOCK (b) BARGE (c) CONES (d) DONOR

9. Direction : In the following question there is a specific relation between the first and second term.
The same relationship exists between third and fourth term. Understanding the relationship find out
the correct alternative to replace the question mark.
25 : 100 :: 81: ?
(a) 648 (b) 468 (c) 864 (d) 684

(Q 10 to 12) Direction : In the following questions there is a specific relation between the first and the
second term. The same relationship exists between the third and the fourth term Understanding the
relationship find out the correct alternative to replace the question mark.
(NTSE-MAHARASHTRA STAGE-1-2018)
10. FILM : ADGH:: MILK : ?
(a) ADGE (b) HDGE (c) HEGF (d) HDGF

169
11. MK : : : JH : ?
121
16 25 64 121
(a) (b) (c) (d)
4 4 100 64

12. ? : DURXQG : : POLICE : SROLFH


(a) ARMOUR (b) AROUND (c) GROUND (d) SHOULD

Q. 13 and 14: Directions - In each of the following questions there is a specific relationship between the
first and second term. The same relationship exists between third and fourth term. Understanding this
relationship, find out the correct alternative to replace the question mark.
(NTSE-MAHARASHTRA STAGE-1-2017)

13 AMK : SPJ : : KNM : ?


(a) CQN (b) BQL (c) CQL (d) BLQ

14 N9M : PI1Q : : VI4T : ?


(a) X17Z (b) X16W (c) XI5Y (d) X16X

(Q. 15 and 16) Directions : In each of the following question there is a specific relationship between the
first and second term. The same relationship exists between the third and fourth term, which will replace the
question mark. Select the correct term from the alternatives given.
(NTSE-MAHARASHTRA STAGE-1-2016)
15. BYXC : DWVE : : ? : HSRI
(a) FVSG (b) FTSG (c) FUTG (d) FUG

16. 2YC24 : 4WE22 : : 6UG20 : ?


(a) 88J18 (b) 8SK18 (c) 8S118 (d) 8RJ18

210 PACE IIT & MEDICAL: Mumbai / Delhi & NCR / Goa / Akola / Kolkata / Nashik / Pune / Bokaro / Dubai
NTSE-MENTAL ABILITY

(Q. 17 to 18)-Directions : In each of the following questions there is a specific relationship between the first
and second term. The same relationship exists between the third and fourth term which will replace the
question mark (?). Select the correct term from the alternatives given.
(NTSE-MAHARASHTRA STAGE-1-2015)

17. AXD : EWB : : ? : JRG


(a) ETH (b) FSI (c) HRK (d) FRJ

18. CD10Q : FG16C : : 1J22O : ?


(a) LM28A (b) LM28Z (c) LM28B (d) LM28C

19. 512 : 44 : : 125 : ?


(a) 55 (b) 33 (c) 23 (d) 34

20. 467 : 9 : : 771 : ?


(a) 9 (b) 8 (c) 6 (d) 15

(Q. 21 to 22) Directions : In each of the following questions there is a specific relationship between the first
and second term. The same relationship exists between the third and fourth term which will replace the
question mark (?). Select the correct term from the alternatives given :
(NTSE-MAHARASHTRA STAGE-1-2014)
21. 23 : 8 : : 25 : ?
(a) 32 (b) 9 (c) 64 (d) 22

22. 531 : 99 : : ? : ?
(a) 451 : 55 (b) 321 : 44 (c) 642 : 66 (d) 212 : 11

23. BYCW : LOKQ : : ? : JQIS


(a) FUEW (b) DWFU (c) DWEU (d) DVFU

24. ACG : ZXT : : HJN ; ?


(a) SQM (b) TRN (c) SQN (d) SOM

(Q. 25 to 26) Directions : In each of the following questions there is a specific relationship between the first
and second term. The same relationship exists between the third and fourth term which will replace the
question mark (?). Select the correct term from the alternatives given.
(NTSE-MAHARASHTRA STAGE-1-2013)
25. EFH : ILM :: ? : ?
(a) ILM : OPQ (b) EGH : IMN (c) OPR : URT (d) OPQ : UBT

26. BJH : EQL : : DNJ : ?


(a) IYO (b) KZQ (c) GKI (d) GUN

(Q. 27 to 29)-Direction : In each of the follo4ng questions there is a specific relationship (between the 1first
and second term. The same relationship exists between the third and the fourth term which will replace the
question mark. Select the correct term from the alternatives given.
(NTSE-MAHARASHTRA STAGE-1-2012)
27. AYXD : ZBCW : : ? : QKLN
(a) MOPJ (b) JKLM (c) JPOM (d) MJKL

211 PACE IIT & MEDICAL: Mumbai / Delhi & NCR / Goa / Akola / Kolkata / Nashik / Pune / Bokaro / Dubai
NTSE-MENTAL ABILITY

28. KHOT : 45 : : VILE : ?


(a) 48 (b) 68 (c) 76 (d) 84

29. AEH : IMP : : BFI : ?


(a) JMP (b) JNQ (c) JNP (d) JMQ

(Q. 30 to 32)- Directions : In each of the following questions there is a specific relationship between the
first and second number. The same relationship exists between the third and the fourth number which will
replace the question mark. Select the correct term from the alternatives given.
(NTSE-MAHARASHTRA STAGE-1-2012)
30. 7 : 50 : : 9 : ?
(a) 80 (b) 66 (c) 83 (d) 84

31. 2149370 : 2854610 : : 2735630 : ?


(a) 7536501 (b) 832150 (c) 880890 (d) 9437102

32. 13 : 42 : : 17 : ?
(a) 72 (b) 92 (c) 38 (d) 19

(Q. 33 to 36)-Directions : In the following questions, a question column containing a group of letters is
given. The group of letters is prepared in a specific manner. Four alternatives are given on right side.
Understanding the arrangement of letters in the question column, find the right alternative, from the given
alternatives which matches the given group. (NTSE-MAHARASHTRA STAGE-1-2012)

Question Column Alternatives


33. DWJQ (a) RPM, VTQ, HFC

34. VTQ (b) AZMN, BYLO, FUHS

35. YBE (c) MQU, AEI, PTX

36. KOS (d) BEH, KNQ, PSV

Answer Key

1. (a) 2. (c) 3. (d) 4. (c) 5. (a) 6. (b) 7. (b) 8. (d) 9. (a) 10. (d)
11.
12. (b) 13. (c) 14. (d) 15. (c) 16. (c) 17. (b) 18. (a) 19. (c) 20. (b)
(bonus)
21. (a) 22. (c) 23. (c) 24. (a) 25. (c) 26. (d) 27. (c) 28. (d) 29. (b) 30. (b)

31. (d) 32. (a) 33. (b) 34. (a) 35. (d) 36. (c)

212 PACE IIT & MEDICAL: Mumbai / Delhi & NCR / Goa / Akola / Kolkata / Nashik / Pune / Bokaro / Dubai
NTSE-MENTAL ABILITY

16. CLASSIFICATION

‘Classification’ means ‘to assort the items of a given group on the basis of certain common quality they
possess and then spot the stranger or odd one out’. In this test, a group of certain items are given, out of
which all except one are similar to one another in some manner. The candidate is required to choose this one
item which does not fit into the given group.

Classification of Word
In these type of questions, certain words/pairs of words are given out of which the words/pairs of words
except one, bear a certain common relationship. The candidate is required to decipher this relationship and
choose the word/the pair in which the words are differently related, as the answer.

Directions : (1 to 5) In each of the following questions, four words are given, out of which three are
same in one way and the fourth one is different from others. Select the odd one.

1. (a) Sun (b) Moon (c) Venus (d) Mars


Sol. (b) All the terms except ‘Moon’ are related to the Solar system.

2. (a) Green (b) Violet (c) Brown (d) Yellow


Sol. (c) Except ‘Brown’ all the colours are present in the rainbow.

3. (a) Milk (b) Fur (c) Milk (d) Rubber


Sol. (d) Only ‘Rubber’ is the tree product.

4. (a) Milk (b) Syrup (c) Squash (d) Cake


Sol. (d) All others are the drinks.

5. (a) Conscience (b) Morality (c) Conduct (d) Weight


Sol. (d) All other terms are used to represent human behavioral personality factors.

Classification of Letter
In these type of questions, usually four groups of letters are given. Three of them are similar to each other in
some manner while one is different and this is to be chosen by the candidate as the answer.

Directions: (6 to 9) In each of the following questions three out of four alternatives contain alphabet
placed in a particular form. Find the one that does not belong to the group.

6. (a) NKMJ (b) FCEB (c) URTQ (d) TQRP


Sol. (d) In all other groups there is a gap of one letter as in the alphabet between third and fourth letter.

7. (a) DW (b) GT (c) KP (d) FR


Sol. (d) In all other pairs of words first and second letters are equidistant from the beginning and end
respectively in the alphabetical series.

8. (a) A8C (b) D22G (c) H42M (d) F34J


Sol. (d) In all other groups number between first and second letter is twice the sun of positions of first
and last letters in the alphabet.

9. (a) KQ14 (b) AY13 (c) MR11 (d) GW15


Sol. (c) In all other groups number at the end is half of the positions of sum of fist and second letters in
the alphabet.
213 PACE IIT & MEDICAL: Mumbai / Delhi & NCR / Goa / Akola / Kolkata / Nashik / Pune / Bokaro / Dubai
NTSE-MENTAL ABILITY

Classification of Number
In these type of questions, certain numbers/groups of numbers are given, out of which all except one share
some common property and hence are alike, while one is different and this number/groups of numbers is to
be chosen as the answer.

Directions: (10 to 15) In the following questions, four numbers/pairs are given out of which three are
alike in some manner while one is different. Choose the one which is different from the rest three.

10. (a) 4 (b) 8 (c) 16 (d) 9


Sol. (b) All other numbers are square of natural numbers.

11. (a) 43 (b) 53 (c) 63 (d) 73


Sol. (c) All other numbers are prime numbers.

12. (a) 3 : 8 (b) 6 : 35 (c) 7 : 50 (d) 1 : 0


Sol. (c) In other numbers second number is one less than the square of first number.

13. (a) 21 : 24 (b) 28 : 32 (c) 14 : 16 (d) 54 : 62


Sol. (d) The ratio among the numbers is 7 : 8

14. (a) 22 : 0 (b) 24 : 12 (c) 23 : 5 (d) 24 : 18


Sol. (d) Second number is the difference of the square of digits of first number.

15. (a) 4, 6, 10, 7 (b) 4, 12, 20, 28 (c) 1, 3, 5, 7 (d) 2, 6, 10, 14


Sol. (a) The difference of consecutive numbers are equal.

214 PACE IIT & MEDICAL: Mumbai / Delhi & NCR / Goa / Akola / Kolkata / Nashik / Pune / Bokaro / Dubai
NTSE-MENTAL ABILITY

EXERCISE

Directions: (1 to 9) In each of the following questions, four words are given, out of which three are
same in one way and the fourth one is different from others. Select the odd one.

1. (a) Gold (b) Silver (c) Bronze (d) Iron

2. (a) Rickshaw (b) Taxi (c) Tonga (d) Cart

3. (a) Camel (b) Goat (c) Cow (d) Dog

4. (a) Yen (b) Lira (c) Dollar (d) Ounce

5. (a) Huge (b) Tiny (c) Heavy (d) Small

6. (a) Teeth (b) Tongue (c) Palate (d) Chin

7. (a) Silk (b) Cotton (c) Nylon (d) Wool

8. (a) Triangle (b) Tangent (c) Square (d) Rhombus

9. (a) Drama (b) Story (c) Poem (d) Novel

Directions: (10 to 24) In each of the following questions three out of four alternatives contain
alphabet/word/number placed in a particular form. Find the one that does not belong to the group.

10. (a) DBF (b) HFK (c) NLP (d) XVZ

11. (a) MrW (b) ChN (c) KpU (d) BgL

12. (a) DFHB (b) KMOJ (c) PRTN (d) XZBV

13. (a) ROQP (b) KHJI (c) VSUT (d) JHIG

14. (a) MADE (b) SHED (c) CEDE (d) DOLT

15. (a) ACE (b) MOQ (c) RTV (d) UWY

16. (a) BDI (b) KMR (c) PRW (d) FHN

17. (a) MLONP (b) HGJIK (c) SRUTV (d) WUYXZ


(NTSE Stage-I/Raj./2007)

18. (a) PRTVN (b) SUWYQ (c) DFHJB (d) NPSUL


(NTSE Stage-I/Raj./2007)

19. (a) VXKN (b) TWIL (c) RUGJ (d) QTFI


(NTSE Stage-I/Raj./2007)

20. (a) GHFIC (b) OPNQM (c) UVTWQ (d) STRUO


(NTSE Stage-I/Raj./2007)
215 PACE IIT & MEDICAL: Mumbai / Delhi & NCR / Goa / Akola / Kolkata / Nashik / Pune / Bokaro / Dubai
NTSE-MENTAL ABILITY

21. (a) YVSR (b) TQNM (c) KHFE (d) NKHG


(NTSE Stage-I/Raj./2007)

22. (a) Ear (b) Nose (c) Throat (d) Tongue


(NTSE Stage-II, 2007)

23. (a) Mathematics (b) Arithmetic (c) Algebra (d) Geometry


(NTSE Stage-II, 2007)

24. (a) CXDW (b) GTHS (c) EVFU (d) QJRK


(NTSE Stage-II, 2007)

Directions: (25 to 70) In the following questions four numbers/pairs/alphabets are given out of which
three are alike in some manner while one is different. Choose the one which is different from the test
three. (NTSE Stage-II, 2007)

25. (a) 2442 (b) 3773 (c) 4545 (d) 6776

26. (a) 27 (b) 125 (c) 1321 (d) 729

27. (a) 9-27 (b) 15-45 (c) 10-30 (d) 20-60

28. (a) 1441 (b) 6996 (c) 5775 (d) 4848

29. (a) 51-28 (b) 37-62 (c) 81-104 (d) 99-76

30. (a) 22,4,5 (b) 34,4,8 (c) 37,4,9 (d) 54,4,13

31. (a) 5 : 25 (b) 3 : 8 (c) 6 : 35 (d) 4 : 15

32. (a) PORQS (b) HGLIM (c) UTWVX (d) ZYBAC


(NTSE Stage-I/Raj./2008)

33. (a) BYWD (b) GTRI (c) KOPM (d) SHFU


(NTSE Stage-I/Raj./2008)

34. (a) KHF (b) HEC (c) PMK (d) EBA


(NTSE Stage-I/Raj./2008)

35. (a) TABLE (b) LATER (c) RATES (d) GREAT


(NTSE Stage-I/Raj./2008)

36. (a) TVS (b) OQN (c) LNK (d) JGI


(NTSE Stage-I/Raj./2008)

37. (a) YXVT (b) GECA (c) VTRP (d) HFDB


(NTSE Stage-II, 2008)

38. (a) CGKO (b) EIMQ (c) GILO (d) BFJN


(NTSE Stage-II, 2008)

216 PACE IIT & MEDICAL: Mumbai / Delhi & NCR / Goa / Akola / Kolkata / Nashik / Pune / Bokaro / Dubai
NTSE-MENTAL ABILITY

39. (a) NMLB (b) NMKB (c) RQPB (d) LKJB


(NTSE Stage-II, 2008)

40. (a) CPQR (b) ERST (c) GTUV (d) XYZK


(NTSE Stage-II, 2008)

41. (a) MNKL (b) IHGJ (c) FEDG (d) QPOR


(NTSE Stage-II, 2008)

42. (a) 64, 8, 27 (b) 349, 625, 524 (c) 512, 1000, 216 (d) 125, 216, 729
(NTSE Stage-II, 2008)

43. (a) 30 (b) 68 (c) 128 (d) 222


(NTSE Stage-II, 2008)

44. (a) 37 : 7 (b) 82 : 10 (c) 168 : 13 (d) 197 : 15


(NTSE Stage-II, 2008)

45. (a) 67, 54 (b) 93, 80 (c) 49, 37 (d) 56, 43


(NTSE Stage-II, 2008)

46. (a) MPV (b) PSY (c) WZF (d) ZCH


(NTSE Stage-I/Raj./2009)

47. (a) EGK (b) LNS (c) SUY (d) ZBF


(NTSE Stage-I/Raj./2009)

48. (a) BFL (b) EIO (c) HLP (d) KOU


(NTSE Stage-I/Raj./2009)

49. (a) LPCG (b) GKHL (c) BFMQ (d) AEOI


(NTSE Stage-I/Raj./2009)

50. (a) ENHO (b) GLHM (c) IJJK (d) KHLI


(NTSE Stage-I/Raj./2009)

51. (a) 1022 (b) 2102 (c) 2002 (d) 3002


(NTSE Stage-I/Raj./2009)

52. (a) 16-96 (b) 21-126 (c) 13-78 (d) 14-70


(NTSE Stage-I/Raj./2009)

53. (a) 4568 (b) 6789 (c) 2345 (d) 5678


(NTSE Stage-I/Raj./2009)

54. (a) AFCJ (b) CHEL (c) KPMT (d) HMJP


(NTSE Stage-II, 2009)

55. (a) TOJD (b) DFHJ (c) NKHE (d) LPTX


(NTSE Stage-II, 2009)

217 PACE IIT & MEDICAL: Mumbai / Delhi & NCR / Goa / Akola / Kolkata / Nashik / Pune / Bokaro / Dubai
NTSE-MENTAL ABILITY

56. (a) 0 (b) 7 (c) 28 (d) 63


(NTSE Stage-II, 2009)

57. (a) 9, 101 (b) 8, 82 (c) 3, 17 (d) 5, 26


(NTSE Stage-II, 2009)

58. (a) 8 5 6 9 (b) 7 8 5 6 (c) 6 7 8 5 (d) 5 6 7 8


(NTSE Stage-II, 2009)

59. (a) EHKN (b) KNRU (c) GJMP (d) MPSV


(NTSE Stage-II, 2011)

60. (a) EY (b) DP (c) BD (d) GT


(NTSE Stage-II, 2011)

61. (a) 130 (b) 60 (c) 24 (d) 6


(NTSE Stage-II, 2011)

62. (a) 11, 132 (b) 9, 90 (c) 8, 56 (d) 6, 42


(NTSE Stage-II, 2011)

63. (a) GEDC (b) AZYX (c) PNML (d) USRQ


(NTSE Stage-I/Raj./2012)

64. (a) SUXB (b) LNQU (c) BDGK (d) JLNQ


(NTSE Stage-I/Raj./2012)

65. (a) 23 (b) 43 (c) 63 (d) 73


(NTSE Stage-I/Raj./2012)

66. (a) Hut (b) Home (c) Palace (d) Restaurant


(NTSE Stage-I/Raj./2012)

67. (a) PROQN (b) DFCEG (c) GIFHE (d) KMJLI


(NTSE Stage-I/Raj./2013)

68. (a) ACEGF (b) FHJLK (c) KMOQP (d) UWYZA


(NTSE Stage-I/Raj./2013)

69. (a) CADGE (b) JHKNL (c) ECFGI (d) XVYBZ


(NTSE Stage-I/Raj./2013)

70. (a) 6253 (b) 1881 (c) 7043 (d) 8210


(NTSE Stage-I/Raj./2013)

71. Which group of letters is different from others? (NTSE Stage-II, 2013)
(a) CBAED (b) IJHGK (c) SRQPT (d) TVWYX

218 PACE IIT & MEDICAL: Mumbai / Delhi & NCR / Goa / Akola / Kolkata / Nashik / Pune / Bokaro / Dubai
NTSE-MENTAL ABILITY

Direction (72 to 75): In questions 16 to 19 three alternatives are alike in a certain way but the rest one
is different. Find out the odd one and write correct answer. (NTSE Stage-I/Raj./2014)

72. (a) AEIF (b) CGKH (c) EILJ (d) GKOL

73. (a) XCJP (b) YEKQ (c) ZGLT (d) AHMV

74. (a) 150 (b) 165 (c) 200 (d) 250

75. (a) Coal (b) Graphite (c) Diamond (d) Gold

76. Which one is different from others? (NTSE Stage-I/Raj./2016)


(a) Haryana (b) Gujarat (c) Rajasthan (d) Shimla

Direction: (77 to 79) In each question three alternatives are alike in a certain way but the rest one is
different. Find out the odd one and write correct answer. (NTSE Stage-I/Raj./2017)

3
77. 10 
2
(a) 23  53 (b) 100   10 (c) 23  53 (d) 2000  2

78. (a) Pacific Ocean (b) Asia (c) Europe (d) Africa

79. (a) Nepal (b) Pakistan (c) Sri Lanka (d) Australia

Answer Key

1. (c) 2. (b) 3. (d) 4. (d) 5. (c) 6. (d) 7. (c) 8. (b) 9. (c) 10. (b)
11. (b) 12. (b) 13. (d) 14. (c) 15. (c) 16. (d) 17. (d) 18. (d) 19. (a) 20. (b)

21. (c) 22. (c) 23. (a) 24. (d) 25. (c) 26. (c) 27. (a) 28. (d) 29. (b) 30. (c)

31. (a) 32. (b) 33. (c) 34. (d) 35. (d) 36. (d) 37. (a) 38. (c) 39. (b) 40. (d)

41. (a) 42. (b) 43. (c) 44. (c) 45. (c) 46. (d) 47. (b) 48. (c) 49. (d) 50. (a)

51. (c) 52. (d) 53. (a) 54. (d) 55. (a) 56. (c) 57. (d) 58. (a) 59. (b) 60. (d)

61. (a) 62. (c) 63. (b) 64. (d) 65. (c) 66. (d) 67. (b) 68. (d) 69. (c) 70. (d)

71. (d) 72. (c) 73. (b) 74. (b) 75. (d) 76. (d) 77. (d) 78. (a) 79. (d)

219 PACE IIT & MEDICAL: Mumbai / Delhi & NCR / Goa / Akola / Kolkata / Nashik / Pune / Bokaro / Dubai
NTSE-MENTAL ABILITY

PREVIOUS YEARS QUESTIONS

Q.1 and 2-Direction: Find the odd term

1. (a) ACEDB (b) HJLIK


(c) TVXWU (d) PRTSQ

2. (a) ZBX (b) VFT


(c) RJO (d) SIQ
(NTSE-MAHARASHTRA STAGE-1-2020)

Q.3 and 4-Directions: In the following questions specific group of letters are given. From the given
alternatives, find out the right letters which matches the given group.

3. GECA ZXVT SQOM


(a) YWUT (b) VTRQ
(c) MKIH (d) LJHF

4. BEIN EHLQ ILPU


(a) NQUZ (b) HKOS
(c) LOSY (d) JMQT

5. BYEV DWHS IRLO


(a) FUKO (b) CXJP
(c) GTDW (d) AZCW
[NTSE-MAHARASHTRA STAGE-1-2020]

Q.6 to 8-Directions: In the following questions specific group of numbers are given. From the given
alternatives, find out the right number which matches the given group

6. 416 749 525


(a) 982 (b) 864
(c) 637 (d) 319

7. 394 648 448


(a) 84 (b) 94
(c) 100 (d) 194

1 2
8. 3 3.2 3.25 3
3 3
(a) 3.5 (b) 5
13
(c) 4.2 (d)
3
[NTSE-MAHARASHTRA STAGE-1-2020]

Q. 9 to 11: Direction: In the following questions a specific group of numbers is given. From the given
alternatives. Find out the correct alternative that matches the given group.

9. 150 576 252


(a) 393 (b) 466
(c) 80 (d) 182
220 PACE IIT & MEDICAL: Mumbai / Delhi & NCR / Goa / Akola / Kolkata / Nashik / Pune / Bokaro / Dubai
NTSE-MENTAL ABILITY

10. 132 736 350


(a) 223 (b) 72
(c) 505 (d) 993

11. 193 454 265


(a) 572 (b) 823
(c) 734 (d) 367

Q.12 and 13: Direction Find the odd term

12. (a) DUFW (b) HQJS


(c) JOLQ (d) AWCZ

13. (a) AEVZ (b) FJQU


(c) CQTX (d) JMOS
[NTSE-MAHARASHTRA STAGE-1-2019]

Q.14 to 16 Direction In each of the following questions find out the group of letters that matches the
given group.

14. AUEFG EOVWX IAPQE


(a) OQRST (b) UEJKL
(c) OKEFG (d) UGHIJ

15. ZXAVT WUESQ TRUPN


(a) VTRPN (b) JHFDB
(c) LJOHF (d) QOMKI

16. BYMN DWJZ GTKP


(a) AZFV (b) CXHS
(c) HSOX (d) EVJP
[NTSE-MAHARASHTRA STAGE-1-2019]
Q.17 and 18: Find the odd term

17. (a) ABDEF (b) JKMNX


(c) GHJKR (d) IJLMT

18. (a) CEG18 (b) KMO42


(c) UWY70 (d) RTV63
[NTSE-MAHARASHTRA STAGE-1-2018]

Q. 19 to 21-Direction: In the following questions a specific group of terms is given. From the given
alternative, find out the correct term that matches the given group.

19. JGDA ZWTQ UROL


(a) LIGD (b) SPMK
(c) LIFC (d) NKHF

20. AGM DJP HNT


(a) GNT (b) EKP
(c) IOV (d) KQW

221 PACE IIT & MEDICAL: Mumbai / Delhi & NCR / Goa / Akola / Kolkata / Nashik / Pune / Bokaro / Dubai
NTSE-MENTAL ABILITY

21. BFYU GKTP FJUQ


(a) AEZW (b) DHWS
(c) IMRO (d) CGXS
[NTSE-MAHARASHTRA STAGE-1-2018]
Q.22 to 24 Direction : In the following questions a specific group of numbers is given. From the given
alternatives, choose the correct alternative that matches the given group

22. 78 26 54
(a) 52 (b) 6
(c) -6 (d) 31

23. 738 4930 2210


(a) 1341 (b) 6877
(c) 222 (d) 518

24. 41 21 69
(a) 89 (b) 87
(c) 107 (d) 105
[NTSE-MAHARASHTRA STAGE-1-2018]
Q.25 and 27 Directions- Find the odd term.

25. (a) 141 (b) 101


(c) 107 (d) 131

26. (a) 6131 (b) 2191


(c) 3312 (d) 3164

27. (a) DHLP (b) FNUB


(c) BDFH (d) KVGR

28. In the following question a specific group of numbers is given. From the given alternatives, find out
the right number which matches the given group.
341, 572, 781
(a) 634 (b) 891
(c) 909 (d) 990
[NTSE-MAHARASHTRA STAGE-1-2017]
Q.29 and 30-Directions:- In the following questions a specific group is given. From the given
alternatives, find out the term which matches the given group.

29. 37, 46, 28


(a) 56 (b) 55
(c) 54 (d) 83

30. 22TB, M23K, QR35


(a) H22K (b) N29O
(c) 35YZ (d) Q47X

31. Find the odd term


(a) B4J (b) L5V
(c) H2D (d) L1H [2016]
222 PACE IIT & MEDICAL: Mumbai / Delhi & NCR / Goa / Akola / Kolkata / Nashik / Pune / Bokaro / Dubai
NTSE-MENTAL ABILITY

Q 32.and 33-Directions:-Find the odd term

32. (a) 749 (b) 694


(c) 568 (d) 487

33. (a) 4321 (b) 3212


(c) 2116 (d) 6211

Q.34 to 37-Direction: - Find the odd term out [2015]

34. (a) AEA8 (b) GK36


(c) RV24 (d) DI41

35. (a) DINR (b) GLQU


(c) AEJN (d) BGLP

36. (a) 8163 (b) 4236


(c) 2632 (d) 5271

37. (a) 103 (b) 241


(c) 211 (d) 133

38. In the following question a specific group of letters is given. From the given alternatives, find out the
right term which matches the given group.
TOL MEK RCP
(a) YAT (b) SFQ
(c) WHV (d) BJX [2015]

Q.39 and 40 -Directions : - Find the odd term out

39. (a) 336 (b) 508


(b) 504 (d) 720

40. (a) 431 (b) 621


(c) 981 (d) 241

Directions: In the following questions a specific group is given. From the given alternatives, find out
the right term which matches the given group.

41. FKR, KPW, DIP


(a) JNU (b) CHO
(c) BGM (d) GMU [2014]

Q42 and 43-Directions : - Find the odd term out

42. (a) EMI (b) BJF


(c) DLH (d) GOJ

43. (a) ABXY (b) FGTU


(c) IJQR (d) CDWX [2014]

223 PACE IIT & MEDICAL: Mumbai / Delhi & NCR / Goa / Akola / Kolkata / Nashik / Pune / Bokaro / Dubai
NTSE-MENTAL ABILITY

Q.44 and 45: Direction: - In the following questions a specific group is given. From the given
alternatives, find out the right term which matches the given group

44. 123 147 173


(a) 201 (b) 225
(c) 169 (d) 144

45. XWVU, SRAI, NMLK


(a) BCDE (b) QRST
(c) MNOP (d) IHGF [2013]

Q.46 to 48

46. (a) 1124 (b) 2349


(c) 5139 (d) 2138

47. (a) 3261 (b) 4331


(c) 3413 (d) 4353

48. (a) 8723 (b) 3467


(c) 8264 (d) 5746

Q.49 and 50-Directions : - In the following questions a specific group is given. From the given
alternatives, find out the right term which matches the given group

49. 657, 954, 369


(a) 468 (b) 835
(c) 782 (d) 665

50. 42, 20, 56


(a) 36 (b) 24
(c) 16 (d) 12 [2012]

Q.51 to 54-Directions:- Find the odd term out

51. (a) AXE (b) CVG


(c) ETJ (d) GRK

52. (a) LJ1 (b) RF7


(c) ZH9 (d) XB11

53. (a) HG (b) JM


(c) VT (d) UR

54. (a) CZHK (b) GALM


(c) BASU (d) XABU

224 PACE IIT & MEDICAL: Mumbai / Delhi & NCR / Goa / Akola / Kolkata / Nashik / Pune / Bokaro / Dubai
NTSE-MENTAL ABILITY

Q.55 to 58- Directions: In the following questions, a question column containing a group of letters is
given. The group of letters is prepared in a specific manner. Four alternatives are given on right side.
Understanding the arrangement of letter in the question column, find the right alternative, from the
given alternatives which matches the given group.

Question Column Alternatives


55. DWJQ (1) RPM, VTQ, HFC

56. VTQ (2) AZMN, BYLO, FUHS

57. YBE (3) MQU, AEI, PTX

58. KOS (d) BEH, KNQ, PSV [2012]

Q.59 and 60-Directions: - In the following questions a specific group is given. Select the correct
alternative which matches the group.

59. zyw, lki, tsq


(a) nmk (b) hgd
(c) xwv (d) fdc

60. IKLN, HJKM, JLMO


(a) GIJL (b) GJIL
(c) GHJL (d) FGHJ

Answer Key

8. 9. 10.
1. (b) 2. (c) 3. (d) 4. (a) 5. (c) 6. (b) 7. (c)
(Bonus) (a, c) (a,b,c)
13. 16. 17. 18.
11. (b) 12. (d) 14. (b) 15. (c) 19. (c) 20. (d)
(Bonus) (Bonus) (a,d) (a, c)
21. (b) 22. (c) 23. (c) 24. (d) 25. (a) 26. (d) 27. (b) 28. (d) 29. (b) 30. (b)
31. (d) 32. (a) 33. (a) 34. (c) 35. (c) 36. (c) 37. (d) 38. (c) 39. (b) 40. (c)
41. (b) 42. (d) 43. (a) 44. (d) 45. (a) 46. (d) 47. (d) 48. (d) 49. (a) 50. (d)
51. (3) 52. (2) 53. (b) 54. (d) 55. (b) 56. (a) 57. (d) 58. (c) 59. (a) 60. (a)

225 PACE IIT & MEDICAL: Mumbai / Delhi & NCR / Goa / Akola / Kolkata / Nashik / Pune / Bokaro / Dubai
NTSE-MENTAL ABILITY

17. NUMBER PYRAMID


Brief review of concepts : The questions are based on the pyramid of numbers from 1 to 100, as given
below.

Many types of questions are possible based on the above pattern. Four instance, geometrical figures like
formation of parallel lines, perpendicular lines, triangles, squares, etc. by taking numbers in order.

1. Fill the blanks from the choices given below.


129 : 145 : : 3811: ?
(a) 3713 (b) 328 (c) 3615 (d) 3614
Sol. (c) There are two groups of numbers. The numbers one the right hand side must have the same
relation as the numbers on the left hand side. 129 and 145, in the above pyramid, form a pattern.

Hence the number in the blank on the right hand side must form same pattern with 3811. Therefore,
the answer is 3615 which forms the pattern.

2. Fill the blank from the choices given below.


2812 : 765 : : 91123 : ?
(a) 121110 (b) 121314 (c) 122132 (d) 303132
Sol. (b) The two numbers on the left hand side form perpendicular lines in the pyramid. Therefore, the
numbers on the right hand side must be of the same pattern. The answer to the above question should
be 121314 to satisfy the same relation.

3. Fill the blank from the choices given below.


234 : 4614 : : 2812 : ?
(a) 3713 (b) 34614 (c) 121314 (d) 131415
Sol. (c) The numbers on the left-hand side form two sides of a square. The numbers have been chosen in
continuity. 2812 and 121314 are in continuity and form the other two sides of the same square.

226 PACE IIT & MEDICAL: Mumbai / Delhi & NCR / Goa / Akola / Kolkata / Nashik / Pune / Bokaro / Dubai
NTSE-MENTAL ABILITY

4. Fill the blank from the choice given below.


507986 : 772112 : : 537689 : ?
(a) 745742 (b) 735841 (c) 716039 (d) 755643
Sol. (a) Hence two numbers on the left-hand side form perpendicular lines. Therefore, the numbers on the
right-hand side must form the same point, taking numbers in order. Hence the answer is 745742.

5. Fill the blank from the choices given below.


322120 : 321920 : : 324342 : ?
(a) 324142 (b) 324132 (c) 323143 (d) 323319
Sol. (a) The numbers on the left-hand side form a triangle. Therefore, the numbers on the right-hand side
must form a triangle with common point 32. Hence, the answer is 324142.

Alphabet Pyramid
Brief review of concepts : The questions are based on the pyramid of Alphabet from a to z, as given below:

Many types of questions are possible based on the above pattern. Four instance, geometrical figures like
formation of parallel lines, perpendicular lines, triangles, squares, etc. by taking Alphabet in order.

6. cgm : mno : : ? : tuv


(a) bfl (b) dhn (c) rst (d) flt
Sol. (d) The alphabet on the left hand side form perpendicular lines in the pyramid. Therefore, the
alphabet on the right hand side must be of the same pattern. The answer to the above question should
be flt to satisfy the same relation.

7. gmu : lmn : : iow : ?


(a) hnv (b) mno (c) nop (d) gmu
Sol. (c) The alphabet on the left hand side form perpendicular lines in the pyramid. Therefore, the
alphabet on the right hand side must be of the same pattern. The answer to the above question should
be nop to satisfy the same relation.

227 PACE IIT & MEDICAL: Mumbai / Delhi & NCR / Goa / Akola / Kolkata / Nashik / Pune / Bokaro / Dubai
NTSE-MENTAL ABILITY

8. xwv:vnh::rst:?
(a) h m n (b) t l f (c) e k j (d) i k r
(e) nop (f) gmu
Sol. (b) The alphabet on the left hand side form perpendicular lines in the pyramid. Therefore, the
alphabet on the right hand side must be of the same pattern. The answer to the above question should
be tlf to satisfy the same relation.

EXERCISE

Directions : (1 to 10) Following questions are based on a pyramid of numbers from 1 to 100 arranged as
shown below. In each question there are two groups of numbers on the left of the sign : : which are related in
some way. The same relationship exists between the third group and the one missing term indicated by a
blank space with a question marks (?). With the help of the given pyramid, find the missing group in each
question amongst the four alternatives given under it.
1
2 3 4
9 8 7 6 5
10 11 12 13 14 15 16
25 24 23 22 21 20 19 18 17
26 27 28 29 30 31 32 33 34 35 36
49 48 47 46 45 44 43 42 41 40 39 38 37
50 51 52 53 54 55 56 57 58 59 60 61 62 63 64
81 80 79 78 77 76 75 74 73 72 71 70 69 68 67 66 65
82 83 84 85 86 87 88 89 90 91 92 93 94 95 96 97 98 99 100

1. 192021 : 213241 : : 414243:?


(A) 433023 (B) 435871 (C) 414039 (D) 435773

2. 494852 : 818084 : : 101123:?


(A) 121321 (B) 494852 (C) 252428 (D) 262747

3. 432946 : 443241 ::? : 706267


(A) 725675 (B) 695972 (C) 705873 (D) 696366

4. 173125 : 364326 : : 5139 : ?


(A) 162110 (B) 626368 (C) 161310 (D) 363126

5. 151933 : 425774 : : 132131 : ?


(A) 435675 (B) 445576 (C) 455477 (D) 304554

6. 232945 : 303132 : : 324258: (?) (NTSE Stage-I/Raj./2008)


(A) 414039 (B) 434445 (C) 333435 (D) 575655

7. 102232 : 233141 : : 252943 : (?) (NTSE Stage-l/Raj./2008)


(A) 284752 (B) 284556 (C) 386366 (D) 284458

228 PACE IIT & MEDICAL: Mumbai / Delhi & NCR / Goa / Akola / Kolkata / Nashik / Pune / Bokaro / Dubai
NTSE-MENTAL ABILITY

8. 295557 : 234547 : : 335961 : (?) (NTSE Stage-I/Raj./2008)


(A) 417173 (B) 194143 (C) 194139 (D) 334142

9. 294653 : 537689 :: 294457 : (?) (NTSE Stage-I/Raj./2008)


(A) 594035 (B) 584134 (C) 577489 (D) 574233

10. 514846 : 454358 : : 857876 : (?) (NTSE Stage-Raj./2008)


(A) 757491 (B) 757358 (C) 757392 (D) 755447

Directions: (11 to 15) Study the arrangement of letters in the form of a pyramid and answer the questions
that follow.
a
b c d
e f g h i
j k 1m n o p
q r s t u y w x y
z
11. cfk : ktz : : ? : ovz
(A) dip (B) cho (C) gmu (D) hnv

12. efg : gmu : : eks : ?


(A) hmn (B) stu (C) ekj (D) kr

13. elu : adi : : ? : abe (NTSE Stage-II, 2007)


(A) fmv (B) dgl (C) hmt (D) inu

14. fit : klm :: iow : ? (NTSE Stage-II, 2007)


(A) hnv (B) mno (C) nop (D) gmu

15. yxp:qrj::ekl:? (NTSE Stage-II, 2007)


(A) h m n (B) i o n (C) e k j (D) i k r

Directions : (16 to 18) Given questions are based on the arrangement of numbers 1 to 49 in the form of a
pyramid. In each question two sets of numbers given on the left of sing (: :) which are related in some way
or other. Same relationship is between two sets on the right with one blank term. Select the correct option
for the blank term. (NTSE Stage-I/Raj./2009)

1
2 3 4
5 6 7 8 9
10 11 12 13 14 15 16
17 18 19 20 21 22 23 24 25
26 27 28 29 30 31 32 33 34 35 36
37 38 39 40 41 42 43 44 45 46 47 48 49

16. 2 6 5 11: 3 7 8 14 : : 5 12 20 29 : ?
(A) 7 14 22 31 (B) 9 15 22 33 (C) 7 12 20 31 (D) 3 6 12 19

17. 5 6 12 19 : 22 23 33 44 : : ? : 16 24 23 33
(A) 12 20 21 31 (B) 12 20 31 43 (C) 14 13 21 20 (D) 15 23 22 31

229 PACE IIT & MEDICAL: Mumbai / Delhi & NCR / Goa / Akola / Kolkata / Nashik / Pune / Bokaro / Dubai
NTSE-MENTAL ABILITY

18. 3 7 12 : 11 19 28 : : ? : 16 24 35
(A) 6 12 20 (B) 8 14 23 (C) 25 35 46 (D) 13 21 31

Directions (19 to 22) : Observe the pyramid of numbers given below and answer the following questions.
28
27 26
23 24 25
22 21 20 19
14 15 16 17 18
13 12 11 10 9 8
1 2 3 4 5 6 7
(NTSE Stage-II Maharashtra/2010)

19. 14, 18; 21, 20:13, 8,?


(A) 2, 5 (B) 15,17 (C) 23, 24 (D) 13, 10

20. 13, 1; 19, 18 ; 27, 23 ; ?


(A) 15, 2 (B) 24,19 (C) 28, 26 (D) 11, 5

21. 3, 11, 16:4, 10, 17; 15, 21, 24;?


(A) 23, 26, 28 (B) 12, 11, 20 (C) 4, 11, 21 (D) 16, 20, 25

22. 1, 2, 12, 13; 16, 17, 19, 20; 5, 6, 8, 9;?


(A) 22, 21, 24, 27 (B) 11, 10, 17, 16 (C) 10, 4, 5, 8 (D) 2, 3, 11, 15

Directions (23 to 26) : Observe the pyramid of letters given below and answer the following questions.

(NTSE Stage-I/Maharashtra/2010)

23. gz, yp, je,?


(A) we (B) fx (C) eb (D) mn

24. pyf : jqz : : dab : ?


(A) omn (B) oml (C) lon (D) lom

25. pwf : jsz : : ktbg : ?


(A) ovdk (B) nucj (C) luck (D) cmun

26. aghb : flmg : : hlmi : ?


(A) rzbt (B) ucew (C) lmnh (D) cghd

230 PACE IIT & MEDICAL: Mumbai / Delhi & NCR / Goa / Akola / Kolkata / Nashik / Pune / Bokaro / Dubai
NTSE-MENTAL ABILITY

Directions : (27 to 31) A pyramid of letters is given below. Study the pyramid and select the correct
alternative to fill in the missing term.

(NTSE Stage-II, 2011)

27. HNPRQ, GIMQL, ZFJLK, ?, PVBDC


(A) QLKDC (B) WAEKD (C) WPVBD (D) IFAVQ

28. FHSIE, ISTRJ, RTCUQ, UCDBV,?


(A) JLKDC (B) LPVBD (C) BDKEA (D) BDCTS

29. YGIM, LOXY, ZFJL,?


(A) GUQK (B) JQVA (C) PWZG (D) KPWZ

30. GYXOW, ZXOLP, FZWPV,?


(A) AWPKQ (B) PVQJE (C) KQUCT (D) EBURI

31. AMONB, YGIHZ,?


(A) LPVBD (B) FAVQJ (C) OSUTP (D) LPVQJ

Directions :(32 to 36) The following questions are based on the arrangement of alphabets in the form of
pyramid. In each question there is some relationship between the two sets of letters on the left of the (:: ) The
same relationship exists between the two terms on the right of which one is missing. Find the missing one
from the given alternatives. (NTSE Stage-I/Raj/2012)

32. BCG : DCG : : EGU : ?


(A) IGU (B) GUJ (C) UIG (D) IUG

33. EJQ : FKR : : HOX : ?


(A) IOW (B) IPY (C) GNU (D) HMT

34. LTSK : MUTL : : (?) : OWXP


(A) LTUM (B) NMUV (C) NVWO (D) NOWV

35. (?) : KRSL : : PYXO : OXWN


(A) RSLK (B) JQRK (C) JKRQ (D) QRKJ

36. JQS : PWY : : KRT: (?)


(A) OVX (B) PWN (C) OWY (D) VOX

231 PACE IIT & MEDICAL: Mumbai / Delhi & NCR / Goa / Akola / Kolkata / Nashik / Pune / Bokaro / Dubai
NTSE-MENTAL ABILITY

Directions : (37 to 41) The following question are based on the arrangement of numbers in the form of a
pyramid. In each question there is some relationship between the two numbers on the left of the (::). The
same relationship exists between the two terms in the right of which one is missing. Find the missing are
from the given alternatives. (NTSE Stage-l/Raj./2013)

37. 132220 : 211412 : : 222931 : (?)


(A) 304345 (B) 302123 (C) 442931 (D) 312022

38. 2873 : 13212014 : : 10242311 : (?)


(A) 29282423 (B) 28274746 (C) 25272824 (D) 29454430

39. 241214 : 752119 : : 14163234 : (?)


(A) 19174139 (B) 20184240 (C) 21194341 (D) 20224244

40. 25224447 : 23204245 : : 11143229 : (?)


(A) 13163431 (B) 24214341 (C) 24214346 (D) 13102421

41. 82224 : 133133 : : 62022 : (?)


(A) 91221 (B) 81312 (C) 153335 (D) 51921

Answer Key

1. (b) 2. (d) 3. (b) 4. (a) 5. (b) 6. (a) 7. (d) 8. (b) 9. (c) 10. (c)
11. (b) 12. (b) 13. (d) 14. (c) 15. (b) 16. (c) 17. (a) 18. (b) 19. (b) 20. (c)

21. (d) 22. (b) 23. (c) 24. (c) 25. (a) 26. (d) 27. (b) 28. (c) 29. (d) 30. (a)

31. (c) 32. (a) 33. (b) 34. (c) 35. (b) 36. (a) 37. (b) 38. (d) 39. (a) 40. (a)

41. (c)

232 PACE IIT & MEDICAL: Mumbai / Delhi & NCR / Goa / Akola / Kolkata / Nashik / Pune / Bokaro / Dubai
NTSE-MENTAL ABILITY

PREVIOUS YEARS QUESTIONS

Q.1 to 3: Directions: Observe the following pyramid and decide which alternative will be in place of
question mark in each of the following questions.

1. oab: zpq : : gab:?


(a) mde (b) bxy
(c) jhi (d) tjk

2. pqcb : utfg : : yzqr: ?


(a) abcd (b) lscb
(c) ekig (d) wvts

3. opnqm : ijtk : : ? : gifke


(a) bncmd (b) gbfce
(c) jfxle (d) ybxsr
[NTSE-MAHARASHTRA STAGE-1-2020]

Q4 and 6 Directions: Observe the pyramid of number and choose the correct alternative which will replace
question mark.

4. 18284041:24344645::20304243:?
(a) 22324443 (b) 21314344
(c) 22324445 (d) 24344647

5. 261728: ? : : 292031: 332231


(a) 281930 (b) 302132
(c) 362534 (d) 352433

233 PACE IIT & MEDICAL: Mumbai / Delhi & NCR / Goa / Akola / Kolkata / Nashik / Pune / Bokaro / Dubai
NTSE-MENTAL ABILITY

6. 37261718:49362524::39271710:?
(a) 39281920 (b) 47352516
(c) 47342322 (d) 46342416
[NTSE-MAHARASHTRA STAGE-1-2020]
Q.7 and 9: Direction : Observe the following pyramid of letter and decide which alternative will replace the
question mark.

7. hab : mgf: : jicd: ?


(a) kled (b) kdel
(c) ldek (d) delk

8. bza : bwy : : bsv:?


(a) bnr (b) bvs
(c) bhm (d) bag

9. wsop : yvqp :: ptw:?


(a) pqr (b) puy
(c) pos (d) pxb
[NTSE-MAHARASHTRA STAGE-1-2019]
Q.10 to 12: Direction-Observe the following pyramid and choose the correct alternative to answer the
equations.

10. 1352 : 13192518 : : 53138:?


(a) 25334132 (b) 25324133
(c) 25413332 (d) 33253241

11. 163044: 213538 ::713143:?


(a) 393420 (b) 203439
(c) 183241 (d) 203440

12. 281627 : 222123 : : 292830:?


(a) 352236 (b) 353622
(c) 362235 (d) 363522
[NTSE-MAHARASHTRA STAGE-1-2019]
234 PACE IIT & MEDICAL: Mumbai / Delhi & NCR / Goa / Akola / Kolkata / Nashik / Pune / Bokaro / Dubai
NTSE-MENTAL ABILITY

Q.13 to 15: Direction: Observe the following pyramid of letters and decide which alternative will be in
place of question mark in each of the following question.

13. acdj : ? : : eilt : ennq


(a) abfg (b) acid
(c) acei (d) abfh

14. vihx : uwxt : : ? : pazq


(a) abde (b) bdez
(c) dbaz (d) azed
15. jktljt : goqngq : : ilsmis:?
(a) hnrmhr (b) hnrnmr
(c) hnrmhn (d) hrmnhn

[NTSE-MAHARASHTRA STAGE-1-2018]

Q16 to 18: Direction: Observe the following pyramid and choose the correct alternative to answer the
Question.

16. 52262839 : 62363447 : : 54283041 : ?


(a) 60343245 (b) 54283042
(c) 60343244 (d) 30463244

17. 2567 : 4987 : : 7256 : ?


(a) 7894 (b) 7489
(c) 7498 (d) 7948

18. 61820 : 82422 : : 121920: ?


(a) 142223 (b) 231524
(c) 191220 (d) 142322
[NTSE-MAHARASHTRA STAGE-1-2018]

235 PACE IIT & MEDICAL: Mumbai / Delhi & NCR / Goa / Akola / Kolkata / Nashik / Pune / Bokaro / Dubai
NTSE-MENTAL ABILITY

Q.19 and 20: Directions – Observe the following pyramid of numerals and decide which alternative will be
in place of question mark in each of the following question.

19. 95761 : 105844 : : ? : 346648


(a) 377149 (b) 353331
(c) 356763 (d) 363840

20. 95670 : 70579 : : 356766 : ?


(a) 663435 (b) 677063
(c) 634623 (d) 587061 [2017]

Q.21 and 22  Directions:- Observe the pyramid of alphabets. Answer the following questions

21. dheb, kqlg, cluk, ?


(a) gihd (b) kqwp
(c) pwdv (d) vdic

22. bhr : cis : : rxem: ?


(a) jnsy (b) szhq
(c) dhms (d) giry [2016]

236 PACE IIT & MEDICAL: Mumbai / Delhi & NCR / Goa / Akola / Kolkata / Nashik / Pune / Bokaro / Dubai
NTSE-MENTAL ABILITY

Q. 23 and 24  Directions:- observer the pyramid of alphabets and answer the questions the follow

23. Which term is related to the terms hgfi, wefx, lrym?


(a) deic (b) pzay
(c) xyzq (d) ifby

24. Find the term that will replace the question mark (?)
fvji : zymn : gfxy:?
(a) cecx (b) dewv
(c) gaym (d) aics
[2015]

Q.25 and 26  Directions:- Observer the following pyramid of alphabets and answer the following
questions

25. onza, nmyz, mlxy, ?


(a) nmst (b) hilm
(c) abcd (d) lkwx

26. aqze : wtxh : : ghno : ?


(a) abce (b) qrfe
(c) jilk (d) ijlh [2014]

237 PACE IIT & MEDICAL: Mumbai / Delhi & NCR / Goa / Akola / Kolkata / Nashik / Pune / Bokaro / Dubai
NTSE-MENTAL ABILITY

Q.27 to 30  Directions: - Observe the pyramid of numerals and decide which alternative will be in place
of question mark?

15
16 14
17 10 13
18 11 9 12
19 12 16 8 11
20 13 14 15 7 10
1 2 3 4 5 6 9
1 2 3 4 5 6 7 8

27. 2, 3, 4, 14; 2, 3, 4, 16; 13, 14, 5, 9,?


(a) 1, 2, 3, 3 (b) 3, 4, 5, 5
(c) 4, 5, 6, 11 (d) 12, 16, 8, 13

28. 3, 2, 20; 3, 4, 3; 5, 5, 6;?


(a) 5,6,10 (b) 2, 3, 2
(c) 1, 2, 1 (d) 3, 14, 16

29. 3, 20, 18, 16 : 5,14, 9, 7 : : 6,15,12,10 : ?


(a) 12, 18, 17, 11 (b) 4, 13, 11, 15
(c) 6, 7, 8, 9 (d) 12, 8, 15, 4

30. 2, 20, 9,12,14;3,13,12,16,15,4,114,16,8,7;?


(a) 4, 3, 14, 15, 5 (b) 13, 19, 18, 11, 16
(c) 14, 12, 9, 8, 15 (d) 4, 15, 8, 11, 10 [2012]

Answer Key
3.
1. (c) 2. (d) 4. (a) 5. (c) 6. (b) 7. (a) 8. (a) 9. (b) 10. (a)
(Bonus)
11. (b) 12. (c) 13. (a) 14. (b) 15. (a) 16. (a) 17. (c) 18. (d) 19.(c) 20.(a)
21. (c) 22. (b) 23 (a) 24. (b) 25. (d) 26. (c) 27.(d) 28. (a) 29. (b) 30. (d)

238 PACE IIT & MEDICAL: Mumbai / Delhi & NCR / Goa / Akola / Kolkata / Nashik / Pune / Bokaro / Dubai
NTSE-MENTAL ABILITY

18. CALENDAR TEST

To find the day of the week on a mentioned date

Certain concepts are defined as under.


 An ordinary year has 365 days.
 In an ordinary year, first and last day of the year are same.
 A leap year has 366 days. Every year which is divisible by 4 is called a leap year. For example
1200, 1600, 1992, 2004, etc. are all leap years.
 For a leap year, if first day is Monday than last day will be Tuesday for the same year.
 In a leap year, February is of 29 days but in an ordinary year, it has only 28 days.
 Year ending in 00’s but not divisible by 400 is not considered a leap year. e.g., 900, 1000, 1100,
1300, 1400, 1500, 1700, 1800, 1900, 2100 are not leap years.
 The day on which calendar started (or the very first day) i.e., 1 Jan, 0001 was Monday.
 Calendar year is from 1 Jan to 31 Dec. Financial year is from 1 April to 31 March.

Odd Days
The number of days exceeding the complete number of weeks in duration is the number of odd days during
that duration.

Counting of Odd Days


 Every ordinary year has 365 days = 52 weeks + 1 day.
 Ordinary year has 1 odd day.
 Every leap year 366 days = 52 weeks + 2 days.
 Leap year has 2 odd days.
 Odd days of 100 years = 555,
Odd days of 200 years = 3,
Odd days of 300 years = 1,
Odd days of 400 days = 0.

 Explanation:
100 years = 76 ordinary years + 24 leap years (The year 100 is not a leap year)
= 76 odd days 2  24 odd days = 124 odd days.
124
Odd days =  5 odd days.
7
 Similarly, 200 years = 10 odd days = 03 odd days
15
300 years =  1 odd day.
7
20  1
400 years =  0 odd day (1 is added as 400 is a leap year)
7
Similarly, 800, 1200, 1600, 2000, 2400 years contain 0 odd days.
 After counting the odd days, we find the day according to the number of odd days.
 Sunday for 0 odd day, Monday for 1 odd day and so on as shown in the following table.

Table : 1 (odd days for week days)

Days Sunday Monday Tuesday Wednesday Thursday Friday Saturday


Odd Days 0 1 2 3 4 5 6

239 PACE IIT & MEDICAL: Mumbai / Delhi & NCR / Goa / Akola / Kolkata / Nashik / Pune / Bokaro / Dubai
NTSE-MENTAL ABILITY

Table : 2 (odd days for months in a year)

Ordinary Year Days Odd Days Leap Year Days Odd Days
January 31 3 January 31 3
February 28 0 February 29 1
March 31 3 March 31 3
April 30 2 April 30 2
May 31 3 May 31 3
June 30 2 June 30 2
Total 181 days 6 Total 182 days 0
July 31 3 July 31 3
August 31 3 August 31 3
September 30 2 September 30 2
October 31 3 October 31 3
November 30 2 November 30 2
December 31 3 December 311 3
Total 184 days 1 Total 184 days 2

Table : 3 (Odd days for every quarter)

Months of years Ist three months IInd three IIInd three IVth three Total year 1 Jan
1Jan to 31 months 1 Apr months 1 July months 1 Oct. to 31 Dec.
March to 30 June to 30 Sep. to 31 Dec.
90/91 365/366
Total days 91 92 92
Ord./Leap Ord./ Leap
6/0 0 1 1 1/2
Odd days
Ord./Leap Odd day Odd day Odd day Ord./ Leap

1. If it was Saturday on 17th December 1982 what will be the day on 22nd December 1984?
Sol. Total number of odd days between 17 Dec 1982 to 17 Dec. 1984 the number of odd days = 1 + 2 = 3.
From 17 to 22 Dec. number of odd days = 5
 3 + 5 = 8 odd days = 1 odd day.
 Saturday + 1 odd day = Sunday.

2. Find the day of the week on 18 July, 1776.


Sol. Here 1600 years have 0 odd day …. (a)
100 years have 5 odd days .…(b)
75 years = (18 leap years + 57 ordinary years)
 18  2  57 1  93 odd days   7 13  2
= 2 odd days ….(c)
Now, the number of days from 1st January to 18th July, 1776
 182  18   28  7  4 days
= 4 odd days ….(d)
Adding, a, b, c & d = 0 + 5 + 2 + 4 = 4 odd days.
Hence, the required day is Thursday.

240 PACE IIT & MEDICAL: Mumbai / Delhi & NCR / Goa / Akola / Kolkata / Nashik / Pune / Bokaro / Dubai
NTSE-MENTAL ABILITY

3. On what dates of October, 1975 did Tuesday fall?


Sol. For determining the dates, we find the day on 1 st Oct, 1975.
1600 years have 0 odd days …(a)
300 years have 1 odd days …(b)
74 years have (18 leap years + 56 ordinary years)
2 18  1 56  92 odd days = 1 odd days …(c)
st st
Days from 1 January to 1 Oct.
= 1 st Jan to 30 June + 1 st July to 1st Oct.
181  31  31  30  1  274 days. …(d)
Adding a, b, c & d = 0 + 1 + 1 + 1 = 3 odd days.
So, Wednesday falls on 1 st Oct.
Hence 7, 14, 21, 28 October will Tuesday fall.

4. Which year will have the same calendar next to 1995.


Sol. The calendar for 1995 and the required year will be the same if day on 1 st January of both the years is
same.
This is possible only if the total odd days from 1st January 1995 to 31st December of the previous
year of required year is 0.
Let the required year is 2006 then, we have 3 leap years (1996, 2000, 2004) and 8 ordinary years
(1995, 1997, 1998, 1999, 2001, 2002, 2003, 2005)
Total odd days =  2  3  1 8  14  0 odd days.
Hence, the required year is 2006.

5. Prove that last day of a century cannot be Tuesday. Thursday or Saturday.


Sol. 100 years have = 5 odd days.
st
 Last day of 1 century is Friday.
200 years have = 10 odd days = 3 odd days.
nd
 Last day of 2 century is Wednesday.
300 years have = 15 odd days = 1 odd day
rd
 Last day of 3 century is Monday.
400 years have  5  4  1  21  0 odd days.
So, the last day of 4th century is Sunday.
Since the order keeps on cycling, we see that the last day of the century cannot be Tuesday, Thursday
or Saturday.

241 PACE IIT & MEDICAL: Mumbai / Delhi & NCR / Goa / Akola / Kolkata / Nashik / Pune / Bokaro / Dubai
NTSE-MENTAL ABILITY

EXERCISE

1. Which of the following year was leap year?


(a) 1000 (b) 1800 (c) 1200 (d) 1400

2. If the day before yesterday was Saturday what day will fall on the day after tomorrow:
(a) Friday (b) Thursday (c) Wednesday (d) Tuesday

3. My brother is 562 days older to me while my sister is 75 weeks older to him. If my sister was born
on Tuesday, on which day was I born:
(a) Sunday (b) Monday (c) Thursday (d) Tuesday

4. Find the day of the week on 26 January, 1950.


(a) Tuesday (b) Friday (c) Wednesday (d) Thursday

5. Find the day of the week on 15 August, 1947.


(a) Tuesday (b) Friday (c) Wednesday (d) Thursday

6. In U.P. on 17th Oct. 1996, the president rule was declared. Find the day of week on that date.
(a) Tuesday (b) Friday (c) Wednesday (d) Thursday

7. Karan was born on Saturday 22 nd March 1982. On what day of the week was he 14 years 7 months
and 8 days of age?
(a) Sunday (b) Tuesday (c) Wednesday (d) Monday

8. On what dates of August 2020 will Monday fall:


(a) 3, 10, 17, 24, 31 (b) 5, 12, 19, 26 (c) 1, 8, 15, 22, 29 (d) None of these

9. On what date of October, 1975 did 3rd Tuesday fall:


(a) 17/10/75 (b) 18/10/75 (c) 21/10/75 (d) 23/10/75

10. The year next to 1990 will have the same calendar as that of the year 1990:
(a) 2002 (b) 2000 (c) 2001 (d) 1996

11. Which two months in a year have always the same calendar?
(a) June, Oct. (b) April, Nov. (c) April, July (d) Oct., Dec

12. There are 30 days in a month and 1 st day of this month is Monday. If alternate Monday starting from
second and each Sunday is a holiday then how many working days are there in the month:
(a) 23 (b) 22 (c) 24 (d) 21

13. If the sixth day of a month is fourth day after Sunday then which day of the weak will be on 19 th day
of that month- (NTSE Stage-I/Raj./2007)
(a) Friday (b) Wednesday (c) Saturday (d) Tuesday

14. If Sripal’s birthday falls on Thursday 20th March, 2000, then on which day of the week his birthday
falls in the year 2001? (NTSE Stage-I/Raj./2008)
(a) Wednesday (b) Friday (c) Saturday (d) Sunday

242 PACE IIT & MEDICAL: Mumbai / Delhi & NCR / Goa / Akola / Kolkata / Nashik / Pune / Bokaro / Dubai
NTSE-MENTAL ABILITY

15. Monday, Tuesday etc. are numbered as 1, 2 and so on. If 8th December 1994 was Monday, what day
was 8th January 1995? Write the number of the day. (NTSE Stage-II, 2009)
(a) 4 (b) 5 (c) 6 (d) 7

16. If your birth day 30th June, 2003 falls on Monday, on what day of the week does your birth day fall
in the year 2005? (NTSE Stage-I/Raj./2013)
(a) Sunday (b) Tuesday (c) Wednesday (d) Thursday

17. On what day of the week India will celebrate its Republic Day on 26th January, 2015?
(NTSE Stage-I/Raj./2013)
(a) Sunday (b) Monday (c) Tuesday (d) Wednesday

18. If 1st October is Sunday, then 1st November will be (NTSE Stage-I/Raj./2014)
(a) Monday (b) Tuesday (c) Wednesday (d) Thursday

19. Which two months in a year have the same calendar? (NTSE Stage-I/Raj./2014)
(a) June, October (b) April, November (c) April, July (d) October, December

20. If the first day of a leap year is Monday, then what day will be on the last day of the year?
(NTSE Stage-I/Raj./2014)
(a) Wednesday (b) Tuesday (c) Thursday (d) Sunday

21. If 14th September, 2013 is Saturday, then what day will be 22nd December, 2014
(a) Sunday (b) Monday (c) Tuesday (d) Wednesday

22. If 14th July of 1995 was Friday, then what was the day on 30th March of 1994?
(a) Sunday (b) Monday (c) Tuesday (d) Wednesday

Answer Key

1. (c) 2. (c) 3. (c) 4. (d) 5. (b) 6. (d) 7. (d) 8. (a) 9. (c) 10. (c)
11. (c) 12. (c) 13. (b) 14. (b) 15. (a) 16. (d) 17. (b) 18. (c) 19. (c) 20. (b)

21. (b) 22. (d)

PREVIOUS YEARS QUESTIONS

1. A school boy was having Deepavali vacation from 11 October to 28 October 2012.It was Monday on
the 10th Day before the start of the vacation. After the vacation, the school excursion was arranged
on the 7th day from the reopening of the school. On which Day was the school excursion arranged.
(a) Sunday (b) Friday
(c) Thursday (d) Tuesday (2017)

2. Kiran was born on 12th September this year. Soham is 12 days younger to Kiran. In the same month
Hindi day was celebrated on Friday. What is the birthday of Sohan?
(a) Wednesday (b) Sunday
(c) Saturday (d) Monday (2016)

243 PACE IIT & MEDICAL: Mumbai / Delhi & NCR / Goa / Akola / Kolkata / Nashik / Pune / Bokaro / Dubai
NTSE-MENTAL ABILITY

3. Shrushti, born on 17th May, celebrates her birthday on Thursday in the year 2007. Again in which
year her birthday will come on Thursday?
(a) 2011 (b) 2012
(c) 2013 (d) 2014 (2015)

4. ‘Environment Day’ falls on Thursday, then on which day ‘Children’s Day’ will fall in the same year?
(a) Friday (b) Saturday
(c) Sunday (d) Monday (2014)

5. Birthday of three siblings falls on 1st August. Ages of Kshama and Rama on 1 st August 2002 were
seven years and three years respectively. If Uma is neither elder to Kshma, nor younger to Rama,
then among the following, which is the birth year of Uma?
(a) 1996 (b) 1994
(c) 1997 (d) 1999 (2013)

6. Soham was born on 25th February 2012. Hasan is younger to Sohan by four days. If 26th January in
that year falls on Thursday, then Hasan’s birthday falls on which day?
(a) Saturday (b) Wednesday
(c) Monday (d) Thursday (2012)

Answer Key
1. (a) 2. (d) 3. (b) 4. (a) 5. (c) 6. (a)

244 PACE IIT & MEDICAL: Mumbai / Delhi & NCR / Goa / Akola / Kolkata / Nashik / Pune / Bokaro / Dubai
NTSE-MENTAL ABILITY

19. DICE

Type-I
General Dice: In a general dice the sum of numbers on the any two adjacent faces is ‘7’.
Standard Dice: In a standard dice the sum of numbers on the opposite faces is ‘7’.

1. Which number is opposite 4 in a standard dice given below?

(a) 1 (b) 3 (c) 5 (d) Can’t be determined


Sol. Clearly, from the standard dice the sum of numbers on the opposite faces is ‘7’, so number opposite
to 4 is 3.

Type-II

2. The figures given below show the two different positions of a dice. Which number will appear
opposite to number 2?

(a) 3 (b) 4 (c) 5 (d) 6


Sol. (c) The above question, where only two positions of a dice are given, can easily be solved with the
following method.

Step I. The dice, when unfolded, will appear as shown in the figure given on the right side.

Step II. Write the common number to both the dice in the middle block. Since common number is 4,
hence number 4 will appear in the central block.

Step III. Consider the figure (i) and write the first number in the anti-clockwise direction of number 4,
(common number) in block I and second number in block II. Therefore, numbers 3 and 2 being
the first and second number to 4 in anticlockwise directions respectively, will appear in block I &
II respectively.

Step IV. Consider figure (ii) and wire first and second number in the antilock-wise direction to number 4,
(common number) in block (III) & (IV). Hence numbers 6 and 5 will appear in the blocks III and
IV respectively.

Step V. Write remaining number in the remaining block. Therefore, number 1 will come in the remaining
block. Now, from the unfolded figures we find that number opposite to 6 is 3, number opposite to
2 is 5 and number opposite to 4 is 1. Therefore, option (c) is our answer.

245 PACE IIT & MEDICAL: Mumbai / Delhi & NCR / Goa / Akola / Kolkata / Nashik / Pune / Bokaro / Dubai
NTSE-MENTAL ABILITY

(Short thick : From the given dice, we will take the common number as the base and then in its
respect move clockwise direction and write as follow:
4–2–3
4–5–6
Here, we find that number opposite to 6 is 3, number opposite to 2 is 5 and number opposite to 4
is remaining number 1. Therefore, option (c) is out answer.)

3. On the basis of two figures of dice, you have to tell what number will be on the opposite face of
number 5?

(a) 1 (b) 2 (c) 4 (d) 6


Sol. (d) The above question where only two positions of a dice are given, can easily be solved with the
following method:
If in the given dice, there are two numbers common, then uncommon numbers will always be
opposite of each other.
Therefore, option (d) is our answer.

Type-III

4. From the following figures of dice, find which number will come in place of ‘?’

(a) 4 (b) 5 (c) 2 (d) 3


Sol. (d) If the above dice is unfolded, it will look like as the figure (i) given below.

Now the number in place of ‘?’ can be obtained by making a slight change in the figure as given
here. Now comparing figure (ii) with third dice as above, we get that number in place of ? is 3.

Type-IV
5. A dice has been thrown four times and produces following results.

246 PACE IIT & MEDICAL: Mumbai / Delhi & NCR / Goa / Akola / Kolkata / Nashik / Pune / Bokaro / Dubai
NTSE-MENTAL ABILITY

Which number will appear opposite to the number 3?


(a) 4 (b) 5 (c) 6 (d) 1
Sol. (a) From the figures (i), (ii) and (iv) we find that numbers 6, 1, 5 and 2 appear on the adjacent
surfaces to the number 3. Therefore, number 4 will be opposite to number 3.

Type-V

6. Which of the following dices is identical to the unfolded figure as shown here?

Sol. (i) From the unfolded figure of dice, we find that number opposite to 2 is 4, for 5 it is 3 and for 1it
is 6. From this result we can definitely say that figure (ii), (iii) and (iv) cannot be the answer figure
as numbers lying on the opposite pair of surfaces are present on the adjacent surfaces.

EXERCISE
1. Which alphabet is opposite D?

(A) E (B) C (C) F (D) A

2. What should be the number opposite 4 ?

(A) 5 (B) 1 (C) 3 (D) 2

3.

Which letter will be opposite to letter D?


(A) A (B) B (C) E (D) F
247 PACE IIT & MEDICAL: Mumbai / Delhi & NCR / Goa / Akola / Kolkata / Nashik / Pune / Bokaro / Dubai
NTSE-MENTAL ABILITY

Directions: (4 to 5) The figure (X) given below is the unfolded position of a cubical dice. In each of the
following questions this unfolded figure is followed by four different figures of dice. You have to select the
figure which is identical to the figure (X).

4.

5.

Directions: (6 to 8) In each of the following questions select the correct option for the question asked.

6. Which number will come opposite to number 2?


(A) 5 (B) 1 (C) 6 (D) 3

7. Which number will come opposite to number 6?


(A) 1 (B) 5 (C) 4 (D) 3

8. Which number will come opposite to number 4?


(A) 3 (B) 5 (C) 1 (D) 2

9. On the basis of two figures of dice, you have to tell what number will be on the opposite face of
number 5 ?

(A) 1 (B) 2 (C) 4 (D) 6

248 PACE IIT & MEDICAL: Mumbai / Delhi & NCR / Goa / Akola / Kolkata / Nashik / Pune / Bokaro / Dubai
NTSE-MENTAL ABILITY

10. Which symbol will appear on the opposite surface to the symbol x?

(A) + (B) = (C)  (D) O

11. Three positions of the same dice are given below. Observe the figures carefully and tell which
number will come in place of ‘?’ (NTSE Stage-I/Raj./ 2013)

(A) 1 (B) 6 (C) 3 (D) 5

12. On the basis of the following figures you have to tell which number will come in place of ‘?’

(A) 2 (B) 3 (C) 6 (D) 4

Direction: (13 to 15) choose from the alternatives, the boxes that will be formed when figure (X) is folded.
(NTSE Stage-I/Raj./ 2013)
13.

249 PACE IIT & MEDICAL: Mumbai / Delhi & NCR / Goa / Akola / Kolkata / Nashik / Pune / Bokaro / Dubai
NTSE-MENTAL ABILITY

14.

15.

Direction: (16) The six faces of a cube have been marked with number 1, 2, 3, 4, 5 and 6 respectively. The
cube is rolled down three times. The three positions are given. Choose the figure that will be formed when
the cube is unfolded.

16.

(A) (B) (C) (D)

250 PACE IIT & MEDICAL: Mumbai / Delhi & NCR / Goa / Akola / Kolkata / Nashik / Pune / Bokaro / Dubai
NTSE-MENTAL ABILITY

17. Which number is opposite 3 in a standard dice given below?

(A) 1 (B) 4 (C) 5 (D) Can’t be determined

18. Which number is opposite 4?

(A) 5 (B) 3 (C) 2 (D) 1

Direction: (19) In the following question four positions of the same dice have been shown. You have to see
these figures and select the number opposite to the number as asked in each question.

19.

Which number is opposite to number 5?


(A) 6 (B) 5 (C) 1 (D) 3

Directions: (20 to 21) Following questions are based on dice test. Numbers from 1 to 6 are marked on
different faces of the :Dice”. (NTSE Stage-I/Raj./ 2007)

20. The number opposite to 4 is:

(A) 1 (B) 2 (C) 5 (D) 6

21. The number opposite to 6 is :

(A) 1 (B) 3 (C) 5 (D) none of these

22. If the sum of the numbers on the opposite faces of dice is 7, then which figure of dice is correctly
marked?

(A) (B) (C) (D)

251 PACE IIT & MEDICAL: Mumbai / Delhi & NCR / Goa / Akola / Kolkata / Nashik / Pune / Bokaro / Dubai
NTSE-MENTAL ABILITY

23. The number opposite to 2 is

(A) 1 (B) 3 (C) 4 (D) 5

24. The number opposite to 3 is

(A) 2 (B) 4 (C) 5 (D) 6

25. The number opposite to 1 is (NTSE Stage-I/Raj./ 2008)

(A) 4 (B) 6 (C) 5 (D) 3

26. If the sum of the numbers on the opposite faces of a dice is ‘7’ then which figure of dice is not
correctly presented? (NTSE Stage-I/Raj./ 2008)

27. Which of the following dices is identical to the unfolded figure as shown here?
(NTSE Stage-I/Raj./ 2008)

252 PACE IIT & MEDICAL: Mumbai / Delhi & NCR / Goa / Akola / Kolkata / Nashik / Pune / Bokaro / Dubai
NTSE-MENTAL ABILITY

28. A dice is thrown four times and its four different positions are given below.
(NTSE Stage-II, 2009)

Which number is opposite to 3 ?


(A) 1 (B) 2 (C) 4 (D) 5
29. A dice is thrown twice and its different positions are shown below. (NTSE Stage-II, 2009)

Which is the number of dots on the face opposite 4 dots?


(A) 6 (B) 3 (C) 2 (D) 1

30. Two positions of a standard dice are shown below. (NTSE Stage-II, 2009)

When 6 is at the bottom, what number will be at the top?


(A) 4 (B) 3 (C) 2 (D) 1
Direction (31 to 34) In the following questions, the figure is folded to form a box. Choose from among the
alternatives the box or boxes that can be formed by folding the figure.
31.

(NTSE Stage-II, 2011)


(A) Only IV (B) Both II and III (C) Only III (D) Both I and IV

32.

(NTSE Stage-II, 2011)


(A) Both II and IV (B) Both I and II (C) IV Only (D) II Only
253 PACE IIT & MEDICAL: Mumbai / Delhi & NCR / Goa / Akola / Kolkata / Nashik / Pune / Bokaro / Dubai
NTSE-MENTAL ABILITY

33.

(NTSE Stage-II, 2011)


(A) II, III and IV (B) III and IV only (C) II and III only (D) I and IV only

34.

(NTSE Stage-II, 2011)


(A) Only I (B) Both II and III (C) Only IV (D) All I, II, III and IV

Directions: (35 to 36) A dice is thrown 3 times and its 3 position are given. Select the alternative which
provides correct answer to the question asked. (NTSE Stage-II, 2011)

35. Which number is opposite 4 ?


(A) 1 (B) 3 (C) 4 (D) 6

36 Which number is opposite 1?


(A) 2 (B) 3 (C) 4 (D) 6

Directions: (37 to 39) Four positions of the same dice have been shown. Select the alternative which
provides correct answer to the question asked. (NTSE Stage-II, 2011)

254 PACE IIT & MEDICAL: Mumbai / Delhi & NCR / Goa / Akola / Kolkata / Nashik / Pune / Bokaro / Dubai
NTSE-MENTAL ABILITY

37. Which number would be opposite to 3?


(A) 1 (B) 4 (C) 5 (D) 6

38. Which number would be opposite to 5?


(A) 2 (B) 3 (C) 4 (D) 6

39. Which number would be opposite to 4?


(A) 2 (B) 3 (C) 4 (D) 6

40.

The number opposite side the face having the number 5 will be (NTSE Stage-I/Raj./2012)
(A) 1 (B) 2 (C) 3 (D) 4

41. The number opposite the face having the no.2 will be

(NTSE Stage-I/Raj./2012)
(A) 1 (B) 4 (C) 3 (D) 5

42. The number opposite the face having the no. 3 will be

(NTSE Stage-I/Raj./2012)
(A) 1 (B) 3 (C) 4 (D) 6

43. The following figures is converted in to a cube. Its incorrect shape will be

(NTSE Stage-I/Raj./2012)

255 PACE IIT & MEDICAL: Mumbai / Delhi & NCR / Goa / Akola / Kolkata / Nashik / Pune / Bokaro / Dubai
NTSE-MENTAL ABILITY

Directions: (44 to 48) : Choose the cube from the options that will unfold to give the figure on the left.
44.

45.

46.

47.

256 PACE IIT & MEDICAL: Mumbai / Delhi & NCR / Goa / Akola / Kolkata / Nashik / Pune / Bokaro / Dubai
NTSE-MENTAL ABILITY

48.

Directions:(49 to 52) In each of the following questions, a diagram has been given which can be folded into
a cube. The entries given in the squares indicate the entries on the face of the cube. In each question a
number or a letter has been given. Of the four alternatives given below it, you have to find the one that
would appear on the face opposite to it in the cube.

49. Which letter is opposite Q?

(A) L (B) M (C) N (D) P


50. Which number/letter is opposite 2?

(A) A (B) C (C) 1 (D) 3


51. Which number/letter is opposite O?

(A) L (B) M (C) N (D) 2

257 PACE IIT & MEDICAL: Mumbai / Delhi & NCR / Goa / Akola / Kolkata / Nashik / Pune / Bokaro / Dubai
NTSE-MENTAL ABILITY

52. Which letter is opposite R?

(A) P (B) S (C) T (D) U


53.

The number opposite side the face having the no. 4 will be (NTSE Stage-I/Raj./2013)
(A) 1 (B) 2 (C) 5 (D) 6

54. The following figure is converted into a cube. It’s four positions (a) (b), (c) and (d) are shown. On
the basis of these select correct alternative. (NTSE Stage-I/Raj./2013)

(A) a only (B) b only (C) a and c only (D) a, b, c and d

55. On the basis of the four positions of a dice given below find the colour of the face opposite ‘Yellow’.

(NTSE Stage-II, 2013)


(A) Indigo (B) Red (C) Pink (D) Blue

56. If the given figure is folded to form a box, which among the boxes below will be formed

(NTSE Stage-II, 2013)

258 PACE IIT & MEDICAL: Mumbai / Delhi & NCR / Goa / Akola / Kolkata / Nashik / Pune / Bokaro / Dubai
NTSE-MENTAL ABILITY

57. Two positions of a dice are shown. Which number will appear on the face opposite the one having 5?

(NTSE Stage-II, 2013)


(A) 1 (B) 2 (C) 4 (D) 6

58.

(NTSE Stage-II, 2013)


When the above is folded into a cube, which is the only cube that can be produced amongst the
following?

(A) (B) (C) (D)

59. The number on opposite side of the face having the number 3 will be (NTSE Stage-I/Raj./2014)

(A) 5 (B) 4 (C) 2 (D) 1


60.

(NTSE Stage-I/Raj./2014)
The number on opposite side of the face having the number 3 will be
(A) 1 (B) 2 (C) 4 (D) 5

61.

(NTSE Stage-I/Raj./2014)
The number on opposite side of the face having the number 4 will be
(A) 1 (B) 2 (C) 3 (D) 4

259 PACE IIT & MEDICAL: Mumbai / Delhi & NCR / Goa / Akola / Kolkata / Nashik / Pune / Bokaro / Dubai
NTSE-MENTAL ABILITY

62.

(NTSE Stage-I/Raj./2014)
The word on opposite side of the face having the word F will be
(A) B (B) C (C) D (D) E

63.

(NTSE Stage-I/Karnataka/2013)

64. In the given two positions of a dice, when 2 is below the dice which number is on the dice?

(NTSE Stage-I/Rajasthan/2016)
(A) 3 (B) 5 (C) 1 (D) 6

65. In the given figure squares are folded and a cube is formed. Then the number opposite to 2 is

(NTSE Stage-I/Rajasthan/2016)
(A) 1 (B) 3 (C) 5 (D) 6

66. The two positions of a dice are shown below. If 1 is at the bottom then what will be on the top?

(NTSE Stage-I/Rajasthan/2017)
(A) 2 (B) 3 (C) 4 (D) 5

260 PACE IIT & MEDICAL: Mumbai / Delhi & NCR / Goa / Akola / Kolkata / Nashik / Pune / Bokaro / Dubai
NTSE-MENTAL ABILITY

67. In the given figures squares are folded and a dice is formed. Then how will it be seen from the
following? (NTSE Stage-I/Rajasthan/2017)

Answer Key

1. (b) 2. (b) 3. (a) 4. (d) 5. (b) 6. (a) 7. (a) 8. (b) 9. (c) 10. (d)
11. (a) 12. (b) 13. (d) 14. (b) 15. (d) 16. (c) 17. (b) 18. (a) 19. (c) 20. (b)

21. (a) 22. (a) 23. (c) 24. (b) 25. (b) 26. (b) 27. (d) 28. (c) 29. (d) 30. (d)

31. (b) 32. (c) 33. (b) 34. (d) 35. (c) 36. (d) 37. (a) 38. (d) 39. (a) 40. (b)

41. (b) 42. (a) 43. (b) 44. (c) 45. (a) 46. (d) 47. (e) 48. (d) 49. (c) 50. (a)

51. (b) 52. (b) 53. (a) 54. (b) 55. (a) 56. (d) 57. (d) 58. (d) 59. (c) 60. (c)

61. (b) 62. (d) 63. (d) 64. (d) 65. (c) 66. (b) 67. (c)

PREVIOUS YEARS QUESTIONS

Q.1 to 3 Direction: The figure given alongside is folded on the given lines to construct a cube. Observe
the figure and answer the following questions by choosing correct alternative.

3 8

5 4

1. From the following which number will be opposite to 8?


(a) 4 (b) 5
(c) 6 (d) 7

261 PACE IIT & MEDICAL: Mumbai / Delhi & NCR / Goa / Akola / Kolkata / Nashik / Pune / Bokaro / Dubai
NTSE-MENTAL ABILITY

2. From the following which number will not be adjacent to 4?


(a) 3 (b) 8
(c) 6 (d) 7

3. From the following which figure is not obtained by folding the proper to form a cube?
(a) (b)

(c) (d)

[NTSE-MAHARASHTRA STAGE-1-2020]

Q.4 to 6 Direction The adjacent figure is folded to form a cube. Observe the figure and answer the
following questions.

- <



O 

4. Which symbol will not be adjacent to the symbol ‘.’


(a) < (b) -
(c)  (d) 

5. Which symbol will be opposite to the symbol  ?


(a)  (b) .
(c) < (d) -

6. Which of the following figure is the figure obtained by folding the paper to form a cube?
(a) (b)

(c) (d)

[NTSE-MAHARASHTRA STAGE-1-201

262 PACE IIT & MEDICAL: Mumbai / Delhi & NCR / Goa / Akola / Kolkata / Nashik / Pune / Bokaro / Dubai
NTSE-MENTAL ABILITY

Q7 to 9: Direction: The adjacent figure is folded to form a cube. Observe the figure and answer the
following questions.

 



 
7. Which of the following symbol will not occur adjacent to  ?
(a)  (b) 
(c)  (d) 
8. Which of the following symbol will be on the opposite surface of the symbols  ?
(a)  (b) 
(c)  (d) 

9. Which of the following figure obtained by folding the paper to form a cube?
(a) (b)

(c) (d)

[NTSE-MAHARASHTRA STAGE-1-2018]

10. If the figure given along side is folded to construct a cube, find out the correct cubical figure from
the given alternative figures.

(a) (b)

(c) (d)

(2017)

263 PACE IIT & MEDICAL: Mumbai / Delhi & NCR / Goa / Akola / Kolkata / Nashik / Pune / Bokaro / Dubai
NTSE-MENTAL ABILITY

Q.11 and 12 Directions- In the following figures, three different position of a cube has been shown.
Observe the figures and answer the questions that follow.

11. Which sign will be on the surface opposite to surface having X sign?
(a) (b)

(c) (d)

12. Which sign will be on the surface opposite to surface having sign
(a) (b)

(c) (d) X
[2017]

13. The following figure is folded to form a cube. Observe the cube. O the following cube figures find
the most appropriate figure.

(a) (b)

(c) (d)

[2016]

Q.14 and 15 Directions: Three positions of a cube are shown in figure. Observe the colours and
answer the following questions.

14. Which coloured surface is opposite to red coloured surface?


(a) Blue (b) Orange
(c) Yellow (d) White
264 PACE IIT & MEDICAL: Mumbai / Delhi & NCR / Goa / Akola / Kolkata / Nashik / Pune / Bokaro / Dubai
NTSE-MENTAL ABILITY

15. Which coloured surface is opposite to yellow coloured surface?


(a) White (b) Red
(c) Orange (d) Green [2016]

Q.16 and 17  Directions: - The figure given below is folded to form a cube. Observe the figure an
answer the questions given ahead.


> 
+ 
<

16. Of the following cube figures, find the most relevant figure.
(a) (b)

(c) (d)

[2014]

17. Which sign will be there on the opposite surface of the sign ‘>’?
(a) – (b) +
(c)  (d) <

Q.18 to 19  Directions:- Three positions of a cube are shown in the figure. Observer them and
answer the following questions.

18. What sign will be on the surface opposite to surface having sign?
(a) (b)

(c) (d)

19. Which sign will be on opposite surface of figure (2)?


(a) (b)

(c) (d)

265 PACE IIT & MEDICAL: Mumbai / Delhi & NCR / Goa / Akola / Kolkata / Nashik / Pune / Bokaro / Dubai
NTSE-MENTAL ABILITY

20. Which two surfaces will be oppoiste to each other?


(a) (b)

(c) (d)

21.

The above figure is folded to form a cube. Observe the cube. Of the following cube figures, find the
non-relevant figure.
(a) (b)

(c) (d)

[2012]

Answer Key
1. (d) 2. (a) 3. (b) 4. (b) 5. (c) 6. (d) 7. (a) 8. (b) 9. (a) 10. (b)
11. (b) 12. (a) 13. (c) 14. (a) 15.(d) 16. (c) 17.(d) 18. (a) 19.(b) 20. (c)

21. (4)

266 PACE IIT & MEDICAL: Mumbai / Delhi & NCR / Goa / Akola / Kolkata / Nashik / Pune / Bokaro / Dubai
NTSE-MENTAL ABILITY

20. CUBES

A cube is three dimensional figure having 8 corners, 6 surfaces and 12 edges. If a cube is painted on all of its
surfaces with any colour and further divided into various smaller cubes, we get following results. Smaller
cubes with three surfaces painted will be present on the corners of the big cube.

Smaller cubes with two surface painted will be present on the edges of the big cube. Smaller cubes with one
surface painted will be present on the surfaces of the big cube. Smaller cubes with no surface painted will be
present inside the big cube.

If a cube is painted on all of its surfaces with a colour and then divided into smaller cubes of equal size then
after separation, number of smaller cubes so obtained will be calculated as under:

Number of smaller cubes with three surfaces painted = 8


Number of smaller cubes with two surfaces painted = (n – 2)  12
Number of smaller cubes with one surface painted = (n– 2)2  6
Number of smaller cubes with no surface painted = (n – 2)3
Where n = No of divisions on the surfaces of the bigger cube
length of edge of big cube
n
length of edge of one smaller cube

Type-I

If a cube is painted on all of its surfaces with single colour and then divided into various smaller cubes
of equal size.

Directions : (1 to 4) A cube of side 4 cm is painted black on all of its surfaces and then divided into various
smaller cubes of side 1 cm each. The smaller cubes so obtained are separated.

4 4 4
Total cubes of obtained =  64
1 1 1

side of big cube 4


Here n = 4
side of small cube 1

267 PACE IIT & MEDICAL: Mumbai / Delhi & NCR / Goa / Akola / Kolkata / Nashik / Pune / Bokaro / Dubai
NTSE-MENTAL ABILITY

1. How many smaller cubes have three surfaces painted ?


(A) 4 (B) 8 (C) 16 (D) 24
Sol. (B) Number of smaller cubes with three surfaces painted = 8

2. How many smaller cubes have two surfaces painted ?


(A) 4 (B) 8 (C) 16 (D) 24
Sol. (D) Number of smaller cubes with two surfaces painted = (n – 2)  12 = (4 – 2)  12 = 24

3. How many smaller cubes have only one surface painted ?


(A) 8 (B) 16 (C) 24 (D) 32
Sol. (C) Number of smaller cubes with one surface painted (n – 2)2  6 = (4 – 2)2  6 = 4  6 = 24

4. How many smaller cubes will have no side painted ?


(A) 18 (B) 16 (C) 22 (D) 8

Sol. (D) Number of smaller cubes with no surface painted = (n – 2)3 = (4 – 2)3 = (2)3 = 8

Type-II
If a cube is painted on all of its surfaces with different colours and then divided into various smaller cubes of
equal size.

Directions: (5 to 7) A cube of side 4 cm is painted black on the pair of one opposite surfaces, blue on the
pair
of another opposite surfaces and red on remaining pair of opposite surfaces. The cube is now divided into
smaller cubes of equal side of 1cm each.

5. How many smaller cubes have three surfaces painted ?


(A) 4 (B) 8 (C) 16 (D) 24

Sol. (B) Number of smaller cubes with three surfaces painted = 8


(These smaller cubes will have all three surfaces painted with different colour blue, black and red.)

6. How many smaller cubes have two surfaces painted ?


(A) 4 (B) 8 (C) 16 (D) 24

Sol. (D) Number of smaller cubes with two surfaces painted = 24. And out of this -
(a) Number of cubes with two surfaces painted with black and blue colour = 8.
(b) Number of cubes with two surfaces painted with blue and red colour = 8.
(c) Number of cubes with two surfaces painted with black and red color = 8.

268 PACE IIT & MEDICAL: Mumbai / Delhi & NCR / Goa / Akola / Kolkata / Nashik / Pune / Bokaro / Dubai
NTSE-MENTAL ABILITY

7. How many smaller cubes have only one surface painted ?


(A) 8 (B) 16 (C) 24 (D) 32

Sol. (C) Number of smaller cubes with one surface painted = 24. And out of this -
(a) Number of cubes with one surface painted with black colour = 8.
(b) Number of cubes with one surface painted with blue colour = 8.
(c) Number of cubes with one surface painted with red colour = 8.
If a cube is painted on its surfaces in such a way that one pair of opposite surfaces is left unpainted.

Directions : (8 to 11) A cube of side 4 cm is painted red on the pair of one opposite surfaces, green on the
pair of another opposite surfaces and one pair of opposite surfaces is left unpainted. Now the cube is divided
into 64 smaller cubes of side 1 cm each.

8. How many smaller cubes have three surfaces painted ?


(A) 0 (B) 8 (C) 16 (D) 20

Sol. (A) Number of smaller cubes with three surfaces painted = 0 (Because each smaller cube at the
corner is attached to a surface which is unpainted.)

9. How many smaller cubes have two surfaces painted ?


(A) 4 (B) 8 (C) 16 (D) 24

Sol. (C) Number of smaller cubes with two surfaces painted = Number of cubes present at the corners +
Numbers of cubes present at 4 edges
= 8 + (n – 2)  4 = 8 + 8 = 16

10. How many smaller cubes have only one surface painted ?
(A) 8 (B) 16 (C) 24 (D) 32

Sol. (D) Number of smaller cubes with one surface painted Number of cubes present at the 8 edges +
number of cubes present at the four surfaces (n – 2)  8 + (n – 2)2  4
= 2  8 + 4  4 = 16 + 16 = 32

11. How many smaller cubes will have no side painted ?


(A) 18 (B) 16 (C) 22 (D) 8

Sol. (B) Number of smaller cubes with no side painted


= Number of cubes on the two unpainted surfaces + number of cubes present inside the cube.
269 PACE IIT & MEDICAL: Mumbai / Delhi & NCR / Goa / Akola / Kolkata / Nashik / Pune / Bokaro / Dubai
NTSE-MENTAL ABILITY

= (n – 2)2  2 + (n – 2)3 = 4  2 + (2)3 = 8 + 8 = 16


If a cube is painted on its surfaces in such a way that one pair of adjacent surfaces is left unpainted.

Directions (12 to 15) A cube of side 4cm is painted red on the pair of one adjacent surfaces, green on the
pair of other adjacent surfaces and two adjacent surfaces ere left unpainted. Now the cube is divided into 64
smaller cubes of side 1 cm each.

12. How many smaller cubes have three surfaces painted ?


(A) 2 (B) 4 (C) 8 (D) 6

Sol. (A) Number of smaller cubes with three surfaces painted = Number of smaller cubes at two corners =
2.

13. How many smaller cubes have two surfaces painted ?


(A) 4 (B) 8 (C) 16 (D) 14

Sol. (D) Number of smaller cubes with two surfaces painted = Number of smaller cubes at four corners +
Number of smaller cubes at 5 edges.
= 4 + (n – 2)  5 = 4 + 2  5
= 4 + 10 = 14

14. How many smaller cubes have only one surface painted ?
(A) 8 (B) 16 (C) 24 (D) 30

Sol. (D) Number of smaller cubes with one surface painted = Number of smaller cubes at four surfaces +
Number of smaller cubes at 6 edges + Number of smaller cubes at two corners.
= (n – 2)2  4+ (n – 2)  6 + 2
= 4  4 + 2  6 + 2 = 16 + 12 = 28 + 2 = 30

15. How many smaller cubes will have no side painted?


(A) 18 (8) 16 (C) 22 (D) 8

Sol. (A) Number of smaller cubes with no surfaces painted = Number of smaller cubes from inside the
big cube + Number of cubes at two surfaces + Number of cubes at one edge.
= (n – 2)3 + (n – 2)2  2 + (n – 2)
= (2) + (2)2 + 2
= 8 + 8 + 2 = 18

270 PACE IIT & MEDICAL: Mumbai / Delhi & NCR / Goa / Akola / Kolkata / Nashik / Pune / Bokaro / Dubai
NTSE-MENTAL ABILITY

EXERCISE

Directions : (1 to 4) A cube of side 5 cm is painted black on all of its surfaces and then divided into various
smaller cubes of side 1 cm each. The smaller cubes so obtained are separated.

1. How many smaller cubes have three surfaces painted ?


(a) 4 (b) 8 (c) 16 (d) 24

2. How many smaller cubes have two surfaces painted ?


(a) 8 (b) 16 (c) 24 (d) 36

3. How many smaller cubes have only one surface painted ?


(a) 27 (b) 36 (c) 54 (d) 62

4. How many smaller cubes will have no side painted


(a) 18 (b) 22 (c) 24 (d) 27

Directions: (5 to 7) A cube of side 3 cm is painted black on the pair of one opposite surfaces, blue on the
pair of another opposite surfaces and red on remaining pair of opposite surfaces. The cube is now divided
into smaller cubes of equal side of 1 cm each.

5. How many smaller cubes have three surfaces painted ?


(a) 4 (b) 8 (c) 16 (d) 24

6. How many smaller cubes have two surfaces painted ?


(a) 4 (b) 8 (c) 16 (d) 12

7. How many smaller cubes have only one surface painted ?


(a) 4 (b) 12 (c) 6 (d) 16

Directions : (8 to 11) A cube of side 15 cm is painted red on the pair of one opposite surfaces, green on the
pair of another opposite surfaces and one pair of opposite surfaces is left unpainted Now the cube is divided
into 125 smaller cubes of side 3 cm each.

8. How many smaller cubes have three surfaces painted?


(a) 0 (b) 8 (c) 16 (d) 20

9. How many smaller cubes have two surfaces painted?


(a) 36 (b) 60 (c) 20 (d) 24

10. How many smaller cubes have only one surface painted?
(a) 54 (b) 36 (c) 24 (d) 60

11. How many smaller cubes will have no side painted?


(a) 64 (b) 45 (c) 22 (d) 27

271 PACE IIT & MEDICAL: Mumbai / Delhi & NCR / Goa / Akola / Kolkata / Nashik / Pune / Bokaro / Dubai
NTSE-MENTAL ABILITY

Directions : (12 to 16) The following questions are based on a solid cube which has been shaded as shown
on pairs of opposite faces. (NTSE Stage-II, 2008)

12. How many small cubes are there in the middle layer?
(a) 6 (b) 8 (c) 9 (d) 12

13. How many small cubes have no face with any shading?
(a) 6 (b) 3 (c) 2 (d) 1

14. How many small cubes are there which have shading only on two faces?
(a) 8 (b) 9 (c) 12 (d) 16

15. How many small cubes are there in which three faces are shaded?
(a) 2 (b) 4 (c) 6 (d) 8

16. How many small cubes are there in which only one face is shaded?
(a) 6 (b) 8 (c) 10 (d) 12

Directions: (17 to 21) The following questions are based on a solid cube with each side measuring 4 cm.
The cube has been shaded as shown on pairs of opposite faces. It is then cut into smaller cubes with each
side measuring 1 cm. Answer the question that follow. (NTSE Stage-II, 2009)

17. How many cubes will have no faces shaded ?


(a) 8 (b) 12 (c) 16 (d) 24

18. How many cubes will have three faces shaded?


(a) 4 (b) 8 (c) 12 (d) 16

19. How many cubes will have only two faces shaded?
(a) 12 (b) 16 (c) 4 (d) 32

20. How many cubes will have only one face shaded?
(a) 24 (b) 16 (c) 32 (d) 20

21. How many cubes will have one face shaded with lines and another shaded with dots (all other faces
are unshaded) ?
(a) 16 (b) 12 (c) 8 (d) 4
272 PACE IIT & MEDICAL: Mumbai / Delhi & NCR / Goa / Akola / Kolkata / Nashik / Pune / Bokaro / Dubai
NTSE-MENTAL ABILITY

Directions : (22 to 23) Seema likes to build blocks from small cubes like the one shown below.
(NTSE Stage-II, 2009)

Seema has lots of small cubes like this one. She uses glue to join cubes together to make other block.

22. How many small cubes will Seema need to make the solid block shown in the given diagram?

23. Seema realizes that she used more smell cubes than she really needed to make a block like the one
shown in the above diagram. She realizes that she could have glued small cubes together to look like
the above diagram, but the block could have been hollow on the inside.
What is the minimum number of cubes she needs to make a block that looks like the one shown in
the above diagram?

Directions: (24 to 28) A cube of 4 cm has been painted on its surfaces in such a way that two opposite
surfaces have been painted blue and two adjacent surfaces have been painted red. Two remaining surfaces
have been left unpainted. Now the cube is cut into smaller cubes of side 1 cm each.

24. How many cubes will have no side painted?


(a) 18 (b) 16 (c) 22 (d) 8

25. How many cubes will have at least red colour on its surfaces?
(a) 20 (b) 22 (c) 28 (d) 32

26. How many cubes will have at least blue colour on its surfaces
(a) 20 (b) 8 (c) 24 (d) 32

27. How many cubes will have only two surfaces painted with red and blue colour respectively?
(a) 8 (b) 12 (c) 24 (d) 30

28. How many cubes will have three surfaces coloured?


(a) 3 (b) 4 (c) 2 (d) 16

Directions: (29 to 33) The outer border of width 1 cm of a cube with side 5 cm is painted yellow on each
side and the remaining space enclosed by this 1 cm path is painted pink. This cube is now cut into 125
smaller cubes of each side 1 cm. The smaller cubes so obtained are now separated.
29. How many smaller cubes have all the surfaces uncloured?
(a) 0 (b) 9 (c) 18 (d) 27

30. How many smaller cubes have three surfaces coloured?


(a) 2 (b) 4 (c) 8 (d) 10

31. How many cubes have at least two surfaces coloured yellow?
(a) 24 (b) 44 (c) 48 (d) 96
273 PACE IIT & MEDICAL: Mumbai / Delhi & NCR / Goa / Akola / Kolkata / Nashik / Pune / Bokaro / Dubai
NTSE-MENTAL ABILITY

32. How many cubes have one face coloured pink and an adjacent face yellow?
(a) 0 (b) 1 (c) 2 (d) 4

33. How may cubes have at least one face coloured?


(a) 27 (b) 98 (c) 48 (d) 121

Directions: (34 to 43) a solid cube has been painted yellow, blue and black on pairs of opposite faces. The
cube is then cut into 36 smaller cubes such that 32 cubes are of the same size while 4 others are of bigger
sizes. Also no faces of any of the bigger cubes is painted blue.

34. How many cubes have at least one face painted blue?
(a) 0 (b) 8 (c) 16 (d) 32

35. How many cubes have only one faces painted?


(a) 24 (b) 20 (c) 8 (d) 12

36. How many cubes have only two faces painted?


(a) 24 (b) 20 (c) 16 (d) 8

37. How many cubes have atleast two faces painted?


(a) 36 (b) 34 (c) 28 (d) 24

38. How many cubes have only three faces painted?


(a) 8 (b) 4 (c) 2 (d) 0

39. How many cubes do not have any of their faces painted yellow?
(a) 0 (b) 4 (c) 8 (d) 16

40. How many cubes have at least one of their faces painted black?
(a) 0 (b) 8 (c) 16 (d) 20

41. How many cubes have at least one of their faces painted yellow or blue?
(a) 36 (b) 32 (c) 16 (d) 0

42. How many cubes have no face painted?


(a) 8 (b) 4 (c) 1 (d) 0

43. How many cubes have two faces painted yellow and black respectively?
(a) 0 (b) 8 (c) 12 (d) 16

Directions: (44 to 47) Some equal cubes are arranged in the form of a solid block as shown in the adjacent
figure. All the visible surfaces of the block (except the bottom) are then painted.

274 PACE IIT & MEDICAL: Mumbai / Delhi & NCR / Goa / Akola / Kolkata / Nashik / Pune / Bokaro / Dubai
NTSE-MENTAL ABILITY

44. How many cubes do not have any of the faces painted?
(a) 27 (b) 8 (c) 10 (d) 12

45. How many cubes have one face painted?


(a) 9 (b) 24 (c) 30 (d) 20

46. How many cubes have only two faces painted?


(a) 0 (b) 16 (c) 20 (d) 24

47. How many cubes have only three faces painted?


(a) 4 (b) 12 (c) 6 (d) 20

Directions : (48 to 52) A cuboid of dimensions (6cm  4cm  1cm) is painted black on both the surfaces of
dimensions (4cm  1cm), green on the surfaces of dimensions (6cm  4cm) and red on the surfaces of
dimensions (6cm  1cm). Now the block is divided into various smaller cubes of side 1 cm. each. The
smaller cubes so obtained are separated.

48. How many cubes will have all three colours black, green and red each at least on one side?
(a) 2 (b) 12 (c) 10 (d) None of these

49. How many cubes will be formed?


(a) 6 (b) 12 (c) 16 (d) 24

50. If cubes having only black as well as green colour are removed then how many cubes will be left?
(a) 4 (b) 8 (c) 16 (d) 30

51. How many cubes will have 4 coloured Sides and 2 sides without colour?
(a) 8 (b) 4 (c) 16 (d) 10

52. How many cubes will have two sides with green colour and remaining sides without any colour?
(a) 12 (b) 10 (c) 8 (d) 4

Directions : (53 to 56) The six faces of a cut are painted in a manner that no two adjacent faces have the
same colour. The three colours used in painting are red, blue and green. The cube is then cut into 36 smaller
cubes in such a manner that 32 cubes are of one size and the rest of a bigger size and each of the bigger
cubes has red side. Answer the following questions. (NTSE Stage-I/Raj./ 2013)

53. How many cubes in all have a red side?


(a) 16 (b) 32 (c) 8 (d) 20

54. How many cubes in all have only one coloured ?


(a) 20 (b) 16 (c) 0 (d) 8

55. How many cubes are colured on three sides?


(a) 20 (b) 16 (c) 8 (d) 0

56. How many cubes are there which have two or more sides painted?
(a) 36 (b) 28 (c) 20 (d) 32

275 PACE IIT & MEDICAL: Mumbai / Delhi & NCR / Goa / Akola / Kolkata / Nashik / Pune / Bokaro / Dubai
NTSE-MENTAL ABILITY

Direction: (57 to 58) A cube is painted blue on all faces and it is cut into 64 small cubes of equal size. Now
answer the following questions. (NTSE Stage-I/Raj./ 2017)

57. How many cubes are not painted on any face buesa fdrus ?
(a) 8 (b) 16 (c) 27 (d) 54

58. How many cubes are there which are painted on one face only?
(a) 8 (b) 16 (c) 24 (d) 32

Answer Key

1. (b) 2. (d) 3. (c) 4. (d) 5. (b) 6. (d) 7. (c) 8. (a) 9. (c) 10. (d)
11. (b) 12. (c) 13. (d) 14. (c) 15. (d) 16. (a) 17. (a) 18. (b) 19. (c) 20. (a)

21. (c) 22. (27) 23. (26) 24. (a) 25. (c) 26. (d) 27. (b) 28. (c) 29. (d) 30. (c)

31. (b) 32. (a) 33. (b) 34. (d) 35. (c) 36. (b) 37. (c) 38. (a) 39. (d) 40. (d)

41. (a) 42. (d) 43. (c) 44. (d) 45. (c) 46. (d) 47. (c) 48. (d) 49. (d) 50. (c)

51. (b) 52. (c) 53. (b) 54. (d) 55. (c) 56. (b) 57. (a) 58. (c)

PREVIOUS YEARS QUESTIONS

Q.1 to 3 Directions: A wooden block of 4  4 dimensions is taken. All faces of block are painted from
outside. As shown in the figure it is cut into smaller cubes. Answer the questions by studying adjoin
figure.

1. How many cubes are there having at least one face painted?
(a) 64 (b) 52
(c) 48 (d) 24

2. If the base layer of the block would be same as the top layer then how many cubes will be in the
block?
(a) 56 (b) 52
(c) 60 (d) 62

3. If the base layer and top layer of block is same then at the most how many faces of the cube will be
painted?
(a) 3 (b) 2
(c) 1 (d) 0
[NTSE-MAHARASHTRA STAGE-1-2020]

276 PACE IIT & MEDICAL: Mumbai / Delhi & NCR / Goa / Akola / Kolkata / Nashik / Pune / Bokaro / Dubai
NTSE-MENTAL ABILITY

Q. 4 to 6-Directions: The following figure is made by arranging some cube having each side 1 unit.
This is painted from all sides. Observe the figure and choose correct alternative for following
questions.

4. Find the number of cubes having maximum number of faces painted.


(a) 1 (b) 2
(c) 3 (d) 4

5. How many cubes are used to make the arrangement as shown in the figure?
(a) 35 (b) 40
(c) 44 (d) 46

6. Find the number of cubes having no face painted.


(a) 0 (b) 1
(c) 2 (d) 3
[NTSE-MAHARASHTRA STAGE-1-2020]

Q.7 to 9 : Direction The bottom and the top surface of a cube, having each side 5 units, is painted
black. The opposite surfaces of the cube are red. Then the cube is cut into smaller cubes having each
side I unit. On the basis of this information choose the correct alternative to answer the question.

7. How many cubes have at least one surface painted?


(a) 125 (b) 116
(c) 100 (d) 98

8. How many cubes have only red surface?


(a) 18 (b) 30
(c) 48 (d) 60

9. How many cubes have surface in both the colours, black and red?
(a) 25 (b) 50
(c) 8 (d) 20

277 PACE IIT & MEDICAL: Mumbai / Delhi & NCR / Goa / Akola / Kolkata / Nashik / Pune / Bokaro / Dubai
NTSE-MENTAL ABILITY

Q. 10 to 12: Direction: The following figure is made by arranging some cubes having each side 1 unit.
The figure is painted from all the outside surfaces. Observe the figure and choose the correct
alternative to answer the questions.

10. Maximum how many faces of a cube are painted?


(a) 5 (b) 3
(c) 4 (d) 2

11. How may cubes have at least two faces coloured?


(a) 12 (b) 20
(c) 28 (d) 48

12. How many cubes have only one face painted?


(a) 4 (b) 16
(c) 24 (d) 64

Q.13 to 15-Direction: The following figure is made by joining some cube of size 1 1 1 unit to each
other. Outer surfaces of the figure are painted. Observe the figure and answer the questions by
choosing the correct alternative.

13. Find the maximum number of faces of a cube that may have been painted.
(a) 5 (b) 4
(c) 3 (d) 2

14. Find the number of cubes having no face painted?


(a) 0 (b) 1
(c) 2 (d) 3

15. How many small cubes are used to form the given figure?
(a) 50 (b) 48
(c) 52 (d) 46
[NTSE-MAHARASHTRA STAGE-1-2018]

278 PACE IIT & MEDICAL: Mumbai / Delhi & NCR / Goa / Akola / Kolkata / Nashik / Pune / Bokaro / Dubai
NTSE-MENTAL ABILITY

Q.16 to 18-Direction: The following figure is made by arranging some cubes having each side 1 unit.
The figure is painted from all the sides. Observe the figure and answer the questions by choosing the
correct alternative.

16. How many cubes are there at the base level?


(a) 18 (b) 24
(c) 36 (d) 216

17. How many cubes are there having three faces painted?
(a) 8 (b) 10
(c) 15 (d) 20

18. How many cubes are there having no face painted?


(a) 3 (b) 2
(c) 0 (d) 4

Q.19 and 20: Directions – In the following figure the arrangement of small blocks is given observe it
and answer the following question.

19. What is the total number of small blocks?


(a) 17 (b) 24
(c) 27 (d) 30

20. Find the total number of blocks whose two surfaces are seen.
(a) 11 (b) 13
(c) 15 (d) 17

Q.21 and 22-Directions:- In the following figure small cubes are arranged in a particular manner as
shown. Observe the arrangement and answer the following questions

279 PACE IIT & MEDICAL: Mumbai / Delhi & NCR / Goa / Akola / Kolkata / Nashik / Pune / Bokaro / Dubai
NTSE-MENTAL ABILITY

21. What is the total number of small cubes?


(a) 52 (b) 49
(c) 47 (d) 45

22. What is the total number of blocks whose three surfaces are seen?
(a) 12 (b) 13
(c) 14 (d) 15 [2016]

Q.23 and 24-Directions: - 125 cubes of small size are arranged as given in the following figure. They
are painted form outside. Observed the figure and answer the questions by choosing the correct
alternative given below.

23. How many cubes have four sides coloured?


(a) 0 (b) 4
(c) 8 (d) 12

24. How many cubes have three sides coloured?


(a) 25 (b) 24
(c) 12 (d) 8 [2015]

Q.25 and 26-Directions: 64 cubes of same size are arranged according to the figure given below. These
cubes are painted from outside. Observe the figure and answer the following questions

25. How many cubes have two sides coloured?


(a) 4 (b) 8
(c) 12 (d) 24

26. How many cubes have no side painted?


(a) 0 (b) 4
(c) 8 (d) 12 [2014]
280 PACE IIT & MEDICAL: Mumbai / Delhi & NCR / Goa / Akola / Kolkata / Nashik / Pune / Bokaro / Dubai
NTSE-MENTAL ABILITY

Q.27 to 30-Directions:- In the following figure, the solid cube is painted on all sides by a single colour.
Observe the given solid and choose the correct alternative for the following questions.

27. Find the total number of blocks whose three surfaces are coloured?
(a) 8 (b) 12
(c) 18 (d) 24

28. Find total number of blocks whose surfaces are coloured


(a) 0 (b) 8
(c) 16 (d) 24

29. Find total number of blocks whose only one surface is coloured.
(a) 8 (b) 12
(c) 16 (d) 24

30. Find total number of blocks whose not even one surface is coloured.
(a) 12 (b) 16
(c) 8 (d) 24

Answer Key

9.
1. (b) 2. (c) 3. (a) 4. (c) 5. (c) 6. (a) 7. (d) 8. (c) 10. (b)
(Bonus)
11. (c) 12. (c) 13. (a) 14. (d) 15. (a) 16. (c) 17. (c) 18. (a) 19. (c) 20. (b)

21. (c) 22. (d) 23. (4) 24 (c) 25. (d) 26. (d) 27. (d) 28. (a) 29.(d) 30. (c)

281 PACE IIT & MEDICAL: Mumbai / Delhi & NCR / Goa / Akola / Kolkata / Nashik / Pune / Bokaro / Dubai
NTSE-MENTAL ABILITY

21. CLOCK TEST

Important Facts

 Minute hand and hour hand coincides once in every hour. They coincide 11 times in 12 hours and 22
times in 24 hours.
 They coincide only one time between 11 to 1 O’clock, at 12 O’clock.
 Minute hand and hour hand are opposite once in every hour. They do it 11 times in 12 hours and 22
times in 24 hours.
 They opposite only one time between 5 to 7 O’clock, at 6 O’ clock.
 Both hands (minute and hour) are perpendicular twice in every hour. 22 times in 12 hours and 44 times
in 24 hours.
 In one minute, hour hand moves 1/2° and minute hand moves 6°. In one hour, hour hand moves 30o and
minute hand moves 360°.
 In an hour, minute hand moves 55 minutes ahead of hour hand.

1. At what time between 3 O’ Clock and 4 O’ Clock will the two hands coincide ?
Sol. At 3 O’clock the distance between the two hands is 15 minutes when they coincide with each other the
distance between the two hands will be 0 min.
So, the time taken (15 + 0 ) = 15 minutes.
 Minute hand is 55 min. ahead of hour hand in 60 min.
60
 Minute hand is 1 min. ahead of hour hand in min.
55
60  15 180 4
 Minute hand is 15 min. ahead of hour hand in   16 min .
55 11 11
4
Hence the right time is 16 minute past 3.
11

2. At what time between 2 O’clock and 3 O’clock will the two hands be opposite?
Sol. At 2 O’clock the distance between the two hands is 10 minutes. When they are at 30 minutes distance,
they are opposite to each other. The time taken (30 + 10) = 40 min.
 Minute hand is 55 min. ahead of hour hand in 60 min.
60
 Minute hand is 1 min. ahead of hour hand in min .
55
60  40 480 7
 Minute hand is 40 minutes ahead of hour hand in   43 min.
55 11 11
7
Hence, the right time is 43 min. past 2.
11

3. At what time between 4 O’clock and 5 O’clock will the hands are perpendicular?
Sol. At 4 O’clock the distance between the two hands is 20 min. When they are at 15 minutes distance,
they are perpendicular to each other.
Case-I : When the time taken (20 – 15) = 5 min.
 Minute hand is 55 min. ahead of hour hand in 60 min.
60  5 60 5
 Minute hand is 5 min. ahead of hour hand in   5 min.
55 11 11

282 PACE IIT & MEDICAL: Mumbai / Delhi & NCR / Goa / Akola / Kolkata / Nashik / Pune / Bokaro / Dubai
NTSE-MENTAL ABILITY

5
Hence, the right time is 5 min. past 4.
11

Case-II: When the time taken (20 + 15) = 35 min.


 Minute hand is 55 min. ahead of hour hand in 60 min.
60  35 420 1
 Minute hand is 35 min. ahead of hour hand in   38 min.
55 11 11
2
Hence, the right time is 38 min. past 4.
11
Mirror Image of Clock

 If the time is between 1 O’clock to 11 O’clock, then to find the mirror image, time is subtracted from
11: 60.
 If the time is between 11 O’clock to 1 O’clock, then to find the mirror image, time is subtracted from
23 : 60.

4. The tine in the clock is 4 : 46, what is the mirror image?


Sol. (11 : 60) – (4 : 46) = 7 : 14

5. The time in the clock is 12: 35, then find its mirror image.
Sol. (23 : 60) – (12 : 35) = 11 : 25.

Angle between Two Hands

 Angle are of two types:


Positive angle : It is obtained by moving from hour hand to minute hand moving in clockwise direction.
Negative angle : It is obtained by moving from minute hand to hour hand.

 Both types of angles are 360o in total. If one angle is known, other can be obtained by subtracting from
360°.

6. At 4 : 30, what is the angle formed between hour hand and minute hand ?
Sol. At 4 O’ clock angle between hour and min. hand is of 120°.
 In 30 min. minute hand make an angle of 180°.
So, the resultant angle is 180° – 120° = 60°.
But in 30 min. hour hand will also cover an angle of 15°.
Hence, the final angle between both hands is 60o – 15°= 45°.

Time Sequence Test


7. A bus for Delhi leaves every thirty minutes from a bus stand. An enquiry clerk told a passenger that
the bus had already left ten minutes ago and the next bus will leave at 9.35 A.M. At what time did the
enquiry clerk give this information to the passenger ?
Sol. Bus leaves after every 30 minutes.
The next bus will leave at 9: 35 A.M.
The last bus left at 9 : 35 – 0 : 30 = 9 :05 A.M.
but clerk said that bus had left 10 minutes earlier.
 9 : 05 + 0 : 10 = 9 : 15 A.M.
283 PACE IIT & MEDICAL: Mumbai / Delhi & NCR / Goa / Akola / Kolkata / Nashik / Pune / Bokaro / Dubai
NTSE-MENTAL ABILITY

EXERCISE
1. At what time are the hands of a clock together between 5 O’clock and 6 O’clock?
3 3
(a) 33 minutes past 5 (b) 28 minutes past 5
11 11
3 3
(c) 27 minutes past 5 (d) 26 minutes past 5
11 11

2. At what between 9 O’clock and 10 O’clock will the hands of a clock be in the straight line, but not
together?
4
(a) 16 minutes past 9 (b) 16 minutes past 9
11
6 9
(c) 16 minutes past 9 (d) 16 minutes past 9
11 11

3. At what between 5 O’clock and 5:30 will the hands of a clock be at right angle?
10 5
(a) 10 minutes past 5 (b) 11 minutes past 5
11 11
10 9
(c) 9 minutes past 5 (d) 10 minutes past 5
11 11

4. If a clock shows 12 : 37 then its mirror image will be?


(a) 11: 37 (b) 11: 23 (c) 01 :23 (d) 21 :23

5. Find the angle between the two hands of a clock of 15 minutes past 4 O’clock.
(a) 38.5° (b) 36.5° (c) 375° (d) None of these

6. At what time between 5 and 6 O’clock the hands of a clock will make an angle of 18°.
(a) 12 minutes past 5 (b) 24 minutes past (c) 36 minutes past 5 (d) 40 minutes Past

7. Ashish leaves his house at 20 minutes to seven in the morning, reaches Kunal’s house in 25 minutes,
they finish their breakfast in another 15 minutes and leave for their office which takes another 35
minutes. At what time do they leave Kunal’s house to reach their office?
(a) 7.40 A.M. (b) 7.20 A.M. (c) 7.45 A.M. (d) 8.15 A.M.

8. The train for Lucknow leaves every two and a half hours from New Delhi Railway Station. An
announcement was made at the station that the train for Lucknow had left 40 minutes ago and the
next train will leave at 18. 00 hrs. At what time was the announcement made?
(a) 15.30 hrs (b) 17.10 hrs (c) 16.00 hrs (d) None of these

9. If the two incorrect watches are set at 12 : 00 noon at correct time, when will both the watches Show
the correct time for the first time given that the first watch gains 1 min in 1 hour and second watch
loses 4 min in 2 hours :
(a) 6 pm, 25 days later (b) 12 : 00 noon, 30 days later
(c) 12 noon, 15 days later (d) 6 am 45 days later

284 PACE IIT & MEDICAL: Mumbai / Delhi & NCR / Goa / Akola / Kolkata / Nashik / Pune / Bokaro / Dubai
NTSE-MENTAL ABILITY

10. The priest told the devotee, “The temple bell is rung at regular intervals of 45 minutes. The last
bell was rung five minutes ago. The next bell is due to be rung at 7.45 a.m.” At what time did the
priest give this information to the devotee?
(a) 7.40 am. (b) 7.05 a.m. (c) 6.55 a.m. (d) None of these

11. How many times are the hands of a clocks perpendicular in a day?
(a) 42 (b) 48 (c) 44 (d) 46

12. A watch, which gains uniformly, is 3 minutes slow at noon on Monday and is 3 minutes 48 seconds
fast at 2 p.m. on the following Monday. What time it was correct?
(a) 2 p.m. on Tuesday (b) 2 p.m. on Wednesday
(c) 3 p.m. on Thursday (d) 1 p.m. on Friday.

13. A monkey climbs 30 feet at the beginning of each hour and rests for a while when he slips back 20
feet before he again starts Climbing in the beginning of the next hour. If he begins his ascent at 8.00 am.,
at what time will he first touch a flag at 120 feet from the ground?
(a) 4 p.m. (b) 5 p.m. (c) 6 p.m (d) None of these

14. Rajeev and Sanjeev are too close friends Rajeev’s watch gains 1 minute in an hour and Sanjeev’s
watch loses 2 minutes in an hour. Once they set both the watches at 12 : 00 noon, with my correct
watch. When will the two incorrect watches of Rajeev and, Sanjeev show the same time together?
(a) 8 days later (b) 10 days later (c) 6 days later (d) can’t be determined

15. At a railway station a 24 hour watch loses 3 minutes in 4 hours. If it is set correctly on Sunday
noon when will the watch show the correct time?
(a) 6 pm after 40 days (b) 12 noon after 75 days
(c) 12 pm after loo days (d) 12 noon after 80 days

16. A Swiss watch is being shown in a museum which has a very peculiar property. It gains as much in
the day as it loses during night between 8 pm to 8 am. In a week how many times will the clock show
the correct time?
(a) 6 times (b) 14 times (c) 7 times (d) 8 times

17. A wrist watch which is running 12 minutes late on a Sunday noon is 16 minutes ahead of the correct
time at 12 noon on the next Sunday. When is the clock 8 minutes ahead of time ?
(a) Thursday 10 am (b) Friday noon (c) Friday 8 pm (d) Tuesday noon

18. A clock loses 2 minutes in a hour and another clock gains 2 minutes in every 2 hours. Both these
clocks are set correctly at a certain time on Sunday and both the clocks stop simultaneously on the
next day with the time shown being 9 am and 10:06 AM. What is the correct time at which they
stopped?
(a) 9:54am (b) 9:44pm (c) 9:46am (d) 9:44am

19. David sets his watch at 6 :10 am on Sunday, which gains 12 minutes in a day. On Wednesday if this
watch is showing 2 : 50 pm. What is the correct time ?
(a) 1: 50 pm (b) 2:10 pm (c) 2 :30 pm (d) 3:30 pm

285 PACE IIT & MEDICAL: Mumbai / Delhi & NCR / Goa / Akola / Kolkata / Nashik / Pune / Bokaro / Dubai
NTSE-MENTAL ABILITY

20. Ramu purchased a second hand Swiss watch which is very costly. In this watch the minute-hand and
3
hour hand coincide after every 65 minutes. How much time does the watch lose or gain per day?
11
(a) 4 min (b) 5 min (c) 4 mm, 20 sec (d) none of these

Direction :(21 to 22) A 12 dial clock has its minute hand defective Whenever it touches dial 12, it
immediately falls down to 6 instead of running smoothly (the hour hand remains unaffected during. that
fall). It was set right at 12 O’clock in the noon.

21. What was the actual time when the minute hand of the clock touched dial for the 5th time?
(a) 2:15 (b) 3:00 (c) 5:15 (d) 6:45

22. If the actual time is 10:10, what is the position of the hour hand in that defective clock ?
(a) Between 2 and 3 (b) Between 4 and 5
(c) Between 10 and 11 (d) Between 3 and 4

23. How many times the two hands of a watch fur wright angle from 12 O’clock noon to 6 O’clock
evening. (NTSE Stage-I/Raj./2007)
(a) 6 (b) 10 (c) 11 (d) 12

24. In a period of 12 hours how many times hours hand and minutes hand of a clock are in a straight
line facing the opposite direction ? (NTSE Stage-I/Raj./2008)
(a) 22 times (b) 11 times (c) 12 times (d) 24 times

25. At the time half past three, the angle between hour and minute hands of the watch will be –
(NTSE Stage-I/Raj./2008)
(a) 75° (b) 60° (c) 90° (d) 105°

26. At what angle are the hands of a clock inclined at 30 minutes past 6? (NTSE Stage-I/Raj./2013)
1o 1o
(a) 7 (b) 11 (c) 15o (d) 23o
2 2

27. A clock is set to show the correct time at 11 a.m. The clock gains 12 minutes in 12 hours what will be
the true time when the watch indicates 1p.m. on the 6th day? (NTSE Stage-I/Raj./2013)
(a) 10 a.m. (b) 11 a.m. (c) 12 noon (d) None of these

28. I left home for bringing milk between 7 am and 8 am. The angle between the hour-hand and the
minute-hand was 90° I returned home between 7 am and 8 am. Then also the angle between the
minute-hand and hour-hand was 90o. At what time (nearest to second) did I leave and return home?
(NTSE Stage-II, 2013)
(a) 7h 18m 35s & 7h 51m 24s (b) 7h 19m 24s & 7h 52m 14s
(c) 7h 20m 42s & 7h 53m 11s (d) 7h 21m 49s & 7h 54m 33s

29. A clock only with dots marking 3, 6, 9 and 12 O’clock position has been kept upside down in front of a
mirror. A person reads the time in the reflection of the clock as 10 : 20. What is the actual time?
(NTSE Stage-I/Haryana./2015)
(a) 8:10 (b) 2:40 (c) 4 : 50 (d) 10: 20
286 PACE IIT & MEDICAL: Mumbai / Delhi & NCR / Goa / Akola / Kolkata / Nashik / Pune / Bokaro / Dubai
NTSE-MENTAL ABILITY

30. Renu went to the market between 7 am and 8 am .The angle between the hour-hand and the minute-hand
was 90o. She returned home between 7 am and 8 am. Then also the angle between the minute-hand and
hour-hand was 90o. At what time (nearest to second) did Renu leave and return home ?
(NTSE Stage-II, 2015)
(a) 7h l8m 35s and 7h 5l m 24s (b) 7h 19m 24s and 7h 52m 14s
(c) 7h 20m 42s and 7h 53m 11s (d) 7h 21m 49s and 7h 54m 33s

31. One watch is 1 minute slow at 1 pm on Tues day and 2 minutes fast at 1 am on Friday when did it show
the correct time? (NTSE Stage-II, 2015)
(a) 5.00 am on Wednesday (b) 9.00am on Wednesday
(c) 5.00 pm on Wednesday (d) 9.00 pm on Wednesday

Answer Key

1. (c) 2. (b) 3. (a) 4. (b) 5. (c) 6. (b) 7. (b) 8. (d) 9. (b) 10. (b)
11. (c) 12. (c) 13. (c) 14. (b) 15. (d) 16. (d) 17. (b) 18. (d) 19. (b) 20. (a)

21. (a) 22. (c) 23. (c) 24. (b) 25. (a) 26. (c) 27. (b) 28. (d) 29. (a) 30. (d)

31. (b)

287 PACE IIT & MEDICAL: Mumbai / Delhi & NCR / Goa / Akola / Kolkata / Nashik / Pune / Bokaro / Dubai
NTSE-MENTAL ABILITY

22. FIGURE PARTITION


The problems on figure partition are based on counting number of figures generated due to partition lines.
 If a square is subdivided into n parts on each side, then the total number of squares formed is given by
n  n  1 2n  1
6
n  n  1 m  m  1
 Total number of rectangles (including squares) in a rectangular of size n  m 
2 2

1. What is the number of straight lines in the following figure?


(a) 11 (b) 14 (c) 16 (d) 17
Sol. (b) The figure is labeled as shown.
Clearly, there are 3 horizontal lines namely AE, LF and KG.
There are 5 vertical lines: AK, BJ, CI, DH and EG. There are 6 slanting lines: LC, KE, IF, LI, AG and
CF. Thus, there are 3 + 5 + 6 = 14 straight lines in the figure.

2. How many squares does the figure have?


(a) 6 (b) 7 (c) 9 (d) 10
Sol. (c) The figure may be labelled as shown :
The squares composed to two components each, are ABKJ, BCLK, CDEL, LEFG, KLGH, and JKHI.
Thus, there are 6 such squares. Only one square, KCEG is composed of four components. Two squares
namely, ACGI and BDFH are composed of eight components each. Thus, there are 2 such
squares. .. There are 6 + 1 + 2 = 9 squares in the given figure.

3. How many parallelograms are there in the figure below?

(a) 14 (b) 15 (c) 16 (d) 18

288 PACE IIT & MEDICAL: Mumbai / Delhi & NCR / Goa / Akola / Kolkata / Nashik / Pune / Bokaro / Dubai
NTSE-MENTAL ABILITY

Sol. (d) We can label the figure as shown.


The simplest parallelograms are ABFE, BCGF, CDHG, EFJI, FGKJ and GHKL. These are 6 in
number. The parallelograms composed of two components each, are ACGE, BDHF, EGKI, FHLJ,
BCKJ,ABJI, and CDLK. Thus, there are 7 such parallelograms. The parallelograms composed of
three components are ADHE and EHLI i.e. 2 in number .The parallelograms composed of four
components are ACKI and BDLJ Le. 2 in number. There is only one parallelograms composed of
six components, namely, ADLI. Thus, there are
6 + 7 + 2 + 2+ 1 = 18 parallelograms in the figure.
Hence,

4. What is the number of rectangles in the following figure?


(a) 6 (b) 7 (c) 8 (d) 9

Sol. (d) The figure is labelled as shown :


Simplest rectangles are AEHG, EF1H, FBKJ, JKCL and GILD. i.e. there are 5 such rectangles.
The rectangles composed of two components each are AFIG and FBCL. Thus, there are 2 such
rectangles. Only one rectangles, namely AFLD is,composed of 3 components and only one
rectangle, namely ABCD is composed of 5 components. Thus, there are 5 + 2 + 1 + 1 = 9
rectangles in the figure.

5. Determine the number of pentagons in the following figure:

(a) 5 (b) 6 (c) 8 (d) 10


Sol. The figure may be labelled as follows:
In this case, six pentagons have been formed by the combination of three triangles and two
rhombuses - ADFHJ, CFHJL, EHJLB, GJLBD, ILBDF and KBDFH. Four other pentagons are
289 PACE IIT & MEDICAL: Mumbai / Delhi & NCR / Goa / Akola / Kolkata / Nashik / Pune / Bokaro / Dubai
NTSE-MENTAL ABILITY

formed by the combination of three triangles and one rhombus - LCFHM, LBEHM, BKFHM and
BLIFM. Thus, there are 10 pentagons in the figure.

EXERCISE
1. How many squares are there in the following figure?

(a) 13 (b) 14 (c) 16 (d) 15

2. Count the number of triangles and squares in the following figure?

(a) 28 triangles, 10 squares (b) 28 triangles, 8 squares


(c) 32 triangles, 10 squares (d) 32 triangles, 8 squares

3. Count the number of squares in the following figure?

(a) 16 (b) 17 (c) 30 (d) 55

4. Count the number of straight lines and triangles in the following figure?

(a) 10 straight lines and 34 triangles (b) 9 straight lines and 34 triangles
(c) 9 straight lines and 36 triangles (d) 10 straight lines and 36 triangles

290 PACE IIT & MEDICAL: Mumbai / Delhi & NCR / Goa / Akola / Kolkata / Nashik / Pune / Bokaro / Dubai
NTSE-MENTAL ABILITY

5. Count the number of triangles in the following figure.

(a) 8 (b) 10 (c) 12 (d) 14

6. How many triangles and squares are there in the following figure?

(a) 28 triangles, 5 squares (b) 24 triangles, 4 squares


(c) 28 triangles, 4 squares (d) 24 triangles, 5 squares

7. Count the number of squares in the following figure?

(a) 15 (b) 21 (c) 24 (d) 26

8. How many squares does the following figure have?

(a) 17 (b) 18 (c) 13 (d) 16

9. How many triangles are there in the figure below?

(a) 5 (b) 6 (c) 8 (d) 10

291 PACE IIT & MEDICAL: Mumbai / Delhi & NCR / Goa / Akola / Kolkata / Nashik / Pune / Bokaro / Dubai
NTSE-MENTAL ABILITY

10. How many triangles are there in the following figure?

(a) 19 (b) 21 (c) 27 (d) 48

11. How many triangles does the following figure contain?

(a) 11 (b) 10 (c) 6 (d) 12

12. How many squares does the figure have?

(a) 17 (b) 12 (c) 13 (d) 15

13. How many rectangles are there in the given figure?

(a) 6 (b) 7 (c) 8 (d) 9

14. How many squares does the figure have?

(a) 10 (b) 11 (c) 12 (d) 14

292 PACE IIT & MEDICAL: Mumbai / Delhi & NCR / Goa / Akola / Kolkata / Nashik / Pune / Bokaro / Dubai
NTSE-MENTAL ABILITY

15. How many triangles are there in the following figure?

(a) 6 (b) 7 (c) 8 (d) 9

16. Find the number of convex pentagons:

(a) 2 (b) 3 (c) 4 (d) 6

17. How many triangles are there in the following figure?

(a) 11 (b) 14 (c) 16 (d) 7

18. How many quadrilateral there in the following figure?

(a) 11 (b) 8 (c) 2 (d) 4

Directions: (19 to 23) How many triangles are there in the following figures? (NTSE Stage-II, 2007)

19.

(a) 10 (b) 16 (c) 14 (d) 12

293 PACE IIT & MEDICAL: Mumbai / Delhi & NCR / Goa / Akola / Kolkata / Nashik / Pune / Bokaro / Dubai
NTSE-MENTAL ABILITY

20.

(a) 21 or more (b) 13 to 20 (c) 6 to 9 (d) 10 to 12

21.

(a) 14 or more (b) 12 to 13 (c) 10 to 11 (d) 7 to 9

22.

(a) 19 (b) 21 (c) 25 (d) 23

23.

(a) 11 or more (b) 10 (c) 9 (d) 8 or less

24. In the following figure, the number of triangles are- (NTSE Stage-I/Raj./2008)

(a) 8 (b) 12 (c) 15 (d) 16

294 PACE IIT & MEDICAL: Mumbai / Delhi & NCR / Goa / Akola / Kolkata / Nashik / Pune / Bokaro / Dubai
NTSE-MENTAL ABILITY

25. In the following figure, how many squares are there? (NTSE Stage-I/Raj./2009)

(a) 8 (b) 11 (c) 10 (d) 9

26. How many triangles are there in the following figure? (NTSE Stage-II, 2009)

(a) 10 (b) 11 (c) 12 (d) 13

27. How many squares are there in the following figure? (NTSE Stage-II, 2009)

(a) 18 (b) 15 (c) 13 (d) 12

28. What will be the number of triangles in the given figure? (NTSE Stage-I/Raj./2012)

(a) 9 (b) 11 (c) 10 (d) 12

29. What will be the number of Hexagonals in the given figure? (NTSE Stage-I/Raj./2013)

(a) 2 (b) 4 (c) 5 (d) 6

295 PACE IIT & MEDICAL: Mumbai / Delhi & NCR / Goa / Akola / Kolkata / Nashik / Pune / Bokaro / Dubai
NTSE-MENTAL ABILITY

30. What will be the number of Parallelogram’s in the given figure? (NTSE Stage-I/Raj./2013)

(a) 15 (b) 17 (c) 13 (d) 16

31. How many triangles are in the given figure? (NTSE Stage-I/Karnataka/2014)

(a) 21 (b) 22 (c) 23 (d) 24

32. How many triangles are there in the figure below? (NTSE Stage-I/Rajasthan/2016)

(a) 5 (b) 6 (c) 8 (d) 10

33. Determine the number of pentagons in the following figure. (NTSE Stage-I/Rajasthan/2017)

(a) 5 (b) 6 (c) 8 (d) 10

34. Determine the number of triangles in the following figure. (NTSE Stage-I/Rajasthan/2017)
(a) 5 (b) 6 (c) 8 (d) 10

296 PACE IIT & MEDICAL: Mumbai / Delhi & NCR / Goa / Akola / Kolkata / Nashik / Pune / Bokaro / Dubai
NTSE-MENTAL ABILITY

Answer Key

1. (d) 2. (c) 3. (c) 4. (c) 5. (c) 6. (a) 7. (c) 8. (b) 9. (d) 10. (c)
11. (d) 12. (a) 13. (d) 14. (b) 15. (c) 16. (d) 17. (c) 18. (a) 19. (b) 20. (b)

21. (a) 22. (c) 23. (a) 24. (d) 25. (b) 26. (d) 27. (b) 28. (d) 29. (c) 30. (b)

31. (b) 32. (d) 33. (b) 34. (d)

PREVIOUS YEARS QUESTIONS


1 Find number of triangles in the given figure

(a) 16 (b) 20
(c) 24 (d) 32

2. Find total number of squares in the following figure

(a) 6 (b) 12
(c) 13 (d) 15
[NTSE-MAHARASHTRA STAGE-1-2020]
3. Find the number of triangles in the adjacent figure

(a) 12 (b) 16
(c) 20 (d) 24

4. Find the number of Squares from the adjacent figure

(a) 6 (b) 11
(c) 13 (d) 10
[NTSE-MAHARASHTRA STAGE-1-2019]
297 PACE IIT & MEDICAL: Mumbai / Delhi & NCR / Goa / Akola / Kolkata / Nashik / Pune / Bokaro / Dubai
NTSE-MENTAL ABILITY

Q.5 and Q6: Direction: Find the number of triangles in the following figures.

5.

(a) 16 (b) 32
(c) 40 (d) 80

6.

(a) 48 (b) 50
(c) 58 (d) 62 [2018]

7. Observe the adjoining figure and answer the following question. Choosing the correct alternative. How
many isosceles trapezium are in the figure?

(a) 16 (b) 10
(c) 8 (d) 14

8. Observe the figure and the number of isosceles traingles which are not equilateral.

(a) 6 (b) 8
(c) 10 (d) 12

298 PACE IIT & MEDICAL: Mumbai / Delhi & NCR / Goa / Akola / Kolkata / Nashik / Pune / Bokaro / Dubai
NTSE-MENTAL ABILITY

Q9. And Q.10 Directions: - Observe the following figure and answre the following questions by choosing
the corrct alternative given below.

9. Find the number of rectangels which are not squares.


(a) 16 (b) 20
(c) 24 (d) 28

10. How many squares are there in the figure?


(a) 0 (b) 3
(c) 10 (d) 13 [2015]

Q11 and 12  Directions: - Observe the following figure and anser the following questions by choosing
the correct alternative given below

11. Find the number of equilateral triangles in the given figure


(a) 32 (b) 18
(c) 5 (d) 0

12. Find the number of parallelograms which are not rectangles from the given figure
(a) 4 (b) 6
(c) 8 (d) 10 [2014]

Q13 to 15  Directions : - Observe the following figure and answer the following questions choosing the
correct alternative.

299 PACE IIT & MEDICAL: Mumbai / Delhi & NCR / Goa / Akola / Kolkata / Nashik / Pune / Bokaro / Dubai
NTSE-MENTAL ABILITY

13. What is the total number of parallelograms which are not rectangles?
(a) 24 (b) 20
(c) 18 (d) 16

14. How many pentagons are there in the figure?


(a) 32 (b) 28
(c) 30 (d) 26

15. What is the total number of isosceles trapezium in the figure?


(a) 4 (b) 8
(c) 12 (d) 20 [2012]

Answer Key

1. 2. 9.
3. (c) 4. (b) 5. (c) 6.(d) 7. (c) 8. (b) 10. (4)
(Bonus) (Bonus) (Bonus)
11. (d) 12. (a) 13. (a) 14.(b) 15.(c)

300 PACE IIT & MEDICAL: Mumbai / Delhi & NCR / Goa / Akola / Kolkata / Nashik / Pune / Bokaro / Dubai
NTSE-MENTAL ABILITY

23. MIRROR IMAGES

Here questions are based on criteria that few figures are given and you have to find out of which one is exact
image of the given figure by the mirror placed aside. This image formation is based on the principle of
‘LATERAL INVERSION’ which implies that size of the image is equal to the size of the object but both sides
are changed. The left portion of the object is seen on the right portion and right portion of the object is seen on
the left portion. For example mirror image of ABC = .

NOTE: There are ‘11’ letters in English Alphabet which have identical mirror images as
A, H, I, M, O,T, U, V, W,X,Y

I. Mirror Images of capital letters & small letters.

II. Mirror Images of Numbers.

Reflection of an object into the mirror is called mirror-image. It is obtained by inverting an object laterally i.e.
towards tile sides. Example of lateral inversions of few figures and words are given below:

301 PACE IIT & MEDICAL: Mumbai / Delhi & NCR / Goa / Akola / Kolkata / Nashik / Pune / Bokaro / Dubai
NTSE-MENTAL ABILITY

Objects having Different Mirror Images:


OBJECTS MIRROR IMAGES

302 PACE IIT & MEDICAL: Mumbai / Delhi & NCR / Goa / Akola / Kolkata / Nashik / Pune / Bokaro / Dubai
NTSE-MENTAL ABILITY

Mirror images of certain words and numbers:

Water Image
Water-image: The reflection of an object as seen in water is called ts water image. It is the inverted
image obtained by turning the object upside down.

303 PACE IIT & MEDICAL: Mumbai / Delhi & NCR / Goa / Akola / Kolkata / Nashik / Pune / Bokaro / Dubai
NTSE-MENTAL ABILITY

NOTE: There are ‘7’ letters ¡n English Alphabet which have identical water-images as
C, D, E , H, I, O ,X.

Water-Images of capital letters

Water-Images of small letters

Water-Images of numbers

EXERCISE

Directions: (1 to 17) In each of the following questions, choose the correct mirror image from
alternatives A, B, C and D of the Word/figure (X).

1. VINAYAKA

2. VERBAL

3. CONSOLIDATE

304 PACE IIT & MEDICAL: Mumbai / Delhi & NCR / Goa / Akola / Kolkata / Nashik / Pune / Bokaro / Dubai
NTSE-MENTAL ABILITY

4. JUDGEMENT

5. TARAIN1014A

6.

7.

8.

305 PACE IIT & MEDICAL: Mumbai / Delhi & NCR / Goa / Akola / Kolkata / Nashik / Pune / Bokaro / Dubai
NTSE-MENTAL ABILITY

9.

10.

11.

12.

306 PACE IIT & MEDICAL: Mumbai / Delhi & NCR / Goa / Akola / Kolkata / Nashik / Pune / Bokaro / Dubai
NTSE-MENTAL ABILITY

13.

14.

15.

16.

307 PACE IIT & MEDICAL: Mumbai / Delhi & NCR / Goa / Akola / Kolkata / Nashik / Pune / Bokaro / Dubai
NTSE-MENTAL ABILITY

17.

Directions (18 to 22) In the following questions you have to visualize the image of the item
(Letter/word/ number/figures) in the mirror. The mirror is below the item. Choose the mirror image
from the alternatives. (NTSE Stage-I/Raj./2008)

18.

19.

20.

21.

308 PACE IIT & MEDICAL: Mumbai / Delhi & NCR / Goa / Akola / Kolkata / Nashik / Pune / Bokaro / Dubai
NTSE-MENTAL ABILITY

22.

Directions: (23 to 25) In the following questions you have to identify the image of the item (Letter/word/
number) in the mirror. The mirror is placed below the item. Choose the true image from the given
alternatives. (NTSE Stage-I/Raj./2009)

23.

24.

25.

309 PACE IIT & MEDICAL: Mumbai / Delhi & NCR / Goa / Akola / Kolkata / Nashik / Pune / Bokaro / Dubai
NTSE-MENTAL ABILITY

Directions: (26 to 27) Find the correct alternative which is the mirror image of the figure given in each
question. (NTSE Stage-II, 2011)

26.

27.

Directions: (28 to 32) In the following questions visualize the image of the correct item.
(NTSE Stage-I/Raj./2012)
28.

29.

310 PACE IIT & MEDICAL: Mumbai / Delhi & NCR / Goa / Akola / Kolkata / Nashik / Pune / Bokaro / Dubai
NTSE-MENTAL ABILITY

30.

31.

32. (NTSE Stage-II, 2013)

33. What is the mirror image of b3k4s I ?

311 PACE IIT & MEDICAL: Mumbai / Delhi & NCR / Goa / Akola / Kolkata / Nashik / Pune / Bokaro / Dubai
NTSE-MENTAL ABILITY

EXERCISE -2

Directions: (1 to 16) In each of the following questions, choose the correct water-image from alternatives
A, B, C and D of the Word/figure (X).

1.

2.

3.

4.

5.

6.

312 PACE IIT & MEDICAL: Mumbai / Delhi & NCR / Goa / Akola / Kolkata / Nashik / Pune / Bokaro / Dubai
NTSE-MENTAL ABILITY

7.

8.

9.

313 PACE IIT & MEDICAL: Mumbai / Delhi & NCR / Goa / Akola / Kolkata / Nashik / Pune / Bokaro / Dubai
NTSE-MENTAL ABILITY

10.

11.

12.

314 PACE IIT & MEDICAL: Mumbai / Delhi & NCR / Goa / Akola / Kolkata / Nashik / Pune / Bokaro / Dubai
NTSE-MENTAL ABILITY

13.

14.

15.

315 PACE IIT & MEDICAL: Mumbai / Delhi & NCR / Goa / Akola / Kolkata / Nashik / Pune / Bokaro / Dubai
NTSE-MENTAL ABILITY

16.

Directions: (17 to 21) In the following questions, you have to visualize the image of the item
(Letter/word/number/figure) in the water. The water source is below the item. Choose the water image
from the alternatives. (NTSE Stage-II, 2007)

17.

18.

19.

20.

316 PACE IIT & MEDICAL: Mumbai / Delhi & NCR / Goa / Akola / Kolkata / Nashik / Pune / Bokaro / Dubai
NTSE-MENTAL ABILITY

21.

Directions: (22 to 24) In the following questions you have to visualize the image of the item (figure, word,
letter, number) in water. The water source is below the item. Choose the water image from the
alternatives. (NTSE Stage-II, 2008)

22.

23.

24.

25. What will be water image of CHICK?

317 PACE IIT & MEDICAL: Mumbai / Delhi & NCR / Goa / Akola / Kolkata / Nashik / Pune / Bokaro / Dubai
NTSE-MENTAL ABILITY

Direction : (26) find the correct mirror image of the given figure, when mirror is placed or night side of
the figure. (NTSE Stage-I/Raj./ 2017)

26

Answer Key

EXERCISE -1

1. (c) 2. (d) 3. (b) 4. (c) 5. (a) 6. (c) 7. (a) 8. (c) 9. (c) 10. (d)
11. (d) 12. (b) 13. (a) 14. (b) 15. (b) 16. (d) 17. (b) 18. (b) 19. (d) 20. (a)

21. (b) 22. (a) 23. (b) 24. (a) 25. (c) 26. (b) 27. (c) 28. (b) 29. (a) 30. (b)

31. (b) 32. (a) 33. (a)

EXERCISE -2

1. (c) 2. (b) 3. (a) 4. (d) 5. (c) 6. (a) 7. (c) 8. (d) 9. (d) 10. (b)
11. (a) 12. (c) 13. (c) 14. (b) 15. (b) 16. (d) 17. (d) 18. (a) 19. (d) 20. (b)

21. (c) 22. (c) 23. (a) 24. (b) 25. (a) 26. (b)

318 PACE IIT & MEDICAL: Mumbai / Delhi & NCR / Goa / Akola / Kolkata / Nashik / Pune / Bokaro / Dubai
NTSE-MENTAL ABILITY

PREVIOUS YEARS QUESTIONS

Q.1 to 2: Directions: Choose the mirror image from the alternatives given for the given question figures.

1. Question figure

Answer figures

(a) (b)

(c) (d)

2. Question figures

Answer figures

(a) (b)

(c) (d)

[NTSE-MAHARASHTRA STAGE-1-2020]

319 PACE IIT & MEDICAL: Mumbai / Delhi & NCR / Goa / Akola / Kolkata / Nashik / Pune / Bokaro / Dubai
NTSE-MENTAL ABILITY

Q.3 and 4: Directions: Choose the mirror image from the alternatives given for the given question
figures.
3. Question figure

(a) (b)

(c) (d)

4. Question figure

Answer figures

(a) (b)

(c) (d)

[NTSE-MAHARASHTRA STAGE-1-2020]

320 PACE IIT & MEDICAL: Mumbai / Delhi & NCR / Goa / Akola / Kolkata / Nashik / Pune / Bokaro / Dubai
NTSE-MENTAL ABILITY

Q.5 and 6: Direction Choose the water image from the alternatives given for the question figure.
5. Question figure

(a) (b)

(c) (d)

6. Question Figure

(a) (b)

(c) (d)

Q.7 and 8: Direction Choose the correct mirror image from the alternatives given for the question figure.
7. Question figure

(a) (b)

(c) (d)

321 PACE IIT & MEDICAL: Mumbai / Delhi & NCR / Goa / Akola / Kolkata / Nashik / Pune / Bokaro / Dubai
NTSE-MENTAL ABILITY

8. Question figure

Answer figure

(a) (b)

(c) (d)
[NTSE-MAHARASHTRA STAGE-1-2019]

Q 9. And 10: Direction: Choose the mirror image from the alternative given for the given question figure

9.

(a) (b)

(c) (d)

10.

(a) (b)

(c) (d)

[NTSE-MAHARASHTRA STAGE-1-2018]

322 PACE IIT & MEDICAL: Mumbai / Delhi & NCR / Goa / Akola / Kolkata / Nashik / Pune / Bokaro / Dubai
NTSE-MENTAL ABILITY

Q.11 and 12: Direction Choose the correct water image from the given alternatives for the given question
figure.
11. Question figure

Answer figures

(a) (b)

(c) (d)

12. Question figure

Answer figures

(a) (b)

(c) (d)
[NTSE-MAHARASHTRA STAGE-1-2018]
13. The following figure is rotated in anticlockwise direction and its mirror image is obtained. Select the
correct mirror image from four alternatives given

(a) (b)

(c) (d) [2017]

323 PACE IIT & MEDICAL: Mumbai / Delhi & NCR / Goa / Akola / Kolkata / Nashik / Pune / Bokaro / Dubai
NTSE-MENTAL ABILITY

14. Choose the correct mirror image of the question figure from the alternatives given below
Question figure:

(a) (b)

(c) (d)

15. Choose the correct water image of the question figure from the alternatives given below:
Question figure:

(a) (b)

(c) (d) [2015]

16. Choose the mirror image of the ‘Question Figure’ from the given alternatives.
Question figure:

(a) (b)

(c) (d)

324 PACE IIT & MEDICAL: Mumbai / Delhi & NCR / Goa / Akola / Kolkata / Nashik / Pune / Bokaro / Dubai
NTSE-MENTAL ABILITY

17. Choose the water image of the ‘Question Figure’ from the given alternatives.
Question Figure

Answer Figure

(a) (b)

(c) (d) [2014]

Q.18 and 19:  Direction:- Question figure given to the left hand side is held vertically on water surface.
How will the reflection of the given figure in water look? Select the correct alternative.

18. Question

(a) (b)

(c) (d)

19. Question Figure:

(a) (b)

(c) (d) [2012]

325 PACE IIT & MEDICAL: Mumbai / Delhi & NCR / Goa / Akola / Kolkata / Nashik / Pune / Bokaro / Dubai
NTSE-MENTAL ABILITY

20. The following figure is rotated in anticlockwise direction through 900 and its mirror image is obtained.
Select the correct mirror image from four alternatives given.
Questions figure

(a) (b)

(c) (d) [2012]

Answer Key

1. (a) 2. (c) 3. (d) 4. (c) 5. (b) 6. (c) 7. (c) 8. (a) 9. (d) 10. (d)
11. (d) 12. (a) 13. (b) 14. (d) 15. (b) 16. (c) 17. (c) 18. (a) 19.(d) 20.(c)

326 PACE IIT & MEDICAL: Mumbai / Delhi & NCR / Goa / Akola / Kolkata / Nashik / Pune / Bokaro / Dubai
NTSE-MENTAL ABILITY

24. PAPER FOLDING


The problems on paper folding involve the process of selecting a figure which would most nearly match the
pattern that would be formed when a transparent sheet carrying designs on either side of a dotted line is folded
along this line. The figure has to be selected from a set of four alternatives (answer or response figures).

1. In the following questions problems, a square transparent sheet with a pattern is given. Figure
out from amongst four alternatives as to how the pattern would appear when the transparent sheet
is folded at the dotted line.

(a) (b) (c) (d)


Sol. (b) Clearly, the lower half of the square sheet has been folded over the upper half. Hence, the bent
line in the lower half will be inverted over the other half so that a ‘V’ shaped figure is formed.

Paper Cutting
The problems on paper cutting contain a set of three figures showing the manner in which a piece
of paper has been folded. In each of the first two figures, a dotted line together with an arrow on it
has been given indicating the line along which the paper is to be folded and the direction of the
fold respectively. In the third figure, there are marks showing the position and nature of the cut made
in the folded sheet. The candidate has to select one of the figures from the set of four answer figures A, B, C
and D, that would most nearly match the pattern when the paper is unfolded. It will be interested to see that the
designs of the cut will appear on each fold made in the paper.

2. Consider the following three figures, marked X, Y, Z showing one fold in X, another in Y and cut in
Z. From amongst the answer figures A, B, C and D, select the one showing the unfolded position of
Z.

(a) (b) (c) (d)


Sol. (c) In figure X, the upper triangular half of the paper has been folded over the lower half. In figure Y,
the paper is refolded to a quarter triangle. In figure Z, a square has been punched in the folded paper.
Clearly, the square will appear in each of the triangular quarters of the paper. Thus, when the
paper is unfolded, four squares will appear symmetrically over it and it will resemble figure (c).

327 PACE IIT & MEDICAL: Mumbai / Delhi & NCR / Goa / Akola / Kolkata / Nashik / Pune / Bokaro / Dubai
NTSE-MENTAL ABILITY

EXERCISE

Directions: (1 to 6) A square transparent sheet with a pattern is given in figure X. Find out from amongst
the alternatives as to how the pattern would appear when the transparent sheet is folded at the dotted
line.

1.

2.

(A) (B) (C) (D)

3.

4.

328 PACE IIT & MEDICAL: Mumbai / Delhi & NCR / Goa / Akola / Kolkata / Nashik / Pune / Bokaro / Dubai
NTSE-MENTAL ABILITY

5.

6.

Directions: (7 to 18) Sheet has been folded in the manner as shown in X, Y and Z respectively and
punched. You have to choose from the alternatives how it will look when unfolded.

7.

8.

329 PACE IIT & MEDICAL: Mumbai / Delhi & NCR / Goa / Akola / Kolkata / Nashik / Pune / Bokaro / Dubai
NTSE-MENTAL ABILITY

9.

10.

11.

12.

13.

330 PACE IIT & MEDICAL: Mumbai / Delhi & NCR / Goa / Akola / Kolkata / Nashik / Pune / Bokaro / Dubai
NTSE-MENTAL ABILITY

14.

15.

16.

17.

(NTSE Stage-II, 2008)

331 PACE IIT & MEDICAL: Mumbai / Delhi & NCR / Goa / Akola / Kolkata / Nashik / Pune / Bokaro / Dubai
NTSE-MENTAL ABILITY

18.

(NTSE Stage-II, 2008)

Directions: (19 to 22) The following questions are related to paper cutting. The questions that follow
contain a set of three figures X, Y and Z, showing a sequence of folding of a piece of paper. Fig. (Z) shows
the manner in which the folded paper has been cut. These three figures are followed by four answer
figures A, B, C and D (II nd Set) from which you have to choose a figure which would most closely
resemble the unfolded from of fig. (Z) (NTSE Stage-I/Raj./2013)

19. Sequence of folding the paper.

Answer Figures

20. Sequence of folding the paper

Answer Figures

332 PACE IIT & MEDICAL: Mumbai / Delhi & NCR / Goa / Akola / Kolkata / Nashik / Pune / Bokaro / Dubai
NTSE-MENTAL ABILITY

21. Sequence of folding the paper

Answer Figures

22. Sequence of folding the paper

Answer Figures

Answer Key

1. (d) 2. (b) 3. (d) 4. (a) 5. (b) 6. (a) 7. (b) 8. (b) 9. (d) 10. (a)
11. (a) 12. (b) 13. (a) 14. (c) 15. (d) 16. (d) 17. (d) 18. (a) 19. (c) 20. (b)

21. (a) 22. (d)

PREVIOUS YEARS QUESTIONS

Q.1 to 2: Directions: A square shaped paper is folded as shown in the figure. The paper when unfolded ill
look like as shown in one of the alternatives. Select the correct alternative.

1. Questions figure

Answer figure

333 PACE IIT & MEDICAL: Mumbai / Delhi & NCR / Goa / Akola / Kolkata / Nashik / Pune / Bokaro / Dubai
NTSE-MENTAL ABILITY

2. Question figure

(a) (b)

(c) (d)

[NTSE-MAHARASHTRA STAGE-1-2020]

Q.3 and 4
Directions: A square piece of paper is folded and cut at specific spots as shown in the figures. The paper
when unfolded will look-like as shown in one of the alternatives. Select the correct alternative.

3. Question figure

Answer figure

(a) (b)

(c) (d)

334 PACE IIT & MEDICAL: Mumbai / Delhi & NCR / Goa / Akola / Kolkata / Nashik / Pune / Bokaro / Dubai
NTSE-MENTAL ABILITY

4. Question figure

(a) (b)

(c) (d)
[NTSE-MAHARASHTRA STAGE-1-2020]
Q.5 and 6
Directions: In the figure given below a transparent square shaped paper is folded along the dotted lines,
which figure will be obtained? Find the figure from the alternative figures given
5 Question Figure

Answer figures

(a) (b)

(c) (d)

6. Question figure

Answer figure

(a) (b)

(c) (d)
[NTSE-MAHARASHTRA STAGE-1-2020]
335 PACE IIT & MEDICAL: Mumbai / Delhi & NCR / Goa / Akola / Kolkata / Nashik / Pune / Bokaro / Dubai
NTSE-MENTAL ABILITY

Q.7and 8: Direction: After folding a square piece of paper it appears as shown in the question figure. The
paper when unfolded will look like as shown in one of the alternatives. Select the correct alternative.
7. Question figure

Answer figure

(a) (b)

(c) (d)

8. Question figure

Answer figure

(a) (b)

(c) (d)
[NTSE-MAHARASHTRA STAGE-1-2020]

336 PACE IIT & MEDICAL: Mumbai / Delhi & NCR / Goa / Akola / Kolkata / Nashik / Pune / Bokaro / Dubai
NTSE-MENTAL ABILITY

Q.9 and 10: Direction In the figure give below, a transparent square shaped paper is folded along the
dotted lines, which figure will be obtained? Choose the correct figure from the given alternatives.

9. Question figure

Answer figure

(a) (b)

(c) (d)

10. Question figure

(a) (b)

(c) (d)
[NTSE-MAHARASHTRA STAGE-1-2020]

337 PACE IIT & MEDICAL: Mumbai / Delhi & NCR / Goa / Akola / Kolkata / Nashik / Pune / Bokaro / Dubai
NTSE-MENTAL ABILITY

Q.11 and 12: Direction A square piece of paper is folded and cut at specific spots as shown in the figure.
The paper when unfolded will look like as one of the alternative given. Choose the correct alternative.

11. Question Figure

Answer Figure

(a) (b)

(c) (d)

12. Question figure

Answer figure

(a) (b)

(c) (d)
[NTSE-MAHARASHTRA STAGE-1-2019]

338 PACE IIT & MEDICAL: Mumbai / Delhi & NCR / Goa / Akola / Kolkata / Nashik / Pune / Bokaro / Dubai
NTSE-MENTAL ABILITY

Q.13 and 14
Direction: In the figure given below a transparent square shaped paper is folded along the dotted lines.
Which figure will be obtained? Find the correct figure from the alternative given.

13. Question figure

Answer figure

(a) (b)

(c) (d)

14 Question figure

Answer figure

(a) (b)

(c) (d)
[NTSE-MAHARASHTRA STAGE-1-2018]

339 PACE IIT & MEDICAL: Mumbai / Delhi & NCR / Goa / Akola / Kolkata / Nashik / Pune / Bokaro / Dubai
NTSE-MENTAL ABILITY

Q.15 and 16
Direction: After folding a square piece of paper, it appears as shown in the question figure. The paper
when unfolded will look like as shown in one of the alternatives. Select the correct alternative.

15.

(a) (b)

(c) (d)

16.

(a) (b)

(c) (d)
[NTSE-MAHARASHTRA STAGE-1-2018]

340 PACE IIT & MEDICAL: Mumbai / Delhi & NCR / Goa / Akola / Kolkata / Nashik / Pune / Bokaro / Dubai
NTSE-MENTAL ABILITY

Q.17 and 18: Direction: A square place of paper is folded and cut at specific spots as shown in the figure.
The paper when unfolded will look like as shown in one of the alternative.

17. Question figure

Answer Figures

(a) (b)

(c) (d)

18. Question figure

(a) (b)

(c) (d)
[NTSE-MAHARASHTRA STAGE-1-2018]

341 PACE IIT & MEDICAL: Mumbai / Delhi & NCR / Goa / Akola / Kolkata / Nashik / Pune / Bokaro / Dubai
NTSE-MENTAL ABILITY

19. A square piece of paper is folded and cut at specific spots as shown in the figure. The paper when
unfolded will look like as shown in area of the alternatives. Select the correct alternative.

(a) (b)

(c) (d) [2017]

20. In the figure given below, a transparent square shaped paper is folded along the dotted lines. What figure
will be obtained? Find the figure from the alternative figures given

(a) (b)

(c) (d) [2017]

21. After folding a square piece of paper, it appears as shown in the left side question figure. The paper
when unfolded will look like as shown in one of the alternatives. Select the correct alternative.

(a) (b)

(c) (d)

342 PACE IIT & MEDICAL: Mumbai / Delhi & NCR / Goa / Akola / Kolkata / Nashik / Pune / Bokaro / Dubai
NTSE-MENTAL ABILITY

22. A square piece of paper is folded and cut at specific spot as shown in the figure. The paper when
unfolded will look like as shown in one of the alternatives. Select the correct alternative
Question Figure

(a) (b)

(c) (d) [2016]

23. Directions:- In the figure given below, a transparent square shaped paper is folded along the
dotted lines. What figure is obtained? Find the figure from the alternative figure given
Question figure

(a) (b)

(c) (d)

24. A folded piece of square paper is shown as question figure. The paper is unfolded. How will it look is
shown in the alternatives. Select the correct alternative.
Question Figure

(a) (b)

(c) (d) [2016]


343 PACE IIT & MEDICAL: Mumbai / Delhi & NCR / Goa / Akola / Kolkata / Nashik / Pune / Bokaro / Dubai
NTSE-MENTAL ABILITY

25. Direction-A square piece of paper is folded and cut at specific spots as shown in the question
figure. The paper when unfolded will look as shown in one of the alternatives. Select the correct
alternative.
Question Figure

(a) (b)

(c) (d)

26. A folded piece of square paper is shown as question figure. When the paper is unfolded, how will it look
is shown in the alternatives. Select the correct alternative:
Question figure

Answer figures:

(a) (b)

(c) (d) [2015]

27. A transparent square sheet of paper is folded on the dotted line as shown in the question figure. Choose
the correct alterantive from the answer figures as how the paper will look when folded:
Question figure

Answer figures:

(a) (b)

344 PACE IIT & MEDICAL: Mumbai / Delhi & NCR / Goa / Akola / Kolkata / Nashik / Pune / Bokaro / Dubai
NTSE-MENTAL ABILITY

(c) (d)

28. Directions – A square piece of paper is folded and cut at specific spots as shown in the figure. The
paper when unfolded will look as shown in one of the alternatives. Select the correct alternative.
Question figure

Answer figures:

(a) (b)

(c) (d) [2014]

29. Directions-A folded piece of square paper is shown as question figure. The paper is ufolded. How
will it loot is shown in the alternatives. Select the correct alaternative.
Question Figure

Answer Figures

(a) (b)

(c) (d) [2014]

345 PACE IIT & MEDICAL: Mumbai / Delhi & NCR / Goa / Akola / Kolkata / Nashik / Pune / Bokaro / Dubai
NTSE-MENTAL ABILITY

30. A square transparent sheet of paper is folded on the dotted line as shown in the question figure. Choose
the correct alternative from the answer figure as how the paper will look when folded.
Question figure

Answer Figure

(a) (b)

(c) (d)
[2014]
31. In the figure given below, a transparent squar shaped paper is folded along the dotted lines. What figure
will be obtained? Find the figure fromt eh alternative figures given
Question Figure

(a) (b)

(c) (d) [2012]

Q.32 and 33  Directions : - A square piece of paper is folded and cut at specific spots as shown in the
figure. The paper when unfolded will look like as shown in one of the alternatives. Select the correct
alternative.

32. Question figure

(a) (b)

346 PACE IIT & MEDICAL: Mumbai / Delhi & NCR / Goa / Akola / Kolkata / Nashik / Pune / Bokaro / Dubai
NTSE-MENTAL ABILITY

(c) (d)

33. Question figure

(a) (b)

(c) (d) [2012]


34. A folded piece of square paper is shown as question figure at left side. The paper is unfolded. How will
it look is shown in the alternatives on right side. Select the correct alternative.
Question Figure

(a) (b)

(c) (d)
[2012]

Answer Key

1. (d) 2. (3) 3. (d) 4. (d) 5. (a) 6. (d) 7. (d) 8. (d) 9. (b) 10. (d)
11. (b) 12. (b) 13. (b) 14. (c) 15. (d) 16. (b) 17. (b) 18. (c) 19.(a) 20.(d)
21. (c) 22. (a) 23 (c) 24. (a) 25. (b) 26. (b) 27.(c) 28. (c) 29.(a) 30. (b)
31. (b) 32. (d) 33. (d) 34. (a)

347 PACE IIT & MEDICAL: Mumbai / Delhi & NCR / Goa / Akola / Kolkata / Nashik / Pune / Bokaro / Dubai
NTSE-MENTAL ABILITY

25. FIGURE COMPLETION


In such problems, a figure following a particular sequence or pattern is given, in which a part usually one-fourth
is left blank. This problem figure is followed by four alternative figures. One is required to choose the one
which best fit into the blank space of problem figure so as to complete the original pattern.

Directions : (1 to 2 ) Selecta figure from the four alternatives, which when placed in the blank space of
figure (X) would complete the pattern.
1.

(a) (b) (c) (d)


Sol. (d) Clearly, figure (d) will complete the pattern when placed in the blank space of figure (X) as shown
below.
2.

(a) (b) (c) (d)


Sol. (c) Clearly, figure (c) will complete the pattern when placed in the blank space of figure (X) as shown
below.

Figure Formation
Direction: (3 to 4) In the following question, find out which of the figure (a), (b), (c) and (d) can be formed
from the pieces given in figure (X).

3.

(a) (b) (c) (d)


Sol. (a) On close observation, we find that only figure (a) is combined figure of (x).
348 PACE IIT & MEDICAL: Mumbai / Delhi & NCR / Goa / Akola / Kolkata / Nashik / Pune / Bokaro / Dubai
NTSE-MENTAL ABILITY

4.

(a) (b) (c) (d)


Sol. (b) On close observation, we find that only figure (b) is combined figure of (x).

EXERCISE

Directions : (1 to 7) Select a figure from the alternatives which when placed in the blank space of (x)
would complete the pattern ?

1.

2.

3.

4.

349 PACE IIT & MEDICAL: Mumbai / Delhi & NCR / Goa / Akola / Kolkata / Nashik / Pune / Bokaro / Dubai
NTSE-MENTAL ABILITY

5.

6.

7.

Direction: (8 to 12) In the following questions a pattern is given with a part missing which ¡s indicated by
a question mark. Find out from the alternative figures the part which would go in place of the question
mark to complete the pattern. (NTSE Stage-II, 2007)

8.

350 PACE IIT & MEDICAL: Mumbai / Delhi & NCR / Goa / Akola / Kolkata / Nashik / Pune / Bokaro / Dubai
NTSE-MENTAL ABILITY

9.

10.

11.

351 PACE IIT & MEDICAL: Mumbai / Delhi & NCR / Goa / Akola / Kolkata / Nashik / Pune / Bokaro / Dubai
NTSE-MENTAL ABILITY

12.

(NTSE Stage-II, 2008)

Direction : (13 to 14) In the following question find out which of the figure (A),(B),(C) and (D) can be
formed from the pieces given in figure (X).

13.

14.

352 PACE IIT & MEDICAL: Mumbai / Delhi & NCR / Goa / Akola / Kolkata / Nashik / Pune / Bokaro / Dubai
NTSE-MENTAL ABILITY

Directions: (15 to 16) Which of the following figures (given as alternatives) would be formed by joining
the four pieces I blocks of figures given below ?

15.

(NTSE Stage-II, 2007)

(A) (B)

(C) (D)

16.

(NTSE Stage-II, 2007)

17. There are four figures, three of which would form a complete square. Choose the three from among
the alternatives. (NTSE Stage-II, 2007)

(A) bcd (B) abc (C) acd (D) abd

353 PACE IIT & MEDICAL: Mumbai / Delhi & NCR / Goa / Akola / Kolkata / Nashik / Pune / Bokaro / Dubai
NTSE-MENTAL ABILITY

Directions: (18 to 22) In the following questions two figures are given as problem figures. Which of the
following alternative figures would be formed if the first figure is superimposed on the second figure?

18.

(NTSE Stage-II, 2007)

19.

(NTSE Stage-II, 2007)

20.

(NTSE Stage-I/Raj.2009)

354 PACE IIT & MEDICAL: Mumbai / Delhi & NCR / Goa / Akola / Kolkata / Nashik / Pune / Bokaro / Dubai
NTSE-MENTAL ABILITY

21.

(NTSE Stage-I/Raj.2009)

22.

(NTSE Stage-I/Raj.2009)

Directions : (23 to 24) In the following questions, a pattern is given with a part missing which is indicated
by a question mark. Find out from the alternative figures the part which would go in place of the
question mark to complete the pattern. (NTSE Stage-I/Raj.2009)

23.

355 PACE IIT & MEDICAL: Mumbai / Delhi & NCR / Goa / Akola / Kolkata / Nashik / Pune / Bokaro / Dubai
NTSE-MENTAL ABILITY

24.

Directions: (25 to 26) Which of the following figures (given as alternatives) would be formed by joining
the two/four pieces/blocks of figures given below ? (NTSE Stage-I/Raj.2009)

25.

(A) (B)

(C) (D)

26.

356 PACE IIT & MEDICAL: Mumbai / Delhi & NCR / Goa / Akola / Kolkata / Nashik / Pune / Bokaro / Dubai
NTSE-MENTAL ABILITY

Directions: (27 to 30) (In each of the following questions design/pattern is given. One small segment is
missing from the design. Select the alternative which will complete the design when placed on the missing
area. (NTSE Stage-II, 2009)

27.

28.

29.

357 PACE IIT & MEDICAL: Mumbai / Delhi & NCR / Goa / Akola / Kolkata / Nashik / Pune / Bokaro / Dubai
NTSE-MENTAL ABILITY

30.

31.

(NTSE Stage-II, 2011)

32.

(NTSE Stage-II, 2011)

358 PACE IIT & MEDICAL: Mumbai / Delhi & NCR / Goa / Akola / Kolkata / Nashik / Pune / Bokaro / Dubai
NTSE-MENTAL ABILITY

33.

(NTSE Stage-II, 2011)

34. Identify which among the pieces given below will not be required to complete the triangular pattern
shown below. (NTSE Stage-II,2013)

(A) q (B) r (C) s (D) t

35. A pattern is given below. You have to identify which among the following pieces will not be required to
complete the pattern. (NTSE Stage-II,2013)

(A) q (B) r (C) s (D) t

359 PACE IIT & MEDICAL: Mumbai / Delhi & NCR / Goa / Akola / Kolkata / Nashik / Pune / Bokaro / Dubai
NTSE-MENTAL ABILITY

36. In this question, among the four answer figures which one can be formed from the cut out pieces given
in question figure?
Question Figure

Answer Figures:

Answer Key

1. (d) 2. (b) 3. (a) 4. (d) 5. (a) 6. (c) 7. (a) 8. (c) 9. (c) 10. (d)
11. (d) 12. (a) 13. (b) 14. (c) 15. (d) 16. (a) 17. (b) 18. (b) 19. (a) 20. (b)

21. (a) 22. (b) 23. (a) 24. (c) 25. (b) 26. (c) 27. (a) 28. (d) 29. (c) 30. (c)

31. (c) 32. (b) 33. (c) 34. (a) 35. (b) 36. (b)

PREVIOUS YEARS QUESTIONS


Q.1 and 2  Directions: Select the correct alternatives which can complete the figures.

1. Question figure

(a) (b)

360 PACE IIT & MEDICAL: Mumbai / Delhi & NCR / Goa / Akola / Kolkata / Nashik / Pune / Bokaro / Dubai
NTSE-MENTAL ABILITY

(c) (d)

2. Question figure

(a) (b)

(c) (d)
[NTSE-MAHARASHTRA STAGE-1-2020]

Q.3 and 4: Direction be following question figure is incomplete. Select the correct alternative that will
complete the figure.

3. Question figure

(a) (b)

(c) (d)

361 PACE IIT & MEDICAL: Mumbai / Delhi & NCR / Goa / Akola / Kolkata / Nashik / Pune / Bokaro / Dubai
NTSE-MENTAL ABILITY

4. Question figure

(a) (b)

(c) (d)
[NTSE-MAHARASHTRA STAGE-1-2019]

Q.5 and 6: Direction: Complete the question figure by choosing the correct answer figure

5.

(a) (b)

(c) (d)

362 PACE IIT & MEDICAL: Mumbai / Delhi & NCR / Goa / Akola / Kolkata / Nashik / Pune / Bokaro / Dubai
NTSE-MENTAL ABILITY

6.

(a) (b)

(c) (d)
[NTSE-MAHARASHTRA STAGE-1-2018]

7. The following question figure given at left side is incomplete. Select the correct alternative which can
complete the figure.

(a) (b)

(c) (d)
[2017]

363 PACE IIT & MEDICAL: Mumbai / Delhi & NCR / Goa / Akola / Kolkata / Nashik / Pune / Bokaro / Dubai
NTSE-MENTAL ABILITY

8. In the following question the given figure is incomplete. Select the correct alternative which can
complete the figure.

(a) (b)

(c) (d) [2012]

Answer Key

1. (d) 2. (b) 3. (b) 4. (a) 5. (c) 6.(a) 7. (c) 8. (d)

364 PACE IIT & MEDICAL: Mumbai / Delhi & NCR / Goa / Akola / Kolkata / Nashik / Pune / Bokaro / Dubai
NTSE-MENTAL ABILITY

26. EMBEDDED FIGURE

A figure (X) is said to be embedded in a figure Y, if figure Y contains figure (X) as its part. Thus problem on
embedded figures contain a figure (X) followed by four complex figures in such a way that figure (X) is
embedded in one of these. The figure containing the figure (X) is your answer.

Directions: (1 to 5) In each of the following questions, choose the alternative figure in which the question
figure (X) is embedded.

1.

(a) (b) (c) (d)


Sol. (b) On close observation, we find that figure (X) is embedded in figure (b) as shown below:

2.

(a) (b) (c) (d)


Sol. (b) On close observation, we find that figure (X) is embedded in figure (b) as shown below:

365 PACE IIT & MEDICAL: Mumbai / Delhi & NCR / Goa / Akola / Kolkata / Nashik / Pune / Bokaro / Dubai
NTSE-MENTAL ABILITY

3.

(a) (b) (c) (d)


Sol. (d) On close observation, we find that figure (X) is embedded in figure (d) as shown below.

4.

(a) (b) (c) (d)


Sol. (d) On close observation, we find that figure (X) is embedded in figure (d) as shown below.

5.

(a) (b) (c) (d)


Sol. (c) On close observation, we find that figure (X) is embedded in figure (c) as shown below.

366 PACE IIT & MEDICAL: Mumbai / Delhi & NCR / Goa / Akola / Kolkata / Nashik / Pune / Bokaro / Dubai
NTSE-MENTAL ABILITY

EXERCISE

Directions : (1 to 9) In each of the following questions, choose the alternative figure in which the question
figure (X) is embedded.

1.

2.

3.

367 PACE IIT & MEDICAL: Mumbai / Delhi & NCR / Goa / Akola / Kolkata / Nashik / Pune / Bokaro / Dubai
NTSE-MENTAL ABILITY

4.

(NTSE Stage-II, 2007)

5.

(NTSE Stage-II, 2007)


6.

(NTSE Stage-II, 2008)

368 PACE IIT & MEDICAL: Mumbai / Delhi & NCR / Goa / Akola / Kolkata / Nashik / Pune / Bokaro / Dubai
NTSE-MENTAL ABILITY

7.

(NTSE Stage-II, 2008)


8.

(NTSE Stage-II, 2008)

9.

(NTSE Stage-II, 2008)

369 PACE IIT & MEDICAL: Mumbai / Delhi & NCR / Goa / Akola / Kolkata / Nashik / Pune / Bokaro / Dubai
NTSE-MENTAL ABILITY

Directions: (10 to 12) In the following questions, a problem figure is given. The problem figure is hidden
in one of the figures given as alternatives. Find the figure in which the problem figure is hidden.
(NTSE Stage-I/Raj./2009)
10.

11.

12.

370 PACE IIT & MEDICAL: Mumbai / Delhi & NCR / Goa / Akola / Kolkata / Nashik / Pune / Bokaro / Dubai
NTSE-MENTAL ABILITY

Answer Key

1. (a) 2. (d) 3. (d) 4. (c) 5. (d) 6. (b) 7. (c) 8. (c) 9. (b) 10. (d)
11. (a) 12. (b)

PREVIOUS YEARS QUESTIONS

Q.1 and 2 Directions: In the given questions a complex figure is given. Find out which of the sample
figures given in the alternatives is hidden in the complex figure

1.

(a) (b)

(c) (d)

2.

(a) (b)

(c) (d)
[NTSE-MAHARASHTRA STAGE-1-2020]

371 PACE IIT & MEDICAL: Mumbai / Delhi & NCR / Goa / Akola / Kolkata / Nashik / Pune / Bokaro / Dubai
NTSE-MENTAL ABILITY

Q.3 and 4: Direction In the given question a complex figure is given. Find out which of the figure given in
the alternatives is hidden in the complex figure.

3.

(a) (b)

(c) (d)

4.

(a) (b)

(c) (d)

[NTSE-MAHARASHTRA STAGE-1-2019]

372 PACE IIT & MEDICAL: Mumbai / Delhi & NCR / Goa / Akola / Kolkata / Nashik / Pune / Bokaro / Dubai
NTSE-MENTAL ABILITY

Q.5 and 6: Direction : In the given question figure complex figure is given. Find out which of the figure
given in the alternatives is hidden in the complex figure?

5.

(a) (b)

(c) (d)

6.

(a) (b)

(c) (d)

[NTSE-MAHARASHTRA STAGE-1-2018]

373 PACE IIT & MEDICAL: Mumbai / Delhi & NCR / Goa / Akola / Kolkata / Nashik / Pune / Bokaro / Dubai
NTSE-MENTAL ABILITY

7. In the figure question a complex figure is given. Find out which of the simple figure given in the
alternatives is hidden in the complex figure?

(a) (b)

(c) (d)

Answer Key

2.
1. (b) 3. (a) 4. (b and d) 5. (c) 6.(d) 7. (b)
(Bonus)

374 PACE IIT & MEDICAL: Mumbai / Delhi & NCR / Goa / Akola / Kolkata / Nashik / Pune / Bokaro / Dubai
NTSE-MENTAL ABILITY

27. NON-VERBAL SERIES

Type-I
In this type of non-verbal test, two sets of figures pose the problem. The sets are called problem Figures and
Answer Figures. Each problem figure changes in design from the preceding one. The answer figure set contains
four figures marked A, B, C, D. You are required to choose the correct answer figure, which would best
continue the series.

Directions : (1 to 10) Study the problem figures carefully and try to establish the relationship between
them from the answer figures. Pick out the figure which most appropriately completes the series.

1. Problem Figures

Answer Figures

(a) (b) (c) (d)


Sol. (d) Note the direction of arrow which changes alternately. The dots are also changing alternately.
Hence, we are looking for a figure in which the arrow points down and the dots and positioned
as problem figure second.

2. Problem Figures

Answer Figures

(a) (b) (c) (d)


Sol. (b) The four boxes are changing position in the following way: At first, middle boxes change
position (diagonally) and extreme boxes remain stationary, then extreme boxes change position
and middle boxes remain stationary and so on.

3. Problem Figures

Answer Figures

(a) (b) (c) (d)


Sol. (c) The same figures rotates up-side-down in alternative figures.

375 PACE IIT & MEDICAL: Mumbai / Delhi & NCR / Goa / Akola / Kolkata / Nashik / Pune / Bokaro / Dubai
NTSE-MENTAL ABILITY

4. Problem Figures

Answer Figures

(a) (b) (c) (d)


Sol. The figures is rotated at 90o (in four directions) and the fifth figure in the series shall be same as
the first figure.

5. Problem Figures

Answer Figures

(a) (b) (c) (d)


Sol. (a) The bigger balls diameter is moving at 90o and smaller balls diameter is moving at 45o,
simultaneously the face of the bigger figure is changing position.

Type-II
The there questions the items in the diagrams either increase or decrease in number.

6. Problem Figures

Answer Figures

(a) (b) (c) (d)


Sol. (c) The small circles are decreasing consecutively and the black dots are increasing.

7. Problem Figures

Answer Figures

(a) (b) (c) (d)

376 PACE IIT & MEDICAL: Mumbai / Delhi & NCR / Goa / Akola / Kolkata / Nashik / Pune / Bokaro / Dubai
NTSE-MENTAL ABILITY

Sol. (d) Signs of Plus are adding up one by one. Problem figure one has one plus sign, problem figure
second has two signs, problem figure third has three signs, the next figure should have four signs to
keep the same pattern.

8. Problem Figures

Answer Figures

(a) (b) (c) (d)


Sol. (d) Here a decreasing trend is followed. In the first figure there are 8 lines cutting through the sides
of the sphere. Second figure has 7 lines. The third figure has 6 lines. To continue the series, fourth
figure should have 5 lines.

Type-III
The qualitative characteristics of various elements in the diagrams change to complete the series.

Rotation Type: The various elements in the diagrams move in a specific manner. They may rotate in clockwise
or anti-clockwise direction.

9. Problem Figures

Answer Figures

(a) (b) (c) (d)


Sol. (d) The sign of plus is rotating clockwise. The pin changes direction alternately.

Type-IV
Multi-Relational Series: These are mixed series in which various elements in diagrams increase or decrease in
number, change/positions in a set pattern.

10. Problem Figures

Answer Figures

(a) (b) (c) (d)


Sol. (c) Note movement of dot which is clockwise and the arrow moves in and out in opposite direction
alternately, the circle and square interchange
377 PACE IIT & MEDICAL: Mumbai / Delhi & NCR / Goa / Akola / Kolkata / Nashik / Pune / Bokaro / Dubai
NTSE-MENTAL ABILITY

EXERCISE

Directions : (1 to 11) Each of the following questions consists of five problem figure. These problem
figures form a series. Find out the one figure from the answer figures that will continue the series.

1. Problem Figures

Answer Figures

2. Problem Figures

Answer Figures

3. Problem Figures

Answer Figures

378 PACE IIT & MEDICAL: Mumbai / Delhi & NCR / Goa / Akola / Kolkata / Nashik / Pune / Bokaro / Dubai
NTSE-MENTAL ABILITY

4. Problem Figures

Answer Figures

5. Problem Figures

Answer Figures

6. Problem Figures

Answer Figures

7. Problem Figures

Answer Figures

379 PACE IIT & MEDICAL: Mumbai / Delhi & NCR / Goa / Akola / Kolkata / Nashik / Pune / Bokaro / Dubai
NTSE-MENTAL ABILITY

8. Problem Figures

Answer Figures

9. Problem Figures

Answer Figures

10. Problem Figures

Answer Figures

11. Problem Figures

Answer Figures

380 PACE IIT & MEDICAL: Mumbai / Delhi & NCR / Goa / Akola / Kolkata / Nashik / Pune / Bokaro / Dubai
NTSE-MENTAL ABILITY

Direction (12 to 15) Each of the following questions consists of four problem figure. These problem
figures form a series. Find out the one figure from the answer figures that will continue the series.

12. Problem Figures

Answer Figures

13. Problem Figures

Answer Figures

14. Problem Figures

Answer Figures

15. Problem Figures

Answer Figures

381 PACE IIT & MEDICAL: Mumbai / Delhi & NCR / Goa / Akola / Kolkata / Nashik / Pune / Bokaro / Dubai
NTSE-MENTAL ABILITY

Directions : (16 to 20) In each of the following questions, there is a set of four figures called problem
figures followed by a set of four other figures called answer figures. Problem figures contain a question
mark. Select a suitable figure from the answer figures which will substitute this question mark so that a
series is formed.

16. Problem Figures

Answer Figures

17. Problem Figures

Answer Figures

18. Problem Figures

Answer Figures

19. Problem Figures

Answer Figures

382 PACE IIT & MEDICAL: Mumbai / Delhi & NCR / Goa / Akola / Kolkata / Nashik / Pune / Bokaro / Dubai
NTSE-MENTAL ABILITY

20. Problem Figures

Answer Figures

Directions : (21 to 26) In each of the following questions there are two sets of figures, one set contents
problem figures, another set contents answer figures. There is a sequence according to which the problem
figures are arranged, select one figure from, the set of answer figures which can be placed in sequence
after the set of problem figures. (NTSE Stage-I/Raj./ 2007)

21. Problem Figures

Answer Figures

22. Problem Figures

Answer Figures

23. Problem Figures

Answer Figures

383 PACE IIT & MEDICAL: Mumbai / Delhi & NCR / Goa / Akola / Kolkata / Nashik / Pune / Bokaro / Dubai
NTSE-MENTAL ABILITY

24. Problem Figures

Answer Figures

25. Problem Figures

Answer Figures

26. Problem Figures

Answer Figures

Directions : (27 to 31) In the following questions, some figures are given in a sequence Find out the figure
from the alternatives which will come In place of the question mark to continue the sequence.
(NTSE Stage-II, 2007)
27.

384 PACE IIT & MEDICAL: Mumbai / Delhi & NCR / Goa / Akola / Kolkata / Nashik / Pune / Bokaro / Dubai
NTSE-MENTAL ABILITY

28.

29.

30.

31.

385 PACE IIT & MEDICAL: Mumbai / Delhi & NCR / Goa / Akola / Kolkata / Nashik / Pune / Bokaro / Dubai
NTSE-MENTAL ABILITY

Directions : (32 to 36) In the foiloWifl9 questions there are two sets of the figures. One set is of problem
figures and another set is of answer-figures. Problem figures are arranged in a sequence. One figure from
the answer figures is to be selected such that it can be placed after the series of problem figures.
(NTSE Stage-I/Raj. 2008)
32. Problem Figures

Answer Figures

33. Problem Figures

Answer Figures

34. Problem Figures

Answer Figures

35. Problem Figures

Answer Figures

386 PACE IIT & MEDICAL: Mumbai / Delhi & NCR / Goa / Akola / Kolkata / Nashik / Pune / Bokaro / Dubai
NTSE-MENTAL ABILITY

36. Problem Figures

Answer Figures

Directions : (37 to 38) Study the pattern of figures given in each question to find out the relationship
among them. One figure is missing. Choose the missing: figure from the alternatives.
(NTSE Stage-II, 2008)

37.

38.

387 PACE IIT & MEDICAL: Mumbai / Delhi & NCR / Goa / Akola / Kolkata / Nashik / Pune / Bokaro / Dubai
NTSE-MENTAL ABILITY

Directions: (39 to 40) Om the following questions, series of figures are given. Find the correct alternative
to continue the series. (NTSE Stage-II, 2008)

39.

40.

Directions : (41 to 45) There are two sets of figures. One is of Problem Figures while another is set of
Answer Figures. There is a sequence according to which the Problem Figures are arranged. One figure of
Answer Figures is to be selected which can be placed in sequence of Problem Figures. Find the correct
option of the selected Answer Figure. (NTSE Stage-I/Raj./2009)

41. Problem Figures

Answer Figures

42. Problem Figures

Answer Figures

388 PACE IIT & MEDICAL: Mumbai / Delhi & NCR / Goa / Akola / Kolkata / Nashik / Pune / Bokaro / Dubai
NTSE-MENTAL ABILITY

43. Problem Figures

Answer Figures

44. Problem Figures

Answer Figures

45. Problem Figures

Answer Figures

Directions: (46 to 49) The figures in each of the questions follow a series. Select the figure form the given
alternatives which would continue the series. (NTSE Stage-II, 2009)

46.

389 PACE IIT & MEDICAL: Mumbai / Delhi & NCR / Goa / Akola / Kolkata / Nashik / Pune / Bokaro / Dubai
NTSE-MENTAL ABILITY

47.

48.

49.

50. Write the correct choice which would be required to continue with the series. (NTSE Stage-II, 2009)

390 PACE IIT & MEDICAL: Mumbai / Delhi & NCR / Goa / Akola / Kolkata / Nashik / Pune / Bokaro / Dubai
NTSE-MENTAL ABILITY

Directions: (51 to 55) The figures in each of the questions follow a series. Select the figure form the given
alternatives which would continue the series. (NTSE Stage-I/Raj./ 2012)

51. Question Figures

Answer Figures

52. Question Figures (NTSE Stage-I/Raj./ 2012)

Answer Figures

53. Question Figures (NTSE Stage-I/Raj./ 2012)

Answer Figures

391 PACE IIT & MEDICAL: Mumbai / Delhi & NCR / Goa / Akola / Kolkata / Nashik / Pune / Bokaro / Dubai
NTSE-MENTAL ABILITY

54. Question Figures (NTSE Stage-I/Raj./ 2012)

Answer Figures

55. Question Figures (NTSE Stage-I/Raj./ 2012)

Answer Figures

Directions: (56 to 59) Two sets of the figures are given. One set of Question-figures and another set is of
Answer-figures. Question-figures are arranged in a sequence. One figure from the Answer figures is to be
selected such that it can be placed after the series of Question-figures. Find the correct serial number of
the selected Answer figure. (NTSE Stage-I/Raj./ 2013)

56. Question Figures

Answer Figures

392 PACE IIT & MEDICAL: Mumbai / Delhi & NCR / Goa / Akola / Kolkata / Nashik / Pune / Bokaro / Dubai
NTSE-MENTAL ABILITY

57. Question Figures

Answer Figures

58. Question Figures

Answer Figures

59. Question Figures

Answer Figures

Direction (60 to 64) : There are two sets of figures. One set contains problem-figures while the other has
answer-figures. There is a sequence according to which the problem-figures are arranged. You have to
select one figure from the set of answer-figures which can be placed in sequence after the set of problem-
figures. Find out the correct figure. (NTSE Stage-I/Raj./ 2014)
60. Problem Figures

Answer Figures

393 PACE IIT & MEDICAL: Mumbai / Delhi & NCR / Goa / Akola / Kolkata / Nashik / Pune / Bokaro / Dubai
NTSE-MENTAL ABILITY

61. Problem Figures

Answer Figures

62. Problem Figures

Answer Figures

63. Problem Figures

Answer Figures

64. Problem Figures

Answer Figures

394 PACE IIT & MEDICAL: Mumbai / Delhi & NCR / Goa / Akola / Kolkata / Nashik / Pune / Bokaro / Dubai
NTSE-MENTAL ABILITY

Direction : In questions 65 to 66 there are two sets of figures. One set contains problem - figures while
the other has answer-figures. There is a sequence according to which the problem –figures are arranged.
You have to select an answer-figure which can be added in sequence with the problem - figures. Choose
the correct figure. (NTSE Stage-I/Raj./ 2017)

65. Problem Figures

Answer Figures

66. Problem Figures

Answer Figures

Answer Key

1. (d) 2. (a) 3. (d) 4. (d) 5. (d) 6. (d) 7. (d) 8. (d) 9. (d) 10. (d)
11. (d) 12. (c) 13. (d) 14. (d) 15. (b) 16. (a) 17. (b) 18. (c) 19. (b) 20. (d)

21. (d) 22. (a) 23. (c) 24. (c) 25. (b) 26. (c) 27. (b) 28. (d) 29. (d) 30. (d)

31. (a) 32. (d) 33. (b) 34. (d) 35. (a) 36. (d) 37. (b) 38. (c) 39. (b) 40. (c)

41. (c) 42. (d) 43. (a) 44. (b) 45. (a) 46. (a) 47. (c) 48. (c) 49. (b) 50. (c)

51. (a) 52. (b) 53. (c) 54. (d) 55. (c) 56. (b) 57. (a) 58. (d) 59. (b) 60. (a)

61. (b) 62. (b) 63. (c) 64. (c) 65. (d) 66. (a)

395 PACE IIT & MEDICAL: Mumbai / Delhi & NCR / Goa / Akola / Kolkata / Nashik / Pune / Bokaro / Dubai
NTSE-MENTAL ABILITY

PREVIOUS YEARS QUESTIONS


Q.1 and 2: Directions: In each of the following, the question figures change in a particular order. Decide
which figure from the given alternatives will replace the question mark.

1. Question Figures

Answer figures

(a) (b)

(c) (d)

2. Question figures

Answer figures

(a) (b)

(c) (d)
[NTSE MAHARASHTRA Stage-1-2020]

Q.3 and 4: Directions : Which symbols will come in the order. Choose the correct alternative.

3. , , , ?


(a)  (b) 
(c)  (d) 

4.   ,  , 
(a) n (b)  
(c)  (d) 
NTSE MAHARASHTRA Stage-1-2020

396 PACE IIT & MEDICAL: Mumbai / Delhi & NCR / Goa / Akola / Kolkata / Nashik / Pune / Bokaro / Dubai
NTSE-MENTAL ABILITY

Q.5 and 6: Direction In the following sequence. Choose the correct term that will replace the question
mark.

. . .
5.
(a) (b)
(c) (d)

6. , , ,  , ?


(a)  (b) 
(c)  (d) 
[NTSE MAHARASHTRA Stage-1-2019]

Q.7 and 8: Direction: In each of the following question the question figures are given in specific order.
Select the correct alternative from the answer figure that will replace the question mark

7. Question Figure

Answer Figure

(a) (b)

(c) (d)

8. Question figure

Answer key

(a) (b)

(c) (d)
[NTSE MAHARASHTRA Stage-1-2019]

397 PACE IIT & MEDICAL: Mumbai / Delhi & NCR / Goa / Akola / Kolkata / Nashik / Pune / Bokaro / Dubai
NTSE-MENTAL ABILITY

Q.9 and 10: Direction: In each of the following questions the question figure change in a particular order.
Decide which figure from the given alternatives will replace the question mark.

9. Question figure


Answer Figure

(a) (b)

(c) (d)

10. Question figure

Answer Figure

(a) (b)

(c) (d)

Q.11 and 12: Direction: In each of the following questions write which symbol in the sequence will
replace the question mark.

11.   ,  ,   , ?


(a)  (b) 
(c)   (d)   

12.  nf ,   ,   nf , ?


(a) fn  (b) nf 
(c) f  n (d) n  f
[NTSE MAHARASHTRA Stage-1-2018]

398 PACE IIT & MEDICAL: Mumbai / Delhi & NCR / Goa / Akola / Kolkata / Nashik / Pune / Bokaro / Dubai
NTSE-MENTAL ABILITY

Q.13 and 14: Directions- In each of the following, the question figures change in a particular order.
Decide which figure from the given alternative will replace the question mark.

13.

(a) (b)

(c) (d)

14.

(a) (b)

(c) (d)
[2017]

Q.15 and 26  Directions: - In each of the following, question figures change in a particular order. Find
out the correct figure from the alternative which will replace the question mark?

15.

(a) (b)

(c) (d)

399 PACE IIT & MEDICAL: Mumbai / Delhi & NCR / Goa / Akola / Kolkata / Nashik / Pune / Bokaro / Dubai
NTSE-MENTAL ABILITY

16.

(a) (b)

(c) (d) [2016]

Q.17 and 18  Directions:- In each of the following, question figures change in a particular order. Find
out the correct figure from the alternatives which will replace the question mark.

17. Question Figure

Answer Figures

(a) (b)

(c) (d)

18.

Answer Figures

(a) (b)

(c) (d) [2015]

400 PACE IIT & MEDICAL: Mumbai / Delhi & NCR / Goa / Akola / Kolkata / Nashik / Pune / Bokaro / Dubai
NTSE-MENTAL ABILITY

Q.19 and 20  Directions:- In each of the following question figures change in a particular order. Find
out the correct figure from the alternatives which will replace the question mark (?)

19.

(a) (b)

(c) (d)

20.

(a) (b)

(c) (d) [2014]

Answer Key

5. 6. 7.
1. (b) 2. (d) 3. (d) 4. (a) 8. (c) 9. (d) 10. (a)
(Bonus) (Bonus) (Bonus)
11. (b) 12. (c) 13. (c) 14. (b) 15. (d) 16. (2) 17. (c) 18. (a) 19.(b) 20.(d)

401 PACE IIT & MEDICAL: Mumbai / Delhi & NCR / Goa / Akola / Kolkata / Nashik / Pune / Bokaro / Dubai
NTSE-MENTAL ABILITY

28. NON-VERBAL ANALOGY

Analogy
In this type of non-verbal test the problem figures are presented in two units. The first unit contains two figures,
one in each square. The second unit contains one figure in the first square and a question mark (?) in the second.
You have to find out from among the figures A, B, C and D as to which one should replace the questions mark
after finding the relationship between the two figures in the first unit of the problem figures. Some examples are
discussed below.

Directions : (1 to 3) In the following questions there are two sets of figures. One set is problem figures and
the second set is a answer figures. There is some relationship between the first and the second figure of
the problem figures set. If there is similar relationship between the third and fourth figures of the same
set, select the correct figure from the set of answer figures for question mark (?).

1. Problem Figures

Answer Figures

(a) (b) (c) (d)


Sol. (c) Problem figure second contains the lower half of problem figure first. Hence, answer figure (c)
replaces the question mark.

2. Problem Figures

Answer Figures

(a) (b) (c) (d)


Sol. (d) The lower LHS figure of portion in problem figure first becomes the upper portion in problem
figure second, shifted to the other side. Similarly RHS figure of the upper portion in problem figure
first becomes the lower portion problem figure second shifted to the other side with one vertical line
therein. The other two halves are deleted.

3. Problem Figures

Answer Figures

(a) (b) (c) (d)


402 PACE IIT & MEDICAL: Mumbai / Delhi & NCR / Goa / Akola / Kolkata / Nashik / Pune / Bokaro / Dubai
NTSE-MENTAL ABILITY

Sol. (a) problem figure first is rotated 180o ACW or CW to obtain problem figure second. Then the
shaded and the unshaded leaves are interchanged. Hence, (a) should replace the question mark.

Directions : (4 to 5) In each of the following problem, a related pair of figures is followed by five
numbered pairs of figures. Select the pair that has a relationship similar to that in the original pair.
4.

(a) (b) (c) (d)


Sol. (a) The first figure of (X) rotate 180o.

5.

(a) (b) (c) (d)


Sol. (b) From figure first to figure second : An element identical to the element in figure first
forms the inner element in figure second. The vertically inverted and enlarged form of the
element in figure first forms the outer element in figure second.

EXERCISE
Directions: (1 to 11) In the following questions there are two sets of figures. One set is problem figures
and the second set is a answer figures. There is some relationship between the first and the second figure
of the problem figures set. If there is similar relationship between the third and fourth figures of the same
set, set the correct figure from the set of answer figures for questions mark

1. Problem Figures

Answer Figures

403 PACE IIT & MEDICAL: Mumbai / Delhi & NCR / Goa / Akola / Kolkata / Nashik / Pune / Bokaro / Dubai
NTSE-MENTAL ABILITY

2. Problem Figures

Answer Figures

3. Problem Figures

Answer Figures

CT TC CT TC

(A) (B) (C) (D)

4. Problem Figures

Answer Figures

(A) (B) (C) (D)

5. Problem Figures

Answer Figures

6. Problem Figures

Answer Figures

404 PACE IIT & MEDICAL: Mumbai / Delhi & NCR / Goa / Akola / Kolkata / Nashik / Pune / Bokaro / Dubai
NTSE-MENTAL ABILITY

7. Problem Figures

Answer Figures

8. Problem Figures

Answer Figures

(A) (B) (C) (D)

9. Problem Figures

Answer Figures

(A) (B) (C) (D)

10. Problem Figures

Answer Figures

11. Problem Figures

Answer Figures

405 PACE IIT & MEDICAL: Mumbai / Delhi & NCR / Goa / Akola / Kolkata / Nashik / Pune / Bokaro / Dubai
NTSE-MENTAL ABILITY

Directions: (12 to 14) In each of the following problems, a related pair of figures is followed by five
numbered pairs of figures. Select the pair that has a relationship similar to that in the original pair.

12.

13.

14.

Directions: (15 to 20) In the following questions there are two sets of figures. One set is problem figure
and the second set is a answer figures. There is some relationship between the first and the second figures
of the problem figures set. If there is similar relationship between the third and fourth figures of the same
set, select the correct figure from the set of answer figures for question mark (?)
(NTSE Stage-I/Raj./2007)

15. Problem Figures

Answer Figures

406 PACE IIT & MEDICAL: Mumbai / Delhi & NCR / Goa / Akola / Kolkata / Nashik / Pune / Bokaro / Dubai
NTSE-MENTAL ABILITY

16. Problem Figures

Answer Figures

17. Problem Figures

Answer Figures

18. Problem Figures

Answer Figures

19. Problem Figures

Answer Figures

20. Problem Figures

Answer Figures

407 PACE IIT & MEDICAL: Mumbai / Delhi & NCR / Goa / Akola / Kolkata / Nashik / Pune / Bokaro / Dubai
NTSE-MENTAL ABILITY

Directions: (21 to 25) In the following questions, there is a relationship between the two figures on the left
of the sign (: :). The same relationship exists between the figures to the right of the sign (: :), of which one
is missing. Find the missing one from the alternatives. (NTSE Stage-II, 2007)

21.

22.

23.

408 PACE IIT & MEDICAL: Mumbai / Delhi & NCR / Goa / Akola / Kolkata / Nashik / Pune / Bokaro / Dubai
NTSE-MENTAL ABILITY

24.

25.

Directions: (26 to 30) In the following questions there are two sets of figures. One set is problem figure
and the second set is a answer figures. There is some relationship between the first and the second figures
of the problem figures set. If there is similar relationship between the third and fourth figures of the same
set, select the correct figure from the set of answer figures for question mark (?)
(NTSE Stage-I/Raj./2008)

26. Problem Figures

Answer Figures

27. Problem Figures

Answer Figures

409 PACE IIT & MEDICAL: Mumbai / Delhi & NCR / Goa / Akola / Kolkata / Nashik / Pune / Bokaro / Dubai
NTSE-MENTAL ABILITY

28. Problem Figures

Answer Figures

29. Problem Figures

Answer Figures

30. Problem Figures

Answer Figures

Directions: (31 to 33) There is some relationship between the two figures/pair of letters/numbers on the
left of the sign (: :). The same relationship exists between the two terms on the right of which one is
missing. Find the missing one from the given alternatives. (NTSE Stage-II, 2008)

31.

410 PACE IIT & MEDICAL: Mumbai / Delhi & NCR / Goa / Akola / Kolkata / Nashik / Pune / Bokaro / Dubai
NTSE-MENTAL ABILITY

32.

33.

Directions: (34 to 35) A pair of figures which are related in some way is given below. Find out the
alternative whose figures are related in the same way as the original pair of figures.
(NTSE Stage-II, 2008)

34.

411 PACE IIT & MEDICAL: Mumbai / Delhi & NCR / Goa / Akola / Kolkata / Nashik / Pune / Bokaro / Dubai
NTSE-MENTAL ABILITY

35.

Directions: (36 to 42) There is some relationship between the two figures on the left of the sign (: :). The
same relationship exists between the two terms on the right of which one is missing. Find the missing one
from the given alternatives. (NTSE Stage-I/Raj./2009)

36.

37.

38. Questions Figures

Answer Figures

412 PACE IIT & MEDICAL: Mumbai / Delhi & NCR / Goa / Akola / Kolkata / Nashik / Pune / Bokaro / Dubai
NTSE-MENTAL ABILITY

39. Questions Figures

Answer Figures

40. Questions Figures

Answer Figures

41. Questions Figures

Answer Figures

42. Questions Figures

Answer Figures

Directions: (43 to 46) In each of the following questions, there is a relationship between the two figure on
the left of the sign (: :). The same relationship exists between the two term on the right of the sign
(: :) of which one is missing. Find the missing one from the given alternatives. (NTSE Stage-II, 2009)

43.

413 PACE IIT & MEDICAL: Mumbai / Delhi & NCR / Goa / Akola / Kolkata / Nashik / Pune / Bokaro / Dubai
NTSE-MENTAL ABILITY

44.

45.

46.

Directions: (47 to 54) In each of the following questions, there is a relationship between the two figure on
the left of the sign (: :). The same relationship exists between the two term on the right of the sign
(: :) of which one is missing. Find the missing one from the given alternatives.

47. Problem Figures

Answer Figures

48. Problem Figures

Answer Figures

414 PACE IIT & MEDICAL: Mumbai / Delhi & NCR / Goa / Akola / Kolkata / Nashik / Pune / Bokaro / Dubai
NTSE-MENTAL ABILITY

49.

(NTSE Stage-II, 2011)

50.

(NTSE Stage-II, 2011)

51.

(NTSE Stage-II, 2011)

415 PACE IIT & MEDICAL: Mumbai / Delhi & NCR / Goa / Akola / Kolkata / Nashik / Pune / Bokaro / Dubai
NTSE-MENTAL ABILITY

52.

(NTSE Stage-II, 2011)

53.

(NTSE Stage-II, 2011)

416 PACE IIT & MEDICAL: Mumbai / Delhi & NCR / Goa / Akola / Kolkata / Nashik / Pune / Bokaro / Dubai
NTSE-MENTAL ABILITY

54.

(NTSE Stage-II, 2011)

Directions: (55 to 59) In the following questions there is some relationship between the two figure on the
left of (: :) the same relationship exists between the two terms on the right, of which one is missing. Find
the missing one from the given alternative. (NTSE Stage-I/Raj./2012)

55. Question Figures

Answer Figures

56. Question Figures

Answer Figures

417 PACE IIT & MEDICAL: Mumbai / Delhi & NCR / Goa / Akola / Kolkata / Nashik / Pune / Bokaro / Dubai
NTSE-MENTAL ABILITY

57. Question Figures

Answer Figures

58. Question Figures

Answer Figures

59. Question Figures

Answer Figures

418 PACE IIT & MEDICAL: Mumbai / Delhi & NCR / Goa / Akola / Kolkata / Nashik / Pune / Bokaro / Dubai
NTSE-MENTAL ABILITY

Directions: (60 to 63) In the following questions there is some relationship between the two figures on the
left of (: :) the same relationship exists between the two terms on the right, of which one is missing. Find
the missing one from the given alternative. (NTSE Stage-I/Raj./2013)

60. Question Figures

Answer Figures

61. Question Figures

Answer Figures

62. Question Figures

Answer Figures

63. Question Figures

Answer Figures

419 PACE IIT & MEDICAL: Mumbai / Delhi & NCR / Goa / Akola / Kolkata / Nashik / Pune / Bokaro / Dubai
NTSE-MENTAL ABILITY

64.

(NTSE Stage-II, 2013)

65.

(NTSE Stage-II, 2013)

420 PACE IIT & MEDICAL: Mumbai / Delhi & NCR / Goa / Akola / Kolkata / Nashik / Pune / Bokaro / Dubai
NTSE-MENTAL ABILITY

66.

(NTSE Stage-I, Karnataka 2014)

Answer Key

1. (c) 2. (b) 3. (c) 4. (c) 5. (b) 6. (b) 7. (d) 8. (d) 9. (d) 10. (a)
11. (a) 12. (d) 13. (a) 14. (c) 15. (b) 16. (d) 17. (b) 18. (c) 19. (b) 20. (b)

21. (b) 22. (c) 23. (a) 24. (c) 25. (d) 26. (a) 27. (a) 28. (b) 29. (b) 30. (d)

31. (a) 32. (d) 33. (b) 34. (c) 35. (c) 36. (c) 37. (a) 38. (b) 39. (a) 40. (a)

41. (d) 42. (b) 43. (a) 44. (c) 45. (d) 46. (b) 47. (c) 48. (c) 49. (a) 50. (b)

51. (d) 52. (c) 53. (b) 54. (b) 55. (a) 56. (b) 57. (a) 58. (c) 59. (d) 60. (d)

61. (a) 62. (b) 63. (c) 64. (c) 65. (c) 66. (c)

421 PACE IIT & MEDICAL: Mumbai / Delhi & NCR / Goa / Akola / Kolkata / Nashik / Pune / Bokaro / Dubai
NTSE-MENTAL ABILITY

PREVIOUS YEARS QUESTIONS

Q1 to 3-Directions: In each of the following questions there is a specific relationship between the first and
second figure. The same relationship exists between the third and fourth figure which will replace the
question mark. Select the correct option from the given alternatives
(NTSE MAHARASHTRA Stage-1-2020)
1. Question figures

Answer Figures

2. Question figures

Answer Figures

3. Question figures

Answer Figures

422 PACE IIT & MEDICAL: Mumbai / Delhi & NCR / Goa / Akola / Kolkata / Nashik / Pune / Bokaro / Dubai
NTSE-MENTAL ABILITY

Q.4 to 6-Direction: In each of the following questions, there is a specific relationship between the first and
the second figure. The same relationship exists between the third and the fourth figure. Find the relation
and choose the correct answer to replace the question mark. (NTSE MAHARASHTRA Stage-1-2019)

4. Question figures

Answer Figures

5. Question figures

Answer Figures

6. Question figures

Answer Figures

423 PACE IIT & MEDICAL: Mumbai / Delhi & NCR / Goa / Akola / Kolkata / Nashik / Pune / Bokaro / Dubai
NTSE-MENTAL ABILITY

Q. 7 to 9-Direction : In the following questions there is a specific relation between the first and the second
figure. The same relationship exists between the third and the fourth figure which will replace the
question mark. Select the correct figure from the alternatives given below each question.
(NTSE MAHARASHTRA Stage-1-2018)
7.

8.

9.

Q. 10 and 11- Directions: In each of the following questions there is a specific relationship between the
first and second figure. The same relationship exists between the third and the fourth figure which will
replace the question mark. Select the correct term from the given alternatives.
(NTSE MAHARASHTRA Stage-1-2017)
10.

424 PACE IIT & MEDICAL: Mumbai / Delhi & NCR / Goa / Akola / Kolkata / Nashik / Pune / Bokaro / Dubai
NTSE-MENTAL ABILITY

11.

Q. 12 and 13-Directions : In the following questions there is a specific relationship between the first and
second figure. The same relationship exists between the third and fourth figure which will replace the
question mark. Select the correct term from the alter natives given.
(NTSE MAHARASHTRA Stage-1-2016)
12.

13.

Q.14 and 15. Directions : In the following questions there is a specific relationship between the first and
the second term. The same relationship exists between the third and fourth term, which will replace the
question mark (?). Select the correct term from the alternatives given.
(NTSE MAHARASHTRA Stage-1-2015)

14.

425 PACE IIT & MEDICAL: Mumbai / Delhi & NCR / Goa / Akola / Kolkata / Nashik / Pune / Bokaro / Dubai
NTSE-MENTAL ABILITY

15.

Q. 16 and 17- Directions :- In each of the following questions there k a specific relationship between the
first and second figure. The same relationship exists between the third and fourth figure which will
replace the question mark (?). Select the correct figure from the alternatives given.
(NTSE MAHARASHTRA Stage-1-2014)
16.

17.

(Q. 18 to 21.)-Directions : In each of the following questions there is a specific relationship between the
first and second figure. The same relationship exists between the third and fourth figure which will
replace the question mark. Select the correct term from the alternatives given.
(NTSE MAHARASHTRA Stage-1-2012)
18.

426 PACE IIT & MEDICAL: Mumbai / Delhi & NCR / Goa / Akola / Kolkata / Nashik / Pune / Bokaro / Dubai
NTSE-MENTAL ABILITY

19.

20.

21.

Answer Key

1. (b) 2. (d) 3. (b) 4. (c) 5. (a, b, c) 6. (c) 7. (c) 8. (d) 9. (b) 10. (d)
11. (a) 12. (d) 13. (c) 14. (a) 15. (b) 16. (c) 17. (a) 18. (d) 19. (b) 20. (d)

21. (a)

427 PACE IIT & MEDICAL: Mumbai / Delhi & NCR / Goa / Akola / Kolkata / Nashik / Pune / Bokaro / Dubai
NTSE-MENTAL ABILITY

29. NON-VERBAL CLASSIFICATION

Classification
In these type of questions, four figures numbered (a), (b), (c) and (d) are given. These are treated both problem
figures as well as the answer figures. Three out of these four figures are related to each other by way of having
some common characteristics and so form a group. Out of these four, you have to identify one figure which
does not belong to group.

1.

(a) (b) (c) (d)


Sol. (c) Both the arrow heads are in the same direction in figure (c). In all other figures, they are in the
opposite direction, Hence (c) is the answer.

2.

(a) (b) (c) (d)


Sol. (d) Between the shaded portion and the arrow, there are two triangles in figure (a), (b), (c) and (d).

3.

(a) (b) (c) (d)


Sol. (c) Figures (a) and (d) form a group. The bars are interchanged. Similarly, figure (b) and (d).
Hence (c) is the odd one out.

EXERCISE

Directions: (1 to 10) Out of the four figures (A), (B), (C) and (D), given in each problem, three are similar
in a certain way. However, one figure is not like the other three. Choose the figure which is different from
the rest.

1.

428 PACE IIT & MEDICAL: Mumbai / Delhi & NCR / Goa / Akola / Kolkata / Nashik / Pune / Bokaro / Dubai
NTSE-MENTAL ABILITY

2.

3.

4.

5.

6.

7.

429 PACE IIT & MEDICAL: Mumbai / Delhi & NCR / Goa / Akola / Kolkata / Nashik / Pune / Bokaro / Dubai
NTSE-MENTAL ABILITY

8.

9.

10.

Directions: (11 to 18) In each of the following questions four figures are given. One of these figures does
not correlate with the rest of the figures. Find the different figure. (NTSE Stage-I/Raj./2007)

11.

12.

13.

430 PACE IIT & MEDICAL: Mumbai / Delhi & NCR / Goa / Akola / Kolkata / Nashik / Pune / Bokaro / Dubai
NTSE-MENTAL ABILITY

14.

15.

16.

17.

18.

Directions: (19 to 23) In the following questions, four figures are given. Three of them are alike in a
certain way and one is different. Find the odd one from the alternatives. (NTSE Stage-II, 2007)

19.

431 PACE IIT & MEDICAL: Mumbai / Delhi & NCR / Goa / Akola / Kolkata / Nashik / Pune / Bokaro / Dubai
NTSE-MENTAL ABILITY

20.

21.

22.

23.

Directions: (24 to 28) In each questions four figures are given. One of them does not correlate with the
rest. Find the different figure. (NTSE Stage-I/Raj./2008)

24.

25.

432 PACE IIT & MEDICAL: Mumbai / Delhi & NCR / Goa / Akola / Kolkata / Nashik / Pune / Bokaro / Dubai
NTSE-MENTAL ABILITY

26.

27.

28.

Directions: (29 to 32) In each of the following questions, there are figures of which three are similar in a
certain way and one is different. Find the one that is different from the rest. (NTSE Stage-II, 2008)

29.

30.

31.

433 PACE IIT & MEDICAL: Mumbai / Delhi & NCR / Goa / Akola / Kolkata / Nashik / Pune / Bokaro / Dubai
NTSE-MENTAL ABILITY

32.

Directions: (33 to 37) In each question four figures are given. One of them does not correlate with the
rest. Find out the different figure. (NTSE Stage-I/Raj./2009)

33.

34.

35.

36.

37.

434 PACE IIT & MEDICAL: Mumbai / Delhi & NCR / Goa / Akola / Kolkata / Nashik / Pune / Bokaro / Dubai
NTSE-MENTAL ABILITY

Directions: (38 to 39) In each of the following questions, four figure are given. Three of them are alike I a
certain way. Find the one which is different from other (NTSE Stage-II, 2009)

38.

39.

Directions: (40 to 44) In each of the following question four figures are given. One of these figures does
not fit with the rest of the figures. Find out that correct serial number. (NTSE Stage-I/Raj./2012)

40.

41.

42.

43.

435 PACE IIT & MEDICAL: Mumbai / Delhi & NCR / Goa / Akola / Kolkata / Nashik / Pune / Bokaro / Dubai
NTSE-MENTAL ABILITY

44.

Directions: (45 to 48) In each of the following questions four figures are given. One of these figures does
not fit with the rest of the figures. Find out that correct serial number. (NTSE Stage-I/Raj./2013)

45.

46.

47.

48.

436 PACE IIT & MEDICAL: Mumbai / Delhi & NCR / Goa / Akola / Kolkata / Nashik / Pune / Bokaro / Dubai
NTSE-MENTAL ABILITY

49. Find the odd one out (NTSE Stage-II, 2013)

Direction: (50 to 54) There are four figures given. One of these does not correlate with the rest of the
figures. Find out that figure. (NTSE Stage-I/Raj./2014)

50.

51.

52.

53.

437 PACE IIT & MEDICAL: Mumbai / Delhi & NCR / Goa / Akola / Kolkata / Nashik / Pune / Bokaro / Dubai
NTSE-MENTAL ABILITY

54.

Direction: (55) Find out the odd figure. (NTSE Stage-I/Raj./2015)

55.

Direction: (56) In the below question there are four figures given. One of these does not correlate with the
rest of the figures. Find out that odd figure. (NTSE Stage-I/Raj./2017)

56.

Answer Key

1. (d) 2. (d) 3. (c) 4. (d) 5. (b) 6. (d) 7. (b) 8. (b) 9. (b) 10. (d)
11. (c) 12. (d) 13. (b) 14. (c) 15. (c) 16. (b) 17. (c) 18. (c) 19. (c) 20. (b)

21. (c) 22. (b) 23. (c) 24. (b) 25. (c) 26. (d) 27. (d) 28. (a) 29. (c) 30. (d)

31. (d) 32. (d) 33. (a) 34. (d) 35. (c) 36. (b) 37. (c) 38. (a) 39. (a) 40. (a)

41. (d) 42. (c) 43. (b) 44. (c) 45. (c) 46. (a) 47. (b) 48. (d) 49. (c) 50. (b)

51. (d) 52. (a) 53. (b) 54. (d) 55. (a) 56. (d)

438 PACE IIT & MEDICAL: Mumbai / Delhi & NCR / Goa / Akola / Kolkata / Nashik / Pune / Bokaro / Dubai
NTSE-MENTAL ABILITY

PREVIOUS YEARS QUESTIONS

Q.1 to 3 Directions: Find the odd figure

1.

(a) (b)

(c) (d)
2.

(a) (b)

(c) (d)

439 PACE IIT & MEDICAL: Mumbai / Delhi & NCR / Goa / Akola / Kolkata / Nashik / Pune / Bokaro / Dubai
NTSE-MENTAL ABILITY

3.

(a) (b)

(c) (d)

Q.4 to 6: Direction Find the odd figure

4.

(a) (b)

(c) (d)

5.

(a) (b)

(c) (d)
440 PACE IIT & MEDICAL: Mumbai / Delhi & NCR / Goa / Akola / Kolkata / Nashik / Pune / Bokaro / Dubai
NTSE-MENTAL ABILITY

6.

(a) (b)

(c) (d)
[NTSE-MAHARASHTRA STAGE-1-2019]
Q.7 to 9: Direction: Find the odd figure

7.

(a) (b)

(c) (d)

8.

(a) (b)

(c) (d)
441 PACE IIT & MEDICAL: Mumbai / Delhi & NCR / Goa / Akola / Kolkata / Nashik / Pune / Bokaro / Dubai
NTSE-MENTAL ABILITY

9.

(a) (b)

(c) (d)
[NTSE-MAHARASHTRA STAGE-1-2018]

Q.10 and 11 Directions – Find the odd figure out

10.

(a) (b)

(c) (d)

11.

(a) (b)

(c) (d) (2017)

442 PACE IIT & MEDICAL: Mumbai / Delhi & NCR / Goa / Akola / Kolkata / Nashik / Pune / Bokaro / Dubai
NTSE-MENTAL ABILITY

Q.12 and 13 Directions: - Find the odd figure

12.

(a) (b)

(c) (d)

13.

(a) (b)

(c) (d)

Q.14 and 15: Directions :- Find the odd term out

14.

(a) (b)

(c) (d)

443 PACE IIT & MEDICAL: Mumbai / Delhi & NCR / Goa / Akola / Kolkata / Nashik / Pune / Bokaro / Dubai
NTSE-MENTAL ABILITY

15.

(a) (b)

(c) (d)

Q.16 to 17  Directions: - Find the odd term out.

16.

(a) (b)

(c) (d)

17.

(a) (b)

(c) (d)
[2014]

444 PACE IIT & MEDICAL: Mumbai / Delhi & NCR / Goa / Akola / Kolkata / Nashik / Pune / Bokaro / Dubai
NTSE-MENTAL ABILITY

Q 18. And 19  Directions: - Find the odd term out.

(a) (b)

(c) (d)

19.

(a) (b)

(c) (d) [2013]

Q.20 and 22  Directions:- Find the odd figure out


20.

(a) (b)

(c) (d)

445 PACE IIT & MEDICAL: Mumbai / Delhi & NCR / Goa / Akola / Kolkata / Nashik / Pune / Bokaro / Dubai
NTSE-MENTAL ABILITY

21.

(a) (b)

(c) (d)

22.

(a) (b)

(c) (d) [2012]

Answer Key

4. 6.
1. (c) 2. (d) 3. (b) 5. (c) 7. (c) 8. (b) 9. (d) 10. (c)
(Bonus) (Bonus)
11. (d) 12. (c) 13. (c) 14. (a) 15. (c) 16. (c) 17. (c) 18. (d) 19(b) 20.(d)
21.(b) 22. (c)

446 PACE IIT & MEDICAL: Mumbai / Delhi & NCR / Goa / Akola / Kolkata / Nashik / Pune / Bokaro / Dubai
NTSE-MENTAL ABILITY

30. LOGICAL SEQUENCE OF WORDS


In this type of questions, certain inter-related words are given and numbered, followed by various sequences of
the numbers denoting them; as alter natives. The candidate is required to arrange these words in a logical
sequence based on a common property and then choose the correctly graded sequence from the given
alternatives.
Some common sequences have been discussed below:

Type-I
Sequence of occurrence of events or various stages in a process:

1. Arrange the following In a meaningful sequence:


1. Consultation 2. Illness 3. Doctor 4. Treatment 5. Recovery
(A) 2,3,1,4,5 (B) 2,3,4,1,5 (C) 4,3,1,2,5 (D) 5,1,4,3,2
Sol. (A) Clearly Illness occurs first. One then goes to a doctor and after consultation with him, undergoes
treatment to finally attain recovery.
Thus, the correct order is 2, 3, 1, 4, 5.

2. Arrange the following in a logical order:


1. Birth 2. Death 3. Funeral 4. Marriage 5. Education
(A)1,3,4,5,2 (B)1,5,4,2,3 (C) 2,3,4,5,1 (D) 4,5,3,1,2
Sol. (B) Clearly, the given words when arranged in the order of various events as they occur in u man’s
life, form the sequence: Birth, Education, Marriage, Death, Funeral.
So the correct order becomes 1, 5, 4, 2, 3.

Type II
Sequence of objects in a class or group, from part to the whole:

3. Arrange the following in a meaningful order, from particular to general:


1. Family 2. Community 3. Member 4. Locality 5. Country
(A) 3,1,2,4,5 (B) 3,1,2,5,4 (C) 3,1,4,2,5 (D) 3,1,4,5,2
Sol. (A) Clearly a member is a part of a family, which in turn is a part of community.
The community lives in a locality which lies within a country.
Thus, the correct order is 3, 1, 2, 4, 5.

4. Arrange the following in a logical order:


1. Shoulder 2. Wrist 3. Elbow 4. Palm 5. Finger
(A) 2,4,5,3,1 (B) 3,1,4,2,5 (C) 3,4,5,2,1 (D) 5,4,2,3,1
Sol. (D) Clearly, we are given the names of parts of a hand, which may be arranged
(i) from top to bottom, i.e., Shoulder, Elbow, Wrist, Palm, Finger, which is 1, 3, 2, 4, 5; or
(ii) from bottom to top, i.e., Finger, Palm, Wrist, Elbow, Shoulder, which is 5,4, 2, 3, 1.
Out of these, the sequence 5, 4, 2, 3, 1 is given in the alternatives provided.

447 PACE IIT & MEDICAL: Mumbai / Delhi & NCR / Goa / Akola / Kolkata / Nashik / Pune / Bokaro / Dubai
NTSE-MENTAL ABILITY

Type-III
Sequence of Increasing/decreasing size, value, intensity etc.:
5. Arrange the following in a logical sequence from small to big:
1. Bungalow 2. Flat 3. Cottage 4. House 5. Palace 6. Mansion
(A) 3,2,1,4,6,5 (B) 3,2,4,1,5,6 (C) 3,2,4,1,6,5 (D) 5, 6,4,1,2, 3
Sol. (C) Clearly, the names of various dwelling units, when arranged in increasing order of their sizes
form the sequence: Cottage, Flat, House, Bungalow, Mansion, Palace.
Thus, the correct order is 3, 2, 4, 1, 6, 5.

6. Arrange the following in a logical order:


1. Gold 2. Iron 3. Sand 4. Platinum 5. Diamond
(A) 2,4, 3,5,1 (B) 3, 2,1,5,4 (C) 4,5,1,3,2 (D) 5.4,3,2,1
Sol. (B) Clearly, the given names when arranged in order of increasing values, i.e., from cheapest to the
most expensive, form the sequence: Sand, Iron, Gold, Diamond, Platinum.
Thus, the correct answer is 3, 2, 1, 5, 4.

7. Arrange the following in a logical order:


1. Euphoria 2. Happiness 3. Ambivalence 4. Ecstasy 5. Pleasure
(A)1,4,2,5,3 (B) 2,1,3,4,5 (C) 3,2,5,1,4 (D) 4,1, 3,2,5
Sol. (C) All the given words stand for ‘joy’, but the intensity increases in the order – Ambivalence,
Happiness, Pleasure, Euphoria, Ecstasy.
Thus, the correct order is 3, 2, 5, 1, 4.

Type-IV
Sequence in which a chain of given objects is formed.

8. Arrange the following in a meaningful sequence :


1. Phrase 2. Letter 3. Word 4. Sentence
(A) 1, 2, 3, 4 (B) 1, 3, 2, 4 (C) 2, 3, 1, 4 (D) 2, 3, 4, 1
Sol. (C) A group of letters makes a word. A group of words makes a phrase. A group of phrases makes
a sentence.
Thus, the correct order is 2, 3, 1, 4.

9. Arrange the following in a logical order:


1. Frog 2. Eagle 3. Grasshopper 4. Snake 5. Grass
(A) 1,3,5,2,4 (B) 3,4,2,5,1 (C) 5,3,1,4,2 (D) 5,3,4,2,1
Sol. (C) Clearly, a grasshopper feeds on grass, a frog feeds on a grasshopper, a snake feeds on a frog and an
eagle feeds on a snake. Thus, a food-chain is formed.
So, the correct order is 5, 3, 1, 4, 2.

EXERCISE

Directions: (1 to 40): In each of the following questions, arrange the given words in a meaningful
sequence and then choose the most appropriate sequence from amongst the alternatives provided below
each question:

1. 1. Honey 2. Flower 3. Bee 4. Wax


(a) 1, 3, 4, 2 (b) 2, 1, 4, 3 (c) 2, 3, 1, 4 (d) 4, 3, 2, 1
448 PACE IIT & MEDICAL: Mumbai / Delhi & NCR / Goa / Akola / Kolkata / Nashik / Pune / Bokaro / Dubai
NTSE-MENTAL ABILITY

2. 1. Site 2. Plan 3. Rent 4. Money


5. Building 6. Construction
(a) 1, 2, 3, 6, 5, 4 (b) 2, 3, 6, 5, 1, 4 (c) 3, 4, 2, 6, 5, 1 (d) 4, 1, 2, 6, 5, 3

3. 1. Reading 2. Composing 3. Writing 4. Printing


(a) 1, 3, 2, 4 (b) 2, 3, 4, 1 (c) 3, 1, 2, 4 (d) 3, 2, 4, 1

4. 1. Sentence 2. Chapter 3. Letter 4. Book


5. Word 6. Paragraph
(a) 4, 2, 1, 6, 5, 3 (b) 4, 2, 6, 1, 5, 3 (c) 4, 6, 1, 2, 3, 5 (d) 4, 6, 2, 5, 1, 3

5. 1. Cut 2. Put on 3. Mark 4. Measure


5. Tailor
(a) 1, 3, 2, 4, 5 (b) 2, 4, 3, 1, 5 (c) 3, 1, 5, 4, 2 (d) 4, 3, 1, 5, 2

6. 1. Police 2. Punishment 3. Crime 4. Justice


5. Judgemnet
(a) 1, 2, 3, 4, 5 (b) 3, 1, 2, 4, 5 (c) 3, 1, 4, 5, 2 (d) 5, 4, 3, 2, 1

7. 1. Country 2. Furniture 3. Forest 4. Wood


5. Trees
(a) 1, 3, 5, 4, 2 (b) 1, 4, 3, 2, 5 (c) 2, 4, 3, 1, 5 (d) 5, 2, 3, 1, 4

8. 1. Elephant 2. Cat 3. Mosquito 4. Tiger


5. Whale
(a) (b) (c) (d)

9. 1. Key 2. Door 3. Lock 4 Room


5. Switch on
(a) 1, 2, 3, 5, 4 (b) 1, 3, 2, 4, 5 (c) 4, 2, 1, 5, 3 (d) 5, 1, 2, 4, 3

10. 1. Book 2. Pulp 3. Timber 4. Jungle


5. Paper
(a) 2, 5, 1, 4, 3 (b) 3, 2, 5, 1, 4 (c) 4, 3, 2, 5, 1 (d) 5, 4, 3, 1, 2

11. 1. College 2. Child 3. Salary 4. School


5. Employment
(a) 1, 2, 4, 3, 5 (b) 2, 4, 1, 5, 3 (c) 4, 1, 3, 5, 2 (d) 5, 3, 2, 1, 4

12. 1. Mother 2. Child 3. Milk 4. Cry


5. Smile
(a) 3, 4, 5, 1, 2 (b) 2, 4, 1, 3, 5 (c) 2, 4, 3, 1, 5 (d) 3, 2, 1, 5, 4

13. 1. Leaf 2. Fruit 3. Stem 4. Root


5. Flower
(a) 3, 4, 5, 1, 2 (b) 4, 1, 3, 5, 2 (c) 4, 3, 1, 2, 5 (d) 4, 3, 1, 5, 2

449 PACE IIT & MEDICAL: Mumbai / Delhi & NCR / Goa / Akola / Kolkata / Nashik / Pune / Bokaro / Dubai
NTSE-MENTAL ABILITY

14. 1. Andhra Pradesh 2. Universe 3. Tirupati 4. World


5. India
(a) 1, 5, 3, 2, 4 (b) 2, 1, 3, 5, 4 (c) 3, 1, 5, 4, 2 (d) 5, 4, 2, 1, 3

15. 1. Travel 2. Destination 3. Payment 4. Berth/Seat number


5. Reservation 6. Availability of berth/seat for reservation
(a) 1, 2, 5, 4, 3, 6 (b) 2, 6, 3, 5, 4, 1 (c) 5, 3, 4, 1, 6, 2 (d) 6, 2, 5, 4, 3, 1

16. 1. Atomic Age 2. Metallic Age 3. Stone Age 4. Alloy Age


(a) 1, 3, 4, 2 (b) 2, 3, 1, 4 (c) 3, 2, 4, 1 (d) 4, 3, 2, 1

17. 1. Post-box 2. Letter 3. Envelope 4. Delivery


5. Clearance
(a) 2, 3, 1, 4, 5 (b) 3, 2, 1, 4, 5 (c) 3, 2, 1, 5, 4 (d) 3, 2, 4, 5, 1

18. 1. Foetus 2. Child 3. Baby 4. Adult


5. Youth
(a) 1, 2, 4, 3, 5 (b) 1, 3, 2, 5, 4 (c) 2, 3, 5, 4, 1 (d) 5, 4, 2, 3, 1

19. 1. Curd 2. Grass 3. Butter 4. Milk


5. Cow
(a) 2, 5, 4, 3, 1 (b) 1, 3, 2, 5, 4 (c) 5, 2, 3, 4, 1 (d) 5, 2, 4, 1, 3

20. 1. Heel 2. Shoulder 3. Skull 4. Neck


5. Knee 6. Chest 7. Thigh 8. Stomach
9. Face 10. Hand
(a) 2, 4, 7, 10, 1, 5, 8, 9, 6, 3 (b) 3, 4, 7, 9, 2, 5, 8, 10, 6, 1
(c) 4, 7, 10, 1, 9, 6, 3, 2, 5, 8 (d) 3, 9, 4, 2, 10, 6, 8, 7, 5, 1

21. 1. Butterfly 2. Cocoon 3. Egg 4. Worm


(a) 1, 3, 4, 2 (b) 1, 4, 3, 2 (c) 2, 4, 1, 3 (d) 3, 4, 2, 1

22. 1. Rainbow 2. Rain 3. Sun 4. Happy


5. Child
(a) 2, 1, 4, 3, 5 (b) 2, 3, 1, 5, 4 (c) 4, 2, 3, 5, 1 (d) 4, 5, 1, 2, 3

23. 1. Yarn 2. Plant 3. Saree 4. Cotton


5. Cloth
(a) 2, 4, 1, 5, 3 (b) 2, 4, 3, 5, 1 (c) 2, 4, 5, 1, 3 (d) 2, 4, 5, 3, 1

24. 1. Study 2. Job 3. Examination 4. Earn


5. Apply
(a) 1, 3, 2, 5, 4 (b) 1, 2, 3, 4, 5 (c) 1, 3, 5, 2, 4 (d) 1, 3, 5, 4, 2

25. 1. Rain 2. Monsoon 3. Rescue 4. Flood


5. Shelter 6. Relief
(a) 1, 2, 3, 4, 5, 6 (b) 1, 2, 4, 5, 3, 6 (c) 2, 1, 4, 3, 5, 6 (d) 4, 1, 2, 3, 5, 6
450 PACE IIT & MEDICAL: Mumbai / Delhi & NCR / Goa / Akola / Kolkata / Nashik / Pune / Bokaro / Dubai
NTSE-MENTAL ABILITY

26. 1. Windows 2. Walls 3. Floor 4. Foundation


5. Roof 6. Room
(a) 4, 1, 5, 6, 2, 3 (b) 4, 2, 1, 5, 3, 6 (c) 4, 3, 5, 6, 2, 1 (d) 4, 5, 3, 2, 1, 6

27. 1. Puberty 2. Adulthood 3. Childhood 4. Infancy


5. Sencescence 6. Adolescence
(a) 2, 4, 6, 3, 1, 5 (b) 4, 3, 1, 6, 2, 5 (c) 4, 3, 6, 2, 1, 5 (d) 5, 6, 2, 3, 4, 1

28. 1. District 2. Village 3. State 4. Town


5. City
(a) 2, 4, 5, 1, 3 (b) 2, 1, 4, 5, 3 (c) 5, 3, 2, 1, 4 (d) 2, 5, 3, 4, 1

29. 1. Presentation 2. Recommendation 3. Arrival 4. Discussion


5. Introduction
(a) 3, 5, 1, 4, 2 (b) 3, 5, 4, 2, 1 (c) 5, 3, 2, 1, 4 (d) 5, 3, 4, 1, 2

30. 1. Cutting 2. Dish 3. Vegetable 4. Market


5. Cooking
(a) 1, 2, 4, 5, 3 (b) 3, 2, 5, 1, 4 (c) 4, 3, 1, 5, 2 (d) 5, 3, 2, 1, 4

31. 1. Never 2. Sometimes 3. Generally 4. Seldom


5. Always
(a) 5, 2, 1, 3, 4 (b) 5, 2, 4, 3, 1 (c) 5, 3, 2, 1, 4 (d) 5, 3, 2, 4, 1

32. 1. Table 2. Tree 3. Wood 4. Seed


5. Plant
(a) 1, 2, 3, 4, 5 (b) 1, 3, 2, 4, 5 (c) 4, 5, 2, 3, 1 (d) 4, 5, 3, 2, 1

33. 1. Doctor 2. Fever 3. Prescribe 4. Diagnose


5. Medicine
(a) 1, 4, 3, 2, 5 (b) 2, 1, 3, 4, 5 (c) 2, 1, 4, 3, 5 (d) 2, 4, 3, 5, 1

34. 1. Income 2. Status 3. Education 4. Well-being


5. Job
(a) 1, 2, 5, 3, 4 (b) 1, 3, 2, 5, 4 (c) 3, 1, 5, 2, 4 (d) 3, 5, 1, 2, 4

35. 1. Hecto 2. Centi 3. Deca 4. Kilo


5. Deci
(a) 1, 3, 4, 5, 2 (b) 1, 5, 3, 4, 2 (c) 2, 5, 3, 1, 4 (d) 5, 2, 1, 4, 3

36. 1. Index 2. Contents 3. Title 4. Chapters


5. Introduction
(a) 2, 3, 4, 5, 1 (b) 3, 2, 5, 1, 4 (c) 3, 2, 5, 4, 1 (d) 5, 1, 4, 2, 3

37. 1. Patient 2. Diagnosis 3. Bill 4. Doctor


5. Treatment
(a) 1, 4, 2, 3, 5 (b) 1, 4, 3, 2, 5 (c) 1, 4, 2, 5, 3 (d) 4, 1, 2, 3, 5

451 PACE IIT & MEDICAL: Mumbai / Delhi & NCR / Goa / Akola / Kolkata / Nashik / Pune / Bokaro / Dubai
NTSE-MENTAL ABILITY

38. 1. Milky way 2. Sun 3. Moon 4. Earth


5. Stars
(a) 1, 4, 3, 2, 5 (b) 2, 3, 4, 5, 1 (c) 3, 4, 2, 5, 1 (d) 4, 3, 2, 5, 1

39. 1. Sea 2. Rivulet 3. Ocean 4. River


5. Glacier
(a) 5, 2, 1, 3, 4 (b) 5, 2, 4, 1, 3 (c) 5, 4, 2, 3, 1 (d) 5, 4, 3, 2, 1

40. 1. Probation 2. Interview 3. Selection 4. Appointment


5. Advertisement 6. Application
(a) 5, 6, 2, 3, 4, 1 (b) 5, 6, 3, 2, 4, 1 (c) 5, 6, 4, 2, 3, 1 (d) 6, 5, 4, 2, 3, 1

41. Which would be the proper order of the following (in ascending order)?
1. Trillion 2. Thousand 3. Billion 4. Hundred
5. Million
(a) 1, 2, 4, 3, 5 (b) 1, 5, 3, 2, 4 (c) 4, 2, 3, 5, 1 (d) 4, 2, 5, 3, 1

42. Arrange the following items from general to particular:


1. Animal 2. Feline 3. Leopard 4. Mammal
5. Vertebrate 6. Cat
(a) 1, 2, 3, 4, 5, 6 (b) 1, 3, 5, 4, 2, 6 (c) 1, 4, 3, 2, 5, 6 (d) 1, 5, 4, 2, 6, 3

Answer Key

1. (c) 2. (d) 3. (d) 4. (b) 5. (d) 6. (c) 7. (a) 8. (c) 9. (b) 10. (c)
11. (b) 12. (b) 13. (d) 14. (c) 15. (b) 16. (c) 17. (c) 18. (b) 19. (d) 20. (d)

21. (d) 22. (b) 23. (a) 24. (c) 25. (c) 26. (b) 27. (b) 28. (a) 29. (a) 30. (c)

31. (d) 32. (c) 33. (c) 34. (d) 35. (c) 36. (c) 37. (c) 38. (c) 39. (b) 40. (a)
41. (d) 42. (d)

452 PACE IIT & MEDICAL: Mumbai / Delhi & NCR / Goa / Akola / Kolkata / Nashik / Pune / Bokaro / Dubai
NTSE-MENTAL ABILITY

31. SEQUENTIAL OUTPUT TRACING


In this type of questions, a message comprising of randomized letters/words or numbers or a combination of
both is given as the input followed by steps of rearrangement to give sequential outputs. The candidate is
required to trace out the pattern in the given rearrangement and then determine the desired output step,
according as is asked in the questions.

Patterns to Look for in the Given Sequence:


1. Arranging the given words in forward/reverse alphabetical order.
2. Arranging the given numbers in ascending/descending order.
3. Writing a particular set of words in the reverse order, stepwise.
4. Changing places of words/numbers according to a set pattern.
The above points are the four possible criteria which you should look for to determine the pattern
in a given rearrangement.

Directions : (1 to 5) Read the following information and then find out the answers to the questions that
follow.
An electronic device rearranges numbers step-by-step in a particular order according to a set of rules.
The device stops when the final result is obtained. In this case the device stops at Step V.

Input : 85 16 36 04 19 97 63 09
Step I : 97 85 16 36 04 19 63 09
Step II : 97 85 63 16 36 04 19 09
Step III : 97 85 63 36 16 04 19 09
Step IV : 97 85 63 36 19 16 04 09
Step V : 97 85 63 36 19 16 09 04
Study the above arrangement carefully and then answer the following questions :

1. Which of the following will be step III for the input below ?
Input : 09 25 16 30 32 18 17 06
(A) 32 30 25 09 16 18 17 06 (B) 32 30 09 25 16 18 17 06
(C) 32 09 25 16 30 18 17 06 (D) 32 30 09 25 16 19 17 06

2. Which is the last step for the input below ?


Input : 16 09 25 27 06 05
(A) Step II (B) Step III (C) Step 1V (D) None of these

3. What is the output of step V for the input below?


Input : 25 08 35 11 88 67 23
(A) 08 11 23 25 35 67 88 (B) 88 67 35 25 23 11 08
(C) 88 67 35 25 23 08 11 (D) none of these

4. Which one of the following would be the last step for the input below?
Input : 03 31 43 22 11 09
(A) Step II (B) Step III (C) Step 1V (D) None of these

453 PACE IIT & MEDICAL: Mumbai / Delhi & NCR / Goa / Akola / Kolkata / Nashik / Pune / Bokaro / Dubai
NTSE-MENTAL ABILITY

5. If the output of Step IV is as given below, what was the input?


Step IV : 92 86 71 69 15 19 06 63 58
(A) 15 19 06 63 58 86 92 69 71 (B) 15 86 19 92 06 69 63 58 71
(C) 86 92 69 71 15 19 06 63 58 (D) Cannot be determined
Sol. (1 to 5) Carefully observing the sample sequence, we observe that in the given arrangement, the
numbers have been arranged in descending order (from left to right) in a sequence, altering the position
of only one number in each step followed by the other numbers as it is. The process continues till the
whole set of numbers is arranged in descending order.

1. Input : 09 25 16 30 32 18 17 06
Step I : 32 09 25 16 30 18 17 06
Step II : 32 30 09 25 16 18 17 06
Step III : 32 30 25 09 16 18 17 06
Hence, the answer is (A).

2. Input : 16 09 25 27 06 05
Step I : 27 16 09 25 06 05
Step II : 27 25 16 09 06 05
The whole set of numbers gets arranged in step II itself. So, Step II is the last step for given
input.
Hence, the answer is (A).

3. Input : 25 08 35 11 88 67 23
Step I : 88 25 08 35 11 67 23
Step II : 88 67 25 08 35 11 23
Step III : 88 67 35 25 08 11 23
Step IV : 88 67 35 25 23 08 11
Step V : 88 67 35 25 23 11 08
Hence, the answer is (B).

4. Input : 03 31 43 22 11 09
Step I : 43 03 31 22 11 09
Step II : 43 31 03 22 11 09
Step III : 43 31 22 03 11 09
Step IV : 43 31 22 11 03 09
Step V : 43 31 22 11 09 03
Clearly, Step V s the last step for the given input.
Hence, the answer is (D).

5. Since the numbers may be rearranged in several possible ways, so it is not possible to
determine the exact input. Hence the answer is (D).

454 PACE IIT & MEDICAL: Mumbai / Delhi & NCR / Goa / Akola / Kolkata / Nashik / Pune / Bokaro / Dubai
NTSE-MENTAL ABILITY

Directions : (6 to 9) Read the following information and then find out the answers to the questions that
follow.
A word arrangement machine, when given an input line of words, rearranges it in every step following a
certain rule. Following is an illustration if an input line of words and various steps of rearrangement.

Input : gone are take enough brought station


Step I : take gone are enough brought station
Step II : take are gone enough brought station
Step III : take are station gone enough brought
Step IV : take are station brought gone enough
And, Step IV is the last step for this input. Now find out an appropriate step in each of the following questions
following the above rule.

6. Input : car on star quick demand fat


What will be the third step for this input?
(A) star quick car demand on fat (B) star car quick demand on fat
(C) star car demand quick on fat (D) None of these

7. Input : ink hurry yet for the victory


Which of the following will be the last step of the above input?
(A) Step III (B) Step IV (C) Step V (D) Step VI

8. If Step III of an input is ‘warden examination town ink garden restore’, what step would be ‘warden
ink town garden restore examination’ ?
(A) Step I (B) Step II (C) Step IV (D) Step V

9. If Step IV of an input is ‘violet for sour height journey medium’, which of the following could be Step II
of that input?
(A) violet for sour height journey medium
(B) violet for journey height sour medium
(C) violet journey height for sour medium
(D) violet for sour journey height medium
Sol. (6 to 9) Observing the given arrangement, we find the following pattern :
If we arrange the words in the given input in the alphabetical order and then label them as 1, 2, 3, 4, 5, 6
then the last step contains the words in the order 6, 1, 5, 2, 4, 3. However, the position of only one word
is altered at each step.

6. Input : car on star quick demand fat


Step I : star car on quick demand fat
Step II : star car quick on demand fat
Step III : star car quick demand on fat
Hence, the answer is (B)

455 PACE IIT & MEDICAL: Mumbai / Delhi & NCR / Goa / Akola / Kolkata / Nashik / Pune / Bokaro / Dubai
NTSE-MENTAL ABILITY

7. Arranging the words in the input in alphabetical order and labeling them 1 to 6, we have :

for hurry ink the victory yet


1 2 3 4 5 6
Clearly, the last step should be: 6, 1, 5, 2, 4, 3 i.e.
yet for victory hurry the ink
In put : ink hurry yet for the victory
Step I : yet ink hurry for the victory
Step II : yet for ink hurry the victory
Step III : yet for victory ink hurry the
Step IV : yet for victory hurry ink the
Step V : yet for victory hurry the ink
Thus, Step V is the last step for the given output.
Hence, the answer is (C)

8. Clearly, Step III has two words rearranged in comparison to the given step, i.e., ‘examination’ and
‘town’. Thus, the given step is two steps behind Step III i.e., Step I.
Hence, the answer is (A)

9. Step IV has four rearranged words - ‘violet’, ‘for’, ‘sour’ and ‘height’. This means that none of these
words automatically came to its place during rearrangement, but each of them was replaced at one
step. Thus, Step II should contain only ‘violet’ and ‘for’ in the first and second place, and each one
of ‘sour’ and ‘height’ must lie at any place after ‘jumping’, so that each of them needs to be
rearranged.
Hence, the answer is (B)

Directions : (10 to 12) Read the following information and then find out the answers to the questions that
follow.
Given an input, a coding machine generates pass codes for six batches everyday as follows :
Input : you should know about type of questions
Pass codes:
Batch I : you questions should of know type about
Batch II : about you type questions know should of
Batch III : about of you should type know questions
And so on till the sixth batch.
The first batch begins work at 10:00 a.m. Each batch works for one hour. There is a rest period of one hour after
the fourth batch’s is over.
Now, answer the following questions :

10. If the input on a day is ‘eight friends are sitting in the circle’, then what will be the pass code for the
batch at 3:00 P.M.?
(A) the circle in friends are sitting eight
(B) circle sitting are the in eight friends
(C) sitting friends the are circle in eight
(D) circle friends sitting eight are in the

456 PACE IIT & MEDICAL: Mumbai / Delhi & NCR / Goa / Akola / Kolkata / Nashik / Pune / Bokaro / Dubai
NTSE-MENTAL ABILITY

11. The pass code for the sixth batch on a day was ‘mark your answer against appropriate serial number’.
What was the input provided to the machine on that day?
(A) number against appropriate serial answer mark your
(B) number your against mark appropriate answer serial
(C) number against serial appropriate answer mark your
(D) your answer number mark serial appropriate against

12. Aniket was to attend the batch at 4:00 P.M. on a day with a pass code ‘sentence awarded by high court
was executed’. However, he was compelled to work in the batch at 12 noon on that day. What was his
pass code then ?
(A) awarded sentence executed high by court was
(B) was executed by awarded court high sentence
(C) by high was sentence court awarded executed
(D) None of these
Sol: (10 to 12) Here, the words have been rearranged following two patterns alternately:
Pattern 1 : The words of the previous step are arranged in the order - first, last, second, second last, third,
third last and so on.
Pattern 2 : The words of the previous step are arranged in the order - last, first, second last, second, third
last, third and so on.
In such questions, where the terms change place according to a set pattern/order, we prepare a
rearrangement draft as shown below :
Input : A B C D E F G
Pass codes :
Batch I : A G B F C E D
(10 am. to 11 a.m.)
Batch II : D A E G C B F
(11 am. to 12 noon)
Batch III : D F A B E C G
(12 noon to 1 P.M.)
Batch IV : G D C F E A B
(1 P.M. to 2 P.M.)
Rest hour
(2 P.M. to 3 P.M.)
Batch V : G B D A C E F
(3 P.M. to 4 P.M.)
Batch VI : F G E B C D A
(4 P.M. to 5 P.M.)

10. We label the words of the given input as per the arrangement draft above and then obtain the
desired output for batch V at 3:00 P.M..

Input : eight friends are sitting in the circle


A B C D E F G
Batch V: G B D A C E F

Hence, the answer is (D).

457 PACE IIT & MEDICAL: Mumbai / Delhi & NCR / Goa / Akola / Kolkata / Nashik / Pune / Bokaro / Dubai
NTSE-MENTAL ABILITY

11. According to rearrangement draft, we have:


Batch VI: mark you answer against appropriate Serial number
F G E B C D A
Input: number against appropriate serial answer mark your
A B C D E F G
Hence the answer is (A).

12. According to rearrangement draft, we have:


Batch VI (4 P.M.):

sentence awarded by high court was executed


F G E B C D A
was sentence executed high by court awarded
D F A B E C G
Hence, the answer is (D).

EXERCISE

Directions : (1 to 4) Study the following information carefully to answer these questions:


A number sorting machine when given an input of numbers, rearranges them in a particular manner
step-by-step as indicated below till all the numbers are arranged. Given below is an illustration of this
arrangement.

Input : 39 121 48 18 76 112 l4 45 63 96


Step I : 14 39 121 48 18 76 112 45 63 96
Step II : 14 39 48 18 76 112 45 63 96 121
Step III : 14 18 39 48 76 112 45 63 96 121
Step IV : 14 18 39 48 76 45 63 96 112 121
Step V : 14 18 39 45 48 76 63 96 112 121
Step VI : 14 18 39 45 48 63 76 96 112 121
And step VI is the last step for this input.

1. What will be step III for the following input?


Input : 68 182 39 93 129 46 21 58
(a) 21 39 68 129 93 46 58 182
(b) 21 39 68 93 129 46 58 182
(c) 21 68 39 93 129 46 58 182
(d) none of these

2. Given below is the fifth step of an input. What will be the third step?
Step V: 17 32 43 82 69 93 49 56 99 106
(a) 17 32 82 43 69 93 49 56 99 106
(b) 17 32 82 69 43 93 49 56 99 106
(c) 17 32 82 69 93 43 49 56 99 106
(d) Cannot be determined

458 PACE IIT & MEDICAL: Mumbai / Delhi & NCR / Goa / Akola / Kolkata / Nashik / Pune / Bokaro / Dubai
NTSE-MENTAL ABILITY

3. What will be the last step for the following input?


Input : 138 63 49 93 89 122 32 71
(a) 32 49 63 71 89 93 122 138
(b) 32 49 71 63 89 93 122 138
(c) 32 49 63 71 93 89 122 138
(d) none of these

4. How many steps will be required for getting the final output for the following input?
Input : 101 85 66 49 73 39 142 25 115 74
(a) Five (b)Six (c) Seven (d) Eight

Directions: (5 to 7) Study the following information to answer the questions given below:
A number arrangement machine when given an input of numbers, rearranges them following a
particular rule in each step. The following is an illustration of input and steps of rearrangement.

Input : 48 245 182 26 99 542 378 297


Step I : 542 48 245 182 26 99 378 297
Step II : 542 26 48 245 182 99 378 297
Step III : 542 26 378 48 245 182 99 297
Step 1V : 542 26 378 48 297 245 182 99
Step V : 542 26 378 48 297 99 245 182
This is the final arrangement and step V is the fast step for this input.
5. What will be the fourth step for an input whose second step is given below?
Stop II : 765 42 183 289 542 65 110 350
(a) 765 42 542 65 183 289 110 350
(b) 765 42 542 350 183 289 65 110
(c) 765 42 542 65 110 183 289 350
(d) Cannot be determined

6. How many steps will be required to get the final output from the following input?
Input : 39 88 162 450 386 72 29
(a) Two (b) Third (c) Four (d) None of these

7. If the first stop of an input is ‘785 198 32 426 373 96 49’, then which of the following step will
be 785 32 426 49 198 373 96’?
(a) Second (b) Third (c) Fourth (d) Fifth

Directions : (8 to 11) Study the following information to answer the questions given below:
A word arrangement machine, when given an input line in words, rearranges them following a particular
rule in each step. The following is an illustration of input and the steps of rearrangement.
Input : Go for to Though By easy To Access at
Step I : Access Go for to Though By easy To at
Step II : Access at Go for to Though By easy To
Step III: Access at By Go for to Though easy To
Step IV : Access at By easy Go for to Though To
Step V : Access at By easy for Go to Though To
Step VI: Access at By easy for Go Though to To
459 PACE IIT & MEDICAL: Mumbai / Delhi & NCR / Goa / Akola / Kolkata / Nashik / Pune / Bokaro / Dubai
NTSE-MENTAL ABILITY

Step VII : Access at By easy for Go Though To to


And step VII is last step for this input.
As per the rules followed in the above steps, find out in the given questions the appropriate step for the given
input.
8. Input: story For around on was He at
Which of the following will be Step IV for the given input?
(a) around at For He on was story
(b) around at For He on story was
(c) around at For He story on was
(d) around at He For story on was

9. Input: every and peer to an for


Which of the following steps would be an and every for peer to’?
(a) II (b) III (c) IV (d) V

10. Input : Together over series on feast the so


Which of the following steps will be the last but one?
(a) II (b) III (c) IV (d) V

11. Input : Over Go For through at one


Which step will be the last step for the above input ?
(a) III (b) V (c) VI (d None of these

Directions: (12 to 15) Read the following information carefully to answer the given questions:
A word and number arrangement machine when given an input line in words and numbers, rearranges
them following a particular rule in each step. The following is an illustration of input and rearrangement.
Input : goal 63 57 home five task 82 17
Step I : 82 goal 63 57 home fie task 17
Step II : 82 five goal 63 57 home task 17
Step III: 82 five 63 goal 57 home task 17
Step IV: 82 five 63 goal 57 home 17 task
And Step IV is the last output.
As per the rules followed in the above steps, find out in each of the following questions the appropriate step for
the given input.

12. Input: host 15 32 page 43 over .mother 92


Which of the following steps will be the last but one?
(a) IV (b) V (c) VI (d) VII

13. Step II of an input is: 67 cat 12 25 dog fight man 42.


Which of the following will be Step V?
(a) 67 cat 42 dog 25 fight 12 man
(b) 67 cat 42 dog 25 12 fight man
(c) 67 cat 42 dog 12 25 fight man
(c) 67 cat 42 12 25 dog fight man

460 PACE IIT & MEDICAL: Mumbai / Delhi & NCR / Goa / Akola / Kolkata / Nashik / Pune / Bokaro / Dubai
NTSE-MENTAL ABILITY

14. Input : world 23 new 47 major 13 62 desk


Which of the following will be Step V for the above input?
(a) 62 desk 47 major world 23 new 13
(b) 62 desk 47 world 23 new major 13
(c) 62 desk 47 major 23 world new 13
(d) 62 desk 47 major 23 new world 13

15. Step III of an input is : 81 boat 73 wheel spike dancer 32 59


How many more steps are required to complete the rearrangement?
(a) Two (b) Three (c) four (d) Five
Directions: (16 to 20) Study the following information and answer the questions given below it:
The admission ticket for an exhibition bears a password which is changed after every clock hour based
on set of words chosen for each day. The following is an illustration of the code and steps of
rearrangement for subsequent clock hours.
The time is 9 a.m. to 3 p.m.
Batch I (9 a.m. to 10 a.m.): is not ready cloth simple harmony burning.
Batch II (10 a.m. to 11 a.m.): ready not is cloth burning harmony simple.
Batch III (11 a.m.to 12 noon): cloth is not ready simple harmony burning.
Batch IV (12 noon to 1 p.m.): not is cloth ready burning harmony simple.
Batch V (1 p.m. to 2 p.m.) : ready cloth is not simple harmony burning and so on.
16. If the password for Batch I was ‘rate go long top we let have’. Which batch will have the password-‘go
rate top long have let we’?
(a) II (b) III (c) IV (d) V

17. Day’s first password, “camel road no, toy say me not”. What will be the password for fourth batch, i.e.,
12 noon to 1 p.m.?
(a) road camel toy no not me say
(b) no road camel toy not me say
(c) toy no road camel not me say
(d) toy camel road no say me not

18. lf Batch II has the password’- ‘came along net or else key lot’, what could be the password for
Batch IV i.e., 12 noon to 1 P.M.?
(a) net or came along else key lot
(b) came or net along lot key else
(c) or net long came lot key else
(d) along net or came else key lot

19. If the password for 11 a.m. to 12 noon was -‘soap shy miss pen yet the she’, what was the password for
Batch I?
(a) pen miss shy soap she the yet
(b) shy miss pen soap yet the she
(c) soap pen miss shy she the yet
(d) miss shy soap pen she the yet

461 PACE IIT & MEDICAL: Mumbai / Delhi & NCR / Goa / Akola / Kolkata / Nashik / Pune / Bokaro / Dubai
NTSE-MENTAL ABILITY

20. If the password for Batch VI, i.e., 2 P.M. to 3 P.M. is -‘are trap cut he but say lap’, what will be the
password for Batch II, i.e., 10 to 11 a.m. ?
(a) trap are he cut lap say but
(b) he cut trap are lap say but
(c) cut he are trap but say lap
(d) are he cut trap lap say but

Directions: (21 to 25) Study the following information and answer the questions given below:
Given an input, a coding machine generates pass codes for six batches everyday, as follows:
Input : see the little squirrels jumping here and there

Pass codes :
Batch I : jumping see here the and little there squirrels
Batch II: the and here little see there jumping squirrels
Batch III : see the there and jumping here squirrels little
Batch IV: and jumping there here the squirrels see little and so on.
The first batch timing is 10:00 a.m. and each batch is one hour’s duration. There is rest period of one hour
after the work for the fourth batch is over.

21. If the pass code on a day for the second batch is ‘are of clouds transformed they bhakti the as’, what will
be the pass code for the batch at 3:00 P.M. on that day?
(a) clouds are bhakti as the they of transformed
(b) of the ‘bhakti clouds are as they transformed
(c) the they clouds are as bhakti transformed of
(d) are of as the they bhakti transformed clouds

22. On a particular day, Mr. X was to begin the work in .the batch at 11:00 a.m. with a pass code ‘he
slowly recedes to his inner apartment intellect’.
However, he came late on that day and hence joined the batch at 12 noon. What was his pass code then?
(a) to intellect recedes apartment slowly inner he his
(b) his he inner slowly apartment recedes intellect to
(c) to his recedes inner slowly apartment he intellect
(d) none of these

23. If the pass code on a day for the batch at 3:00 p.m. was ‘it is only the mind that creates problems’, what
was the pass code for the batch at 1:00 P.M. on that day?
(a) mind the problems creates only is that
(b) is the that problems it only mind creates
(c) creates mind only its is the that, problems
(d) mind it that is the problems only creates

24. On a day, the pass code for the first batch was ‘nobody can help us in solving our problems’
Write the input of the day In the reverse order of its words ?
(a) our in help nobody can us solving problems
(b) can us solving problems nobody help in our
(c) our in help nobody problem solving us can
(d) problems solving us can nobody help in our

462 PACE IIT & MEDICAL: Mumbai / Delhi & NCR / Goa / Akola / Kolkata / Nashik / Pune / Bokaro / Dubai
NTSE-MENTAL ABILITY

25. The pass code for the batch immediately before the rest hour was ‘there is no permanent solution
for mental problems’. What was the input for the pass code on that day?
(a) mental solution permanent for is problems there no
(b) mental solution problems is for permanent there no
(c) mental problems solution for is permanent there no
(d) none of these

Directions: (26 to 30) Study the following information to answer the questions given below:
An exhibition is open for public since 9 a.m. till 3 P.M. and again since 4 P.M. till 10 PM. In a day, there
are 12 batches of 1 hour each, the entry ticket bears a pass code made up of seven words, which changes
every hour following a particular rule. The pass code 4 P.M.to 10 P.M. are same as those for respective
hours during 9 a.m. to 3 P.M., i.e., the pass code for 4 P.M. to 5 P.M. is same as that of 9 a.m.to 10 a.m.
and so on. Following is an illustration of the code and steps of rearrangement for subsequent clock hours.

First batch : 9 a.m. to a. m. (4 p.m. to 5 p.m.)


Pass code : dig more and your will find water
Second batch: 10 a.m. to 11 a.m. (5 p.m. to 6 p.m.)
Pass code: and dig find your water will more
Third batch : 11 a.m. to 12 moon (6 p.m. to 7 p.m.)
Pass code : find and will you more water dig and so on.

26. If the pass code for the second batch is ‘do not play the near water dirty’, what will be the pass code for
2 p.p. to 3 p. m. batch?
(a) near dirty not the play do water
(b) dirty near play the not do water
(c) dirty near not the play do water
(d) near dirty not the play water do

27. If the pass code for third batch is ‘at study sleep and night child good’, which batch will have the
pass code ‘child sleep night and study good at’?
(a) Second (b) Fourth (c) Fifth (d) Sixth

28. If the pass code for 7 p.m. to 8 p.m. batch is ‘pen with write pencil nice time day’, what will be the
pass code for 11 a.m. to 12 noon?
(a) day with nice pencil write pen time
(b) day nice with pencil write pen time
(c) nice day with pencil write pen time
(d) none of these

29. If the pass code for 5 p.m. to 6 p.m. is ‘out in above over the field end’, what will be the pass
code for 1 p.m. 2 p.m.?
(a) the field end over out in above
(b) field end the over out in above
(c) field the end over out above in
(d) the field end over out above in

463 PACE IIT & MEDICAL: Mumbai / Delhi & NCR / Goa / Akola / Kolkata / Nashik / Pune / Bokaro / Dubai
NTSE-MENTAL ABILITY

30. If the pass code for the batch 4 p.m. to 5 p.m. is ‘go to office in time every day’, what will be the
pass code for 2 p.m. to 3 p.m. batch?
A) to day go in every office time
(B) day to go.in every office time
(C) to go day in every office time
(D) to go in day every office time

Direction (Question 31-33) : Given an input, a machine generates pass codes for the six batches each
day as follows :
Input: these icons were taken out from the sea.
Pass Codes
Batch I : from sea the out taken were icons these
Batch II : from icons these were taken out the sea
Batch III : from icons out sea the taken were these
Batch IV : from icons out sea these were taken the (NTSE Stage-II, 2015)

31. What will be the pass code for the Batch V on a day, if the input is ‘four of the following five form a
group”?
(a) a five following form four group the of
(b) a five following form group the of four
(c) a five following form four of the group
(d) a five following form four group of the

32. If the pass code for the Batch IV on a day was ‘back go here people who settle want to’ , what
was the pass code for the Batch V on that day?
(a) back go here people settle who want to
(b) back go here people to want settle who
(c) back go here people settle to want who
(d) cannot be determined

33. The pass code for the Batch I on a day was ‘he so used to sell the surplus items’. What was the
input on that day?
(a) items surplus the sell to used so he
(b) he items surplus the sell to used so
(c) so used to sell the surplus items he
(d) cannot be determined .

Answer Key

1. (b) 2. (d) 3. (a) 4. (d) 5. (a) 6. (d) 7. (c) 8. (c) 9. (b) 10. (d)
11. (d) 12. (a) 13. (b) 14. (c) 15. (c) 16. (c) 17. (a) 18. (d) 19. (b) 20. (c)

21. (d) 22. (b) 23. (b) 24. (c) 25. (d) 26. (c) 27. (c) 28. (d) 29. (a) 30. (a)

31. (a) 32. (c) 33. (d)

464 PACE IIT & MEDICAL: Mumbai / Delhi & NCR / Goa / Akola / Kolkata / Nashik / Pune / Bokaro / Dubai
NTSE-MENTAL ABILITY

32. ARITHMETICAL REASONING

1. Ten children take part in a tournament. Each one has to play every other one. How many games must
they play?
(A) 10 (B) 15 (C) 30 (D) 45
Sol. Clearly, we will consider the following matches :
(i) matches of first player with other 9 players;
(ii) matches of second player with 8 players other than the first player;
(iii) matches of third player with 7 players other than the first and second players.
And so on
So, number of matches played during the tournament = 9 + 8 + 7 + 6 + 5 + 4 + 3 + 2 + 1 = 45,
Hence, the answer is (D).

2. There are some benches in a classroom. If 4 students sit on each bench, then 3 benches are left
unoccupied. However, if 3 students sit on each bench, 3 students are left standing. How many students
are there in the class?
(A) 36 (B) 48 (C) 56 (D) 64
Sol. Let there be X students in the class.
X 
  3.
When 4 students sit on each bench, number of benches =  4 
 X 3
 
When 3 students sit on each bench, number of benches =  3  .
X   X 3
  3     3x  36  4x  12
4   3 
 x  48
Hence, number of students in the class = 48.
So, the answer is (B).

3. A, B, C and D play a game of cards. A says to B, “If I give you 8 cards, you will have as many as C
has and I shall have 3 less than what C has. Also, if I take 6 cards from C, I shall have twice as many as
D has”. If B and D together have 50 cards, how many cards has A got?
(A) 40 (8) 37 (C) 27 (D) 23
Sol. Clearly, we have:
B+8=C ... (i)
A–8=C–3 ... (ii)
A + 6 = 2D ... (iii)
B + D = 50 ...(iv)
Putting C = A – 5 from (ii) into (i), we have:
B + 8 = A – 5 or A – B = 13 ...(v)
Putting D = 50 – B from (iv) into (iii), we have:
A + 6 = 100 – 2B or A + 2B = 94 ...(vi)
Solving (v) and (vi), we get: B = 27 and A = 40.
 A has 40 cards. Hence, the answer is (A).

465 PACE IIT & MEDICAL: Mumbai / Delhi & NCR / Goa / Akola / Kolkata / Nashik / Pune / Bokaro / Dubai
NTSE-MENTAL ABILITY

4. At a farm, there are hens, cows and bullocks and the keepers to look after them. There are 69 heads less
than legs; the number of cows is twice the number of bullocks; the number of cows and hens is the same
and there is one keeper per ten birds, hens and cattle. The total number of hens plus cows and bullocks,
and their keepers does not exceed 50. How many cows are there?
(A) 16 (B) 14 (C) 12 (D) 10
Sol. Let H. C. B and K represent the number of hens, cows, bullocks and keepers respectively.
There, as given, we have:
H + C + B + K < 50 …(i)
C = 2B ... (ü)
C=H ... (iii)
H C B
K
10 …(iv)
From (ii), (iii) and (iv), we have :
10K = H + C + B  10K = 2C + B = 2  2B + B
 10K = 5B  B =2K.
Thus B = 2K, C = 2B = 4K, H = C = 4K.
Total number of heads = H + C + B + K.
Total number of legs = 2H + 4C + 4B + 2K.
 (2H + 4C + 4B + 2K) – (H + C + B + K) = 69
 H + 3C + 3B + K = 69
 4K + 12K + 6K + K = 69
 23K = 69  K = 3.
Hence, number of cows = C = 4k
=  4  3  12
So, the answer is (C).

5. Aruna cut a cake into two halves and cuts one half into smaller pieces of equal size. Each of the small
pieces is twenty grams in weight. If she has seven pieces of the cake in all with her how heavy was the
original cake?
(A) 120 grams (B) 140 grams (C) 240 grams (D) 280 grams

Sol. The seven pieces consist of 6 smaller equal pieces and one half cake piece.
Weight of each small piece = 20 g.
So, total weight of the cake = [2  (20  6)] g
= 240 g.
Hence, the answer is (C).

6. Consider the diagram given below:

466 PACE IIT & MEDICAL: Mumbai / Delhi & NCR / Goa / Akola / Kolkata / Nashik / Pune / Bokaro / Dubai
NTSE-MENTAL ABILITY

Five hundred candidates appeared in an examination comprising of tests in English, Hindi and
Mathematics. The diagram gives the number of
who failed in different tests. What is
the percentage of candidates who failed in at least two subjects ?
(A) 0.078 (B) 1.0 (C) 6.8 (C) 7.8
Sol. Clearly, number of candidates who failed in at least two subjects
= number of candidates who failed in two or more subjects.
= (10 + 12 +12 + 5) = 39.
 39 
 100  %  7.8%
 Required percentage =  500 

7. In a group of persons travelling in a bus, 6 persons can speak French, 15 can speak Spanish and 6 can
speak English. In that group, none can speak any other language. If 2 persons in the group can speak two
Language and one person can speak all the three language, then how many persons are there in the
group ?
(A) 21 (B) 22 (C) 23 (D) 24
Sol. Let circles X, Y and Z represent persons who can speak French, Spanish and English respectively.

French-speaking persons -
=a+b+d+e=6 …(i)
Spanish-speaking persons
= b + c + e + f =15 … (ii)
English-speaking persons
=d+e+f+g=6 ... (iii)
Persons speaking 2 languages
=b+d+f=2 …(iv)
Persons speaking all 3 languages
=e=1 …(v)
From (i) and (v), we have:
a+b+d=5 ...(vi)
From (ii) and (v), we have:
b + c + f= 14 ...(vii)
From (iii) and (v), we have :
d+f+g=5 ...(viii)
Subtracting (iv) from (vi), we get :
a–f=3 ...(ix)
Adding (vii) and (viii), we get :
b + c + d + 2f + g =19 …(x)

467 PACE IIT & MEDICAL: Mumbai / Delhi & NCR / Goa / Akola / Kolkata / Nashik / Pune / Bokaro / Dubai
NTSE-MENTAL ABILITY

Adding (ix) and (x), we get:


a + b + c + d + f + g = 22 or
a + b + c + d + e + f + g =23 ( e =1)
 Total number of persons in the group = 23.
Hence, the answer is (C).

8. In a town, 65% people watched the news on television, 40% read a newspaper and 25% read a
newspaper and watched the news on television also. What percent of the people neither watched
the news on television nor read a newspaper?
(A) 5 (B) 10 (C) 15 (D) 20
Sol. Let the total number of people be 100.
Let circle X represent people who watched television and Y represent people who read news paper.

Then, A + B = 65, B + C = 40, B = 25.


Solving, we get: A = 40, B = 25, C = 15.
 Number of persons who neither watched television nor read newspaper
= 100 – (A + B + C)
= 100 – (40 + 25 +15)
= 100 – 80 = 20.
So, required percentage = 20%.

9. A shepherd had 17 sheep. All but nine died. How many was he left with ?
(A) Nil (B) 8 (C) 9 (D) 17
Sol. ‘All but nine died’ means ‘All except nine died’
i.e. 9 sheep remained alive.
Hence, the answer is (C).

EXERCISE

1. 90 gifts were distributed in a party. Everyone presented a gift to another. How many persons were there
in the party?
(a) 9 (b) 10 (c) 81 (d) 89

2. At the end of a business conference the ten people present all shake hands with each other once. How
many handshakes will there be altogether?
(a) 10 (b) 45 (c) 55 (d) 90

3. What is the product of all the numbers in the dial of a telephone?


(a) 1,58,480 (b) 1,59,450 (c) 1,59,480 (d) None of these

4. A bird shooter was asked how many birds he had in the bag. He replied that there were all sparrows but
six, all pigeons but six and all ducks but six. How many birds he had in the bag in all?
(a) 9 (b) 18 (c) 27 (d) 36
468 PACE IIT & MEDICAL: Mumbai / Delhi & NCR / Goa / Akola / Kolkata / Nashik / Pune / Bokaro / Dubai
NTSE-MENTAL ABILITY

5. A group of 1200 persons consisting of captains and soldiers is travelling in a train. For every 15 soldiers
there is one captain. The number of captains in the group is-
(a) 85 (b) 80 (c) 75 (d) 70

6. A man wears socks of two colours-black and brown. He has altogether 20 blacks socks and 20 brown
socks in a drawer. Supposing he has to take out the socks in the dark, how many must he take out to be
sure that he has a matching pair?
(a) 2 (b) 3 (c) 20 (d) 21

7. The total number of digits used in numbering the pages of a book having 366 pages is-
(a) 732 (b) 990 (c) 1098 (d) 1305

8. A printer numbers the pages of a book starting with 1 and uses 3189 digits in all. How many pages does
the book have?
(a) 1000 (b) 1074 (c) 1075 (d) 1080

9. A motorist knows four different routes from Kota to Jaipur. From Jaipur to Delhi he knows three
different routes and Delhi to Ranchi he knows two different routes. How many routes does he know
from Kota to Ranchi?
(a) 4 (b) 8 (c) 12 (d) 24

10. The number of eggs in a basket doubles every minute. The basket is full of eggs in one hour. When was
the basket one-fourth full? Give your answer in minutes counting from start.
(a) 59 (b) 58 (c) 30 (d) 15

11. An enterprising businessman earns an income of Rs. 1 on the first day of his business. On every
subsequent day, he earns an income which is just double of that made on the previous day. On the 10 th
day of business, his income is-
(a) Rs. 29 (b) Rs. 210 (c) Rs. 10 (d) Rs. 102

12. A tailor had a number of shirt pieces to cut from a roll of fabric. He cut each roll of equal length into 10
pieces. He cut at the rate of 45 cuts a minute. How many rolls would be cut in 24 minutes?
(a) 32 rolls (b) 54 rolls (c) 108 rolls (d) 120 rolls

13. A girls counted in the following way on the fingers of her left hand. She started by calling the thumb 1,
the index figure 2, middle finger 3, ring finger 4, little finger 5 and then reversed direction calling the
ring finger 6, middle finger 7 and so on. She counted up to 1994. She ended counting on which finger?
(a) Thumb (b) Index finger (c) Middle finger (d) Ring finger

14. A monkey climbs 30 feets at the beginning of each hour and rests for while when he slips back 20 feet
before he again starts climbing in the beginning of the next hour. If he begins his ascent at 8:00 a.m., at
what time will he first touch a flag at 120 feet from the ground?
(a) 4 P.M. (b) 5 P.M. (c) 6 P.M. (d) None of these

15. If a clock takes seven second to strike seven, how long will it take to strike ten?
(a) 7 seconds (b) 9 seconds (c) 10 seconds (d) None of these

469 PACE IIT & MEDICAL: Mumbai / Delhi & NCR / Goa / Akola / Kolkata / Nashik / Pune / Bokaro / Dubai
NTSE-MENTAL ABILITY

16. If 100 cats kill 100 mice in 100 days, then 4 cats would kill 4 mice in how many days?
(a) 1 day (b) 4 days (c) 40 days (d) 100 days

17. On Children’s Day, sweets were to be equally distributed among 175 children in a school. Actually on
the Children’s Day, 35 children were absent and therefore each child got 4 sweets extra. Total how
many sweets were available for distribution?
(a) 2400 (b) 2480 (c) 2680 (d) 2800

18. Mr. X, a mathematician, defines a number as connected with 6 ' if it divisible by 6 or if the sum of its
digits is 6, or if 6 is one of the digits of the number. Other numbers are all ‘not connected with 6 ' . As
per this definition, the number of integers from 1 to 60 (both inclusive) which are not connected with 6
is-
(a) 18 (b) 22 (c) 42 (d) 43

19. An egg vendor calls on his first customer and sells half his eggs and half an egg. To the second
customer, he sells half of what he has left with and half an egg, and to the third customer, he sells half of
what he was then left with and half an egg. However, he did not break any egg. If in the end, the vendor
was left with three eggs did he have initially?
(a) 26 (b) 31 (c) 39 (d) None of these

20. At a dinner party every two guests used a bowl of rice between them every three guests used a bowl of
dal between them and every four used a bowl of meat between them. There were altogether 65 dishes.
How many guests were present at the party?
(a) 60 (b) 65 (c) 90 (d) 100

21. There are 50 students admitted to a nursery class. Some students can speak only English and some can
speak only Hindi. Ten students can speak both English and Hindi. If the number of students who can
speak English is 21, then how many students can speak Hindi, how many can speak only Hindi and how
many can speak only Hindi and how many can speak only English?
(a) 39, 29 and 11 respectively (b) 37, 27 and 11 respectively
(c) 28, 18 and 11 respectively (d) 21, 11 and 29 respectively

22. In a group of 15 people, 7 read French, 8 read English while 3 of them read none of these two. How
many of them read French and English both?
(a) 0 (b) 3 (c) 4 (d) 5

23. Consider the Venn diagram given below:

The number in the Venn diagram indicates the number of persons reading the newspaper. The diagram
is drawn after surveying 50 persons. In a population of 10,000 how many can be expected to read at
least two newspapers?
(a) 5000 (b) 5400 (c) 6000 (d) 6250
470 PACE IIT & MEDICAL: Mumbai / Delhi & NCR / Goa / Akola / Kolkata / Nashik / Pune / Bokaro / Dubai
NTSE-MENTAL ABILITY

24. Out of a total of 120 musicians in a club, 5% can play all the three instruments-guitar, violin and flute. It
so happens that the number of musicians who can play any two and only two of the above instruments is
30. The number of musicians who can play the guitar alone is 40. What is the total number of those who
can play violin alone or flute alone?
(a) 30 (b) 38 (c) 44 (d) 45

Directions: (25 to 27) The diagram given below shows the number of students who got distinction in three
subjects out of 500 students. Study the diagram carefully and answer the questions that follow:

25. What is the percentage of students who got distinction in two subjects?
(a) 8% (b) 9% (c) 10% (d) 12%

26. What is the percentage of students who got distinction?


(a) 28% (b) 35% (c) 38% (d) 40%

27. The percentage of students distinction marks in Mathematics is


(a) 17.8% (b) 18.6% (c) 19.2% (d) 20.6%

Directions: (28 to 30) Study the following information given below and answer the questions that follow:
A publishing firm publishes newspapers A, B and C. In an effort to persuade advertisers to insert
advertisements in these newspapers, the firm sends out the following statement to possible advertisers.
A survey of representative sample of the whole population shows that-
Newspaper A is read by 26%
Newspaper B is read by 25%
Newspaper C is read by 14%
Newspaper A and B is read by 11%
Newspaper B and C is read by 10%
Newspaper C and A is read by 9%
Newspaper C only is read by 0%

28. The percentage of readers who read all the three newspapers is-
(a) 1 (b) 4 (c) 5 (d) 6

29. The percentage of readers who read A and B but not C, is-
(a) 2 (b) 4 (c) 5 (d) 6

30. The percentage of readers who read at least one of the three newspapers is-
(a) 40 (b) 50 (c) 60 (d) 65
471 PACE IIT & MEDICAL: Mumbai / Delhi & NCR / Goa / Akola / Kolkata / Nashik / Pune / Bokaro / Dubai
NTSE-MENTAL ABILITY

31. Two candles are of different lengths and thicknesses. The short and the long ones can burn respectively
for 5 hour and 3.5 hour. After burning for 2 hour, the lengths of the candles become equal in length.
What fraction of the long candle’s height was the short candle initially? (NTSE Stage II/2014)
2 5 3 4
(a) (b) (c) (d)
7 7 5 5

32. Mother was asked how many gifts she had in the bag. She replied that there were all dolls but six, all
cars but six, and all books but six. How many gifts had she in all? (NTSE Stage II/2014)
(a) 9 (b) 18 (c) 27 (d) 36

33. In a school 120 boys have registered for a singles carom tournament. Each match eliminates one player.
How many matches are to be organized to determine the champion? (NTSE Stage II/2014)
(a) 60 (b) 61 (c) 119 (d) 120

34. Twenty four teams are divided into 4 group of six teams each. Within each group the teams play each
other exactly once. The winners of each group then play in the semifinals. Winners of the semifinals
play in the finals and losers for the 3rd place. How many matches are played?
(NTSE Stage II/2014)
(a) 60 (b) 63 (c) 64 (d) 66

35. A work can be completed by 40 workers in 40 days. If 5 workers leave every 10 days, in how many days
work will be completed? (NTSE Stage II/2015)
(a) 55, 66 (b) 56, 44 (c) 56, 66 (d) 58, 66

36. A boat starts with the speed of 1 km per hour. After every 1 km, the speed of boat becomes twice. How
much will be the average speed of the boat at the end of journey of 2.5 km? (NTSE Stage II/2015)
2.5 2.5 2.5 2.5
(a) (b) (c) (d)
1.5125 1.75 1.625 1.50

37. The sum of the incomes of A and B is more than that of C and D taken together. The sum of incomes of
A and C is the same as that of B and D taken together. Moreover, A earns half as much as the sum of the
incomes of B and D. Whose income is highest? (NTSE Stage II/2015)
(a) A (b) B (c) C (d) D

38. A tank is filled by three pipes with each pipe having uniform flow. The first two pipes operating
simultaneously fill the tank in the same time during in which the tank is filled by the third pipe alone.
The second pipe fills the tank 5 hours faster than the first pipe and 4 hours slower than the third pipe.
The time required by the first pipe to fill the tank is: (NTSE Stage II/2015)
(a) 6 hours (b) 10 hours (c) 15 hours (d) 30 hours

472 PACE IIT & MEDICAL: Mumbai / Delhi & NCR / Goa / Akola / Kolkata / Nashik / Pune / Bokaro / Dubai
NTSE-MENTAL ABILITY

Answer Key

1. (b) 2. (b) 3. (d) 4. (a) 5. (c) 6. (b) 7. (b) 8. (b) 9. (d) 10. (b)
11. (a) 12. (d) 13. (b) 14. (c) 15. (d) 16. (d) 17. (d) 18. (d) 19. (b) 20. (a)

21. (a) 22. (b) 23. (b) 24. (c) 25. (a) 26. (c) 27. (a) 28. (c) 29. (d) 30. (a)

31. (b) 32. (a) 33. (c) 34. (c) 35. (c) 36. (b) 37. (b) 38. (c)

PREVIOUS YEARS QUESTIONS


1. Ten years before ratio of ages of Ram and Shyam was 1 : 7, ten years after ratio of their ages is
1 : 3. Find the present age of Ram.
(a) 10 years (b) 20 years (c) 30 years (d) 70 years

2. From the above information, what will be the age of Shyam after 10 years?
(I) 70 years (2) 80 years (3) 90 years (4) 30 years
(NTSE-MAHARASHTRA STAGE-1-2020)

(Q. 3 and 4: Direction) -Ten years ago the ratio of Sunil’s and Anil's ages was 1 : 7. Ten years hence the
ratio of their ages will be 1 : 2. Then

3. Find Sunil's present age.


(a) 14 years (b) 40 years (c) 70 years (d) 28 years

4. What was Anil’s ace ten years before


(a) 4 years (b) 28 years (c) 24 years (d) 32 years
(NTSE-MAHARASHTRA STAGE-1-2019)

(Q. 5 & 6) : Direction : Ten years ago the ratio of the ages of Ramesh and Suresh was 1 : 5. Ten years hence
the ratio of their ages will be 3 : 5 then

5. Find the present age of Ramesh ?


(a) 14 years (b) 10 years (c) 40 years (d) 24 years

6. How old was Suresh ten years ago ?


(a) 9 years (b) 20 years (c) 40 years (d) 30 years
(NTSE-MAHARASHTRA STAGE-1-2018)

Answer Key

1. (b) 2. (c) 3. (a) 4. (b) 5. (a) 6. (b)

473 PACE IIT & MEDICAL: Mumbai / Delhi & NCR / Goa / Akola / Kolkata / Nashik / Pune / Bokaro / Dubai
NTSE-MENTAL ABILITY

33. LOGICAL DEDUCTION


The phenomenon of deriving a logically certain aspect from a single statement or a set of given statements, is
known as Logical deduction. Here, each question consists of one or more statements, on the basis that a
deduction has to be made. There are several parts of logical deduction, here we are taking four types of logical
deduction in which a variety of statements or combinations of statements are given and we have to deduct a
logical thing from them.
1. Statement and Assumptions
2. Statement and Conclusions
3. Statement and Course of action
4. Statement and Argument

STATEMENT AND ASSUMPTIONS

In these questions, a statement is followed by two or more assumptions. The candidate is required to assess the
given statement and decide which of the given assumptions is implicit in the statement. To understand the
pattern of these questions, it is very essential to know what the terms statement and assumption stand for.

Whenever we communicate with others, we make several statements. Therefore, a statement is a part of our
speech or what we speak to others.

In the process of communication, most of the ideas remain unuttered. As a result, these ideas may not find
expression in the statement. However, these unuttered messages are very much implied in the statement.
Implied message in a statement is known as an assumption. Therefore, an assumption is something that can be
supposed
or assumed on the basis of a given statement.

Here we are giving below a few examples with explanations to make the students acquainted with the pattern of
these questions and the method to tackle them.

(Directions for Q. No. 1 to 4): In each of the questions below is given a statement followed by assumptions
numbered I and II. Consider the statement and decide which of the given assumptions is implicit.
Give your answer as
(A) if only assumption I is implicit
(B) if only assumption II is implicit
(C) if neither I nor II is implicit
(D) if both I and II are implicit

1. Statement:
Patient’s condition would improve after this operation.
Assumptions:
I. The patient can be operated upon in this condition.
II. The patient can’t be operated upon in this condition.
Sol. It is very much implied in the statement that the patient is in a position to be operated upon.
Therefore, assumption I is implicit. Hence option (A) is the correct answer.

474 PACE IIT & MEDICAL: Mumbai / Delhi & NCR / Goa / Akola / Kolkata / Nashik / Pune / Bokaro / Dubai
NTSE-MENTAL ABILITY

2. Statement; The best evidence of India’s glorious past is the growing popularity of Ayurvedic medicines
in the West.
Assumptions:
I. Ayurvedic medicines are not popular in India.
II. Allopathic medicines are more popular in India.
Sol. The statement given in the question states only about the place which Ayurvedic medicine had
occupied in the past on account of its increasing Popularity in West and this signifies neither non-
popularity of Ayurvedic medicines in India nor speaks of popularity of Allopathic medicines.
Hence, none of the assumptions is implicit in the statement, Hence, (C) is the correct answer.

3. Statement : Most of the classical dance themes are based on stories of Gods and avatars.
Assumptions:
I. Classical arts maintain their heritage by sticking to traditions.
II. New themes are not interesting.
Sol. Stories of Gods and avatars carry the reflection of our tradition, Therefore, the statement implies that
the classical art endeavors to maintain their heritage by sticking to traditions. Hence, assumption I is
clearly implicit. But it is not given anywhere in the statement that new themes are not interesting.
Therefore, assumption is not implicit. Hence, (A) is the correct answer.

4. Statement :”If you want to give any advertisement,give it in newspaper ‘X’- A tells B.
Assumptions:
I. B wants to publicize his products.
II. Newspaper ‘X’ has a wide circulation.
Sol. The word ‘if ’ in the statement denotes that B may or may not be willing to publicize his products.
Statement further suggests that advertisements should be given in newspaper ‘X’. This means that ‘X’
will help advertise better. In other words, newspaper ‘X’ has a wide circulation. So, only assumption II
is implicit. Hence, (B) is the correct answer.

STATEMENT AND CONCLUSIONS


This chapter is a mere extension of the previous chapter - Statement and Assumptions. Here, students are
required to approach the questions for solution from a different angle. This approach basically emphasizes the
need for being more systematic and logical in drawing inferences.

Unlike the previous chapter, here the statement is followed by conclusions instead of assumptions. The
questions of this type are designed to test the candidate’s ability to interpret a statement. Therefore, the
consequent effect of a statement has to be analyzed before reaching the correct conclusion. Conclusion is the
direct meaning of the given statement.

In view of the differences between two chapters, the candidates are therefore required to go through the
examples given here and read the explanations following each question. It will enable them to understand the
logical approach required to draw valid conclusions.

475 PACE IIT & MEDICAL: Mumbai / Delhi & NCR / Goa / Akola / Kolkata / Nashik / Pune / Bokaro / Dubai
NTSE-MENTAL ABILITY

Directions : (5 to 7) In each of the following questions, a statement is followed by two conclusions I and II.
Give your answer as :
(A) if only conclusion I follows,
(B) if only conclusion II follows,
(C) if neither I nor II follows, and
(D) if both I and II follow.

5. Statement: Parents are prepared to pay any price for an elite education to their children.
Conclusions:
I. All parents these days are very well off.
II. Parents have an obsessive passion for perfect development of their children through good schooling.
Sol. It may be concluded from statement that since parents want a perfect development of their children
through good schooling, therefore, they are prepared to pay any price for a good education.
But the statement does not give sense of the parents being very well off Hence, only conclusion II
follows. Therefore, the correct answer is (B).

6. Statement : The Constitutional amendment carried out just last month prohibits the employment of child
labour in any organisation.
Conclusions:
I. The employees must now abide by this amendment to the constitution.
II. Children below 14 years of age will now be engaged in acquiring education.
Sol. It is clearly deduced from the statement that consequent upon the amendment prohibiting the child
labour, the employer must abide by the amendment. But it is not mentioned that the amendment gives
any provision for acquiring education. Hence, only conclusion I follows.
Therefore, the correct answer is (A).

7. Statement : Any young man, who makes dowry as a condition for marriage, discredits himself and
dishonours womanhood.
Conclusions;
I. Those who take dowry in marriage should be condemned by society.
II. Those who don’t take dowry in marriage, respect womanhood.
Sol. Clearly, the statement declares dowry as an evil practice and reflects its demerits. Thus, conclusion
I follows. Also, it is given that those who take dowry, dishonour womanhood. This implies that those
who don’t take dowry respect women hood. So conclusion II follows. Hence, the correct answer is (D).

STATEMENT AND COURSE OF ACTION


These types of questions are designed to test a candidate's decision making ability. Course of action is a
decision which a person makes invariably in his life following a problem. In these questions a problem is given
in the form of a statement and is followed by two or more decisions in the form of courses of action.
To solve the questions of this type, the candidates are advised to keep in mind following points :
Correct course of action should either lessen the problem or improve the situation created by the problem.

Simple problem must have a simple course of action and not a complex one which may create more problems
than to solve or reduce it. Course of action should be feasible and should relate with the practical aspect of life.
A clear understanding about the type of question and method of solution can be developed by way of solving a
variety of questions given in this chapter, subsequently. Here are a few examples with answers and
explanations.

476 PACE IIT & MEDICAL: Mumbai / Delhi & NCR / Goa / Akola / Kolkata / Nashik / Pune / Bokaro / Dubai
NTSE-MENTAL ABILITY

Directions: (8 to 10) In each of the questions below is given a statement followed by two courses of action
numbered I and II. A course of action is a step or administrative decision to be taken for improvement,
follow up or further action in regard to the problem, policy etc. On the basis of the information given in
the statement you have to assume everything in the statement to be true, then decide which of the two
given courses of action logically follow.

Give your answer as :


(A) if only I follows.
(B) if only II follows.
(C) if neither I nor II follows.
(D) if both I and II follow.

8. Statement : The officer-in-charge of a company had a hunch that some money was missing from the
safe.
Courses of Action :
I. He should get in recounted with the help of the staff and check it with the balance sheet.
II. He should inform the police.
Sol. Here a hunch about the money being lost does not provide any ground for the incident having actually
taken place, and hence action to inform the police will not lessen the problem but is more likely to
aggravate the situation.
A prudent step is to recount the money and tally with the books of account. Hence, course of action
I is appropriate. Therefore, (A) is the correct answer.

9. Statement: Many cases of cholera were reported from a nearby village.


Courses of Action :
I. The question should be raised in the Legislative Assembly.
II. A team of doctors should be rushed to the village.
Sol. Discussion of the problem in the Legislative Assembly is feasible but does not provide any immediate
relief to the victims. Hence, I is not the right course of action to be chosen for the problem.
However, immediate medical aid will definitely mitigate the problem to a large extent. Hence, course of
action II is the remedial step. Therefore, (B) is the correct answer.

10. Statement: Youngsters are often found staring at obscene posters.


Courses of Action:
I. Children should be punished and penalized if they are found doing so.
II. Any display of such material should be banned.
Sol. Punishment to the children is not a judicious step in the direction of dealing such problem. However,
putting a complete ban can help to solve the problem by way of preventing children to get such
exposure. Hence, (B) is the correct answer.

STATEMENT AND ARGUMENT


In these questions a statement is followed by two arguments. Candidates are required to distinguish between the
strong and weak arguments.
Generally, both the arguments are contrary to each other and refer to the positive and negative results of the
action as mentioned in the statement. A strong argument is that which touches the practical and real aspect of
the situation as described in the statement. A weak argument is very simple, superfluous, ambiguous and long
drawn one.

477 PACE IIT & MEDICAL: Mumbai / Delhi & NCR / Goa / Akola / Kolkata / Nashik / Pune / Bokaro / Dubai
NTSE-MENTAL ABILITY

Following points should be taken into consideration while choosing a strong argument.
1. A strong argument should give the realistic diagnosis of the situation described in the statement.
2. A strong argument should give the deep analysis of the topic dealt within the statement.
3. A strong argument should relate with the statement and be supported up by facts or established notions.
4. A strong argument should not be mere reiteration of the situation given in the statement.
Following examples will help the students to have an understanding of the logic used to solve these questions.

Directions : (11 to 13) Study the following instructions carefully and then answer the questions that
follow.
In making decisions about important questions it is desirable to be able to distinguish ‘strong’ and ‘weak’
arguments so far as they relate to the questions. Weak’ arguments may not be directly related to the
question and may be of minor importance or may be related to the trivial aspect of the question. Each
question below is followed by two arguments numbered I and II, You have to decide which of the
arguments is a ‘strong’ argument and which is a ‘weak’ argument.
Give your answer as :
(A) if only argument I is strong
(B) if only argument II is strong
(C) if neither I nor II is strong`
(D) if both I and II are strong

11. Statement: Should school education be made free in India?


Arguments:
I. Yes, this is the only way to improve the level of literacy.
II. No, it will add to the already heavy burden on the exchequer.
Sol. Argument I is not strong because word ‘only’ makes the argument weak as it is not the only real and
practical solution to improve the level of literacy. Argument II is strong as it describes the practical
problem which may arise out of the decision of making education free in India. Hence, (B) is the
correct answer.

12. Statement : Should there be complete ban on manufacture of fire crackers in India ?
Arguments:
I. No, this will render thousands of workers jobless.
Il. Yes, the fire, cracker manufacturers use child labour to a large extent.
Sol. Both the arguments refer to the practical consequences of the action mentioned in the statement and
hence, are strong. Thus, (D) is the correct answer.

13. Statement : Should luxury hotels be banned in India ?


Argument;
I. Yes, these are places from where international criminals operate.
II. No, affluent foreign tourists will have no place to stay.
Sol. The luxury hotels are symbols of country’s development and a place for staying the affluent foreign
tourists. So, argument II is a strong one.
Argument I is a weak argument because ban on luxury hotels is not a way to end the international
criminals. (B)
Note: Sometimes we have to face a paragraph and by that paragraph we have to find out the
statement among the given alternatives which follows most logically. This type of questions are called

478 PACE IIT & MEDICAL: Mumbai / Delhi & NCR / Goa / Akola / Kolkata / Nashik / Pune / Bokaro / Dubai
NTSE-MENTAL ABILITY

THEME DETECTION.
Each of the following question contains a small paragraph followed by four statements. Find the
statement which follows the paragraph most logically.

14. Though the waste of time or the expenditure on fashions is very large, yet fashion have come to stay.
They will not go, come what may, However, what is now required is that strong efforts should be made
to displace the excessive craze for fashion from the minds of these youngsters.
(A) Fashion is the need of the day.
(B) The excessive craze for fashion is detrimental to one’s personality.
(C) The hoard for fashion should be done away with so as not to let down the constructive development.
(D) Work and other activities should be valued more than the outward appearance.
Sol. Constructive development like good thoughts, behavior, values comes from mind which decides our
personality. Craze for fashion distract our mind from goals and other incentives. So option (C)
logically follows.

15. The future of women in India is quite bright and let us hope that they will justify their abilities by rising
to the occasion. Napoleon was right when he declared that by educating women we can educate the
whole nation, because a country can never rise without the contribution of 50% of their population.
(A) India is striving hard for the emancipation of women
(B) All women should be well educated.
(C) A nation can progress only when women are given equal rights and opportunities as men.
(D) Women ought to be imparted full freedom to prove their worth and contribute to the progress of
the nation.
Sol. The paragraph indicated that equal rights are given to women along with men, so option (D) follows
logically.

EXERCISE

Directions: (1 to 7) In each of the questions below is given a statement followed by two assumptions
numbered I and II. An assumption is something supposed or taken for granted. You have to consider the
statement and the following assumptions and decide which of the assumptions is implicit in the statement.
Give your answer as
(A) if only assumption I is implicit
(B) if only assumption II is implicit
(C) if neither I nor II is implicit
(D) if both I and II are implicit

1. Statement : It is desirable to put the child in the school at the age of 5 years or so.
Assumptions:
I. At that age the child reaches appropriate level of development and is ready to learn.
II. The schools do not admit children after six years of age.

2. Statement : If it does not rain throughout this month, most farmers would be in trouble this year.
Assumptions:
I. Timely rain is essential for farming.
II. Most of the farmers are generally dependent on rains.

479 PACE IIT & MEDICAL: Mumbai / Delhi & NCR / Goa / Akola / Kolkata / Nashik / Pune / Bokaro / Dubai
NTSE-MENTAL ABILITY

3. Statement : A advises B - “If you want to study English, join institute ‘Y.
Assumptions:
I. ‘B’ listens to A’s advice.
Il. Institute ‘Y’ provides good coaching for English.

4. Statement : Everybody loves reading adventure stories.


Assumptions:
I. Adventure stones are the only reading material.
Il. Nobody loves reading any other material.

5. Statement: The function will start at 3 pm. You are requested to take your seats before 3 pm
Assumptions:
I. If an invitee is not in his seat before 3 pm, the function will not start.
Il. Function will start as scheduled.

6. Statement : A warning in a train compartment -“To stop train pull chain. Penalty for improper use
Rs 500.”
Assumptions:
I. Some people misuse the chain.
II. On certain occasions, people may want to stop a running train.

7. Statement : The income tax rules need to be amended so that there is more incentive for the people to
declare their actual wealth.
Assumptions:
I. The income tax rules are not proper.
II. Some people do not declare their actual wealth.

Directions : (8 to 10) In each of the questions below is given a statement followed by three assumptions
numbered I, II and III. An assumption is something supposed or taken for granted. You have to consider
the statement and the following assumptions and decide which of the assumptions is implicit in the
statement, then decide which of the answers (A), (B), (C), and (D) is a correct answer and indicate it on
your answer sheet.

8. Statement : Ashok decided to leave office at 4.00 pm to catch a flight to Bangalore departing at
6.00 pm.
Assumptions:
I. The flight to Bangalore may be delayed.
II. He may be able to reach airport well before 6.00 pm
III. He may get adequate time to reach for a vehicle to go to the airport.
(A) All are implicit (B) Only II and III are implicit
(C) None is implicit (D) Only II is implicit

9. Statement : Ramesh decided to get the railway reservation in February for the journey he wants to
make in May to Chennai.
Assumptions:
I. The railway issues reservations three months in advance.
II. There are more than one train to Chennai
III. There will be vacancy in the desired class.
480 PACE IIT & MEDICAL: Mumbai / Delhi & NCR / Goa / Akola / Kolkata / Nashik / Pune / Bokaro / Dubai
NTSE-MENTAL ABILITY

(A) Only II and III are implicit (B) Only I is implicit


(C) All are implicit (D) Only I and III are implicit

10. Statement: The school authority decided to open a Summer school this year in the school compound for
the students in the age group of 7-14 years.
Assumptions:
I. All the students will attend the Summer school.
II. All the parents will prefer to remain in city than going out of town for enabling their children to
attend the Summer school.
III. Those who can’t afford to go out of the station, will send their children to Summer school.
(A) Only II is implicit (B) Only II and III are implicit
(C) None is implicit (D) Only III is implicit

Directions : (11 to 12) In each of the questions given below, one or more statement (s) is/are followed by
inferential conclusions. The conclusion, which can be derived without supposing anything else, i.e.,
without adding anything extra to the statement (s), is your answer.

11. Statement: State Government imposes a monthly tax on the salary of all the employees. The tax
amount varies according to the income slab. The Central Government also imposes tax on the
same income, which is known as the income tax.
This is against the theory of taxation.
Conclusions:
(A) A regular collection, irrespective of the income slab, by the Central Government is the income
tax.
(B) The Central Government should not collect any tax on income.
(C) Income tax should not vary according to the income slabs.
(D) Tax should be imposed only once on an individual.

12. Statement : A forest has as many Sandal trees as It has Ashoka trees. Three-fourths of the trees are
old and half of the trees are at the flowering stage.
Conclusions:
(A) All Ashoka trees are at the flowering stage.
(B) All sandal trees are at the flowering stage.
(C) At least one-half of the Ashoka trees are old.
(D) One half of the Sandal trees are at the flowering stage.

Directions : (13 to 20) In each of the questions below a statement is given followed by two conclusions
numbered I and II. You have to take the statement to be true. Read both the conclusions and decide
which of the two or both follow from the given statement.
Give your answer as:
(A) If only conclusion I follows.
(B) If only conclusion II follows.
(C) If neither I nor II follows.
(D) If both conclusions I and II follow.

481 PACE IIT & MEDICAL: Mumbai / Delhi & NCR / Goa / Akola / Kolkata / Nashik / Pune / Bokaro / Dubai
NTSE-MENTAL ABILITY

13. Statement : The majority of Indian labourers belong to unorganised sector and most of them
earn very low.
Conclusions:
I. The labourers belonging to organised sector have better benefits and stability.
II. Some labourers belonging to unorganized sector have regular and certain income.

14. Statement: Company ‘X has a record in making cameras with quality and modernisation to ensure
that you are losing not even a single shot despite bad weather.
Conclusions:
I. No other company has any recognition in camera industry.
II. Any common man can take good photographs using camera ‘X’.

15, Statement : In diabetes, there is an excess of sugar in the body. Our body needs sugar for energy.
The quantity of sugar in excess of body’s requirement is excreted through urine.
Conclusions:
I. The excessive consumption of sugar is likely to lead to diabetes.
II. Consumption of sugar should be avoided.

16. Statement : Any students, who does not behave properly while in the school, brings bad name to
himself and also to the school.
Conclusions:
I. Such students should be removed from the school.
II. Strict discipline does not improve behaviour of the students.

17. Statement : Until our country achieves Economic equality and Political freedom, Democracy would
be meaningless.
Conclusions:
I. Political freedom and Democracy go hand in hand.
II. Economic equality leads to real Political freedom and Democracy.

18. Statement: In the Art Gallery, paintings of a artist are displayed for the public. The prices of the
paintings are too high for a common man.
Conclusions:
I. Common man purchases paintings from other places.
II. Costly paintings are generally displayed at the art gallery.

19. Statement : Doctors have comprehensive knowledge of human structure.


Conclusions:
I. Anatomy is one of their subjects in curriculum.
II. Other subjects also deal with human structure.

20. Statement: Cases of bride-burning for dowry are not uncommon.


Conclusions:
I. Inspite of anti-dowry law, the ill practice still continues.
II. The punishment inflicted on the party concerned is not hard enough.

482 PACE IIT & MEDICAL: Mumbai / Delhi & NCR / Goa / Akola / Kolkata / Nashik / Pune / Bokaro / Dubai
NTSE-MENTAL ABILITY

Directions : (21 to 25) In each of the questions below is given a statement followed by two courses of
action. Course of action is a step for administrative decision to be taken for improvement, follow up or
further action in regard to the problem, policy etc. On the basis of the information given in the statement,
you have to assume everything in the statement to be true, then decide which of the given suggested
courses of action is/are logically worth pursuing.
Give your answer as:
(A) if only I follows
(B) if only Il follows
(C) if neither I nor li follows
(D) if both I and li follow

21. Statement : About 30% to 40% of children who are enrolled, do not attend the school on any given
day.
Courses of Action :
I. More schools should be started.
II. Reasons for this absenteeism should be found out,

22. Statement : The experts group on Technical Education has stressed that Computer Education
should be provided to children from primary school itself. It should be implemented in urban and rural
schools, simultaneously.
Courses of Action :
I. Government should issue instructions to all schools for Computer Education.
II. At least one teacher of each school should be trained in Computer operations for teaching
children.

23. Statement: Every year, at the beginning or at the end of the Monsoon, we have some cases of
conjunctivitis, but this year, it seems to be a major epidemic, witnessed after nearly four years.
Courses of Action :
I. Precautionary measures should be taken after every four years to check the epidemic.
II. People should be advised to drink boiled water during the Monsoon season.

24. Statement: There has been a significant drop in the water level of all the lakes supplying water to
the city.
Courses of Action :
I. The water supply authority should impose a partial cut in supply to tackle the situation.
II. The Government should appeal to all the residents through mass media for minimal use of water.

25. Statement : A very large number of students have failed in the final high school examinations due to
wrong questions in one of the subjects.
Courses of Action :
I. All the students who have failed in that subject should be allowed to take supplementary
examination.
II. All those who are responsible for the error should be suspended and an enquiry should be initiated
to find out the facts.

483 PACE IIT & MEDICAL: Mumbai / Delhi & NCR / Goa / Akola / Kolkata / Nashik / Pune / Bokaro / Dubai
NTSE-MENTAL ABILITY

Directions : (26 to 28) In each of the question below is given a statement followed by three courses of
action numbered I, II and III. A course of action is a step or administrative decision to be taken for
improvement follow up, or further action in regard to the problem, policy, etc. On the basis of
information given in the statement, you have to assume everything in the statement to be true.
Then decide which of the three given suggested course (s) of action logically follow(s) worth pursuing.
Then decide which of the alternatives (A), (B), (C), or (D) is correct.

26. Statement : If the faculty members also join the strike, there is going to be a serious problem.
Courses of Action :
I. The faculty members should be persuaded not to go on strike.
II. Those faculty members who join the strike should be suspended
III. The management should not worry about such small things.
(A) Only I follows
(B) Only I and II follow
(C) Only II and III follow
(D) All follow

27. Statement : Every year thousands of eligible students don’t get admission in colleges, both in
urban and rural areas, after passing their school leaving certificate examination.
Courses of Action :
I. More colleges should be set up in both urban and rural areas.
II. The number of schools in both urban and rural areas should be reduced.
III. None school should offer vocational courses to equip students for taking up their vocation after
completing their school education
(A) Only I follows
(B) Only II and III follow
(C) All follow
(D) Only I & III follow

28. Statement: Large number of students have failed in the recently held SSC final examination due to
their performance in the English language paper.
Courses of Action :
I. The Government should immediately issue a circular to all the schools to appoint competent
English language teachers.
II. The Government should immediately instruct all the schools to send their English language
teachers for refresher courses to be conducted by the Government.
III. The Government should instruct the examining body to lower the difficulty level of the English
language Paper in the future examination.
(A) Only I follows (B) Only III follows
(C) Only II and III follow (D) All follow

Directions : (29 to 37) In making decisions about important questions, it is desirable to distinguish
between a strong argument and a Weak’ argument, A ‘strong’ argument must be both important and
directly related to the question. A Weak’ argument may not be directly related to the question and may
be of minor importance or may be related to the trivial aspect of the question. Each question below is
followed by two arguments, numbered I and II. You have to decide which of the arguments is ‘strong’
and which is ‘weak’.

484 PACE IIT & MEDICAL: Mumbai / Delhi & NCR / Goa / Akola / Kolkata / Nashik / Pune / Bokaro / Dubai
NTSE-MENTAL ABILITY

Give your answer as:


(A) if only argument I is strong.
(B) if only argument II is strong,
(C) if neither I nor II is strong,
(D) if both I and II are strong. 1

29. Statement : Should there be a ban on product advertisements ?


Arguments :
I. No, it is an age of advertising. Unless your advertisement is better than that of your other
competitors, the products will not be sold.
II. Yes, the money spent on advertising is very huge and it inflates the cost of the product.

30. Statement : Should the Government levy tax on agricultural income also ?
Arguments:
I. Yes, that is the only way to fill Government’s coffer.
II. No, 80% of our Population live in rural areas.

31. Statement : Should public holidays be declared on demise of important national leaders ?
Arguments:
I. No, such unscheduled holidays hamper national progress.
II. Yes, people would like to pay their homage to the departed soul.

32. Statement : Should new universities be established in India ?


Arguments:
I. No, we have not yet achieved the target of literacy.
II. No, we have to face the problems of highly educated but unemployed youths.

33. Statement: Should small States be formed out of bigger States in India ?
Arguments:
I. Yes, there will be greater administrative convenience.
II. No, it will be a jeopardize the national integration.

34. Statement : Should all the Government owned educational institutions be given to private sector?
Arguments:
I. Yes, there will be upgradation of educational standard in these institutions.
II. Yes, the educational standard of these institutions will decrease.

35. Statement : Should the private companies be allowed to operate passenger train services in India ?
Arguments:
I. Yes, this will improve the quality of service in Indian Railways as it will have to face severe
competition.
II. No, the private companies may not agree to operate in the non-profitable sectors.

36. Statement : Are marriages based on well- matched horoscopes more successful than other marriages?
Arguments:
I. Yes. there is no possibility of rifts or ups and downs as the horoscopes have already been studied
well.
II. No. such marriages are rather dull.
485 PACE IIT & MEDICAL: Mumbai / Delhi & NCR / Goa / Akola / Kolkata / Nashik / Pune / Bokaro / Dubai
NTSE-MENTAL ABILITY

37. Statement: Should age of retirement be brought down?


Arguments:
I. Yes, such a decision on the part of the Government would open new job opportunities to
youngsters.
II. No, people often marry late and their children don’t get settled early.
Directions (38 to 39) : In each of the following questions two statements are given followed by two
concision I and II.
38. Statements: (NTSE Stage-Il, 2011)
In a sports club all the members are not players but all of them are rich.
Mr. P’ is a member of the sports club.
Conclusions:
I. Mr. ‘P’ plays the sport.
II. Mr. ‘P’ is rich.
(A) Only I follows
(B) Only II follows
(C) Both I and II follow
(D) Neither I nor II follows
39. Statements: (NTSE Stage-II, 2011)
Engineers marry only teachers.
Rash mi is a teacher.
Conclusions:
I. Rashmi is married to an Engineer
II. Rashmi is not married to an Engineer
(A) Only I follows (B) Only II follow
(C) Both I and II follow (D) Neither I nor II follow
40. Vimla used to board the train from Metro Station A for going to her office. Since Station A is a
terminus, she had no problem in getting a seat. Ever since she shifted to Locality B she finds it difficult
to get a seat, as by the time the train reaches Locality B it becomes crowded. Find the statement among
the alternatives which must be true as per the given information.
(NTSE Stage-II, 2013)
(A) Vimla would prefer to take a bus rather than the metro.
(B) Vimla’s travel to office has become less corn for table ever since she has shifted.
(C) Commuters staying in and around Locality B would demand metro services originating from
station near Locality B.
(D) Vimla would took for a job close to her home
41. Ramesh started going for regular morning walks for controlling his blood sugar level. He did so for
a month and also started taking Yoga lessons, without going for any pathological examination. He
underwent pathological test after two months and found that the blood sugar level has come down.
Presuming that he had not changed his food habits during these two months, which statement among the
alternative given below follows most logically? (NTSE Stage-II, 2013)
(A) Blood sugar level comes down after doing regular morning walk.
(B) Blood sugar level comes down after doing Yoga.
(C) Blood sugar level comes down on doing regular morning walk and Yoga
(D) Regular morning walk. Yoga or both may bring down sugar level despite not changing food habits

486 PACE IIT & MEDICAL: Mumbai / Delhi & NCR / Goa / Akola / Kolkata / Nashik / Pune / Bokaro / Dubai
NTSE-MENTAL ABILITY

Directions (42 to 44): In each of the following questions a statement is given followed by two conclusion
I and II. (NTSE Stage-l /Raj./2014)

42. Statement : Adversity makes a man wise.


Conclusions:
I. The poor are wise
II. Man learns from bitter experience.
(A) Only conclusion I is true
(B) Only conclusion II is true
(C) Both conclusions I and II are true
(D) Neither conclusion I nor conclusion II are true.

43. Statement: Fortune favours the brave.


Conclusions:
I. Risk is necessary for success.
II. Cowards die many times before their death.
(A) Only conclusion I is true
(B) Only conclusion III is true
(C) Both conclusion I and II are true
(D) Neither conclusion I nor conclusion II are true

44. Statement : Morning walks are good for health.


Conclusions:
I. All healthy people go for morning walks.
II. Evening walks are harmful
(A) Only conclusion I is true
(B) Only conclusion II is true
(C) Both conclusions I and II are true
(D) Neither conclusion I nor conclusion II are true

Directions (45) : In each of the following questions two statements are given followed by two concision
I and II.

45. Statements:
(i) Without rains the crops will not be good,
(ii) The crops were good
Conclusions:
I. There were rains
II. Crops were good due to good fertilizers
(A) Only conclusion I is true
(B) Only conclusion II is true
(C) Both conclusion I & II are true
(D) Neither conclusion I nor conclusion II is true (NTSE Stage-I/Raj./2015)

487 PACE IIT & MEDICAL: Mumbai / Delhi & NCR / Goa / Akola / Kolkata / Nashik / Pune / Bokaro / Dubai
NTSE-MENTAL ABILITY

Answer Key

1. (a) 2. (d) 3. (d) 4. (c) 5. (b) 6. (d) 7. (b) 8. (b) 9. (b) 10. (c)
11. (d) 12. (c) 13. (b) 14. (c) 15. (a) 16. (c) 17. (a) 18. (c) 19. (c) 20. (d)

21. (b) 22. (d) 23. (b) 24. (d) 25. (d) 26. (a) 27. (d) 28. (d) 29. (d) 30. (c)

31. (a) 32. (c) 33. (d) 34. (a) 35. (a) 36. (c) 37. (d) 38. (b) 39. (d) 40. (b)

41. (d) 42. (b) 43. (d) 44. (d) 45. (a)

488 PACE IIT & MEDICAL: Mumbai / Delhi & NCR / Goa / Akola / Kolkata / Nashik / Pune / Bokaro / Dubai
NTSE-MENTAL ABILITY

34. DATA SUFFICIENCY

Directions (1 to 8): Each of the questions below consists of a question and two statements numbered I and
II given below it. You have to decide whether the data provided in the statements are sufficient to answer
the question.
Read both the statements and give answer
(A) if the data in statement I alone are sufficient to answer the question, while the data in statement II alone are
not sufficient to answer the question;
(B) if the data in statement II alone are sufficient to answer the question, while the data in statement I alone are
not sufficient to answer the question;
(C) if the data either in statement I alone or in statement II alone are sufficient to answer the question; and
(D) if the data given in both statements I and II together are necessary to answer the question.

1. Rajan and Madan start from a common point and walk straight in the opposite directions. What will be
the distance between them at the expiry of three hours?
I. Rajan and Madan walk at the speeds of 4km/hr and 6 km/hr respectively.
Il. Madan walks one and a half times faster than Rajan.
Sol. (A): From I, we find that:
distance between Rajan and Madan in 3 hours = [(4 + 6)  3) km = 30 km.
3 
 x
From II, we conclude that if Rajan’s speed is x km/hr, then madan’s speed =  2  km/hr. But, the
actual speed of none of them cannot be as certained.

2. It is 8.00 p.m., when can Hemant get next bus for Ramnagar from Dhanpur?
I. Buses for Ramnagar leave after every 30 minutes, till 10 p.m.
II. Fifteen minutes ago, one bus has left for Ramnagar.
Sol. (D) : II reveals that the previous bus had left at 7.45 p.m. As given in I, the next bus would leave after 30
minutes i.e. at 8.15 p.m.

3. What will be the total weight of 10 poles, each of the same weight?
I. One-forth of the weight of each pole is 5 kg.
Il. The total weight of three poles is 20 kilograms more than the total weight of two poles.
Sol. (C): From I, we conclude that weight of each pole =  4  5 kg = 20 kg.
So, total weight of 10 poles = (20  10) kg = 200 kg.

4. Vinod’s and Javed’s salaries are in the proportion of 4 : 3 respectively. What is Vinod’s salary?
I. Javed’s salary is 75% that of Vinod’s salary.
Il. Javed’s salary is Rs. 4500.
Sol. (B) : Statement I is merely an interpretation of the information contained in the question.
However, Vinod’s salary can be ascertained from II as follows;
Let Vinod’s and Javed’s salaries be 4x and 3x respectively.
Then, 3x = 4500 or x =1500.
Vinod’s salary = 4x = Rs. 6000.

489 PACE IIT & MEDICAL: Mumbai / Delhi & NCR / Goa / Akola / Kolkata / Nashik / Pune / Bokaro / Dubai
NTSE-MENTAL ABILITY

5. How many New Year’s greeting cards were sold this year ¡n your shop ?
I. Last year 2935 cards were sold.
II. The number of cards sold this year was 1.2 times that of last year.
Sol. (D): From both I and II, we find that the number of sold this year (2935  1 .2) = 3522.

6. How many speeches were delivered in the two days programme?


I. 18 speakers were invited to give at least one speech, out of which one-sixth of the speakers could not
come.
II. One -third of the speakers gave two speeches each.
Sol. (D): From I, we find that number of speakers who attended programme = 18  1 of 18 = 15.
6
From II, we find that one -third of 15 i.e. 5 speakers gave 2 speeches each, while each of the remaining
10 speakers delivered only one speech.
So, total number of speeches delivered = (5  2+10  1) 20.

7. What is the monthly salary of Prashant?


I. Prashant gets 15% more than Sumit while Sumit gets 10% less than Lokesh.
II. Lokesh’s monthly salary is Rs 2500.
Sol. (D): From both I and II, we find that:
Prashant’s salary = 115% of (90% of Rs 2500) = Rs 2587. 50.

8. How many pages of book X did Robert read on Sunday?


I. The book has 300 pages out of which two-thirds were read by him before Sunday.
II. Robert read the last 40 pages of the book on the morning of Monday.
Sol. (D) : From I and II, we find that Robert read  300 2  i.e. 200 pages before Sunday and the last 40 pages
 
 3
on Monday. This means that he read [300 – (200+40)] i.e. 60 pages on Sunday.

EXERCISE

Directions (1 to 10): Each of the questions below consists of a question and two statements numbered I
and II given below it. You have to decide whether the data provided in the statements are sufficient to
answer the question.
Read both the statements and give answer
(a) If the data in statement I alone are sufficient to answer the question, while the data in statement II
alone are not sufficient to answer the question;
(b) If the data in statement II alone are sufficient to answer the question, while the data in statement I
alone are not sufficient to answer the question;
(c) If the data either in statement I alone or in statement II alone are sufficient to answer the question;
and
(d) If the data given in both statements I and II together are necessary to answer the question.
(e) If the data given in both statements I and II together are not sufficient to answer the question.

1. In what proportion would Raj, Karan and Altaf distribute profit among them?
I. Raj gets two-fifth of the profit.
II. Raj and Altaf have made 60% of the total investment

490 PACE IIT & MEDICAL: Mumbai / Delhi & NCR / Goa / Akola / Kolkata / Nashik / Pune / Bokaro / Dubai
NTSE-MENTAL ABILITY

2. On which day in April is Gautams birthday?


I. Gautam was born exactly 28 years after his mother was born.
II. His mother will be 55 years 4 months and 5days on August 18 this year.

3. Total money with Naresh and Ajay is 28 percent of that with Usman. How much money is Ajay
having?
I. Usman has got Rs 75000.
II. The ratio of money of Naresh to money held by Ajay is 1 : 3.

4. Can Ritesh retire from office X in January 2006 ,


with full pension benefits?
I. Ritesh will complete 30 years of service in office X in April 2000 and desires to retire.
II. As per office X rules, an employee has to complete minimum 30 years of service and attain
age of 60. Ritesh has 3 years to complete age of 60.

5. When is Manohar’s birthday this year?


I. It is between January 13 and 15, January 13 being Wednesday.
II. It is not on Friday.

6. What is the value of 144 $ 16  7 # 9?


I. $ means ,  means  and # means +.
II. 16 $ 4  2 # 2 = 10

7. How many visitors saw the exhibition yesterday?


I. Each entry pass holder can take up to three persons with him/her.
II. In all, 243 passes were sold yesterday.

8. How many doctors are practising in this town?


I. There is one doctor per seven hundred residents.
II. There are 16 wards with each ward having as many doctors as the number of wards.

9. The area of a playground is 1600 square metres.


What is its perimeter?
I. It is a perfect square playground.
II. It costs Rs. 3200 to put a fence around the playground at the rate of Rs. 20 per metre.

10. Rajeev’s monthly salary is Rs. 4000. What is Atul’s monthly salary?
I. Atul gets Rs 500 more than the average salary of his and Rajeev’s.
II. Average of Rajeev’s and Atul’s salary is Rs. 4500.

491 PACE IIT & MEDICAL: Mumbai / Delhi & NCR / Goa / Akola / Kolkata / Nashik / Pune / Bokaro / Dubai
NTSE-MENTAL ABILITY

Directions : (11 to 15) In each of the following a question is followed by three statements marked I, II and
III. Decide which of the statements are sufficient to answer the question. Choose your answer from the
given alternatives. (NTSE Stage-II, 2009)

11. How much does A have?


Statements:
I. B has half of what A and C together have,.
II. A has half of what C has.
III. A. B and C together have Rs. 360.
(a) Only II and III are sufficient (b) All of I, II and III are required
(c) Only I and II are sufficient (d) Only I and III are sufficient

12. How long would A take to complete the work?


Statements:
I. B is half as efficient as A.
II. B and C together complete the work in 8 days
III. A and B together complete the work in 4 days
(a) Only III is sufficient (b) I and II are sufficient
(c) All of I, II and III are necessary (d) Only I and III are sufficient

13. What is the present age of the son ?


Statements:
I. Five years ago, father was twice as old as the son.
II. Now father is 18 years older than the son.
III. The age of father and mother taken together is
30 years more than the son.
(a) Only I and III are sufficient (b) Only II and III are sufficient
(c) Only I and II are sufficient (d) All of 1, II and HT are not sufficient

14. How much do 2 apples and 3 oranges cost?


Statements:
I. 3 apples and 5 oranges cost Rs. 25.
II. 5 apples and 7 oranges cost Rs. 39
III. 1 apple, 1 orange and 1 guava cost Rs. 8
(a) Only I and II are sufficient (b) Only I and III are sufficient
(c) Only II and III are sufficient (d) All of I, II and III are necessary.

15. What is the distance between A and D?


Statements:
I. B is 8 km East of A and 5 km North of C.
II. D is located 5 kms South of E.
III. E is located 4 km East of B and 5 km North of D
(a) Only I and II are sufficient (b) Only I and III are sufficient
(c) Only II and III are sufficient (d) All of I, II and III are required

492 PACE IIT & MEDICAL: Mumbai / Delhi & NCR / Goa / Akola / Kolkata / Nashik / Pune / Bokaro / Dubai
NTSE-MENTAL ABILITY

16. What is Mohan’s age? (NTSE Stage-II, 2010)


Statements:
I. In 15 years Mohan will be twice as old as Ram would be.
II. Ram was born 5 years ago.
(a) Only I is sufficient. (b) Only II is sufficient.
(c) Both I and II are required. (d) Both I and II are not sufficient.

17. Who is a better singer D or F? (NTSE Stage-II, 2010)


Statements:
I. F sings better than both G and S.
II. Neither S nor F singe so well as D.
(a) Only I is sufficient. (b) Only II is sufficient.
(c) Both I and II are required. (d) Both I and II are not sufficient.
18. Vinod and Pramod are standing together on a sunny day. Vinods shadow is 10 feet long and
Pramods shadow is 9 feet long. How tall is Pramod? (NTSE Stage-II, 2011)
Statement:
I. Vinod is 6 feet tall
II. Vinod is standing 2 feet away from Pramod.
(a) Statement I alone is sufficient to answer the problem.
(b) Statement I along is sufficient to answer the problem
(c) Statement I and II both are needed.
(d) Statement I and II both are not sufficient.

19. Rohit took a test had 60 questions numbered from 1 to 60 . How many questions did he answer
correctly in the. second half of the test?
Statement: (NTSE Stage-II, 2011)
I. The number of questions he answered correctly in the second half of test was 7 less than the
number he answered correctly in the first half of the test.
II. He answered 5/6 of the odd numbered questions correctly and 4/5 of the even numbered
correctly.
(a) Statement I along is sufficient answer the problem
(b) Statement II along is sufficient to answer the problem.
(c) Statements I and II both are needed.
(d) Statements I and II both are not sufficient.

20. Train A leaves town X for town Y and travels at a constant speed. At the same time train B leaves
town Y for town X and also travels at a constant speed. Town Z is between X and Y. Towns X, Z
and Y lie on a straight line. Which train has greater speed ? (NTSE Stage-II, 2011)
Statements:
I. Train B arrives at town Z before train A.
II. Town Z is closer to X than Y
(a) Statement I alone is sufficient to answer the problem.
(b) Statement II alone is sufficient to answer the problem.
(c) Statement I and II both are needed.
(d) Statement I and li both are not sufficient

493 PACE IIT & MEDICAL: Mumbai / Delhi & NCR / Goa / Akola / Kolkata / Nashik / Pune / Bokaro / Dubai
NTSE-MENTAL ABILITY

21. Who is the father of M? (NTSE Stage-II,2011)


Statements:
I. P and Q are brothers.
II. Q’s wife is the sister of M’s wife
(a) Statement I alone is sufficient to answer the problem
(b) Statement II alone is sufficient to answer the problem
(c) Statements I and II both are needed.
(d) Statements I and II both are not sufficient

22. Question given below has a problem and two statements I & II. Decide if the information given
In the statement is sufficient for answering the problem. (NTSE Stage-II, 2013)
K, R, S and T are four players in Indian Cricket team. Who is the oldest among them?
I: The total age of K & T together is more than that of S
II: The total age of R & K together is less than that of S
(a) Data in statement I alone is sufficient
(b) Data in statement II alone is sufficient
(c) Data n both statement together is sufficient
(d) Data in both statement together is not sufficient

Answer Key

1. (d) 2. (d) 3. (d) 4. (d) 5. (a) 6. (c) 7. (e) 8. (b) 9. (c) 10. (c)
11. (b) 12. (d) 13. (c) 14. (a) 15. (b) 16. (c) 17. (b) 18. (a) 19. (c) 20. (c)

21. (d) 22. (d)

494 PACE IIT & MEDICAL: Mumbai / Delhi & NCR / Goa / Akola / Kolkata / Nashik / Pune / Bokaro / Dubai
NTSE-MENTAL ABILITY

35. DATA REDUNDANCY


This section consists of Problems in which questions from topics such as Coding-Decoding Blood Relations,
Puzzle Test, Direction Sense Test, Ranking and Time Sequence Test, Arithmetical Reasoning etc. are given,
each question is followed by certain statements containing facts and clues to solve the question. The candidate
is required to find out which of the given statement/s is/are not required to answer the question and hence can be
dispensed with.

Directions : (1 to 5) Each of the following questions consists of information in three statements. Study the
question and the statements and decide which of the statement/s is/are not required to answer the
question and hence can be dispensed with.

1. How many boys are there in the class?


I. The total number of students in the class is 200.
II. The ratio of boys to girls in the class is 2 : 3.
III. The difference in the number of boys and girls is 40.
(A) Only II
(B) Only III
(C) Either (A) or (B)
(D) Anyone of the three statements can be dispensed with
Sol. (D) The question can be answered using any set of two statements. Hence, any one of the three
statements can be dispensed with.

2. In the adjoining figure find ABC .

I. ACD =120o
II. BAC = 40o
III. AB = AC
(A) Only I
(B) Only II
(C) Only III
(D) Anyone of the three statements can be dispensed with
Sol. (D) The question can be answered using any set of two statements. Hence, any one of the three
statements can be dispensed with.

3. What is the average salary of 15 employees ?


I. Average salary of 7 clerical cadre (out of the 15 employees) is Rs. 8500.
II. Average salary of 5 officer cadre (out of the 15 employees) is Rs. 10000.
III. Average salary of the 3 sub-staff employees (out of the 15 employees) is Rs. 2500.
(A) None (B) Only I (C) Only II (D) Only III

495 PACE IIT & MEDICAL: Mumbai / Delhi & NCR / Goa / Akola / Kolkata / Nashik / Pune / Bokaro / Dubai
NTSE-MENTAL ABILITY

Sol. (A) I. gives, total salary of 7 clerks = Rs. (8500  7) = Rs. 59500.
II. givens, total salary of 5 officers = Rs. (10000  5) = Rs. 50000
III. gives total salary of 3 sub = staff members = R. (2500  3) = Rs. 7500.
Total salary of 15 employees = Rs. (59500 + 50000 + 7500) = Rs, 117000.
 11700 
 
Average salary = Rs.  15  Rs.7800.
All given statements are needed. Hence, none is redundant.

4. What is the ratio of the present ages of Anna and her mother?
I. The sum of the ages of Anna, her mother and her father is 62.
II. Five years ago, Anna’s age was one-fifth of her father’s age.
III. Two years ago, the sum of the ages of Anna and her father was 36.
(A) I or II only (B) II or III only (C) I or III only (D) All I, II and III are required.
Sol. (D) I. A + M + F = 62.
1
 A  5   F  5
II. 5
III. (A – 2) + (F – 2) = 36
From II and III we may get A and F.
putting these values in I, we get M.
Thus, all I, II and III are required to get the answer.

5. What is the marked price of the suitcase ?


I. When a discount of 15% is offered, the profit earned is 10.5%.
II. The cost price of the suitcase is Rs. 1500.
III. The marked price is 30% above the cost price.
(A) I only
(B) Either I or III
(C) Any one of the three
(D) All I, II and III are required
Sol. (B) I. Let C.P. be Rs. x. Then, M.P. 130% of x Rs. 13x
10

EXERCISE

Directions : (1 to 14) Each of the following questions consists of information in three statements. Study
the question and the statements and decide which of the statement/s is/are not required to answer the
question and hence can be dispensed with.

1. What will be the ratio between ages of Sam and Albert after 5 years?
I. Sam’s present age is more than Albert’s present age by 4 years.
II. Albert’s present age is 20 years.
III. The ratio of Albert’s present age to Sam’s present age is 5 : 6.
(a) I or II or III only (b) II only (c) III only (d) I or III only

496 PACE IIT & MEDICAL: Mumbai / Delhi & NCR / Goa / Akola / Kolkata / Nashik / Pune / Bokaro / Dubai
NTSE-MENTAL ABILITY

2. What is the difference between the present ages of Ayush and Deepak?
I. The ratio between Ayush’s present age and ish age after 8 years is 4 : 5
II. The ratio between the present ages of Ayush and Deepak is 4 : 3
III. The ratio between Deepak’s present age and his age four years ago is 6 : 5
(a) Any two of I, II and III (b) I and III only
(c) Any one of the three (d) Any I, II and III are required

3. What is the percent profit earned by the shop keeper on selling the articles in his shop?
I. Labelled price of the articles sold was 130% of the cost price.
II. Cost price of each article was Rs. 550.
III. A discount of 10% on labelled price was of fered.
(a) Only I (b) Only II
(c) Only III (d) All the three are required

4. Three friends P, Q and R started a partnership business investing money in the ratio of 5 : 4 : 2
respectively for a period of 3 years. What is the amount received by P as his share in the total
profit?
I. Total amount invested in the business in Rs. 22,000.
3
II. Profit earned at the end of 3 years is of the total investment.
8
III. The average amount of profit earned per year is Rs. 2750.
(a) I or II or III (b) Either III only, or I and II together
(c) Any two of the three (d) All I, II and III are required

5. What will be the percentage share of Y in the profit earned by X, Y and Z together?
I. X, Y and Z invested a total amount of Rs. 25,000 for a period of two years.
II. The profit earned at the end of 2 years is 30%.
III. The amount invested by Y is equal to the amount invested by X, Z and Z together.
(a) I and II only (b) II and III only
(c) Any two of the three (d) All I, II and III are required

6. 8 men and 14 women are working together in a field. After working for 3 days, 5 men and 8 women
leave the work. How many more days will be required to complete the work?
I. 19 men and 12 women together can complete the work in 18 days.
II. 16 men can complete two-third of the work in 16 days.
III. In a day, the work done by three men is equal to the work done by four women.
(a) I only (b) II only (c) III only (d) I or II or III

7. At what time will the train reach city X from city Y?


I. The train crosses another train of equal length of 200 metres and running in opposite direction
in 15 seconds.
II. The train leaves city Y at 7.15 a.m. for city x situated at a distance of 558 km.
III. The 200 metres long train crosses a signal pole in lo seconds.
(a) I only (b) II only (c) III only (d) I or III only

497 PACE IIT & MEDICAL: Mumbai / Delhi & NCR / Goa / Akola / Kolkata / Nashik / Pune / Bokaro / Dubai
NTSE-MENTAL ABILITY

8. What would be the difference between the simple interest and the compound interest on a sum of
money at the end of four years?
I. The rate of interest ¡s 5 p.c.p.a.
II. The sum fetches total of Rs. 2000 as simple interest at the end of 8 years.
III. The difference between the simple interest and the compound interest at the end of 2 years is
Rs. 12.50.
(a) II only (b) III only
(c) II or III only (d) All I, II and III are required

9. Mr. Gupta borrowed a sum of money on com pound interest. What will be the amount to be
repaid if he is repaying the entire amount at the end of 2 years?
I. The rate of interest fetched on the same amount in one year is Rs. 600.
II. Simple interest fetched on the same amount in one year is Rs. 600.
III. The amount borrowed is 10 times the simple interest in 2 years.
(a) I only (b) III only (c) I or II only (d) I or III only

10. What is the total compound interest earned at the end of 3 years?
I. Simple interest earned ion that amount at the same rate and for the same period is Rs. 4500.
II. The rate of interest is 10 p.c.p.a.
III. Compound interest for 3 years is more than the simple interest for that period by Rs. 465.
(a) I and II only (b) II and III only (c) I and III only (d) Either II or III only

11. What is the rate of interest per annum?


I. The amount becomes Rs. 11,025 with compound interest after 2 years
II. The same amount with simple interest becomes Rs. 11,000 after 2 years.
III. The amount invested is Rs. 10,000.
(a) I or II only (b) II or III only (c) I or III only (d) I or II or III only

12. What is the cost of painting the two adjacent walls of a hall at Rs. 5 per m2 which has no windows
or doors?
I. The area of the hail is 24 sq.m.
II. The breadth, length and height of the hall are in the ratio of 4 : 6 : 5 respectively
III. Area of one walls 30 sq. m.
(a) I only (b) II only (c) III only (d) Either I or III

13. What is the area of the given right-angled triangle?


I. Length of the hypotenuse is 5 cm.
II. Perimeter of the triangle is four times its base.
III. One of the angles of the triangle is 60o.
(a) II only (b) III only (c) II or III only (d) II and III both

14. What will be the cost of painting the four walls of a room with length, width and height 5 m, 3 m
and 8 m respectively? The room has one door and one window.
I. Cost of painting per sq. m is Rs. 25.
II. Area of window is 2.25 sq. m which is half of the area of the door.
III. Area of the room is 15 sq. m.
(a) I only (b) II only (c) III only (d) II or III only

498 PACE IIT & MEDICAL: Mumbai / Delhi & NCR / Goa / Akola / Kolkata / Nashik / Pune / Bokaro / Dubai
NTSE-MENTAL ABILITY

Answer Key

1. (a) 2. (c) 3. (b) 4. (b) 5. (a) 6. (d) 7. (a) 8. (c) 9. (d) 10. (d)
11. (d) 12. (d) 13. (c) 14. (c)

499 PACE IIT & MEDICAL: Mumbai / Delhi & NCR / Goa / Akola / Kolkata / Nashik / Pune / Bokaro / Dubai

You might also like